You are on page 1of 497

Section - A

Income Tax
Act Basics
Syllabus
Paper 7 : Direct Taxation

Syllabus Structure

A Income Tax Act Basics 10%

B Heads of Income and Computation of Total Income and Tax 70%


Liability

C Tax Management, Administration Procedure and ICDS 20%

OBJECTIVES
To gain knowledge about the direct tax laws in force for the relevant
previous year and to provide an insight into procedural aspects for
assessment of tax liability for various assessees.

Learning Aims
The syllabus aims to test the student’s ability to :
 Understand the basic principles underlying the Income Tax Act,
 Compute the taxable income of an assessee,
 Analyze the assessment procedure

Skill sets required


Level B : Requiring the skill levels of knowledge, comprehension,
application and analysis.

7.3
Note : Subjects related to applicable statutes shall be read with
amendments made from time to time.
Section A : Income Tax Act Basics 10%
1. Introduction to Income Tax Act, 1961
2. Income which do not form part of Total Income (Section 10, 11
to 13A)
Section B : Heads of Income and Computation of Total Income 70%
and Tax Liability
3. Heads of Income and Computation of Total Income under
various heads
4. Clubbing Provisions, Set off and Carry forward of Losses,
Deductions
5. Assessment of Income and tax liability of different persons
Section C : Tax Management, Administrative Procedures and 20%
ICDS
6. TDS, TCS and Advance Tax
7. Administrative Procedures
8. ICDS

Section A : Income Tax Act Basics (10 Marks)


1. Introduction to Income tax Act, 1961
a. Constitutional Validity
b. Basic Concepts and definitions
c. Capital and Revenue Receipts
d. Basis of charge and scope of total income
e. Residential status and incidence of tax (excluding section 9A)
2. Income, which do not form part of total income [Sec. 10, 11 to 13A]

7.4
Section B : Heads of Income and computation of total income and
tax liability [70 marks]
3. Heads of income and computation of total income under various
heads
a. Salaries
b. Income from House Property
c. Profits and Gains of Business or Profession (excluding 42 to
44DB but including sections 43A, 43B, 43AA, 44AB, 44AD,
44ADA and 44AE)
d. Capital Gains
e. Income from Other Sources
4. Clubbing Provisions, set off and carry forward of losses, deductions
a. Income of other persons included in assessee’s total income
b. Aggregation of income and set off and carry forward of losses
c. Deductions in computing total income
d. Rebate and Reliefs
e. Applicable Rates of tax and tax liability
5. Assessment of income and tax liability of different persons
a. Taxation of individual (including AMT but excluding non-resident)
b. HUF
c. Firms, LLP and Association of Persons
d. Co-operatives Societies
Section C : Tax Management, Administrative Procedure and ICDS
[20 Marks]
6. TDS, TCS and Advance Tax
a. Tax Deduction at Source (excluding sections relevant to
non-residents)
b. Tax Collected at Source
c. Advance Tax
7. Administrative Procedures
a. Return & PAN
b. Intimation
c. Brief concepts of Assessment u/s 140A, 143 and 144
8. ICDS
a. Basic Concepts of ICDS

7.5
Examination Trend Analysis

Paper 7 Direct Taxation

Question Paper Based Contents of Last Five Examinations


Years Q. No. Chapter Page
No. Name No.
2016 1. (a) 21 Objective Questions 450
Dec. (b) 21 " " " 451
(c) 21 " " " 452
(d)(i) 8 Capital Gains 213
(ii) 5 Income under head Salaries 97
(iii) 15 Tax Deducted at Source 362
2. (b) 6 Income from House Property 124
(c) 5 Income under head Salaries 97
3. (a) 5 " " " 98
(b) 8 Capital Gains 213
4. (a)(i) 4 Income, which do not form Part of Total
Income 72
(ii) 2 Residential Status 46
(iii) 4 Income, which do not form Part of Total
Income 72
(b) 12 Deductions in Computing Total Income 276
(c) 7 Profits & Gains of Business or Profession 173
5. (a) 7 " " " " " 174
(b) 15 Tax Deducted at Source 362
(c) 11 Set Off & Carry Forward of Losses 259
6. (a) 17 Advance Tax 376
(b) 14 Assessment of Various Persons 319
(c) 7 Profits & Gains of Business or Profession 174
2017 1. 21 Objective Questions 453
June 2. (a) 2 Residential Status 46
(b) 5 Income under head Salaries 99
3. (a) 7 Profits & Gains of Business or Profession 175
(b) 14 Assessment of Various Persons 321
4. (b) 14 " " " " 323
5. (a) 14 " " " " 325
(b) 11 Set Off & Carry Forward of Losses 260
7.6
6. (a) 14 Assessment of Various Persons 326
7. (b) 16 Advance Tax 372
8. (a) 20 Income Computation and Disclosure
Standards 405
(b) 19 Assessment Procedure 395
(c) 4 Income, which do not form Part of Total
Income 58
(d) 18 Return and Pan 378
2017 1. 21 Objective Questions 457
Dec. 2. (a) 6 Income from House Property 126
(b) 14 Assessment of Various Persons 328
3. (a) 14 " " " " 329
(b) 15 Tax Deducted at Source 363
4. (a) 14 Assessment of Various Persons 331
(b) 5 Income under head Salaries 101
5. (a) (i) 8 Capital Gains 215
(ii) 8 " " 215
(iii) 9 Income from other Sources 227
(iv) 10 Clubbing of Income 247
(v) 8 Capital Gains 215
(vi) 8 " " 215
(b) 9 Income from other Sources 228
6. (a) 11 Set Off & Carry Forward of Losses 261
(b) 12 Deductions in Computing Total Income 277
7. (b) 19 Assessment Procedure 390
8. (a) 20 Income Computation and Disclosure
Standards 405
(b) 18 Return and Pan 378
(c) 18 " " " 378
(d) 17 Advance Tax 369
2018 1. 21 Objective Questions 461
June 2. (a) (i) 14 Assessment of Various Persons 333
(ii) 1 Basic Concepts 32
(b) 6 Income from House Property 127
3. (a) 14 Assessment of Various Persons 334
(b) 15 Tax Deducted at Source 364
4. (a) 14 Assessment of Various Persons 338
(b) 3 Agricultural Income 55
5. (a) 7 Profits & Gains of Business or Profession 177

7.7
(b) 18 Return and Pan 391
6. (a) 14 Assessment of Various Persons 341
(b) 11 Set Off & Carry Forward of Losses 263
7. (a) (i) 9 Income from other Sources 228
(ii) 9 " " " " 228
(iii) 14 Assessment of Various Persons 345
(iv) 9 Income from other Sources 228
(v) 9 " " " " 228
(b) 14 Assessment of Various Persons 345
8. (a) 18 Return and Pan 379
(b) 19 Assessment Procedure 396
(c) 20 Income Computation and Disclosure
Standards 406
(d) 18 Return and Pan 379
2018 1. 21 Objective Questions 465
Dec. 2. (a) 2 Residential Status 48
(b) 6 Income from House Property 130
3. (a) 5 Income under head Salaries 103
(b) 3 Agricultural Income 56
4. (a) 8 Capital Gains 216
(b) 11 Set Off & Carry Forward of Losses 264
5. (a) 7 Profits & Gains of Business or Profession 180
(b) 14 Assessment of Various Persons 346
6. 14 " " " " 347
7. (a) 9 Income from other Sources 230
(b) 17 Advance Tax 372
8. (a) 18 Return and Pan 381
(b) 18 " " " 381
(c) 20 Income Computation and Disclosure
Standards 406
(d) 19 Assessment Procedure 396

7.8
7.9
Frequency Table Showing Distribution of Marks
Chap. Years 14 14 15 15 16 16 17 17 18 18
No. Chapter Name Total Ave.
June Dec. June Dec. June Dec. June Dec. June Dec.
1. Basics Concepts 3 5 3 11 1.1
2. Residential Status 3 4 5 2 8 9 31 3.1
3. Agricultural Income 2 5 6 13 1.3
4. Income, which do not form Part of 3 3 4 4 5 19 1.9
Total Income

5. Income under head Salaries 10 7 4 11 12 7 8 9 68 6.8

6. Income from House Property 12 6 3 7 8 9 6 7 6 64 6.4


7. Profits & Gains of Business or 4 4 26 12 7 15 6 9 10 93 9.3
Profession

8. Capital Gains 5 21 15 12 7 9 8 8 85 8.5


9. Income from other Sources 5 1 2 5 8 8 29 2.9
10. Clubbing of Income 4 10 2 16 1.6
11. Set Off and Carry Forward of Losses 3 3 3 5 5 6 8 5 7 45 4.5

12. Deductions in Computing Total 11 4 7 22 2.2


Income

13. Rebate and Reliefs

14. Assessment of Various Persons 6 9 5 10 22 6 34 25 39 20 176 17.6

7.10
Chap. Years 14 14 15 15 16 16 17 17 18 18
Total Ave.
No. Chapter Name June Dec. June Dec. June Dec. June Dec. June Dec.
15. Tax Deducted at Source 4 4 3 6 6 8 31 3.1
16. Tax Collected at Source

17. Advance Tax 3 2 4 6 5 7 27 2.7

18. Return and Pan 3 3 1 5 18 16 10 56 5.6


19. Assessment Procedure 5 7 5 5 5 27 2.7
20. Income Computation and Disclosure 3 5 5 5 18 1.8
Standards

21. Objective Questions 11 13 13 15 15 25 25 25 25 167 16.7

7.11
Frequency Table Showing Marks of Compulsory Questions
Chap. Years 14 14 15 15 16 16 17 17 18 18
Total Ave.
No. Chapter Name June Dec. June Dec. June Dec. June Dec. June Dec.
1. Basics Concepts

2. Residential Status 4 4 0.4


3. Agricultural Income 2 2 0.2
4. Income, which do not form Part of Total 0 0.0
Income

5. Income under head Salaries 2 2 0.2


6. Income from House Property

7. Profits & Gains of Business or 1 6 7 0.7


Profession

8. Capital Gains 4 2 6 0.6


9. Income from other Sources 1 1 0.1
10. Clubbing of Income 2 2 0.2
11. Set Off and Carry Forward of Losses

12. Deductions in Computing Total Income

13. Rebate and Reliefs

14. Assessment of Various Persons

7.12
Chap. Years
14 14 15 15 16 16 17 17 18 18
No. Total Ave.
June Dec. June Dec. June Dec. June Dec. June Dec.
Chapter Name
15. Tax Deducted at Source 1 1 0.1
16. Tax Collected at Source

17. Advance Tax 2 2 0.2


18. Return and Pan 1 1 0.1
19. Assessment Procedure 2 2 0.2
20. Income Computation and Disclosure
Standards

21. Objective Questions 11 13 13 15 15 25 25 25 25 167 16.7

7.13
Short Notes

Distinguish Between

7.14
Legends for the Graphs

Descriptive

Practical
Important Updates

Tax Rate for AY 2018-19


1. For Resident Very Senior Citizen i.e. 80 years or more at any time
during PY [Born before April 1, 1938]:
Income Tax Rate
0 - 5,00,000 Nil
5,00,001- 10,00,000 20%
More than 10,00,000 30%
2. For Resident Senior Citizen i.e. 60 years or more but less than 80
years at any time during PY [Born during April 1, 1938 and March
31, 1958]:
Income Tax Rate
0 - 3,00,000 Nil
3,00,001- 5,00,000 5%
5,00,001-10,00,000 20%
More than 10,00,000 30%

3. For Other Individual & HUF:


Income Tax Rate
0 - 2,50,000 Nil
2,50,001- 5,00,000 5%
5,00,001-10,00,000 20%
More than 10,00,000 30%
Other Individuals means:
(A) A male or female whether resident or non- resident below 60 years.
(B) Non-resident very senior or senior citizen.
4. Firm & Limited Liability Partnership:
Flat tax rate of 30%.

7.15
7.16 O Scanner CMA Inter Gr. I Paper 7 (2016 Syllabus)

5. Domestic Company:
Whether the Total Turnover or the Gross Receipts in 25%
previous year 2015-16 does not exceed ` 50 crores.

All other domestic company 30%

6. Foreign Company:
Flat tax rate of 40%.
Surcharge:
Assessee Threshold Limit (If Total Rate of
Income Exceeds) Surcharge
HUF Individual exceeds 50 lakh but does not
exceed. 1 Crore 10%
Individual / HUF exceeds ` 1 Crore 15%
Firm/LLP 1 Crore 12%
Domestic Company 1 Crore 7%
Domestic Company 10 Crores 12%
Foreign Company 1Crore 2%
Foreign Company 10 Crores 5%

Rebate [Section 87A]:


A resident individual having total income upto ` 3.5 lacs shall be eligible
for a rebate of ` 2,500, restricted to the amount of tax payable by him.
Rebate u/s 87A is not available to any other assessee. This rebate is
available before charging education cess.
Health and Education Cess: For all the above assessees @ 4% of the
Total Tax Payable.
Rounding off of Income [Section 288A]:
Taxable income is rounded off to the nearest multiple of ` 10.
Important Updates O 7.17

For Instance:
1. If the taxable income is ` 4,65,294.95 then the income shall be taken as
` 4,65,290.
2. If income is ` 4,65,495, it shall be taken as ` 4,65,500.
Rounding off of Tax [Section 288B]:
Tax payable or the refund due shall be rounded off to the nearest multiple of
` 10.
The Income Declaration Scheme, 2016
An opportunity provided to persons who have not paid full taxes in the past
to come forward and declare the undisclosed income and pay tax, surcharge
and penalty totaling in all to 45% of such undisclosed domestic income
declared.
The scheme is effective from 1st June 2016 to 30th Sep. 2016. The scheme
is made applicable in respect of undisclosed income of any financial year
upto 2015-16.
It is provided under the scheme that where the income chargeable to tax is
declared in the form of investment in any asset, the cost of acquisition of
such asset shall be deemed to be the fair market value.
It is proposed that following cases shall not be eligible for the scheme:
 where notices have been issued under section 142(1) or 143(2) or 148
or 153A or 153C, or
 where a search or survey has been conducted and the time for issuance
of notice under the relevant provisions of the Act has not expired, or
 where information is received under an agreement with foreign countries
regarding such income,
 cases covered under the Black Money Act, 2015, or
 persons notified under Special Court Act, 1992, or
 cases covered under Indian Penal Code, the Narcotic Drugs and
Psychotropic Substances Act, 1985, the Unlawful Activities
(Prevention) Act, 1967, the Prevention of Corruption Act, 1988.
Special Rates of Income Tax:
On Short-term Capital Gain Covered u/s 111A (Listed 15%
Share/ Unit)
On Long-term Capital Gain (Listed Share/ Unit) Exempt
7.18 O Scanner CMA Inter Gr. I Paper 7 (2016 Syllabus)

On Long-term Capital Gain covered by Proviso to Section


112 10%
(Listed Bond/ Debenture)
On Long-term Capital Gain covered u/s 112 (Other Capital 20%
Asset)
On Winning of Lotteries, Crossword Puzzles, Card Game,
Races etc u/s 115BB 30%
TDS Rates:

Sr. Section Nature of payment Threshold Rate of


No. Limit TDS

1. Sec. 192 Payment of salary [normal Basic Normal


tax rates are applicable – Exemption Rate
SC : 15% (if net income
exceeds ` 1 crore), Health &
edu. Cess @ 4%]

2. Sec. 192A Payment of taxable 50,000.00 10%


accumulated balance of
provident fund

3. Sec. 193 Interest on securities —

a. i n t e r e s t on (a) 5,000.00 10%


debentures/securities
for money issued by or
on behalf of any local
authority/statutory
corporation, (b) listed
debentures of a
company [not being
listed securities in
demat form], (c) any
Important Updates O 7.19

security of the Central


or State Government
[i.e., 8% Savings
(taxable) Bonds, 2003,
but not any other
Government security]

b. any other interest on 10,000.00 10%


securities (including
interest on non-listed
debentures)
c. Interest on 7.75% GOI
S a vin g s (Taxab le )
Bonds, 2018. 10,000 10%

4. Sec. 194 Dividend


a. deemed dividend under NA Exempt
section 2(22)(e)
b. any other dividend NIL NIL
5. Sec. 194A Interest other than interest 10,000/5,000/ 10%
on securities 50,000
The threshold limit of
` 1 0 , 000 h a s b e e n
increased (w.e.f.
01/04/2018) to ` 50,000 if
the receipient of interest is
Senior citizen.
6. Sec. 194B Winnings from lottery or 10,000.00 30%
crossword puzzle or card
game or other game of any
sort
7. Sec. 194BB Winnings from horse races 10,000.00 30%
7.20 O Scanner CMA Inter Gr. I Paper 7 (2016 Syllabus)

8. Sec. 194C Payment or credit to a


resident contractor/sub
CONTRACTOR
a. payment/credit to an 1%
individual or a Hindu
undivided family 30,000 per
single contract
b. payment/credit to any or 1 lakh in 2%
person other than an aggregate
individual or a Hindu
undivided family
9. Sec. 194D Insurance commission
– payment/credit to an 15,000.00 5%
individual or a Hindu
undivided family
– payment/credit to any 15,000.00 5%
person other than an
individual or a Hindu
undivided family
10. Sec. 194DA Payment in respect of life 1,00,000.00 1%
insurance policy (applicable
from October 1, 2014)
11. Sec. 194E Payment to a non resident N.A. 20%
foreign citizen
sportsman/entertainer or
non resident sports
association
12. Sec. 194EE Payment in respect of 2,500.00 10%
deposits under National
Saving Scheme, 1987
13. Sec. 194F Payment on account of NIL 20%
repurchase of units of MF or
UTI
Important Updates O 7.21

14. Sec. 194G Commission on sale of 15,000.00 5%


lottery tickets
15. Sec. 194H Commission or brokerage 15,000.00 5%
16. Sec. 194I Rent —
a. rent of plant and 1,80,000.00 2%
machinery
b. rent of land or building 1,80,000.00 10%
or furniture or fitting
17. Sec. 194IA P a y m e n t / c r e d i t o f 50,00,000.00 1%
consideration to a resident
transferor for transfer of any
immovable property (other
than rural agricultural land)
Sec. 194IB R e n t P a ya b le b y a n 50,000 5%
Individual/HUF not covered P.M.
U/s 194I
Sec. 194IC Payment of consideration NA 10%
(not being in kind) under
joint development
agreement or other similar
agreement.
18. Sec. 194J Professional fees, technical 30,000.00 10%
fees, royalty or Note: With
remuneration to a director effect from
June 1,
2017 the
rate of
TDS would
be 2% in
case of
payee
engaged in
business
7.22 O Scanner CMA Inter Gr. I Paper 7 (2016 Syllabus)

of
operation
of call
center.
10%
19. Sec. 194LA Payment of compensation 2,50,000.00 10%
on acquisition of certain
immovable property
20. Sec. 194LB Payment/credit by way of N.A. 5%
interest by infrastructure
debt fund
21. Sec. Payment of the nature NIL 10%
194LBA(1) referred to in section
1 0 (2 3 FC) or se ct io n
10(23FC)(a) (with effect
from June 1, 2016) or
section 10(23FCA) by
business trust to resident
unit holders.
22. Sec. Payment of the nature N.A. 5%
194LBA (2) referred to in section
10(23FC) by business trust
to unit holders (applicable
from October 1, 2014).
23. Sec. Payment in respect of units NIL 10%
194LBB of investment fund
specified in Section115UB.
24. Sec. Payment in respect of an –
194LBC(1) investment in a
securitisation trust specified
in clause (d) of the
Explanation occurring after
section 115TCA (with effect
from June 1, 2016).
Important Updates O 7.23

– if recipient is an NIL 25%


individual or a Hindu
undivided family
– if recipient is any other Nil 30%
person
– if recipients is company NIL 40%
25. Sec. 194LC Payment/credit of interest by N.A. 5%
an Indian specified company
on foreign currency
approved loan/long term
infrastructure bonds (with
effect from October 1, 2014,
any bond) from outside India
26. Sec. 194LD I n t e r e s t o n a r u p e e N.A. 5%
denominated bond of an
Indian company or
Government security (from
June 1, 2013)
27. Sec. 195 Payment/credit of other sum N.A. –
to a non resident
a. income of foreign N.A. 20%
exchange assets
payable to an Indian
citizen.
b. income by way of long- N.A. 10%
term capital gains
referred to in section
115E or section
112(1)(c)(iii)b.
c. short-term capital gains N.A. 15%
under section 111A
7.24 O Scanner CMA Inter Gr. I Paper 7 (2016 Syllabus)

d. any other long-term N.A. 20%


capital gains [not being
covered by 196D
section 10(33), 10(36)
and 10(38)d].
e. income by way of N.A. 5%
interest payable by
Government/Indian
concern on money
borrowed or debt
incurred by Government
or Indian concern in
foreign currency (not
being interest referred
to in section 194LB or
194LC.
(f) royalty N.A. 10%
(g) royalty [not being N.A. 10%
royalty of the nature
referred to in (f) supra]
[see Note 6] –
where the agreement is N.A. 10%
made after March 31, 1961
but before April 1, 1976
where the agreement is N.A. 10%
made on or after April 1,
1976
(h) fees for technical N.A. –
services [see Note 7] –
where the agreement is N.A. 10%
made after February 29,
1964 but before April 1,
1976
Important Updates O 7.25

Where the agreement is N.A. 10%


made on or after April 1,
1976
(i) any other income N.A. 30%
Sec. 196 Interest/Dividend or other NIL
sum payable to Govt. RBI or
Certain Corporation.
28. Sec. 196B Payment/credit of income N.A. 10%
from units (including long
term capital gains on
transfer of such units) to an
offshore fund.
29. Sec. 196C Payment/credit of interest of N.A. 10%
foreign currency bonds or
GDR (including long term
capital gains on transfer of
such bonds) (not being
dividend referred to in
section 115 O.
30. Sec. 196D Payment/credit of income N.A. 20%
from securities (not being
dividend, short term or long
term capital gain) to Foreign
Institutional Investors.
7.26 O Scanner CMA Inter Gr. I Paper 7 (2016 Syllabus)

TCS RATES:
S. Nature of Goods/ Contract % of Purchase
No. Price

1. Alcoholic Liquor for human consumption 1%

2. Tendu leaves 5%

3. Timber or any other forest produce not being tendu 2.5%


leaves

4. Scrap 1%

5. Parking lot, Toll Plaza, Mining and quarrying (shall 2%


not Include mining & quarrying of mineral oil i.e.
petroleum and natural gas)

6. Sale of minerals being coal or lignite or iron ore if the 1%


same was not purchased by buyer of personal
consumption or for manufacturing, processing or
producing article or things or Generation of power.

7. Sale of Bullion exceeding ` 2 lakhs in cash (including 1%


coin)

8. Sale of Jewellery exceeding ` 5 lakhs in cash 1%

9. Sale of motor vehicle over ` 10 lakhs 1%

10. Sale of any goods (other than bullion/jewellery or 1%


provision of any service over 2 lakhs)
1 BASIC CONCEPTS
THIS CHAPTER INCLUDES
 Basic Concepts  Capital and Revenue Receipts
 Important Definitions and Expenditure
 Rates of Income Tax
Marks of Objective, Short Notes, Distinguish Between, Descriptive & Practical Questions
Legend
Objective Short Notes Distinguish Descriptive Practical

For detailed analysis Login at www.scannerclasses.com


for registration and password see first page of this book.

7.27
7.28 O Scanner CMA Inter Gr. I Paper 7 (2016 Syllabus)

DISTINGUISH BETWEEN
2012 - Dec [2] (b) Explain the difference between ‘Total Income’ and ‘Gross
Total Income’. (2 marks)
Answer:
GTI Sec. 2(45) Total Income Sec. 5
1. Sum total of all 5 heads 1. GTI (-)deduction u/s chap. VI A
[Salary, House Property, Business
Profession, Capital Gain, Other
Sources]
2. GTI is not rounded off 2. Rounded off u/s 288A
3. No tax is calculated on GTI 3. Tax is calculated on TI
Space to write important points for revision

DESCRIPTIVE QUESTIONS

2009 - Dec [1] {C} (d) (iv) A company which has its head office in India
operated in Pakistan declared dividend subject to remittance from Pakistan.
During the previous year relevant to the assessment year, the remittance
could not be recovered from Pakistan. What is the tax liability in the hands
of shareholder? Discuss. (2 marks)
Answer:
As head office of Company is in India. So register of members will also be
maintained in India. Income from dividend will be considered as income
deemed to accrue on arise in India and hence taxable.
As dividend from domestic company is exempt U/s 10(34) subject to an
amount of ` 10 Lakhs. Hence this dividend will be exempt from tax.
Space to write important points for revision
[Chapter  1] Basic Concepts O 7.29

2010 - June [8] (b) State the Elements/Sources of Income Tax Law.
(3 marks)
Answer:
The elements/sources of Income Tax Law are:
(i) The Income Tax Act, 1961.
(ii) Finance Act.- Annual
(iii) The Income Tax Rules, 1962.
(iv) Circulars/notifications from CBDT.
(v) Judicial Decisions.
Space to write important points for revision

2010 - Dec [5] (b) One of the exceptions to the rule that the income of the
previous year shall be assessed in the subsequent assessment year is the
shipping business of non-resident. Discuss briefly the assessment aspect of
such income from shipping business. (5 marks)
Answer:
Section 172-Shipping Business of Non Resident:
Where a ship, belonging to or chartered by a non-resident, carries
passengers, livestock, mail or goods shipped at a port in India, the ship is
allowed to leave the port only when the tax has been paid or satisfactory
arrangement has been made for payment thereof, 7.5% of the freight paid
or payable to the owner or the charterer or to any person on his behalf,
whether in India or outside India on account of such carriage is deemed to
be his income which is charged to tax in the same year in which it is
earned.[Section- 44B]
Section 172 will not apply if time-charterer carries his own goods. When
the time charterers carried their own cargo, they served their own interests
and this kind of self service was not contemplated for the purpose of
assessment, and Section 172 would not be attracted - Lima Leitao & Co.
Ltd. v. Union of India.
Space to write important points for revision
7.30 O Scanner CMA Inter Gr. I Paper 7 (2016 Syllabus)

2013 - Dec [2] (a) What are the circumstances in which previous year and
assessment year will be the same? (3 marks)
Answer:
Previous year and the assessment year will be same in the following cases:
1. Shipping business of non-resident. (Section 172)
2. Persons leaving India. (Section 174)
3. AOP or BOI or Artificial juridical person formed for a particular event or
purpose. (Section 174A)
4. Persons likely to transfer property to avoid tax. (Section 175)
5. Discontinued Business. (Section176)
Space to write important points for revision

2014 - June [5] (c) Explain the term “substantial interest” defined in Section
2(32) and its application in at least two situations. (3 marks)
Answer:
Substantial Interest
(a) For company- If individual along with relatives hold not less than 20%
equity shares beneficially.
(b) For others- If individual along with relatives is entitled to atleast 20% of
income.
Application
An individual is chargeable to tax in respect of any salary, commission, fees
or any other remuneration received by the spouse from a concern in which
the individual has substantial interest.
(i) But that portion of salary etc., of spouse which is due to application of
technical or professional knowledge or experience shall not be
clubbed.
(ii) If husband and wife both have substantial interest in the concern and
• both are receiving remuneration because of interest in the concern
• then the remuneration of both shall be clubbed in the hands of that
spouse whose total income is greater, before clubbing such
income.
Space to write important points for revision
[Chapter  1] Basic Concepts O 7.31

2016 - June [2] (a) State the situations in which the income of the assessee
can be assessed in the previous year itself, instead of in the assessment
year. (5 marks)
Answer:
Incomes which are taxed in the assessment year itself
The income of an assessee for a previous year is charged to income-tax in
the assessment year, following the previous year. However, in certain cases,
the income is taxed in the previous year in which it is earned. These
exceptions have been made to protect the interests of revenue. The
exceptions are as follows:
(i) Where a ship, belonging to or chartered by a non-resident, carries
passengers, livestock, mail or goods shipped at a port in India, the
ship is allowed to leave the port only when the tax has been paid or
satisfactory arrangement has been made for payment thereof. 7.5%
of the freight paid or payable to the owner or the charterer or to any
person on his behalf, whether in India or outside India on account of
such carriage is deemed to be his income which is charged to tax in
the same year in which it is earned.
(ii) Where it appears to the Assessing Officer that any individual may
leave India during the current assessment year or shortly after its
expiry and he has no present intention of returning to India, the total
income of such individual for the period from the expiry of the
respective previous year up to the probable date of departure from
India is chargeable to tax in that assessment year.
(iii) If an AOP/BOI etc. is formed or established for a particular event or
purpose and the Assessing Officer apprehends that AOP/BOI is likely
to be dissolved in the same year or, in the next year, he can made
assessment of the income up to the date of dissolution as income of
the relevant assessment year.
(iv) During the current assessment year, if it appears to the Assessing
Officer that a person is likely to charge, sell, transfer, dispose of or
otherwise part with any of his assets to avoid payment of any liability
under this Act, the total income of such person for the period from the
expiry of the previous year to the date, when the Assessing Officer
commences proceedings, is chargeable to tax in that Assessment
year.
7.32 O Scanner CMA Inter Gr. I Paper 7 (2016 Syllabus)

(v) Where any business or profession is discontinued in any assessment


year, the income of the period from the expiry of the previous year
upto the date of such discontinuance may at the discretion of the
Assessing Officer, be charged to tax in that assessment year.
Space to write important points for revision

2018 - June [2] (a) (ii) Explain the following concepts


 Tax Planning
 Tax Avoidance
 Tax Evasion (3 marks)
Answer:
Tax Planning: It means arranging the financial activities in such a way that
maximum tax benefits are enjoyed by making use of all beneficial provisions
in the tax laws which entitle the assessee to get certain rebates and reliefs.
This is permitted and not frowned upon by law.
Tax Avoidance: The line of demarcation between tax planning and tax
avoidance is very thin and blurred. There could be elements of malafide
motive involved in tax avoidance also. Any planning which, though done
strictly according to legal requirements defeats the basic intention of the
Legislature behind the statute could be termed as instance of tax avoidance.
Tax Evasion: It refers to a situation where a person tries to reduce his tax
liability by deliberately suppressing the income or by inflating the expenditure
showing the income lower then the actual income and resorting to various
types of deliberate manipulations. An assessee guilty of tax evasion is
punishable under the relevant laws.
Space to write important points for revision

PRACTICAL QUESTIONS
2008 - Dec [5] (a) Kamlesh was working as a crew member on an Indian
ship plying in foreign waters. During the year ended 31.03.2019, the ship did
not touch the Indian coast, except for 180 days. State the residential status
for the assessment year 2019-20 and taxability of his salary. (2 marks)
[Chapter  1] Basic Concepts O 7.33

Answer:
Any individual, who is a citizen of India, leaving India in any year for the
purpose of employment or as a member of crew of an Indian ship, is
considered as resident in India only when his stay in India during that
previous year is 182 days or more.
But in the given situation Mr. Kamlesh stays in India only for 180 days
during the previous year. Hence, he is non resident in India.
Income received from salary is considered as income received and
accrue outside India. Hence, it is not taxable in India.
Space to write important points for revision

2011 - Dec [4] (a) BIRLA Ltd., a cement manufacturing company, entered
into an agreement with a supplier for purchase of additional cement plant.
One of the conditions in the agreement was that if the supplier failed to
supply the machinery within the stipulated time, the company would be
compensated at 5% of the price of the respective portion of the machinery
without proof of actual loss. The company received ` 8.50 lakhs from the
supplier by way of liquidated damages on account of his failure to supply the
machinery within the stipulated time. What is the nature of liquidated
damages received by BIRLA Ltd. from the supplier of plant for failure to
supply machinery to the company within the stipulated time-a capital receipt
or a revenue receipt ? (5 marks)
Answer:
As per the decision of Apex Court in CIT. Vs. Saurashtra Cement Ltd.
(2010), it was decided that receipts for procuring the capital asset are of
capital nature. The Apex Court affirmed the decision of the Gujarat High
Court holding that the damages were directly and intimately linked with the
procurement of a capital asset i.e., the cement plant, which leads to delay
incoming into existence of the profit-making unit. It was not a receipt in the
course of profit making. Therefore, the amount received by the assessee
towards compensation for sterilization of the profit earning source, not in the
ordinary course of business, is a capital receipt in the hands of the assessee.
Therefore in this case, the liquidated damages of ` 8.50 lakhs received by
Birla Ltd., from the supplier of plant for failure to supply machinery to the
company within the stipulated time is a capital receipt.
Space to write important points for revision
7.34 O Scanner CMA Inter Gr. I Paper 7 (2016 Syllabus)

2012 - June [2] (a) Following details are furnished by Mr. Appaji for the year
ended 31.03.2019: `
(i) Profit on sale of shares in Indian company, sold in India
but proceeds received in France 30,000
(ii) Dividend from a Korean company received in France 50,000
(iii) Rent from property in Sri Lanka deposited in Sri Lanka
but later remitted to India through approved banking
channel-Gross 1,00,000
(iv) Dividend from ABC (P) Ltd. 20,000
(v) Income from nursery in Gujarat 40,000
Compute the total income of Mr. Appaji if he is
(i) Resident and ordinarily resident;
(ii) Resident but not ordinarily resident;
(iii) Non-resident. (9 marks)
Answer:
Computation of total income of Mr. Appaji for the assessment year
2019-20
ROR RNOR NR
Profit on sale of shares in Indian company,
sold in India but proceeds received in France 30,000 30,000 30,000
Dividend from a Korean company received in
France 50,000 Nil Nil
Rent from property in Sri Lanka deposited in
Sri Lanka but later remitted to India through
approved banking channel 70,000* Nil Nil
Dividend from ABC (P) Ltd. Exempt Exempt Exempt
Income from nursery in Gujarat Agri Agri Agri
income income income
Total Income 1,50,000 30,000 30,000

Note: * Taking ` 1,00,000 (Gross) as NAV, standard deduction u/s 24(a)


is applicable.
Space to write important points for revision
2 RESIDENTIAL STATUS
THIS CHAPTER INCLUDES
 Re sidential Status of:  Income deemed to be
Individuals, HUF, Firm, AOP, received in India
Company, Others  Income deemed to accrue or
 Incidence of Tax [Sec. 5] arise in India [Sec. 9]
 Income received in India
Marks of Objective, Short Notes, Distinguish Between, Descriptive & Practical Questions
Legend
Objective Short Notes Distinguish Descriptive Practical

For detailed analysis Login at www.scannerclasses.com


for registration and password see first page of this book.

7.35
7.36 O Scanner CMA Inter Gr. I Paper 7 (2016 Syllabus)

DISTINGUISH BETWEEN
2012 - Dec [2] (a) State the difference between residential status of a
company and that of others. (2 marks)
Answer:
Rules to determine residential status of Companies [Sec. 6(3)]
A person being a company shall be said to be resident in India in any
previous year if:
1. It is an Indian Company, or
2. Its place of effective management at any time in that year, is in India.
Note:
1. A company cannot be “ordinarily” or “not ordinarily resident”.
2. Place of Effective management to mean the place where key
management and commercial decisions that are necessary for the
conduct of the entity’s business as a whole, are, in substance made.
Space to write important points for revision

DESCRIPTIVE QUESTIONS

2012 - June [3] (a) State how the residential status of a company is
determined under the Income-tax Act, 1961. (3 marks)
Answer:
As per Sec. 6(3)
A company is said to be a resident in India in any previous year, if:
(i) It is an Indian company as defined under section 2(26) of the Act; or
(ii) during the relevant previous year, its place of effective management,
in that year, is in India.
A company is said to be Non- resident in India in any previous year if:
(i) it is not an Indian company; or
(ii) during the relevant previous year, its place of effective management
in that year, is outside India.
[Chapter  2] Residential Status O 7.37

Note: For the purpose of this clause “Place of effective management” means
a place where key management and commercial decisions that are
necessary for the conduct of business of an entity as a whole are, in
substance made.
Space to write important points for revision

2015 - June [4] (d) Mr. Bharat, an engineering graduate, born and brought
up in India, got employment in USA in August, 2018. By what date he should
leave India, in order to become a non-resident? By that, what tax advantage
he will get? (3 marks)
Answer:
Planning for residential status:
A person who leaves India for employment if remains for less than 182 days
during the financial year of leaving, he will be a non-resident.
Mr. Bharat must leave India before 29th September, 2018 to be non-resident
for the financial year 2018-19.
When he plans his departure in such a way that he becomes non-resident,
his income accruing or arising outside India will not be subjected to tax in
India. His income accruing or arising in India alone will be liable to tax in
India.
Space to write important points for revision

PRACTICAL QUESTIONS

2009 - Dec [6] (d) X got an employment in Singapore during the previous
year 2018-19. He left for Singapore on August 9, 2018. He is an Indian
Citizen. Determine the residential status for the Assessment Year 2019-20.
(2 marks)
Answer:
Any individual, who is a citizen of India, leaving India in any year for the
purpose of employment or as a member of crew of an Indian ship, is
considered as resident in India only when his stay in India during that
previous year is 182 days or more.
7.38 O Scanner CMA Inter Gr. I Paper 7 (2016 Syllabus)

But in the given situation Mr. X stays in India only for 131 days during the
previous year. Hence, he is non resident in India.
Space to write important points for revision

2010 - Dec [4] (a) Following are the details of income of Mr. Subramani for
the financial year 2018-19:
Income from property in Sri Lanka remitted by the tenant
to the assessee in India through SBI ` 2,10,000
Profit from business in India ` 1,00,000
Loss from business in Sri Lanka (whose control
and management of business wholly remained in India) ` 80,000
Dividend from shares in foreign companies
received outside India ` 60,000
Interest on deposits in Indian companies ` 1,20,000
Determine the total income in terms of the Income-tax Act, 1961 in the
following situations:
(i) Resident and ordinarily resident of India;
(ii) Resident but not ordinarily resident of India;
(iii) Non-resident. (9 marks)
Answer:
Computation of Taxable Income under various Residential Status
Particulars Resident Resident Non
and but not Resident
Ordinarily Ordinarily
Resident Resident
Income from property in Sri Lanka 2,10,000 2,10,000 2,10,000
Profit from business in India 1,00,000 1,00,000 1,00,000
Loss from business in Sri
Lanka(Control & Management (80,000) (80,000) Not
wholly in India) Taxable
Dividend from shares in foreign co. 60,000 Not Not
received outside India Taxable Taxable
[Chapter  2] Residential Status O 7.39

Interest on deposits in Indian 1,20,000 1,20,000 1,20,000


Companies
Total Income 4,10,000 3,50,000 4,30,000
Space to write important points for revision

2012 - Dec [2] (d) Mr. A furnishes the following particulars of his income
during the previous year 2018-19:
(i) Income from agriculture in Bangladesh, received thereof ` 2,00,000
and sub-sequently remitted to India.
(ii) Gift of ` 52,000 received in foreign currency from a relative in India.
(iii) Arrears of salary ` 70,000 received in India from a former employer in
England.
(iv) Income from property received abroad but later on remitted to India
` 3,20,000. (` 1 lakh used in Bahrain for educational expenses and ` 2
lakhs remitted in India later).
(v) Profit from business outside India managed from India ` 90,000 and
received outside India.
Find out the gross total income of Mr. A for the assessment year 2019-20 if
A is (i) Resident and ordinarily resident (ii) Resident but not ordinarily
resident and (iii) Non-resident. (7 marks)
Answer:
Particulars Res. & Res. but Non
ord. not ord. resident
resident resident (NR)
(ROR) (NOR)
Income from agriculture in Bangladesh, 2,00,000 - -
received there but later on remitted to India
Gift received from a relative in India - - -
[exempt u/s 56(2)(VII)]
Arrears of salary received in India from a 70,000 70,000 70,000
former employer in England
Income from property received outside India 3,20,000 - -
but later on remitted to India
7.40 O Scanner CMA Inter Gr. I Paper 7 (2016 Syllabus)

Profit from business outside India, managed 90,000 90,000 -


from India.
Gross Total Income 6,80,000 1,60,000 70,000
Space to write important points for revision

2013 - June [2] (a) Mr. Jeff, a citizen of USA came to India for 80 days, 90
days, 110 days and 130 days in the financial years 2015-16, 2016-17, 2017-
18 and 2018-19 respectively. Determine his residential status for the
Assessment Year 2019-20. (3 marks)
(b) Compute the total income of Mr. Taylor, UK citizen and a non-resident
for the Assessment Year 2019-20 from the following details furnished by
him.
`
(i) Income from business carried out in Mumbai (60%
received in USA) 5,00,000
(ii) Capital gain from sale of shares of Zenith Private
Limited, an Indian company. Sale proceeds were
received in UK 3,50,000
(iii) Rent from a house property in New Jersey collected
there, but later remitted to India through normal banking
channel 12,00,000
(iv) Dividend received from MNO Limited, an Indian
Company 2,50,000
(v) Royalty received in UK from PQR Limited, an Indian
company for use of trade mark for its business
operation in India 6,00,000
(vi) Interest on loan received in UK from S&T Limited, an
Indian company. The loan was used by S&T Limited for
its business carried on in Dubai. 3,00,000
(7 marks)
[Chapter  2] Residential Status O 7.41

Answer:
(a) As per Section 6 an individual is a resident in India in any previous year,
if he fulfils any of the following two conditions:
(i) He is present in India in that previous year for 182 days or more.
(ii) He was present in India within 4 years preceding that previous year
for 365 days or more and for 60 days or more in that previous year.
In this case Jeff was physically present in India for less than 182 days in
previous year 2018-19.
AY 2019 –20 – FY 2018-19 = 130 days
4 Previous Years
FY 2014 – 15 NIL
FY 2015 – 16 80
FY 2016 – 17 90
FY 2017 – 18 110
280 days
Hence, non-resident. He was present in India for 130 days (more than
60 days in the previous year and he was physically present in India for
280 days (80 + 90 + 110) i.e. less than 365 days in 4 previous years
preceding the previous year 2018 –19. Hence, he does not fulfill the
second condition and Jeff is non-resident in India for the Assessment
Year 2019–20.
Answer:
(b)

Particulars `
Income from business carried out in India is income from
business connection in India and deemed to accrue or arise in
India. 5,00,000
Capital gain is deemed to accrue or arise in India as shares of
Indian company are capital assets situated in India. Place of
receipt of consideration is immaterial. 3,50,000
Rent from house property situated in New Jersey being an
income from source outside India is not taxable. Subsequent
remittance of rent to India does not alter the position. ---
7.42 O Scanner CMA Inter Gr. I Paper 7 (2016 Syllabus)

As Dividend from Indian company is exempt U/s 10(34) subject


to an amount of ` 10 Lakhs. ---
Royalty received from the Indian company is deemed to accrue
or arise in India, as the patent was used by the Indian company
for its business in India. 6,00,000
Interest on loan received from the Indian company is not
deemed to accrue or arise in India as the amount of loan was
used by the Indian company for its business carried out outside
India ---
Total Income 14,50,000
Space to write important points for revision

2013 - June [3] (b) Mr. Rajput, aged 82 years gives you the following
information for the previous year 2018-19:
`
(i) Interest on fixed deposits with banks 4,80,000
(ii) Long-term capital gain on sale of land 50,000
(iii) Short-term capital gain on sale of shares 20,000
(securities transactions tax paid)
Compute tax payable by Mr. Rajput for the Assessment year 2019-20 in
cases (i) he is resident; (ii) he is non-resident. (4 marks)
Answer:
As per the proviso to Section 112(1)(a) if the following conditions are
satisfied :
(i) The taxpayer is a resident individual or a resident HUF. He or it may
be ordinarily resident or not ordinarily resident.
(ii) Taxable income - Long-term Capital Gain is less than the amount of
basic exemption limit
The following shall be deducted from long-term capital gain :
Exemption limit – (Net Income or taxable income including Long-term Capital
Gain – Long-term Capital Gain)
[Chapter  2] Residential Status O 7.43

(i) As Mr. Rajput is a resident, the relief u/s 112 is available


`
Basic Exemption Limit 5,00,000
Taxable income including Long-term Capital Gain 5,30,000
Long-term Capital Gain 50,000
Relief u/s 112 20,000
Computation of tax payable : `
Tax on income other than capital gain ` 4,80,000 Nil
Tax on long-term capital gain on sale of land i.e.,
` 50,000 – 20,000 i.e., `30,000 @ 20%. 6,000
Tax on short-term capital gain on sale of shares @ 15%
as per Section 111A as STT is paid. 3,000
Tax 9,000
Health & education cess @ 4% 360
Total Tax 9,360
(ii) As Mr. Rajput is a non-resident
Computation of tax payable : `
Tax on income other than capital gain ` 4,80,000 11,500
Tax on long-term capital gain on sale of land i.e.,
` 50,000 @ 20%. 10,000
Tax on short-term capital gain on sale of shares @ 15%
as per Section 111A as STT is paid. 3,000
Tax 24,500
Health & Education cess @ 4% 980
Total Tax 25,480
Space to write important points for revision

2015 - Dec [1] {C} Answer the following questions with brief
reasons/workings:
(c) Mr. David, a citizen of Spain came to India for the first time in previous
year 2014-15 and stayed for 100 days in that year. During the previous
years 2015-16, 2016-17, 2017-18 and 2018-19 he stayed in India for
120 days, 110 days, 80 days and 90 days respectively. What is the
residential status of Mr. David for the assessment year 2019-20?
(2 marks)
7.44 O Scanner CMA Inter Gr. I Paper 7 (2016 Syllabus)

(f) X. Limited is an Indian company. However, it carries on business in USA.


All the shareholders are residents of USA. The Board Meetings and
Annual General Meetings are held outside India. What is the residential
status of X. Limited? (2 marks)
Answer:
(c) As per Section 6 an individual is a resident in India in any previous year
if he fulfills any of the following two conditions:
(i) He is present in India in that previous year for 182 days or more.
(ii) He was present in India within 4 year preceding that previous year
for 365 days or more and for 60 days or more in that previous year.
In case of Mr. David, he was physically present in India for less than 182
days in previous year 2018-19 but, he is physically present in India for
410 days in 4 year preceding that previous year and he also present for
more than 60 days (i.e. 90 days) in previous year. Hence, Mr. David is
resident in India in A.Y. 2019-20.
(f) As per Section 6(3)
A company is said to be a resident in India in any previous year if:
(i) It is an Indian Company as defined under section 2(26) of the Act;
or
(ii) during the relevant previous year, its place of effective management,
in that year, is in India.
X Limited is an Indian Company. Therefore, X Limited is a resident in
India even if the business is carried on outside India and the meeting of
the board and shareholders are held outside India.
Note: For the purpose of this clause “Place of effective management”
means a place where key management and commercial decisions that
are necessary for the conduct of business of an entity as a whole are, in
substance made.
Space to write important points for revision

2016 - June [4] (a) Following are the transactions related to Mr. Kiran
Kumar, a resident but not ordinarily resident in India during the previous year
2018-19. Compute Gross Total Income of Mr. Kiran Kumar for the
assessment year 2019-20.
[Chapter  2] Residential Status O 7.45

Particulars `
Income from agriculture in Sri Lanka (received in Sri Lanka and
subsequently remitted to India) 4,00,000
Arrears of salary received in India from a former employer in USA 2,50,000
Rent from house property located outside India and received
outside India (` 2,00,000 is used in Bahrain for the educational
expenses of his son studying there and the balance ` 30,00,000
subsequently remitted of India) 5,00,000
Income from business in Japan which is managed and controlled
from India (` 90,000 received in India and balance ` 3,10,000
received outside India) 4,00,000
(5 marks)
Answer:
Computation of Gross Total Income of Mr. Kiran Kumar, a resident but not
ordinarily resident for the assessment year 2019-20.
Particulars `
Income from agriculture in Sri Lanka managed and controlled in –
Sri Lanka is not liable to tax in view of provision of Section 5(1)
Subsequent remittance of income to India does not alter the
position.
Arrear of salary received in India from a former employer in USA. 2,50,000
Income from house property located outside India is not an –
income accruing or arising in India or deemed to accrue or arise
in India. Hence rent is not liable to tax in India.
Income from business in Japan which is managed and controlled 4,00,000
from India is taxable in India in view of provision of Section 5(1)
Place of receipt is not material
Gross Total Income 6,50,000
Space to write important points for revision
7.46 O Scanner CMA Inter Gr. I Paper 7 (2016 Syllabus)

2016 - Dec [4] (a) Discuss, with brief reason, the taxability or otherwise of
the following under the Income-tax Act:
(ii) Mr. Ram kumar, a citizen of India employed by the Government of
India, left India for the first time on 10.02.2018 to USA for foreign
assignment. He did not visit India during previous year 2018-19. He
has been paid ` 5,00,000 towards allowances in USA. (2 marks)
Answer:
Allowances and perquisites paid or allowed as such outside India by the
Government to a citizen of India for rendering service outside India is exempt
under section 10(7). Accordingly, allowance of ` 5,00,000 paid outside for
rendering services there would not be liable to tax.
Space to write important points for revision

2017 - June [2] (a) Mr. Ramesh, an Indian citizen, gives you the following
information for the year ended 31.03.2019.
`
Business income in Mumbai 2,50,000

Rental income from property let out in London 5,40,000


(Converted in Indian rupees)

Fixed deposit interest in India from LMN Bank 60,000

Fixed deposit interest from Bank of England 40,000


(Converted in Indian rupees)

Business consultancy income from Essex Ltd. in Hampshire 75,000


(England), being a company incorporated in Delhi having
branch office in England. The business is managed from Delhi.
(Converted in Indian rupees)

Agricultural income from land located in Malaysia 90,000


(Converted in Indian rupees)

Income from nursery at Alwar, Rajasthan 1,40,000


[Chapter  2] Residential Status O 7.47

Mr. Ramesh returned to India on 15.06.2018 after remaining in England for


10 years. During the last 4 years he was in India for 100 days only.
Determine the residential status of Mr. Ramesh for the assessment year
2019-20 and compute his total income chargeable to tax in India by giving
reason for treatment of each item.
Note: Ignore Double Taxation Avoidance Agreement (DTAA). (8 marks)
Answer:
Computation of Total Income of Mr. Ramesh
for the A.Y. 2019-20
Particulars Amount `
Rental Income from property let out in London —
Income from Business or profession
Business Income in Mumbai 2,50,000
Business consultancy income from Essex Ltd. in
Hampshire (England) 75,000 3,25,000
Income from other Sources
Fixed Deposit interest from Bank in India from LMN
Bank 60,000
Fixed Deposit interest from Bank of England —
Agricultural income from land located in Malaysia — 60,000
Gross Total Income 3,85,000
Less: Deductions under chapter VI-A NIL
Total Income 3,85,000
Add: Agricultural Income from land in India Income 1,40,000
from Nursery at Alwar Rajasthan
Aggregate Income 5,25,000

Notes:
1. Residential status of Mr. Ramesh for Assessment Year 2019-20 the
Residential Status of Mr. Ramesh will be not ordinary resident because
he has not satisfied both the additional conditions of Section 6(1).
7.48 O Scanner CMA Inter Gr. I Paper 7 (2016 Syllabus)

2. Rental Income from property let out in London will not be taxable in India
because his residential status is not ordinary resident in India.
3. Fixed deposit interest from Bank of England will not be taxable India
because his residential status is not ordinary resident in India.
4. Business Consultancy Income from Essex Ltd. in Hampshire (England)
is taxable in India because the business is managed from Delhi India.
5. Agricultural Income from land located in Malaysia is not taxable in India
because his residential status is not ordinary resident in India.
6. Income from nursery at Alwar, Rajasthan will be treated as Agricultural
Income on the assumption that the land is owned by Mr. Ramesh and
Agricultural activities are carried out by himself. (which is exempt)
Space to write important points for revision

2018 - Dec [2] (a) Mr. Barun furnishes you the following information for the
year ended 31st March, 2019:
SI. Particulars `
No.
(i) Pension received in India from a former employer in United 1,80,000
Kingdom (UK)
(ii) Income from business in Singapore (Controlled from India) 1,00,000
(iii) Interest on company deposit in Singapore (credited in bank 80,000
account held there)
(iv) Profit from business in Kolkata controlled from UK 2,00,000
(v) Income from tea cultivation in Sri Lanka 3,00,000
(vi) Income from property in Singapore but received in Malaysia 2,50,000
Compute the total income of Mr. Barun, where he is (i) an ordinarily resident
in India; (ii) a resident but not ordinarily resident in India, and (iii) a non-
resident. (9 marks)
[Chapter  2] Residential Status O 7.49

Table Showing Marks of Compulsory Questions


Year 14 14 15 15 16 16 17 17 18 18
J D J D J D J D J D
Practical 4
Total 4
3 AGRICULTURAL INCOME
THIS CHAPTER INCLUDES
 Agricultural Income  Treatment of Partly Agricultural
 Instances of Agricultural (Agro) & Partly Non-Agricultural
Income Income [Rule 7]
 Instances of Non-agricultural  Impact of Agricultural Income
(Non - agro) Income on tax computation
Marks of Objective, Short Notes, Distinguish Between, Descriptive & Practical Questions
Legend
Objective Short Notes Distinguish Descriptive Practical

For detailed analysis Login at www.scannerclasses.com


for registration and password see first page of this book.

7.50
[Chapter  3] Agricultural Income O 7.51

DESCRIPTIVE QUESTIONS

2013 - June [7] (a) State whether the following are agricultural income or
non-agricultural income:
(i) Where owner himself performs slaughter tapping and them sells the
rubber.
(ii) Conversion of sugar cane into gur. (2 marks)
Answer:
(i) Where owner himself performs slaughter tapping and then sells the
rubber, it is Agricultural income.
(ii) Conversion of sugar cane into Gur - Non Agricultural income as it
involves manufacturing activity which is of business nature.
Space to write important points for revision

PRACTICAL QUESTIONS
2009 - June [3] (b) Mrs. Vasudha is running a cotton ginning factory. Raw
cotton is grown in the lands owned by her and the same is used for ginning
in her factory. The ginned cotton is sold subsequently for ` 12,00,000. The
following data are also available;
`
Cost of cultivation 4,00,000
Selling price of raw cotton when sent to the ginning factory 6,00,000
Expenses of ginning factory 3,40,000
You are required to ascertain the agricultural income and business income
of Mrs. Vasudha. (5 marks)
Answer:
Rule 7 is applicable to the given situation. Where the agricultural produce
grown by the assessee is used for further processing and generating income,
the agricultural income will be determined with reference to the market value
on the date they are taken for further processing. Subsequent income
generated is taken as business income.
7.52 O Scanner CMA Inter Gr. I Paper 7 (2016 Syllabus)

Income is computed on above basis thus:


`
Sale price of cotton when sent for ginning 6,00,000
Less: Cost of cultivation 4,00,000
Agricultural income 2,00,000
Sale price of ginned cotton (finished product) 12,00,000
Less: Cost of raw material (input) 6,00,000
Manufacturing expenses (cost of ginning) 3,40,000 9,40,000
Profits and gains of business or profession 2,60,000
Space to write important points for revision

2013 - Dec [5] (b) Manmohan owns a tea estate in Assam. He also owns a
nursery wherein he grows plants and sells them. He furnishes the following
particulars:
`
(i) Profit from sale of green tea leaves 1,75,000
(ii) Profit from manufacturing of tea grown in the garden
owned by him 7,00,000
(iii) Profit from sale of plants from nursery 1,00,000
Compute tax payable by Manmohan for the Assessment Year 2019-20.
(6 marks)
Answer:
In the case of nursery plant, question is silent about whether sapling or
seedling process activity has been undertaken or not. So, it is required as
per the question that answer should be in both alternatives. Because if
sapling or seedling process has been undertaken then it is agricultural
Income otherwise not.
Alternative - 1
Computation of Taxable Income for the Assessment Year 2019-20
Nature of Business Agl. Inc. Non-
Agl. Inc.
Profit from sale of green leaves grown in own garden
being agricultural Income is exempted under Section
10(1) 1,75,000 -
[Chapter  3] Agricultural Income O 7.53

Profit from growing and manufacturing of tea (60%


agricultural income and 40% non-agricultural income) 4,20,000 2,80,000
Profit from sale of plants from nursery (agricultural
income) 1,00,000 -
Total Income 6,95,000 2,80,000

Computation of Tax Liability:


`

(a) Total Income (Agricultural Income + Non-agricultural Income)


[6,95,000 + 2,80,000] 9,75,000
(b) Tax on (a) above 1,07,500
(c) Total of (Agricultural Income + Basic Exemption Limit)
[6,95,000 + 2,50,000] 9,45,000
(d) Tax on (c) above 1,01,500
(e) Tax Payable (b) - (d) 6,000
Add: Health and Education Cess @ 4% 240
Total Tax Liability 6,240

Note: It is assumed that sapling & seedling process has been undertaken
for nursery plant.

Alternative - 2
Computation of Taxable Income for the Assessment Year 2019-20
Nature of Business Agl. Inc. Non-Agl.
Inc.
Profit from sale of green leaves grown in own garden
being agricultural Income is exempted under Section
10(i) 1,75,000 -
Profit from growing and manufacturing of tea (60 %
agricultural income and 40% non-agricultural income) ,20,000 2,80,000
7.54 O Scanner CMA Inter Gr. I Paper 7 (2016 Syllabus)

Profit from sale of plants from nursery (non-


agricultural income) - 1,00,000
Total Income 5,95,000 3,80,000

Computation of Tax Liability:


`
(a) Total Income (Agricultural Income + Non-agricultural Income)
[5,95,000 + 3,80,000] 9,75,000
(b) Tax on (a) above 1,07,500
(c) Total of (Agricultural Income + Basic Exemption Limit)
[5,95,000 + 2,50,000] 8,45,000
(d) Tax on (c) above 81,500
(e) Tax Payable (b) - (d) 26,000
Add: Health and Education Cess @ 4% 1040
Total Tax Liability 27,040

Note - It is assumed that sapling & seedling process has not been
undertaken for nursery plant.
Space to write important points for revision

2016 - June [1] {C} (d) Answer the following question with brief
reasons/working:
(i) Rajesh has earned an income of ` 45,000 from letting out his rural
agricultural lands for a movie shooting. Will this income be regarded
as agricultural income and hence exempt? (2 marks)
Answer:
Rent earned from letting out the agricultural land is not rent or revenue
derived from the agricultural land. As per Section 2(1A), any income derived
from any building owned and occupied by the receiver of the rent or revenue
of any such land, or occupied by the cultivator or the receiver of rent-in-kind,
of any land with respect to which, or the produce of which, any process
mentioned in the section alone, is regarded as rent for the purpose of this
[Chapter  3] Agricultural Income O 7.55

section. Rent from letting out to a movie company will not fall in this category.
The land was not used for agricultural purposes, but for movie shooting.
The impugned income is not agricultural income and hence is not exempt.
Space to write important points for revision

2018 - June [4] (b) State with brief reasons whether the following are
agricultural income either in whole or in part:
(i) Purchase of standing sugarcane crop by Mr. Amin for ` 2 lakhs and
after cutting the canes, selling them for ` 2,50,000.
(ii) Income from milk dairy run by Mr. Raj in his agricultural lands
` 50,000.
(iii) Income from sale of plants ` 1,00,000 earned by Mr. Jain who
maintains a nursery by name Soundarya Nursery.
(iv) Income from sale of rubber ` 3,20,000 realised by Mr. Ram Nair who
owns rubber estate and cultivates rubber.
(v) Income from gracing of cattles allowed in the land owned by Mr.
Richard ` 60,000. (1 × 5 = 5 Mark)
Answer:
(i) Purchase of standing sugarcane crop by Mr. Amin for ` 2 lakhs and
after cutting the canes, selling them for ` 2,50,000 is not an
Agricultural Income because he has not done the basic agricultural
activities.
(ii) Income from milk dairy run by Mr. Raj in his agricultural land ` 50,000
is not an agricultural Income, it is an Income from Business.
(iii) Income from sale of plants ` 1,00,000 earned by Mr. Jain who
Maintains nursery by name Soundarya Nersery is an Agricultural
Income because it is derived by performing basic agricultural Activities.
(iv) Income from sale of rubber ` 3,20,000 realized by Mr. Ram Nair who
owns rubber Estate and Cultivates rubber is partly agricultural Income,
65% of such Income will be treated as agricultural income and 35% of
such income shall be income liable to tax as business income.
(v) Income from gracing of cattles allowed in the land owned by
Mr. Richard ` 60,000 is not an agricultural Income.
Space to write important points for revision
7.56 O Scanner CMA Inter Gr. I Paper 7 (2016 Syllabus)

2018 - Dec [3] (b) Mr. Manish, a resident in India, has the following incomes
for the year ended 31st March, 2019:
Income from sale of tea grown and manufactured in India ` 4,00,000
Income from growing and manufacturing rubber in India ` 5,00,000
Income from agricultural operations in Sri Lanka
(cultivated paddy) ` 1,00,000
Income derived from sale of coffee grown, cured, roasted
and grinded in India ` 2,00,000
Determine the quantum of income which is regarded as agricultural income
and non-agricultural income in the hands of Mr. Manish for the assessment
year 2019-20. (6 marks)

Table Showing Marks of Compulsory Questions

Year 14 14 15 15 16 16 17 17 18 18
J D J D J D J D J D

Practical 2

Total 2
4 INCOME, WHICH DO NOT FORM
PART OF TOTAL INCOME
THIS CHAPTER INCLUDES
 Incomes Not Included in Total  Income of Political Parties
Income (Section 10)
 Income of Trusts or Institutions
from Contribution
Marks of Objective, Short Notes, Distinguish Between, Descriptive & Practical Questions
Legend
Objective Short Notes Distinguish Descriptive Practical

For detailed analysis Login at www.scannerclasses.com


for registration and password see first page of this book.
7.57
7.58 O Scanner CMA Inter Gr. I Paper 7 (2016 Syllabus)

SHORT NOTES
2009 - June [7] Write a short note on the following:
(a) Amendment by the Finance Act, 2008 to “Charitable purpose” as defined
in Section 2(15) of the Income-tax Act, 1961. (5 marks)
Answer:
Charitable Purpose [Sec. 2(15)]
“Charitable Purpose” includes
 Relief of the poor,
 Education,
 Yoga
 Medical relief,
 Preservation of environment (including watersheds, forests and wildlife)
and preservation of monuments or places or objects of artistic or historic
interest, and
 The advancement of any other object of general public utility.
The purpose of Institution shall not be charitable if the advancement of any
other object of general public utility involves carrying on of any activity in the
nature of trade, commerce or business, for a cess or fee or any other
consideration where the aggregate value of such receipts exceeds 20% of
total receipt of the trust in the previous year, irrespective of the nature of use
or application, or retention, of the income from such activity.
Space to write important points for revision

2017 - June [8] Write a short note on the following:


(c) Provisions of Equalization levy as per the Finance Act, 2016.
(5 marks)
Answer:
Provisions of equalization levy as per the finance Act 2016:
In terms of the recommendations of the committee on taxation of e-
commerce constituted by the CBDT on taxation of E-commerce, with effect
from 01-06-2016, new chapter VIII has been inserted to provide for as under:
[Chapter  4] Income, which do not form.... O 7.59

1. Charge of Equalization Levy: On and from the date of commencement


of this Chapter VIII, there shall be charged on equalization levy at the
rate of 6% of the amount of consideration for and specified service
received or receivable by a person, being a non resident from:
(i) a person resident in India and carrying on business or
profession; or
(ii) a non-resident having a permanent establishment in India.
2. When equalization levy is not chargeable: Under section 165(2), the
equalization levy shall not be charged where:
(i) The non-resident providing the specified service has a
permanent establishment in India and the specified service is
effectively connected with such permanent establishment.
(ii) The aggregate amount of consideration for specified service
received or receivable in a previous year by the non resident
from a person resident in India and carrying on business or
profession, or from a non-resident having a permanent
establishment in India, does not exceed one lakh rupees; or
(iii) Where the payment for the specified, service by the person
resident in India, or the permanent establishment in India is not
for the purposes of carrying out business or profession.
Space to write important points for revision

DISTINGUISH BETWEEN
2012 - Dec [5] (c) State the difference between Exemption u/s 10 and
Deduction under Chapter VIA of the Income-tax Act, 1961. (3 marks)
Answer:
Difference between Exemption u/s 10 and Deduction under Chapter VIA

Exemption u/s 10 Deduction under Chapter VIA


(i) Exemption doesn't form part of (i) Deduction form part of a total
total income income.
7.60 O Scanner CMA Inter Gr. I Paper 7 (2016 Syllabus)

(ii) Expenditure in relation to (ii) Expenditure in relation to


exempt income is not these incomes is deductible.
deductible. (iii) Deduction is normally allowed
(iii) Income is normally exempts based on payment or
subject to certain conditions. fulfillment of specified
conditions.
Space to write important points for revision

DESCRIPTIVE QUESTIONS
2008 - Dec [6] (d) What are the types of income of a political party exempt
from income-tax under Section 13A of the Income-tax Act, 1961? Are
conditions required to be fulfilled in this regard? (3 marks)
Answer:
Incomes which are exempt (Section 13A)
The following incomes derived by a political party are not included in
computing its total income:
(i) Income which is chargeable under the head ’Income from house
property; or
(ii) Income chargeable under the head ‘Income from other sources’.
(iii) Any income by way of voluntary contribution from any person.
(iv) Any income by way of capital gains.
Requisite conditions: The exemption of the above income shall be
available only when the following conditions are satisfied:
(i) The political party keeps and maintains such books of accounts and
other documents as will enable the Assessing Officer to properly
reduce its income therefrom;
(ii) In respect of each such voluntary contribution other than contribution
by way of electoral bond in excess of ` 20,000, such political party
keeps and maintains a record of such contribution and the name and
address of the person who has made such contribution.
[Chapter  4] Income, which do not form.... O 7.61

(iii) The accounts of the political party are audited by a Chartered Accountant.
Provided also that such political party furnishes a return of income for
the previous year in accordance with the provisions of sub-section (4B)
of section 139 on or before the due date under that section.
(Amendment by Finance Act, 2017)
Explanation – For the purposes of this section, “political party” means
a political party ristered under section 29A of the Representation of the
People Act, 1951.
(iv) The treasurer of such political party or any person authorized by the
political party in this behalf must submit a report under Section 29C(3)
of the Representation of People Act, 1951 for the relevant financial year.
Political party for the purpose of this section means a political party
registered under Section 29A of the Representation of People Act, 1951.
Space to write important points for revision

2009 - Dec [1] {C} (d) (ii) Income earned by a mutual concern from mutual
activities is not taxable. Comment. (2 marks)
Answer:
A person cannot earn income from himself. In case of mutual activities, the
fund and a surplus arises which is distributed to the contributors of the fund,
such surplus cannot be called income.
So, income earned by a mutual concern from mutual activities is not the
part of taxable income. Hence, it is not taxable.
Space to write important points for revision

2009 - Dec [7] (b) Amount received from superannuation fund on resignation
before specified age is exempt from income tax. Comment. (2 marks)
Answer:
As per Section 10(13) any payment from an approved superannuation fund
shall be exempt if it is made by retirement at or after a specified age or on his
becoming incapacitated prior to such retirement, to the extent to which such
payment does not exceed the contributions made prior to the
commencement of this act (i.e. 1.4.1961) and any interest thereon.
Space to write important points for revision
7.62 O Scanner CMA Inter Gr. I Paper 7 (2016 Syllabus)

2010 - June [1] {C} (b) (ii) Income earned by an Association of Trade Unions
is not taxable — Comment. (2 marks)
Answer:
Any income chargeable under the head 'Income from House Property' and
'Income from other sources' of a Registered Trade Union within the meaning
of the Indian Trade Unions Act, 1926, formed primarily for the purposes of
regulating the workmen and the relations between workmen and the
employers or between the workmen is exempt from income tax and also of
a Federation of such Union. Hence the statement is not correct.
Space to write important points for revision

2010 - June [4] (c) Discuss whether there is any exemption for voluntary
contributions received by electoral trusts. Can an assessee giving such
donation claim the same as deduction? (5 marks)
Answer:
Exemption for voluntary contributions received by electoral trusts
[Section 13B]
Voluntary contribution received by an electoral trust is treated as income
under Section 2(24) (iia). By virtue of Section 13B donation received by an
electoral trust will not be chargeable to tax if the following conditions are
satisfied:
(i) The electoral trust is approved by the central board of direct taxes in
accordance with the scheme made by the Central Government.
(ii) The trust distributes to a registered political party during the year, 95%
of the aggregate donations received by it during the year along with
the surplus if any, brought forward from any earlier previous year.
(iii) The electoral trust should function in accordance with the rules made
by the Central Government.
The Finance (No. 2) Act, 2009 has widened the scope of deductions under
these sections by allowing deduction to also the contribution/donation made
to the electoral trusts as may be approved by the CBDT in accordance with
the scheme to be made by the Central Government.
So the assessee giving such donation can claim the deduction.
Space to write important points for revision
[Chapter  4] Income, which do not form.... O 7.63

2010 - June [6] (a) Kidwai Club is a private members club which provides
entertainment to its members by offering accommodation, library, reading
room, etc. The club also earned income by letting out its marriage hall to
non-members by making them as temporary members. The club contends
that the “doctrine of mutuality” would apply in such a case and hence, its
income would not be taxable. Discuss the correctness or otherwise of the
contention of the assessee club. (4 marks)
Answer:
Concept of mutuality: The concept of mutuality means that the contributors
and the beneficiaries are identical. Since one cannot make profit by dealing
with oneself, there is no taxable profit involved wherever such concept
applies.
In this case, however, the principle of mutuality does not apply since
non-members are made temporary members only for the purpose of letting
out the marriage hall.
This issue came up before the Kerala High Court, in CIT Vs. Trivandrum
Club (2006) 153 Taxman 481. The High Court observed that the marriage
hall was being rented out to non-members by making them temporary
members only for the purpose of letting out the marriage hall and the amount
received from the non-members formed part of the assessee club.
The principle of mutuality, therefore, would not apply in such a case.
Thus the contention of the assessee club is incorrect.
Space to write important points for revision

2011 - June [3] (c) State the conditions under which the political party gets
tax exemption under Section 13A in respect of certain types of
income/receipts. (3 marks)
Answer:
Exemption u/s 13A is available to registered political parties only. The
following incomes of the political parties are exempt from tax-
1. Income from House Property
2. Income under head Capital Gains
3. Income from other sources.
4. Voluntary Contribution received from any person
7.64 O Scanner CMA Inter Gr. I Paper 7 (2016 Syllabus)

Condition to be fulfilled by the political party to avail exemption u/s


13A, if
(a) It keeps and maintains such books of accounts and other documents as
would enable the Assessing Officer to properly deduce its income
therefrom.
(b) In respect of each such voluntary contribution other than contribution by
way of electoral bond in excess of ` 20,000, such political party keeps
and maintains a record of such contribution and the name and address
of the person who has made such contribution.
(c) Its accounts are audited by a Chartered Accountant (CA).
(d) Its treasurer or any authorised person must submit a report u/s 29C(3)
of Representation of People Act, 1951 to election Commission
providing contributions exceeding ` 20,000 and listing separately
contribution from companies and Government companies.
Space to write important points for revision

2012 - June [2] (c) Is any income of Khadi and Village Industries Board
exempt from tax? If not, state taxable and exempted items of income.
(2 marks)
Answer:
As per Section 10(23BB) any income of an authority, whether known as
Khadi and Village Industries Board established in a State by or under a State
or Provincial Act for the development of khadi or village industries in the
State is exempt from tax.
Therefore, the entire income of Khadi and Village Industries Board is
exempt from tax regardless of the nature or source of income.
Space to write important points for revision

2013 - Dec [2] (b) What are the conditions for claiming exemption u/s
10(10C) of the Income-tax Act, 1961 relating to Voluntary Retirement
Compensation? (3 marks)
Answer:
Condition for claiming exemption under Section 10(10C) are:
(i) Compensation is received at the time of voluntary retirement or
termination or voluntary separation;
[Chapter  4] Income, which do not form.... O 7.65

(ii) Compensation is received in accordance with the scheme of voluntary


retirement/ separation which is framed in accordance with prescribed
guidelines;
(iii) Maximum amount of exemption is ` 5,00,000;
(iv) An individual who has retired under the voluntary retirement scheme
should not be employed in another Company of the same
management;
(v) He should not have received any other voluntary retirement
compensation before from any other employer and claimed exemption;
(vi) The person who has avail a relief under Section 89 in respect of
compensation for voluntary retirement or separation or termination of
service, will not be able to claim any exemption under Section
10(10C) for the same assessment year or any other assessment year.
Space to write important points for revision

2014 - June [2] (a) There exists no difference in the treatment of income
claimed as exempt u/s 10 with those entitled to deduction under chapter VI-A
of the Income Tax Act, 1961. Do you agree with the statement? Justify your
answer. (3 marks)
Answer:
The statement is not correct. There are differences between the two.
The differences in the treatment of income claimed u/s 10 with those
Chapter VI-A of the Income Tax Act, 1961 are as follows:
Exemption u/s 10 Deduction under Chapter VI-A
Income exempt does not form part Income forms part of total income
of the total income
Expenditure in relation to income Expenditure in relation to income
exempt not deductible deductible
It will not enter in the calculation of It is a deduction from gross total
gross total income income. Hence, the impugned
income might enter the calculation
upto gross total income.
7.66 O Scanner CMA Inter Gr. I Paper 7 (2016 Syllabus)

Income exemption is normally Deduction is normally allowed


subject to certain conditions based on payment or fulfillment of
certain conditions
Space to write important points for revision

2014 - Dec [6] (c) State the conditions to be satisfied by political party to
avail income-tax exemption. (3 marks)
Answer:
Section 13A deals with tax exemption for incomes of political parties.
Any income of a political party which is chargeable under the head “income
from house property” or “Income from other sources” or “Capital gains” or
any income by way of voluntary contributions received by a political party
from any person shall not be included in the total income of the previous year
of such political party.
(a) such political party keeps and maintains such books of accounts and
other documents as would enable the Assessing Officer to properly
reduce its income therefrom;
(b) in respect of each such voluntary contribution other than contribution by
way of electoral bond in excess of ` 20,000, such political party keeps
and maintains a record of such contribution and the name and address
of the person who has made such contribution.
(c) the accounts of such political party are audited by an accountant as
defined in the Explanation below sub-section (2) of section 288; and
(d) no donation exceeding two thousand rupees is received by such political
party otherwise than by an account payee cheque drawn on a bank or
an account payee bank draft or use of electronic clearing system through
a bank account or through electoral bond.
Provided also that such political party furnishes a return of income for the
previous year in accordance with the provisions of sub-section (4B) of
section 139 on or before the due date under that section.
Space to write important points for revision
[Chapter  4] Income, which do not form.... O 7.67

PRACTICAL QUESTIONS

2011 - Dec [2] (c) RAJASTHALI Ltd. furnishes you the following information
for the year ended 31.3.2019:
`
Total turnover of Unit A located in Special Economic Zone 100 lakhs
Profit of the business of Unit A 30 lakhs
Export turnover of Unit A 50 lakhs
Total turnover of Unit B located in Domestic Tariff Area (DTA) 200 lakhs
Profit of the business of Unit B 20 lakhs
Compute deduction under Section 10AA of the A.Y. 2019-20. (3 marks)
Answer:
Under Section 10AA, 100% of the profit derived from export of articles or
things or services is deductible for a period of 5 consecutive assessment
years and 50% of profits are deductible for next 5 years and 50% of the
profits are further deductible from 11th year to 15th year provided an
equivalent amount is debited to profit and loss account of the previous year
and credited to Special Economic Zone Re-investment allowance Reserve
Account. As per Section 10AA(7), the profit derived from export of articles
or things or services shall be the amount which bears to the profits of the
business of the undertaking, being the Unit, the same proportion as the
export turnover in respect of articles or things or services bears to the total
turnover of the business carried on by the undertaking.
Deduction under Section 10AA =

= = 15 lakhs

Space to write important points for revision


7.68 O Scanner CMA Inter Gr. I Paper 7 (2016 Syllabus)

2011 - Dec [7] (b) CHERRY Ltd. is running two industrial undertakings, one
in a SEZ (Unit S) and another in a normal area (Unit N). The brief
summarized details for the year ended 31.3.2019 are as under:
(` in lacs)
S N
Domestic turnover 10 100
Export turnover 120 Nil
Gross profit 20 10
Less : Expenses and depreciation 7 6
Profits derived from the unit 13 4
The brought forward business loss pertaining to Unit N is ` 2 lacs. Briefly
compute the business income of the assessee. (7 marks)
Answer:
Computation of business income of CHERRY Ltd.
Particulars `
Total profit derived from Units S ` 13,00,000
Total profit derived from Units N ` 4,00,000 17,00,000
Less: Exemption under Section 10AA
[See Working Note below) 12 ,00,000 5,00,000
Less: Brought forward business loss 2,00,000
Business Income 3,00,000
Working Note:
Computation of exemption under Section 10AA in respect of Unit S located
in a SEZ -
Domestic turnover 10,00,000
Export turnover 1,20,00,000
Total turnover 1,30,00,000
Profit gained from 13,00,000
Exemption under Section 10AA
Profit × = 13,00,000 × =12,00,000
[Chapter  4] Income, which do not form.... O 7.69

Note: Under section 10AA, 100 % profit derived from export of articles
or things or from services is eligible for deduction, assuming that
F.Y.2018-19 falls within the first five year period commencing from the
year of manufacture or production of articles or things or provision of
services by the Unit in SEZ.
Space to write important points for revision

2012 - June [2] (b) Ms. Monisha, a Sikkimese woman, married Mr. Atul of
Surat in June, 2008. She has income from let out property at Sikkim being
` 10,000 per month. She is employed in a bank at Surat and her salary
income for the year ended 31.03.2019 was computed at ` 3,12,000.
Determine her total income. What would be your answer if she got married
on 30th June, 2013? (4 marks)
Answer:
As per Section 10(26AAA) income accruing to a Sikkimese woman from any
source in the State of Sikkim or by way of dividend or interest on securities
is exempt from tax.
Therefore, the income from let out property at Sikkim is exempt from tax.
However, her salary income from bank employment of ` 3,12,000 being the
total income would be chargeable to tax.
As per proviso to Section 10(26AAA) the exemption the following income,
which accrue or arises to Sikkimese shall be exempt from income tax-
(a) income from any source in the state of Sikkim: or
(b) income by way of dividend or interest on securities.
However, this exemption will not be available to a Sikkimese woman who, on
or after 01.04.2010, marries a non Sikkimese individual.
Hence, the exemption in respect of let out property income would not apply
if she got married on 30.06.2013, to a non-Sikkimese individual.
Statement of total Income
Income from salary ` 3,12,000
Income from House Property `1,20,000
Less- Statutory deduction ` 36,000 ` 84,000
Total Income ` 3,96,000
Space to write important points for revision
7.70 O Scanner CMA Inter Gr. I Paper 7 (2016 Syllabus)

2013 - June [5] (b) Decide the exemption/taxability of following


receipts/recipients:
(i) Educational scholarship of ` 10,000 received from a charitable trust by
a college student.
(ii) Rental income earned by a registered trade union.
(iii) Co-operatives formed for promoting the interest of schedule tribes.
(iv) Dividend received from Indian companies by resident individuals and
tax on such dividend paid by the company u/s. 115-O.
(v) Amount received by a non-resident towards compulsory acquisition of
urban agricultural land in India by Central Government. (5 marks)
Answer:
(i) Educational scholarship is exempt in the hands of recipient regardless
of the amount or source of scholarship [Section 10(16)].
(ii) Rental income of registered trade union is exempt from tax. [Section
10(24)].
(iii) Income of co-operative society meant for promoting the interests of
members of schedule tribes is exempt [Section 10(27)]
(iv) Dividend from Indian companies is exempt from tax if dividends so
paid by the company were subjected to tax under Section 115-O.
Section 115 BBDA
Where the total income of a specified assessee, resident in India,
includes income in aggregate exceeding ` 10 lakhs rupees, by way of
dividends declared, distributed or paid by a domestic company or
companies, the income tax payable shall be the aggregate of–
(a) The amount of income tax calculated on the income by way of
such dividends in aggregate exceeding ` 10 lakhs rupees, at the
rate of 10%, and
(b) The amount of income tax with which the assessee would have
been chargeable had the total income of the assessee been
reduced by the amount of income by way of dividends.
(v) Amount received towards compulsory acquisition of urban agricultural
land is exempt under Section 10(37) if the acquisition if such
compensation is received after March 31, 2004 and the agricultural
land was used by the assessee (or by any of his parents) for
agricultural purposes during 2 years immediately prior to transfer. The
residential status of the recipient has no bearing on the benefit of
exemption.
Space to write important points for revision
[Chapter  4] Income, which do not form.... O 7.71

2013 - Dec [1] {C} (a) Answer the following sub-divisions briefly in the light
of the provisions of the Income-tax Act, 1961:
(vii) Giant Oil Inc. sold crude oil to HPCL, a company in India. The sale was
made within India. Is the income arising from such sale liable to tax?
(1 mark)
Answer:
As per Section 10(48), the income from sale of crude oil by foreign company
to any person in India is exempt from tax provided income received in India
in Indian currency by a foreign company on account of sale of crude oil to
any person in India.
Space to write important points for revision

2015 - Dec [2] (a) MNO Ltd. commenced commercial production of its unit
located in Special Economic Zone (SEZ) from 01.04.2009. It furnishes the
following information for the year ended 31.03.2019:

Particulars ` in lakhs
Total sales 100
Export sales 50
Profit earned 30
Compute the deduction under section 10AA and income chargeable to tax
for the assessment year 2019-20. Your computation must be supported by
reasons. (4 marks)
Answer:
U/s 10AA, 100% of the profit derived from export of articles or services is
deductible for a period of 5 consecutive assessment years and 50% of profit
are deductible for next 5 years and 50% of the profit are further deductible
from 11th year to 15th year provided on equivalent amount is debited to profit
and loss account of the previous year and credited to special economic zone
reinvestment allowance reserve account.
As per Section 10AA(7), the profit derived from export of things or
services shall be the amount which bears to the profit of the business of the
undertaking, being the unit, the same proportion as the export turnover in
respect of articles or things or services bears to the total turnover of the
business carried on by the undertaking:
7.72 O Scanner CMA Inter Gr. I Paper 7 (2016 Syllabus)

Deduction u/s 10AA = × 50%

= × 50%
Deduction u/s 10AA will be ` 7.5 lakhs.
Space to write important points for revision

2016 - Dec [4] (a) Discuss, with brief reasons, the taxability or otherwise of
the following under the Income-tax Act:
(i) Agricultural income of ` 1,50,000 earned from land situated in Bihar by
Mrs. Bhagyashree, a non-resident.
(iii) An amount of ` 4,00,000 withdrawn by Mr. Prakash, a resident
individual from Public Provident Fund as per relevant rule.
(2 × 2 = 4 marks)
Answer:
(i) Income from agricultural operation carried out on land situated in India
is exempt under section 10(1) of the Income-tax Act both in case of
residents and non-residents. In view of above, agricultural income of
` 1,50,000 from land situated in Bihar will not be liable to tax.
(iii) Amount withdrawn from Public Provident Fund is exempt under
section 10(11) of the Income-tax Act. Hence, withdrawal of ` 4,00,000
by Mr. Prakash from Public Provident Fund as per relevant rule would
not be chargeable to tax in his hands.
Space to write important points for revision
Section - B
Heads of Income and
Computation of Total
Income and Tax
Liability
5 INCOME UNDER HEAD SALARIES
THIS CHAPTER INCLUDES
 Meaning and elements of  Deductions from Salary
Salary  Perquisites
 Allowances  Pension
 Death-Cum-Retirement  Retrenchment Compensation
Gratuity  Leave Salary
 Profits in Lieu of Salary
Marks of Objective, Short Notes, Distinguish Between, Descriptive & Practical Questions
Legend
Objective Short Notes Distinguish Descriptive Practical

For detailed analysis Login at www.scannerclasses.com


for registration and password see first page of this book.

7.75
7.76 O Scanner CMA Inter Gr. I Paper 7 (2016 Syllabus)

DISTINGUISH BETWEEN
2009 - Dec [8] (b) Distinguish between foregoing of salary and surrender of
salary. (4 marks)
Answer:
Distinguish between foregoing of salary and surrender of salary
Foregoing of salary:
Once salary has been earned by an employee, it becomes taxable in his
hands though he may subsequently waive the right to receive the same from
the employer. The waiver of salary by the employee would be treated as
application of the income and salary though waived would be taxable in his
hands.
Surrender of salary: Any salary surrendered by the employee to the Central
Government, under the Voluntary Surrender of Salaries (exemption from
taxation) Act, 1961, will not be included while computing his taxable income,
whether he is a private sector/public sector or Government employee.
Space to write important points for revision

DESCRIPTIVE QUESTIONS
2009 - Dec [5] (b) An employee instructs his employer to pay a certain
portion of his salary to a charity and claims it as exempt as it is diverted by
overriding charge or title. Comment. (2 marks)
Answer:
Section 15 taxes salary on “due” basis, even if it is not received. If,
therefore, an employee foregoes his salary, it does not mean that salary so
foregone is not taxable. Once salary has accrued to an employee, its
subsequent waiver does not make it exempt from tax liability. Such voluntary
waiver or foregoing by an employee of salary due to him is merely an
application of income and is nonetheless chargeable to tax.
Space to write important points for revision
[Chapter  5] Income under head Salaries O 7.77

2010 - Dec [4] (b) The question whether a particular income is income from
salary or is income from business depends upon whether the contract is a
contract of service or is a contract for service. Discuss. Also explain with the
help of one example. (6 marks)
Answer:
Contract of Service: In case of contract of service, there must be
employer-employee relationship. The work is done by employee on the
instructions of employer. Employee has to follow the instructions of
employer. He is bound to follow the employer. Control and management of
work wholly and exclusively lie with the employer. Employer has to decide
about the manner of execution of the work. Employee is compensated for his
services rendered for employer on monthly basis or in lump sum or in any
other mode as per mutual agreement between employer and employee.
Example: Mr. A working for ABC Ltd. He is giving consultancy services to
various clients and clients are paying service charges to ABC Ltd. ABC Ltd
pays a fix remuneration to A. As there is employer-employee relationship
exists between Mr. A and ABC Ltd. Hence Mr. A’s remuneration will be taxed
as salary.
Contract for Service: Under contract for service a person offers his services
to any person and he pays directly to the service provider. Manner of
execution is decided by the service provider. Control of the work regarding
what should be done is with the client and how it should be done is with the
service provider. He is remunerated for his services by the client.
Example: A is working for various organizations and is remunerated directly
by these organizations. Hence receipts will be considered business income.
Space to write important points for revision

2011 - June [3] (b) State the treatment of the following perquisites for
income tax purpose in the hands of the employee:
(i) Firm (employer) offers special discount on its product sales to
employees.
(ii) Employee is allowed to use the laptop of the employer, at his
residence.
(iii) Reimbursement of air travel expense to an employee's wife.
(iv) Assessee is given interest-free loan for purchase of car.
(1 mark each)
7.78 O Scanner CMA Inter Gr. I Paper 7 (2016 Syllabus)

Answer:
(i) Special discount offered on the firm’s products to employees is not
taxable as perquisite.
(ii) Use of laptop is not treated as perquisite.
(iii) Reimbursement of travel expenses to an employee’s wife is taxable
as perquisite.
(iv) Loan given to employee for purchase of car will be taxable as
perquisite.
Space to write important points for revision

2013 - June [8] (d) What are the conditions to be fulfilled by an employee to
get his accommodation in a hotel that will not be a taxable perquisite?
(2 marks)
Answer:
Accommodation provided in a hotel will not be a taxable perquisite if the
following two conditions are fulfilled.
(i) The period of such accommodation does not exceed 15 days.
(ii) Such accommodation has been provided on the transfer of the
employees from one place to another.
Space to write important points for revision

2013 - Dec [2] (c) “Accommodation provided in a hotel will not be a taxable
perquisite” in the hands of employee. Is it correct? Briefly explain.
(2 marks)
Answer:
Please refer 2013 - June [8] (d) on page no. 78
Space to write important points for revision

2014 - Dec [2] (a) What are “profits in lieu of salary” as per Section 17(3) of
the Income Tax Act, 1961? (3 marks)
[Chapter  5] Income under head Salaries O 7.79

Answer:
As per Section 17(3), “profits in lieu of salary” includes –
(i) The amount of any compensation due to or received by the assessee
from his employer or former employer at or in connection with the
termination of his employment or the modification of the terms and
conditions, relating thereto;
(ii) Any payment (other than gratuity, commuted pension, compensation
received under the Industrial Disputes Act or any other Act, etc., any
payment from a provident fund to which the Provident Funds Act,
1925 applies, accumulated balance in recognized provident fund,
payment from approved superannuation fund or house rent allowance)
to the extent to which it does not consist of contributions by the
assessee or interest thereon or any sum received under a Keyman
insurance policy including bonus allocated under such policy.
Space to write important points for revision

PRACTICAL QUESTIONS

2008 - Dec [2] (a) Answer the following questions briefly:


(i) Gratuity of ` 1,20,000 is received in August, 2018 by Mr. M, a legal
heir of Mr. R aged 45 years who died on June 28, 2018. Is it taxable?
(2 marks)
Answer:
Gratuity is not taxable in the hands of the deceased employee as it becomes
due and is paid after death. It is also not taxable in hands of the legal heir as
it does not par take character as income in hands of legal heir. It forms a part
of the estate of the deceased.
Space to write important points for revision
7.80 O Scanner CMA Inter Gr. I Paper 7 (2016 Syllabus)

2010 - June [6] (b) Mrs. Hemalatha, working in a public sector company,
opted for voluntary retirement scheme and received ` 7 lakh as VRS
compensation. She claimed ` 5 lakhs as exemption under section 10(10C)
and in respect of the balance amount of ` 2 lakh, she claimed relief under
section 89(1). Advise Mrs. Hemalatha as to the correctness of the aforesaid
tax treatment. (7 marks)
Answer:
Exemption u/s 10(10C) as well as relief u/s 89
An employee opting for voluntary retirement scheme receives a lump sum
amount in respect of his balance period of service. This amount is in the
nature of advance salary.
Under section 10(10C), an exemption of `5 lakh is provided in respect
of such amount to mitigate the hardship on account of the employee going
into the higher tax bracket consequent to receipt of the amount in lump sum
upon voluntary retirement.
However, some tax payers have resorted to claiming both the exemption
under section 10(10C) (upto `5 lakh) and relief under section 89 (in
respect of the amount received in excess of `5 lakh). This tax treatment has
been supported by many Court/Tribunal judgments.
Therefore, in order to convey the true legislative intention, Section 89
has been amended to provide that no relief shall be granted in respect of any
amount received or receivable by an assessee on his voluntary retirement
or termination of his service, in accordance (in the case of a public sector
company), if exemption under section 10(10C) in respect of such
compensation received on voluntary retirement or termination of his service
or voluntary separation has been claimed by the assessee in respect of the
same assessment year or any other assessment year.
Correspondingly, Section 10(10C) has been amended to provide that
where any relief has been allowed to any assessee under section 89 for
any assessment year in respect of any amount received or receivable on his
voluntary retirement or termination of service or voluntary separation, no
exemption under section 10(10C) shall be allowed to him in relation to that
assessment year or any other assessment year.
[Chapter  5] Income under head Salaries O 7.81

Therefore, in view of the above amendment, Mrs. Vasudha’s tax


treatment is incorrect. She has to either opt for exemption of upto ` 5 lakh
under section 10(10C) or relief under section 89(1), but not both. But
exemption u/s 10(10C) will be more beneficial to her.
Sec. 89 - Relief when salary etc., is paid in arrears or in advance.
Space to write important points for revision

2010 - Dec [2] (b) Parvez (P) Ltd. confers of electricity benefit to its
employee Mr. Ashwin. Annual consumption as per meter reading was 3200
units. Determine the value of perquisite in the following cases:
(i) Electricity meter is in the name of Mr. Ashwin and the rate of electricity
is ` 5 per unit paid by Parvez(P) Ltd. to the State Electricity Board
(Give brief reason). (2 marks)
(ii) Electricity meter is in the name of Parvez(P) Ltd. and the rate of
electricity charged by the State Electricity Board is ` 5 per unit. Is this
chargeable in all situations? (2 marks)
(iii) Parvez(P) Ltd. is a power generating company. The manufacturing
cost is ` 1.90 per unit but supplied to public @ ` 5 per unit. However,
it charges ` 1 per unit from its employees. (1 mark)
Answer:
(i) Electricity meter is in the name of employee and expenses of
electricity are met by the employer hence is a perquisite and taxable
in the hands of all employees whether specified employees or non
specified employees.
Taxable amount is 3200 units × ` 5 = ` 16,000
(ii) Electricity meter is in the name of employer and expenses of electricity
are met by the employer hence is a perquisite and taxable in the
hands of only specified employees and not taxable for non-specified
employees.
Taxable amount is 3200 units × ` 5 = ` 16,000
(iii) Employer is the manufacturer of the facility given to the employee.
Value of the perquisite is the manufacturing cost to the employer after
deducting the amount if any if recovered by the employer from the
employee. Perquisite is taxable for all employees, whether specified
or non specified.
7.82 O Scanner CMA Inter Gr. I Paper 7 (2016 Syllabus)

Amount of perk is 3200 units × ` 1.90= ` 6,080


Less: Amount Recovered 3200×1= ` 3,200
Taxable amount is 6080 – 3200= ` 2,880
Space to write important points for revision

2011 - June [3] (a) Mrs. Vasudha retires on 15th Feb., 2019, after serving
for 30 years and 5 months. She gets ` 10,70,000 as gratuity. Her salary
details are as under:
FY 2018-19 Salary ` 1,00,000 p.m. D.A. 50% of salary. 50% thereof taken
for retirement benefits.
FY 2017-18 Salary ` 90,000 p.m. D.A. 50% of salary. 50% thereof forms
part of retirement benefits.
Compute the taxable amount of gratuity in her hands in the following
situations:
(i) She retires from Government service;
(ii) She retires from seasonal factory in a private sector, covered under
Payment of Gratuity Act, 1972. (6 marks)
Answer:
(i) As per Section 10(10)(i) of Income Tax Act 1961, gratuity received by
a Government employee is fully exempt.
(ii) As an employee of a seasonal factory, in a private sector, covered
under the Payment of Gratuity Act,1972.
Computation of taxable Gratuity
Particulars ` `
Amount of Gratuity received 10,70,000
Less:
Exemption u/s 10(10)(ii)
Least of the following
(i) Actual amount received 10,70,000
(ii) 7/26 x last drawn salary x no. of years of
completed service or part thereof in the
excess of 6 months 12,11,538
[30 x 7/26 x 1,50,000]
[Chapter  5] Income under head Salaries O 7.83

(iii) Maximum limit


Amount of gratuity exempt (Min. of above
three) 10,00,000 10,00,000
Taxable Gratuity 70,000
Note: Salary means Basic Salary & DA (DA provided under terms of
employment or not always included when employee covered under
payment of gratuity act)
Space to write important points for revision

2011 - June [6] (b) Johnson is provided 2 cars by his company ABC Ltd.
Other pertinent details are as under:
Particulars Car 1 Car 2
Cost of the Car (`) 6,00,000 4,00,000
Cubic capacity of engine Above 1.6 litres Less than 1.6 litres
Running & repairs/maintenance 72,000 45,000
(borne by the company)
Drivers salary (borne by
the company) 39,000 33,000
Compute the perquisite value of car on the assumption that Car 1 is meant
for both official and personal use and that Car 2 is meant for exclusive
personal use. (5 marks)
Answer:
Valuation of Perquisite of Car
Car - I (Capacity above 1.6 litre)
Particulars Amount
Driver’s salary @ 900 pm (prescribed) 10,800
Running & maintenance exp. @ 2400 pm (prescribed) 28,800
Value of perquisite 39,600
7.84 O Scanner CMA Inter Gr. I Paper 7 (2016 Syllabus)

Car - II (Capacity less than 1.6 litre)


Particulars Amount
Driver’s salary (Actual) 33,000
Running and maintenance (Actual) 45,000
Normal bear & tear cost @ 10% of cost of car (prescribed) 40,000
Value of perquisite 1,18,000

Note: It is assumed that repair & maintenance does not include


depreciation of car (i.e. bear and tear cost)
Space to write important points for revision

2012 - June [3] (c) Mr. David is employed in Vertex Co. Ltd. Mumbai. His
salary details for the financial year 2018-19 are given below:
(i) Basic salary ` 70,000 per month;
(ii) Dearness allowance 50% of basic salary (Eligible for retirement
benefits);
(iii) House rent allowance ` 10,000 per month (Rent paid by employee
` 15,000 per month);
(iv) Bonus ` 30,000 per annum;
(v) Both David and his employer contribute ` 15,000 each per month
towards pension scheme notified in Section 80 CCD.
Calculate the total income of Mr. David for the assessment year 2019-20.
Determine the amount of contribution to the pension scheme eligible for
deduction in the hands of employer. (9 marks)
Answer:
Computation of total income of Mr. David for the assessment year
2019-20
`
Basic salary (70,000 x 12) 8,40,000
Dearness allowance 50% of basic salary
( 50% of 8,40,000) 4,20,000
[Chapter  5] Income under head Salaries O 7.85

Bonus 30,000
House Rent allowance (Ref. W. Note) 66,000
13,56,000
Employers contribution to Pension fund of employee 1,80,000
Gross Salary 15,36,000
Less: Standard Deduction 40,000
Gross Total Income 14,96,000
Less: Deduction under section 80CCD
Own contribution upto 10% of salary 1,26,000
Employers contribution upto 10% of salary
but limited in view of Section 80 CCE 1,26,000 1,50,000
Total Income 2,52,000 13,46,000
Working Note:
House rent allowance received 1,20,000
Less: Exemption U/s. 10(13A)
(a) Actual house rent allowance received 1,20,000
(b) Rent paid over 10% of salary
` 1,80,000 - ` 1,26,000 = 54,000
(c) 50% of salary = 6,30,000
Least of above is exempt 54,000
66,000
Salary for HRA includes basic salary, dearness allowance (if forming
part of retirement benefits) and commission based on fixed percentage
of turnover.
In the hands of employer: Any sum paid by the assessee as an employer
by way of contribution towards a pension scheme referred to Section 80
CCD on account of an employee is deductible to the extent it does not
exceed 10% of the salary of the employee in the previous year.
In view of the above, though the employer has contributed ` 1,80,000 to the
pension fund account of the employee David, the deduction under section
36(1)(iva) cannot exceed 10% of the salary of the employee. Salary for this
purpose means basic salary plus dearness allowance.
7.86 O Scanner CMA Inter Gr. I Paper 7 (2016 Syllabus)

Hence, Salary = 8,40,000 + 4,20,000 = ` 12,60,000 of which 10% is


deductible against the business income of the employer. The excess
contribution of ` 54,000 (` 1,80,000 - ` 1,26,000) is inadmissible expenditure
in the hands of employer.
Space to write important points for revision

2012 - Dec [2] (c)‘R’ who resides at Delhi, gets ` 6,00,000 as basic salary.
He receives ` 1,70,000 as house rent allowance. Rent paid by him is
`1,80,000. Find out the amount of taxable HRA for the assessment year
2019-20. (4 marks)
Answer:
Computation of Taxable HRA for the Previous Year 2019-20
Particulars Amount Amount
` `
Amount Received during the previous year towards
HRA 1,70,000
Less: Exemption u/s 10(13A) – least of the followings:
(a) Actual amount of HRA Received 1,70,000
(b) 50% of Salary 3,00,000
(c) Rent paid 1,80,000
Less: 10% of Salary 60,000 1,20,000 1,20,000
50,000
Salary for HRA = Basic Salary + DA (forming part of retirement benefits) +
Commission (based on fixed percentage of turnover) = ` 6,00,000.
Space to write important points for revision

2012 - Dec [3] (b) Excel Ltd. allotted 1000 (sweat) equity shares of ` 10 each
to Mr. Rao, General Manager. The fair market value of the shares computed
in accordance with the method prescribed under the Income-tax Act/Rules
was ` 500 per share, whereas it was allotted at ` 300 per share.
What is the perquisite value of sweat equity shares allotted to Mr. Rao?
In case these shares are sold subsequently, what would be their cost of
acquisition in the hands of Mr. Rao? (6 marks)
[Chapter  5] Income under head Salaries O 7.87

Answer:
As per Section 17(2)(vi), the fair market value of sweat equity shares on the
date on which the option is exercised by the assessee is considered as
value of perquisite as reduced by the amount actually paid or recovered from
him in respect of such shares.
Fair market value of 1,000 equity shares @ ` 500 each 5,00,000
Less: Amount recovered from Mr. Rao 1,000 x 300 each 3,00,000
Value of perquisite of sweat equity shares allotted to Mr. Rao 2,00,000
If these shares are sold subsequently, as per Section 49(2AA). Where the
capital gain arises from the transfer of specified security or sweat equity
shares which was treated as perquisite in the hands the employee, the cost
of acquisition of such security or shares shall be the fair market value which
has been taken into account for the purposes of valuation of perquisite.
Hence, cost of acquisition for Mr. Rao shall be ` 5,00,000.
Space to write important points for revision

2013 - June [6] (a) ‘X’ a resident of Bengaluru receives ` 20,00,000 as basic
salary. In addition he gets ` 6 lakhs as dearness allowance (forming part of
basic salary), 3.5% commission on sales made by him (sale made by X
during the previous year in ` 80,00,000); ` 2,40,000 is paid to him as house
rent allowance. He however pays ` 2,80,000 as house rent. Determine the
quantum of HRA exempt from tax. (4 marks)
Answer:
Out of the H.A. received i.e., ` 2,40,000/-, the least of the following would be
exempt
(a) ` 11,52,000/- being 40% of salary, i.e., basic salary, dearness pay and
commission: ` 28,80,000/-
(b) ` 2,40,000 being the house rent allowance.
(c) NIL, being the excess of rent paid (i.e., ` 2,80,000/- over 10% of salary,
i.e., ` 2,88,000/-. As least of the three is NIL, the entire house rent
allowance is taxable.
Space to write important points for revision
7.88 O Scanner CMA Inter Gr. I Paper 7 (2016 Syllabus)

2013 - Dec [1] {C} (a) Answer the following sub-divisions briefly in the light
of the provisions of the Income-tax Act, 1961:
(ii) A Government employee received gratuity of ` 16 lakhs upon
retirement, in September 2018. How much is taxable? (1 mark)
Answer:
As per Section 10 (10) gratuity of a person being a government employee
is exempt without any monetary limit.
Space to write important points for revision

2013 - Dec [5] (a) Raja joined TCI Limited on 1st June, 2018. Emoluments
paid and benefits allowed by the company to Raja are as follows:
`
Basic Salary 40,000 p.m.
Dearness Allowance 15,000 p.m.
Incentive 30,000 p.m.
A furnished accommodation at Mumbai belonging to the
company is provided free. Cost of furniture therein ` 3,00,000
Motor car (with engine cc less than 1.6 litres) owned by the
company along with a chauffeur for official and personal
purposes
Salary of sweeper paid by the company 1,000 p.m.
Education provided for Raja’s son without any fees. Cost of
providing education by the school is 750 p.m.
Admission fee for corporate membership of a club paid 1,20,000
by the company. Bills for club facilities were paid by Raja.
House building loan of ` 10,00,000 was given by the company
to Raja on 1st December, 2018 at interest rate of 5% p.a.
No repayment was made during the year.
Compute the income of Raja chargeable under the head “Salaries”.
(7 marks)
[Chapter  5] Income under head Salaries O 7.89

Answer:
Assessee : Mr. Raja Assessment Year : 2019-20
Computation of Income from Salary
Particulars Amount (`)
Basic Pay (` 40,000 x 10 months) 4,00,000
Add: Dearness Allowance (` 15,000 x 10 months) 1,50,000
Add: Bonus (` 30,000 x 10 months) 3,00,000
Add: Taxable Value of Perquisite related to:
Furnished accommodation (Note 1) [1,27,500 + 25,000] 1,52,500
Motor car provided by employer [(` 1,800 + ` 900) x 10
months] 27,000
Salary of sweeper [` 1,000 x 10 months] 10,000
Educational facilities -Note 2 Nil
Value of concessional housing loan - Note 3 19,167
Corporate Membership Fees - Note 4 Nil
Gross Income from Salary 8,80,158
Less: Standard Deduction 40,000
Income from Salary 8,40,158
Note 1: Salary for the purpose of computing taxable value of furnished
accommodation:
Particulars Amount (`)
Basic Salary 4,00,000
Dearness Allowance 1,50,000
Bonus 3,00,000
8,50,000
Assuming, Mr. Raja stays in a city where population is more than 25,00,000
as per 2001 census, value of unfurnished accommodation = 15% of salary
= 15% of ` 8,50,000
= ` 1,27,500
7.90 O Scanner CMA Inter Gr. I Paper 7 (2016 Syllabus)

Value of furniture provided = 10% p.a. of actual cost


= 10% of ` 3,00,000 x 10 / 12
= ` 25,000
(Assuming, value of furniture given in the problem represents actual cost.)
Note 2: Computation of taxable value of perquisite - related to educational
facility.
Since tuition fee per month is less than ` 1,000.
Amount of perquisite = Nil
Note 3: Computation of taxable value of perquisite - related to interest free
housing loan.
Value of Perquisite = Interest @ 10.75% p.a. less Actual interest charged
= (10.75% - 5%) x ` 10,00,000 x 4/12 = ` 19,167
Note 4: Corporate membership fees of a club are for the business of the
employer of the assessee, will not be treated as perquisite in the hand of
employee.
Space to write important points for revision

2014 - June [3] (a) Mr. Raghu joined a company at Chennai on 01.07.2018
and was paid the following emoluments :
(i) Basic salary ` 50,000 per month.
(ii) Dearness allowance 50% of basic salary (eligible for retirement
benefits).
(iii) Furnished accommodation owned by company was provided at
Chennai.
(iv) Value of furniture in the accommodation ` 2,00,000 (cost).
(v) Motor car owned by the employer (with engine capacity less than 1.6
litres) given for exclusive personal use. Self-driven by Raghu.
Expenses incurred by employer on its running and maintenance
` 55,950.
(vi) Educational facility for two children provided free of cost. The school
is owned by the company. Tuition fee per month ` 600 and ` 1200
respectively.
[Chapter  5] Income under head Salaries O 7.91

(vii) Annual membership fee for Gymkhana Club paid by the employer
` 20,000.
Compute the income from salary of Mr. Raghu for the assessment
year 2019-20. (7 marks)
Answer:
Computation of income from salary of Mr. Raghu for A.Y. 2019 - 20
` `
Basic Salary ` 50,000×9 4,50,000
Dearness Allowance @ 50% of Basic Salary 2,25,000
Value of Furnished Accommodation @ 15% of
Salary
In this problem, Salary = Basic
Salary + D.A.(forming part of retirement benefit)
= 4,50,000 + 2,25,000 = 6,75,000 x 15% 1,01,250
Add: Value of perquisite for furniture provided
(2,00,000 x 10% x 9/12) 15,000 1,16,250
Add: Value of perquisite for motor car - running
and maintenance expenses incurred by employer
but car exclusively used for personal use 55,950
Add: Value of perquisite for educational facility
provided to children of employee in a school
owned by employer
Child 1 - Tuition Fee ` 600 per month (since not
exceeding ` 1,000 per month) Exempted
Child 2 - Tuition Fee ` 1,200 per month
(exceeding ` 1,000 per month) . Taxable Value
of perquisite per month = ` (1,200-1,000) =
` 200 × 9 months 1,800 1,800
Add: Club Membership fee paid by employer 20,000
Gross Income from Salary 8,69,000
Less: Standard deduction 40,000
8,29,000
Space to write important points for revision
7.92 O Scanner CMA Inter Gr. I Paper 7 (2016 Syllabus)

2014 - June [6] (d) Mr. Rahim, Director in a MNC Ltd. is entitled to a motor
car (1.8 litres.) to be used for both official and private purposes.
Discuss the taxability of perquisite, if
(i) The car is owned by the employee, expenses paid by employer and
it is a Chauffeur-driven car.
(ii) The car is owned by Mr. Rahim, expenses incurred ` 30,000 and
chauffeur is paid a salary of ` 90,000 provided by the employer.
(3 marks)
Answer:
As per Notification No. 24 dated 18.12.2009 the taxable value of perquisite
will be:
Amount
(`)
(i) Actual expenses paid by the employer - 39,600
` 39,600 (i.e. ` 3,300 × 12)
(ii) Computation of taxable value of perquisite
Amount of expenses 30,000
Add: Salary to chauffeur 90,000
1,20,000
Less: Value of perquisite if the car was owned by the
employer (` 2,400 pm + ` 900 pm for Chauffeur = ` 3,300
×12 = ` 39,600) 39,600
Taxable value of perquisite 80,400
Tutorial Note:
Had the car been owned by the employer in case of the above, case (i), the
calculations would have been as follows:
Expenditure incurred + salary of chauffeur (as per income-tax guidelines)
= ` 2,400 pm + ` 900 pm
= ` 3,300 pm
= ` 39,600 per year.
Space to write important points for revision
[Chapter  5] Income under head Salaries O 7.93

2014 - Dec [2] (b) Mr. Mahim was retrenched from service of ABC Limited.
He received retrenchment compensation amounting to ` 8,75,000. Amount
of compensation determined under the Industrial Disputes Act, 1948 is
` 4,80,000. The scheme of retrenchment is not approved by the Central
Government. Compute the taxable retrenchment compensation.
(4 marks)
Answer:
Computation of Taxable Retrenchment Compensation:
Particulars ` `
Amount of retrenchment compensation received 8,75,000
Less: Exemption under section 10(10B)
(i) Actual amount received 8,75,000
(ii) Amount determined under the Industrial
Disputes Act, 1948 4,80,000
(iii) Maximum Limit 5,00,000
Least of above is exempt 4,80,000
Taxable Retrenchment Compensation 3,95,000
Space to write important points for revision

2015 - June [4] (a) Mr. Ashwin finance manager in Beta Ltd. gives you the
following information:
(i) A rent-free accommodation is provided by the employer at Bangalore
by taking the accommodation on lease basis whose rent was ` 20,000
per month.
(ii) He is provided with a motor car (cubic capacity of engine more than
1.6 litres) both for official and personal use. The expenses on running
and maintenance are met by the employee.
Assume annual salary for the purpose of perquisite valuation as ` 6,00,000.
You are requested to compute the perquisite value in the hands of Mr.
Ashwin. (4 marks)
7.94 O Scanner CMA Inter Gr. I Paper 7 (2016 Syllabus)

Answer:
Computation of perquisite value in the hands of Mr. Ashwin:
Particulars ` `

Rent-free accommodation:

Rent paid by the employer at ` 20,000 per month × 12 240000

15% of salary i.e., 15% of ` 6,00,000 90000

-Whichever is less is taken as the value of


perquisite

Perquisite value of rent-free accommodation (A) 90000

Perquisite value of motor car:

Cubic capacity of engine more than 1.6 litres and the expenses 10800
are met by the employee, hence the perquisite value ` 900 per
month.(` 900 × 12) (B)

Total perquisite value in the hands of Mr. Ashwin (A) + (B) 1,00,800
Space to write important points for revision

2015 - Dec [2] (b) Mr. Sridhar an employee of XY Ltd. received ` 8 lakhs as
leave salary on his retirement on 28.02.2019. Average salary drawn during
last 10 months ` 35,000. Last drawn salary is ` 40,000. He rendered service
of 24 years and 7 months. Leave taken while in service 9 months. Leave
entitlement as per employer’s rules is 1 months for each completed year of

service. Calculate the taxable leave salary for the assessment year 2019-20.
(4 marks)
[Chapter  5] Income under head Salaries O 7.95

Answer:
Taxable leave salary of non government employee received at his retirement
will be as under:
Least of the following is exempt:
(i) Notified limit 3,00,000
(ii) Leave salary actually received 8,00,000
(iii) Cash equivalent of leave standing at the credit of the
employee (maximum 30 days for every completing
year) × 35,000 5,25,000

(iv) 10 months salary (10 × 35,000) 3,50,000


Leave salary exempt 3,00,000
` 5,00,000 (8,00,000 - 3,00,000) will be taxable leave salary in hands of Mr.
Sridhar.
Space to write important points for revision

2015 - Dec [3] (b) Mr. Harbhajan employed in Gama Ltd. furnishes you the
following information for the year ended 31.03.2019:
(i) Basic salary upto 31.12.2018 ` 60,000 per month.
(ii) Basic salary from 01.01.2019 ` 70,000 per month.
Note: Salary is due and paid on the last day of every month.
(iii) Dearness Allowance @ 40% of basic salary.
(iv) Bonus equal to one month salary paid in February, 2019 on basic
salary and DA applicable for that month.
(v) Employer’s contribution to Provident Fund account of the employee at
15% of basic salary.
(vi) Profession tax paid ` 5,000 of which ` 2,000 was paid by employer.
(vii) Facility of laptop and computer was provided to Harbhajan both for
official and personal use. Cost of laptop ` 35,000 and computer
` 25,000 acquired by the company on 01.01.2019.
7.96 O Scanner CMA Inter Gr. I Paper 7 (2016 Syllabus)

(viii) A motor car owned by the employer is provided to the employee meant
for both official and personal use from 01.12.2018. Running expenses
fully met by the employer which amounts to ` 35,000. The motor car
(cubic capacity of engine exceeds 1.6 liters) was self driven by
Mr. Harbhajan.
Compute the salary income chargeable to tax in the hands of
Mr. Harbhajan for the assessment year 2019-20 . (7 marks)
Answer:
Computation of ‘income from salary’ of Mr. Harbhajan
(i) Basic salary
60,000 × 9 = 5,40,000
70,000 × 3 = 2,10,000 7,50,000
(ii) D.A. @ 40% 3,00,000
(iii) Bonus 98,000
(iv) Professional tax paid by employer 2,000
(v) Employer’s contribution in PF 31,500
(vi) Value of perquisites (Laptop & Computer) Nil
(vii) Value of perquisites (Motor car) 9,600
Gross salary in hands of Mr. Harbhajan 11,91,100
Less: Standard deduction 40,000
Taxable salary 11,51,100
Working Notes:
1. In case professional tax is paid by employer on behalf of employee, the
amount paid shall be included in taxable gross salary of employee, then
deduction can be claimed.
2. Employer’s contribution in PF in exempted upto 12%, excess of 12%
shall be taxable in hands of employee. Hence, 3% (15% - 12%) will be
taxable in hands of Mr. Harbhajan.
[Chapter  5] Income under head Salaries O 7.97

3. As per section 17(2)(viii) taxable value of perquisite for use of Laptop &
Computer shall be Nil.
4. As per Rule 3(2), if the motor car (whose engine cubic capacity is 1.6
liters) is owned by employer and is used for both official and personal
purpose by the employee, the value of perquisites would be ` 2,400 per
months.
Space to write important points for revision

2016 - Dec [1] {C} (d) Answer the following question with brief
reason/working:
(ii) Mr. Ajit is employed with XY Co. Ltd. at Mumbai from 01.04.2018. The
company took accommodation on lease basis which cost ` 3 lakhs per
annum. Mr. Ajit is eligible for salary plus DA of ` 1 lakh per month. The
employer’s annual contribution to the recognized provident fund
account of Mr. Ajit is ` 1,20,000. What is the perquisite value of
accommodation liable to tax in the hands of Mr. Ajit? (2 marks)
Answer:
Value of Perquisite will be: (Rent free Accommodation)
(a) 15% of Salary (12,00,000 × 15%) ` 1,80,000
Or
(b) Actual Lease Rent paid by employer ` 3,00,000
Which ever is lower i.e. ` 1,80,000
Space to write important points for revision

2016 - Dec [2] (c) An employee has been given a laptop purchased on
01.04.2018 for ` 40,000, which he is allowed to take home and use. What is
the value to be treated as perquisite while computing income under the head
‘salaries’? (2 marks)
Answer:
Laptop given by employer to the employee is exempt perquisite. Therefore,
value of perquisite will be NIL.
Space to write important points for revision
7.98 O Scanner CMA Inter Gr. I Paper 7 (2016 Syllabus)

2016 - Dec [3] (a) Mr. Nitin is the marketing manager of M&M Ltd., Mumbai.
From the following details compute the salary income of Mr. Nitin for the
assessment year 2019-20:
(i) Basic salary (per month) ` 60,000
(ii) Dearness allowance = 50% of basic salary
(iii) Motor car owned by employer given to employee. Entire running
expenses are met by the employer and the car is used for both official
and personal purpose by the employee. The engine cubic capacity is
above 1.6 litres.
(iv) Provident fund contribution of both employer and employee at 15% of
basic salary.
(v) Accommodation owned by the employer is given to the employee. A
sum of ` 5,000 per month is deducted towards accommodation from
the salary of employee.
(vi) Life insurance premium on policy taken by employee paid by the
employer during the year ` 45,000.
(vii) The employer provides free education facility for Mr. Nitin’s daughter
in a school maintained by the employer. Cost of education in similar
school is ` 800 per month.
(viii) Cost of lunch provided by the employer during office hours ` 18,000.
(8 marks)
Answer:
Computation of Income from Salary of Mr. Nitin for A.Y. 2019-20:
(i) Basic Salary 7,20,000
(ii) DA 3,60,000
(iii) Value Motor Car. (2,400 × 12) 28,800
(iv) Contribution to Provident fund in excess of
12% of Salary (employer contribution) 21,600
(v) Value of concessional Rent free
accommodation
15% of Salary 1,08,000
Less: Amount of Rent deducted from salary
5,000 × 12 60,000 48,000
[Chapter  5] Income under head Salaries O 7.99

(vi) Life Insurance Premium 45,000

(vii) Free education facility to daughter


(because cost in less than 1,000 in similar
school, office) Nil

(viii) Free Lunch Provided during hours Exempt


(because it is less than ` 50 per day i.e.
50 × 365 = 18,250)

Gross Salary 12,23,400

Less: Standard Deduction 40,000

Taxable salary 11,83,400


Space to write important points for revision

2017 - June [2] (b) Mr. Raghu is employed with Yes Power Co. Ltd. as
General Manager (Finance) at Kolkata. He furnishes you the following
information for the year ended 31.03.2019.
`
Basic salary (per month) 40,000

Dearness allowance (per month) eligible for retirement 30,000


benefits

Rent-free accommodation is provided.

A car was provided to him from 01.06.2018 (engine cubic


capacity more than 1.6 litres). It is used both for official and
personal purposes. Running expenses are fully met by
employer. Mr. Raghu drives the car himself.

Provident fund contribution of both employer and employee


12% of basic pay and dearness allowance.
7.100 O Scanner CMA Inter Gr. I Paper 7 (2016 Syllabus)

Fixed tiffin allowance (per annum) 20,000

Fixed medical allowance (per annum) 30,000

Credit card annual fee paid by employer (used for personal 7,000
purposes)

Only son of Mr. Raghu is given free education in the school


run by the employer. Cost of education is ` 1,500 per month.

Loan taken by Mr. Raghu from provident fund during the year 50,000
Compute the total income of Mr. Raghu for the assessment year 2019-20.
(7 marks)
Answer:
Computation of Total Income
of Mr. Raghu
for the A.Y. 2019-20
Particulars Amount Amount
(`) (`)
Income from Salary
Basic Salary (40,000 × 12) 4,80,000
Dearness Allowance (30,000 × 12) 3,60,000

Allowances
Fixed tiffin allowance 20,000
Fixed Medical Allowance 30,000
Value of Perquisites
Value of Rent free accommodation i.e. 15% of
salary (4,80,000 + 3,60,000 + 20,000 + 30,000)
× 15% 1,33,500
Value of Car (2,400 × 10) 24,000
Credit Card Annual fees paid by employer 7,000
[Chapter  5] Income under head Salaries O 7.101

Cost of Education (1,500 × 12) 18,000


ITO Vs Director, Delhi Public School (2007)
Contribution to Provident fund (Not More than
12% of salary) NIL 10,72,500
Gross Salary 10,72,500
Less: Standard deduction 40,000
Gross Total Income 10,32,500
Less: Deduction under chapter VI-A
u/s 80C contribution to provident fund
(4,80,000 + 3,60,000) × 12% 1,00,800
Total Income 9,31,700
Space to write important points for revision

2017 - Dec [4] (b) Mr. Mohan is sales manager in Steel King (P) Ltd. at
Chennai. During the financial year 2018-19, he gets the following
emoluments from his employer:
Particulars `
Basic salary upto 30.09.2018 20,000 p.m.
From 01.10.2018 30,000 p.m.
Dearness allowance @ 50% basic salary [it is not
eligible retirement benefits]
Transport allowance 2,000 p.m.
Children education allowance (for 2 children) 1,000 p.m.
Tiffin allowance (actual expenses ` 9,000) 15,000
Tax paid on employment 3,000
Contribution to recognized provident fund by the
employer @ 15% of basic salary.
7.102 O Scanner CMA Inter Gr. I Paper 7 (2016 Syllabus)

An unfurnished accommodation taken on lease by the


employer was given to the employee for the whole year.
Lease rent paid by the employer ` 1,80,000. Amount
recovered from the employee ` 2,000 per month.
Domestic servant salary reimbursed by the employer as
per employment agreement. 5,000 p.m.
Compute the salary income of Mr. Mohan for the assessment year 2019-20.
(8 marks)
Answer:
Computation of Income from salary of Mr. Mohan
Basic Salary 20,000 × 6 = ` 1,20,000
30,000 × 6 = ` 1,80,000 3,00,000

Dearness Allowance @ 50% of Basic Salary 1,50,000

Transport Allowance 24,000

Children Education Allowance 12,000


Less: Exempt u/s 10(14)/(ii) 2,400 9,600

Tiffin Allowance (fully taxable) 15,000

Tax paid on employment 3,000

(Contribution to Recognised Provident fund in excess of 12% of


salary (3,00,000 × 3%) 9,000

Concessional Rent free accommodation

Lease Rent paid by employer 1,80,000


or
15% of salary (3,00,000+4,800+9,600+15,000) × 15% 49,410
Which ever is less 49,410
Less: Amount paid (2,000 × 12) 24,000 25,410
[Chapter  5] Income under head Salaries O 7.103

Reimbursement of Domestic
Servant salary (5,000 × 12) 60,000

Gross Salary 5,96,010

Less: Standard deduction 40,000

Less: Deduction u/s 16(iii) Tax paid on employment 3,000

Income from salary 5,53,010


Space to write important points for revision

2018 - Dec [3] (a) Mr. Subramani is Senior Manager (Finance) of VKS Steel
Ltd. The particulars of his emoluments for the year ended 31.03.2019 are
given below:
Basic Salary ` 60,000 per month
Dearness Allowance ` 40,000 per month (30% is for retirement
benefit)
Annual performance Incentive ` 1,80,000
House Rent Allowance ` 10,000 per month
Mr. Subramani pays rent of ` 20,000 per month for a flat occupied from 1st
November, 2018 at Erode, Tamil Nadu.
He received gift voucher of ` 6,000 from the employer on the occasion of his
marriage anniversary.
The employer provided him a motor car (cubic capacity of the engine
exceeds 1.6 litres) without chauffeur with effect from 1st December, 2018.
Running and maintenance expenses of ` 30,000 were fully borne by the
employer. The car is used by Mr. Subramani both for official and private
purposes.
The employer paid the following premiums for Mr. Subramani.
(i) Medical insurance premium ` 12,000
(ii) Life insurance premium ` 15,000
(iii) Accident insurance premium ` 10,000
7.104 O Scanner CMA Inter Gr. I Paper 7 (2016 Syllabus)

Tax on employment paid to Erode Municipal Corporation by Mr. Subramani


` 5,000.
Compute the income chargeable to tax under the head “Salaries” in the
hands of Mr. Subramani for Assessment Year 2019-20. (9 marks)

Table Showing Marks of Compulsory Questions

Year 14 14 15 15 16 16 17 17 18 18
J D J D J D J D J D

Practical 2

Total 2
6 INCOME FROM HOUSE PROPERTY
THIS CHAPTER INCLUDES
 Basis of Charge  Recovery of Unrealized Rent
 Deemed Owner  Arrears of Rent
 Let Out House Property  Annual Value
 Self-Occupied House Property  Deductions from Annual Value
 Composite Rent  Treatment of Loss
Marks of Objective, Short Notes, Distinguish Between, Descriptive & Practical Questions
Legend
Objective Short Notes Distinguish Descriptive Practical

For detailed analysis Login at www.scannerclasses.com


for registration and password see first page of this book.

7.105
7.106 O Scanner CMA Inter Gr. I Paper 7 (2016 Syllabus)

DESCRIPTIVE QUESTIONS
2008 - Dec [1] {C} (c) Provide very brief answers to the following:
(iii) In a house belonging to Janak, his cousin is living without paying any
rent. Janak says that since his relative is residing there without paying
any rent, there is no rental income chargeable to income-tax. Is he
correct ? (2 marks)
Answer:
Janak is not correct: If Janak or his parents are residing in the house, it will
be a self-occupied house whose NAV (Net Annual Value) is nil. Where a
cousin lives in the house, the fair rent which the house will fetch if let out, will
have to be taken as the NAV and income from house property will have to
be computed in the manner laid out in Section 23 and 24.
Space to write important points for revision

2008 - Dec [7] (b) How will you treat unrealized rent allowed as a deduction
in the past and realized in a subsequent year when the assessee is no more
the owner of the property. (3 marks)
Answer:
Where the assesses could not realize rent from a property let to a tenant and
the same was allowed as deduction and subsequently the assessee has
realized any amount in respect of such rent, the amount so realized shall be
deemed to be income chargeable under the head “Income from house
property” and accordingly charged to income-tax as the income of that
previous year in which such rent is realized whether or not the assessee is
the owner of that property in the previous year.
Standard deduction @ 30% to be allowed from arrears of rent or unrealised
rent u/s 25A(2).
Space to write important points for revision

2009 - June [8] (e) Is it possible for the net annual value of a house property
to be negative? What will be tax treatment if income under the head “Income
from house property” is negative? (2 + 2 = 4 marks)
[Chapter  6] Income from House Property O 7.107

Answer:
Net Annual value being a negative figure:
In the case of any let out house property, if the municipal tax paid by the
landlord is more than the gross annual value, then the net annual value can
be negative. This situation can arise where municipal taxes of earlier years
are paid during the current year.
Tax treatment of loss under the head “Income from house property”:
If there is a loss under the head “Income from house property”, it can be set-
off against income under any other head during the current year (no loss can
be set-off against winning from lotteries, races, etc.). If it is not possible to
set-off the loss (fully or partly). it can be carried forward to the next year for
being set off against the incomes under the head “income from house
property”. Loss from house property can be carried forward for a maximum
period of 8 years for set-off against income from house property.
Space to write important points for revision

2012 - June [5] (b) State the conditions to be fulfilled for claiming deduction
of unrealized rent while computing the income from house property.
(5 marks)
Answer:
Treatment of Unrealized Rent [explanation to Section 23(1)]: Rule 4
Unrealized Rent means the rent not paid by the tenant to the owner and the
same shall be deducted from the Actual Rent Receivable from the property
before computing income from that property, provided the following
conditions are satisfied:
1. The tenancy is bonafide;
2. The defaulting tenant has vacated, or steps have been taken to compel
him to vacate the property;
3. The defaulting tenant is not in occupation of any other property owned
by the assessee;
4. The assessee has taken all reasonable steps to institute legal
proceeding for the recovery of unrealized rent or satisfies the Assessing
Officer that such legal proceeding would be useless.
Space to write important points for revision
7.108 O Scanner CMA Inter Gr. I Paper 7 (2016 Syllabus)

2015 - June [2] (d) How is the unrealized rent dealt with in annual value
determination of a house property under the Income-tax Act, 1961? What are
the conditions to be satisfied in this regard? (3 marks)
Answer:
Treatment of Unrealized Rent [explanation to Section 23(1)]:
Rule 4
The amount of unrealized rent is not to be includible while determining the
annual value of a property.
However, the following conditions are to be satisfied for excluding the
unrealized rent.
(i) The tenancy must be bona fide.
(ii) The defaulting tenant has vacated or steps have been taken to compel
him to vacate the property.
(iii) The defaulting tenant is not in the occupation of any other property of
the assessee; and
(iv) The assessee has taken all steps to institute legal proceedings for the
recovery of unpaid rent or satisfies the Assessing Officer that the legal
proceedings would be useless.
Space to write important points for revision

PRACTICAL QUESTIONS

2008 - Dec [5] (b) Amalesh owns a house property which is let-out for
` 6,500 per month. The fair rent of the property is ` 90,000. Municipal taxes
paid during the year for each half year is ` 3,200. The tenant has spent
`10,000 towards repairs of the property during the year. Compute the income
from house property for the assessment year 2019-20. (2 marks)
[Chapter  6] Income from House Property O 7.109

Answer:
Computation of Income from House Property of Mr. Amalesh for the
Assessment Year 2019-20
`
Gross Annual Value 90,000
(Expected or Actual rent received whichever is more)
Less: Municipal tax paid 6,400
Net Annual Value 83,600
Less: Deduction under section 24
30% of the net annual value 25,080
Income from house property 58,520
Space to write important points for revision

2009 - Dec [8] (d) X and Y are co-owners of two houses with equal share
of both the houses. While the first house is used by them for their residence,
the second house is let to a tenant at a monthly rent of ` 2,500. The other
relevant particulars of the houses for the year 2018-19 are as follows:
First House Second House
Construction completed on 30.06.2008 31.03.2010
Municipal Tax ` 2,000 ` 2,500
Insurance Premium ` 2,500 ` 2,500
Interest on loan ` 10,000 ` 9,000
Compute income from house property of X and Y for the relevant
assessment year. (4 marks)
Answer:
Computation of income from house property

House I House II
Gross Annual Value - 30,000
Less: Municipal tax paid - 2,500
Net Annual value - 27,500
Less: Standard deduction @ 30% - 8,250
interest on loan 10,000 9,000
Income from House property (10,000) 10,250
7.110 O Scanner CMA Inter Gr. I Paper 7 (2016 Syllabus)

Working Notes:
(i) Gross Annual Value -
Rent Received or Receivable (2500 × 12) = ` 30,000
(ii) Income from house II ` 10,250
Income from house I ` (10,000)
So, total income ` 250
X and Y are co-owners with equal share of both houses. So, taxable income
For X = (250/2) ` 125
For Y = (250/2) ` 125

Note: Annual premium is non deductible expense.


Space to write important points for revision

2010 - June [6] (c) In the following cases, state the head of income under
which the relevant receipt is to be assessed, alongwith reasons:
(ii) Ashok has constructed a house on a leasehold land. He has let-out
the said property and has treated the rent from such property under
the head "Income from other sources" and deducted expenses on
repairs, security charges, insurance and collection charges in all
amounting to 35% of receipts. (2 marks)
Answer:
In this case, the receipt is assessable as “Income from house property” since
ownership of land is not a pre-requisite for assessment of income under this
head. 30% of Net Annual Value (Gross annual value less Corporation tax
actually paid) is allowable as a deduction under section 24.
Space to write important points for revision

2010 - Dec [3] (a) Ramesh owns a house at Hyderabad. Its municipal
valuation is ` 24,000. He incurred the following expenditure in respect of the
house property:
(i) Municipal Tax at 20%;
(ii) Fire insurance premium ` 2,000; and
(iii) Land revenue ` 2,400.
He had taken bank loan of ` 25,000 at 16% per annum on April 1, 2016; the
whole amount is still unpaid.
[Chapter  6] Income from House Property O 7.111

The house was completed on April 1, 2018.


Find the income from house property for the assessment year 2019-20 for
the following situations:
(i) If the assessee uses house for self-occupation throughout the
previous year, and
(ii) If the house is let out for residential purpose on monthly rent of ` 2,500
from April 1, 2018 to December 31, 2018 and self occupied for the
remaining period. (5 marks)
Answer:
Name of Assessee Ramesh
Place of Residence Hyderabad
Computation of Income from House Property
(i) When property is Self occupied throughout the year

Annual value of house property Nil


Less: Interest on borrowed capital (WN) (5,600)
Loss from house property 5,600

Working Note:
Current Year Interest 25,000 × 16% = ` 4,000
Pre construction period interest 25,000 ×16% × 2 = ` 8,000
1/5th of pre construction period = 8,000 × 1/5 = `1,600
Total interest = 4,000 + 1,600 = ` 5,600

(ii) When property is let out for part of year and self occupied for part
of the year:
Particulars Amount (`)
Municipal value 24,000
Expected Rent (2,500 × 12) 30,000
Actual rent received 22,500
Gross Annual Value (higher of above 3) 30,000
7.112 O Scanner CMA Inter Gr. I Paper 7 (2016 Syllabus)

House let out for 9 month and self occupied for 3 months
Gross annual value for 9 months (30,000 × 9/12)
Less: Municipal Taxes (24,000 × 20% = 4,800 × 9/12) 22,500
Net annual value 3,600
Less: Deductions u/s 24 18,900
Standard deduction @ 30% 5,670
Interest on Loan(4,000 +1,600 = 5,600 × 9/12) 4,200
Income from house property for let out period 9,030
Annual value for self occupied period Nil
Less: Interest on loan for 3 months (5,600 × 3/12) 1,400
Loss from house property (1,400)
Total income from house property[9,030 - 1,400] 7,630
Space to write important points for revision

2011 - June [4] (a) Mrs. Vimala commenced construction of house meant for
residential purpose on 01.11.2016. She raised a loan of ` 10 lakhs @ 11%
per annum from a bank. Finding that there was over run in the cost of
construction, she raised a further loan of ` 5 lakhs from her friend at 15%
rate of interest per annum on 1.10.2018. The construction was completed by
February, 2019.
Compute the amount of interest allowable exemption under section 24 of the
income-tax Act, 1961 in the following cases:
(i) The house was meant for self occupation from 01.03.2019
(ii) The house was to be let out from 01.03.2019
Is there any deduction available u/s. 80C towards principal repayment in
respect of above loans? (6 marks)
Answer:
Case I: If the house was meant for self occupation - Section 24 of Income
Tax Act, 1961 says that the maximum tax deduction allowed of self occupied
house property is subjected to a maximum limit of ` 2,00,000.
Hence, the maximum amount eligible for deduction u/s 24 would be
limited to ` 2,00,000 but the actual interest payable is ` 1,78,667 (which is
less than ` 2,00,000). Only ` 1,78,667 is deductible.
[Chapter  6] Income from House Property O 7.113

Case II: Section 24 also states that if the house property for which the
house loan has been taken is not self occupied, no maximum limit has been
prescribed and the taxpayer can take deduction of the whole interest
amount.
So, the whole amount of ` 1,78,667 is deductible u/s 24.
U/s 80C, the amount of principal repayment toward bank loan is qualified
for deduction loan taken from friend is not deductible u/s 80C.
Working Notes:
1. Pre-construction interest upto 31.03.18
(` 10,00,000 × 11% × 17/12) = 1,55,833
2. Amount allowable for each year 31,167
Interest on first loan for the year 18-19
(` 10,00,000 × 11%) = 1,10,000
Interest on friend’s loan
(5,00,000 × 15% × 6/12) 37,500
1,78,667
Space to write important points for revision

2012 - Dec [6] (a) Mr. Nitin completed construction of a residential house on
01.04.2018.
Interest paid on loans borrowed for the purpose of construction during the 30
months prior to completion was ` 60,000.
The house was let-out on a monthly rent of ` 18,000.
Annual corporation tax paid is ` 35,000.
Interest paid during the year is ` 25,000.
Amount spent on repairs is ` 6,000.
Fire insurance premium paid ` 3,000 p.a.
The property was vacant for 4 months.
Annual letting value as per corporation records is ` 1,50,000.
He had also received arrears of rent of ` 36,000 during the year, which had
not been not charged to tax in the earlier year.
Compute the income under the head “Income from House Property” for the
assessment year 2019-20. (8 marks)
7.114 O Scanner CMA Inter Gr. I Paper 7 (2016 Syllabus)

Answer:
Computation of Income from House Property of Mr. Nitin
for the Assessment Year 2019-20
Particulars Amount Amount
` `
Gross Annual Value u/s 23(1) (c) Ref. Note 1. 1,44,000
Less: Municipal Taxes Paid 35,000
Net Annual Value (NAV) 1,09,000
Less: Deduction u/s 24
(a) Standard deduction i.e.30% of NAV 32,700
(1,09,000 × 30%)
(b) Interest on Borrowed Capital
Current Year 25,000
1/5th of Pre-construction Period (60,000 × 1/5) 12,000 37,000 69,700
39,300
Arrear Rent 36,000
Less: Standard deduction 30% of Arrears of Rent
Received 10,800 25,200
Net Income from House Property 64,500

Note (1): Computation of Gross Annual Value


Municipal value 1,50,000
Actual rent (18,000 x 12) 2,16,000
Higher of the two 2,16,000
Less: Vacancy allowance as property was vacant for four months
(2,16,000/12) × 4 76,000
Gross Annual Value 1,44,000
Space to write important points for revision
[Chapter  6] Income from House Property O 7.115

2013 - June [3] (a) Mr. Sridhar constructed his house on a plot of land
acquired by him in Kolkata. The house has two floors of equal size. He
started construction of the house on 1st April, 2017 and completed
construction on 30th June, 2018. He occupied the ground floor on 1st July,
2018 and let out the first floor at a rent of ` 20,000 per month on the same
date. However, the tenant vacated the first floor on 31st January, 2019 and
Mr. Sridhar occupied the entire house from 1st February, 2019 to 31st March,
2020.
Other information
(i) Fare Rent of each floor ` 1,20,000 per annum
(ii) Municipal value of each floor ` 80,000 per annum
(iii) Municipal tax paid ` 10,000
(iv) Repair expenses ` 5,000
Mr. Sridhar obtained a housing loan of ` 15 lacs at interest of 10% per
annum on 1st July, 2017. He did not repay any part of the loan till 31st March,
2019.
Compute income from house property in the hands of Mr. Sridhar for the
Assessment year 2019-20. (8 marks)
Answer:
Computation of income from house property of Mr. Sridhar for
Assessment Year 2019-20

Particulars Ground Floor First


(Self- Occu- Floor
pied) `
Gross Annual Value Nil 1,40,000
Less: Municipal Tax 5,000
Net Annual Value Nil 1,35,000
Less: Deduction under section 24 (30% of net
annual value) 40,500
Interest on loan:
Current years interest (15,00,000 x 10% =
` 1,50,000) 75,000 75,000
7.116 O Scanner CMA Inter Gr. I Paper 7 (2016 Syllabus)

Pre-construction periods interest (15,00,000


x 10% x 9/12 = ` 1,12,500 to be allowed in
5 equal installments 1,12,500/5 = ` 22,500) 11,250 11,250
Aggregate deduction 86,250 1,26,750
Income from house property (each floor) (86,250)
Loss under the head “income from house
property” 78,000 8,250

Notes: Annual letting value is the higher of fair rent and municipal value.
However, as the construction of the house was completed on 30th June,
2018, annual letting value should be considered for 9 months.
Fair Rent = ` 1,20,000 x 9/12 = ` 90,000
Municipal Value = ` 80,000 x 9/12 = ` 60,000
Annual letting value = ` 90,000
Actual Rent = ` 20,000 x 7 = ` 1,40,000
Gross Annual Value = Higher of annual letting value or actual rent
= ` 1,40,000
Space to write important points for revision

2013 - Dec [2] (d) Mr. Lal is the owner of a commercial property let out at
` 60,000.00 per month. The Corporation tax on the property is ` 30,000.00
annually, 60% of which is payable by the tenant. This tax was actually paid
on 15.04.2019. He had borrowed a sum of ` 40.00 lakhs from his cousin,
resident in Singapore (in dollars) for the construction of the property on which
interest at 8% is payable. He has also received arrears of rent of ` 80,000.00
during the year, which was not charged to tax in the earlier years. What is
the property income of Mr. Lal for the assessment year 2019-20?
(5 marks)
[Chapter  6] Income from House Property O 7.117

Answer:
Assessee: Mr. LAL: Previous Year 2018 - 19 - Assessment Year 2019-20
Computation of income from house property

Particulars ` `
Let out: So, Annual value u/s (i) (a)/ (b)
Actual rent ` 60,000.00 X 12 7,20,000.00
Less: Municipal taxes paid during the
Financial Year 2017 - 18 Nil
Net Annual value (NAV) 7,20,000.00
Less: Deduction u/s 24
(a) 30% of NAV of ` 7,20,000 X 30% 2,16,000.00
(b) Interest on housing loan
(Note) ` 40,00,000 X 8% 3,20,000.00 (-) 5,36,000.00
Income from house property before
considering arrears of rent 1,84,000.00
Arrears of rent received 80,000.00
Less: Deduction u/s 25A(2) - 30% of arrears
received 24,000.00 56,000.00
Net income from house property 2,40,000.00

Note:
(i) It is presumed that the tax has been deducted at source on the
amount of interest payable outside India.
(ii) Corporation tax not deducted because it was paid in F.Y. 2019-20
not in 2018-19.
Space to write important points for revision

2014 - June [2] (c) Ram and Shyam are members of a firm “R and S” and
also joint owners (50% each) of a two storied house property (of equal area),
the details of which are as follows:
(i) Ground Floor - let out at a monthly rent of ` 30,000/-
7.118 O Scanner CMA Inter Gr. I Paper 7 (2016 Syllabus)

(ii) First Floor-used for partnership business of Ram and Shyam.


(iii) Ram and Shyam received the following amounts in respect of another
property which they had sold it on 31.03.2018 :
• Unrealised Rent of same property pertaining to FY 2018-19 -
` 50,000/-
• Arrears of rent of sold out property pertaining to FY 2017-18 -
` 1,00,000/-
(iv) Municipal taxes paid for the entire house property - ` 15,000/- p.a
(v) Interest on borrowings for the entire house property (Joint loan taken
from HDFC)- ` 3,00,000/-
Compute the income from house property and also explain how such
income will be assessed in the hands of R and S. (6 marks)
Answer:
Income from House property
Actual rent (30,000 × 12 – 50,000) 3,10,000
 Municipal Taxes (half of 15,000) 7,500
NAV 3,02,500
 Deduction u/s 24
Standard deduction @ 30% 90,750
Interest on borrowed capital 1,50,000 2,40,750
(half of 3,00,000) 61,750
+ Arrears of rent of FY 2017-18 70,000
(1,00,000  30%) 1,31,750
AS R and S are joint owners (50% each) ` 65,875 (50% of 1,31,750) is
taxable in R’s hand and ` 65,875 is taxable in S’s hand.
Space to write important points for revision

2014 - June [4] (c) Mr. Nitin owns two houses, both of which are occupied
by him for residential purpose. The details are given below :
House-I House-II
Fair rent 7,20,000 6,30,000
Municipal value 5,00,000 5,00,000
[Chapter  6] Income from House Property O 7.119

Standard rent 6,00,000 6,00,000


Date of completion 01.01.2004 01.07.2010
Municipal tax paid 10% 12%
Date of loan 01.07.2000 01.05.2007
Interest on loan for the financial year 2018-19 1,10,000 1,70,000
Compute his income from house property and advise which house should be
opted by him as self occupied. (6 marks)
Answer:
Computation of income from house property of Mr. Nitin for the A. Y. 2018-19
House-I House-II
Option-I Let out Self occupied
Gross annual value 6,00,000 Nil
Less: Municipal tax 50,000 Nil
5,50,000 Nil
Less: Deduction U/s.24
Statutory deduction @ 30% 1,65,000 Nil
Interest on loan 1,10,000 1,70,000
Income from house property 2,75,000 (1,70,000)
Income from house property = ` 1,05,000

House - I House - II
Option - II Set occupied Let out
Gross annual value Nil 6,00,000
Less: Municipal tax Nil 60,000
Nil 5,40,000
7.120 O Scanner CMA Inter Gr. I Paper 7 (2016 Syllabus)

Less: Deduction U/s. 24


Statutory deduction @ 30% Nil 1,62,000
Interest on loan (taken before 01.04.2001) 30,000
Interest on loan 1,70,000
Income from house property (30,000) 2,08,000
Income from house property = ` 1,78,000

Note: The assessee must opt for option - I i.e., taking House- I as let out
and House-II as self occupied. In which case the income from house
property chargeable to tax would be ` 1,05,000.
Space to write important points for revision

2014 - Dec [2] (c) Mr. Singhania constructed a residential house property in
Kanpur. Construction was completed on 1st April, 2018. The house was
vacant from 1st April, 2018 to 30th June, 2018. The house was let out at rent
of ` 7,500 per month from 1st August, 2018. Mr. Singhania obtained loan for
the purpose of construction. Interest paid on such loan during two years prior
to completion of construction amounted to ` 30,000. Interest paid during the
year 2018-19 is ` 16,000. Fire Insurance premium paid is ` 2,000. Municipal
value of the property has been assessed at ` 40,000. Annual corporation tax
paid ` 3,000.
Compute income under the head “Income from House Property” for
Assessment year 2019-20. (6 marks)
Answer:
Computation of Income from house property of Mr. Singhania for
Assessment Year 2019-20.

Particulars ` `

Municipal Value 40,000

Annual rent (` 7,500 x 12) 90,000


[Chapter  6] Income from House Property O 7.121

Higher of Municipal Value and Annual rent 90,000

Less: Vacancy allowance (` 90,000 x 3/12) 22,500

Gross Annual Value 67,500

Less: Corporation tax paid 3,000

Net Annual Value 64,500

Less: Deductions under section 24

(i) Standard deduction at 30% of net annual


value (` 64,500 × x 30%) 19,350

(ii) Interest on loan for prior period (` 30,000 / 5) 6,000

(iii) Interest on loan for current year 16,000 41,350

Income from house property 23,150


Space to write important points for revision

2015 - Dec [5] (a) Raja is the owner of a residential house property having
two independent floors of equal size in Chennai. The ground floor of the
property has been let out to a tenant at rent of ` 15,000 per month from 1st
June, 2018. The first floor of the property is occupied by Raja for his
residential purpose.
Other particulars relating to the property are as follows:

Particulars `

Standard Rent 3,20,000 p.a.

Municipal valuation 3,80,000 p.a.

Fair rent 3,70,000 p.a.

Annual municipal tax (50% paid) 57,000


7.122 O Scanner CMA Inter Gr. I Paper 7 (2016 Syllabus)

Interest on loan taken for construction of property for


the year 2017-18 30,000

Annual insurance premium 5,000


Compute income from house property of Raja for the Assessment Year
2019-20. (7 marks)
Answer:
Calculation of income from house property (let out for 10 months)
Gross annual value 3,20,000
 Municipal tax paid 57,000 × (28,500)
Net annual value 2,91,500
 Standard deduction @ 30% (87,450)
 Interest on loan 30,000 × (25,000)
Income from let of property 1,79,050
Income from self occupied house (1 April to 31 May 2018)
Gross annual value Nil
 Interest on loan (5,000)
Loss from self occupied house (5,000)
Net income taxable under income from house property
Income from let out property 1,79,050
Loss from self occupied house (5,000)
Taxable under head ‘income from house property’ 1,74,050
Working Notes:
Calculation of gross annual value:
(a) Municipal value or fair rent 3,80,000
(whichever is higher)
(b) Standard Rent 3,20,000
(c) Expected rent whichever is lower in (a) and (b) 3,20,000
(d) Actual rent received 15,000 × 10 (June 18 to March 19) 1,50,000
Gross annual value [whichever is higher in (c) and (d)] = 3,20,000
Space to write important points for revision
[Chapter  6] Income from House Property O 7.123

2016 - June [5] (c) Mr. Kamal Hasan has two independent residential flats
in an apartment, both of them being of identical size. First flat is
self-occupied and the second flat is occupied by his daughter, from whom he
does not receive any rent.
For each flat, the relevant annual rent details are as under:
Particulars `
Municipal Value 6,00,000
Fair Rent 5,70,000
Standard Rent 5,16,000
Municipal Tax (fully paid) 11% of municipal valuation
Pre-construction period interest (third year) 1,50,000
Interest on housing loan for current year 1,65,000
(25% unpaid)
Fire Insurance Premium 2,500
Ceiling amount 2,00,000
Compute income of Mr. Kamal Hasan under the head “income from house
property” for assessment year 2019 – 20. (8 marks)
Answer:
Computation of Income from House Property of Mr. Kamal Hasan for
the assessment year 2019–20
Where a house property is occupied by assessee’s daughter, even though
no rent is charged/ received, income from house property has to be charged
to tax.
Flat 1 Self–occupied ` `
Net Annual value Nil
Less: Interest 30,000
Pre- Construction 1/5 Current year 1,65,000
Total 1,95,000
Ceiling amount 2,00,000
Income from this house: Loss (A) 1,95,000
7.124 O Scanner CMA Inter Gr. I Paper 7 (2016 Syllabus)

Flat 2 Deemed let out


Gross Annual Value 5,16,000
[Higher of Municipal value or Fair value, but
restricted to standard rent]
Less: Municipal taxes 66,000
Net annual value 4,50,000
Less: Deduction u/s 24
Standard deduction 30% of NAV 1,35,000
th
For pre-construction (1/5 ) 30,000
For current year 1,65,000
Total deductions 3,30,000
Income from this house property (B) 1,20,000
Income chargeable under the head “Income from (A+B) -75,000
house property”

Note: No deduction is available for Fire Insurance Premium.


Space to write important points for revision

2016 - Dec [2] (b) Mr. Ashok owns two buildings which are let out during the
financial year 2018-19. The relevant details are as under:
Particulars House-I House-II
Residential Commercial
` `
Municipal valuation 1,80,000 3,60,000
Standard rent 1,50,000 3,00,000
Actual rent 2,40,000 6,00,000
Municipal taxes - paid 20,000 30,000
Municipal taxes - outstanding 10,000 15,000
[Chapter  6] Income from House Property O 7.125

Interest on moneys borrowed - paid 60,000 20,000


Interest on moneys borrowed - outstanding 1,00,000 60,000
Housing loan principal repaid to bank 50,000 30,000
You are requested to compute income of Mr. Ashok under the head “Income
from House property” for the assessment year 2019-20. (9 marks)
Answer:
Computation of Income from House Property of Mr. Ashok for the A.Y.
2019-20:
House I House II
(i) Expected Rent Standard Rent or Municipal
Valuation 1,50,000 3,00,000
(ii) Actual Rent 2,40,000 6,00,000
(Which ever is More)
Gross Annual Value 2,40,000 6,00,000
Less: Municipal Tax paid 20,000 30,000
Net Annual Value 2,20,000 5,70,000
Less: Deduction u/s 24 (66,000) (1,71,000)

(iii) Interest on Money borrowed (Paid + (1,60,000) (80,000)


Outstanding)
Income from House Property (6,000) 3,19,000
Income from house property 3,19,000 - ` 6,000 3,13,000
Less: Principal amount repaid for residential 50,000
building
Principal repaid for commercial building-not Nil
eligible
Total Income 2,63,000
Space to write important points for revision
7.126 O Scanner CMA Inter Gr. I Paper 7 (2016 Syllabus)

2017 - Dec [2] (a) Surbhi has two houses, both of which are self-occupied.
You are required to compute Surbhi’s income form house property for the
Assessment Year 2019-20 and suggest which house should be opted by
Surbhi to be assessed as self-occupied so that her tax liability is minimum.
The particulars of these are given below:

(Value in `)
Particulars House - I House - II
Municipal Valuation per annum 1,30,000 1,15,000
Fair Rent per annum 1,10,000 1,70,000
Standard rent per annum 1,00,000 1,65,000
Date of completion 31-03-2001 31-03-2003
Municipal taxes payable during the year 12% 8%
(paid for House II only)
Interest on money borrowed for repair of – 55,000
property during current year
(6 marks)
Answer:
Under section 23(4), where both the houses are self-occupied, then, at the
option of the assessee, any one of the houses shall be treated as self
occupied and the other house/houses be deemed to be let out. For the
purpose of Tax planing, the assessee should select the house as self
occupied in such a way that it minimizes the tax burden of the assessee.
H1 Self- H2 Let out
occupied
Option - A NIL 1,65,000
Gross Annual value (for the let out house
higher of municipal value and fair rent but not
exceeding standard rent)
Less: Municipal tax NIL 9,200
[Chapter  6] Income from House Property O 7.127

Net Annual value NIL 1,55,800


Less: Deduction u/s 24 NIL
(a) Standard deduction @ 30% 4,6,740
(b) Interest on borrowed capital 55,000
Income from House Property NIL 54,060
Option - B H-1 Let out H2 Self-occ.
Gross Annual value (for the let out house
higher of municipal value and fair rent but not
exceeding standard rent) 1,00,000 NIL
Less: Municipal tax paid NIL NA
Net Annual value 1,00,000 NIL
Less: Deduction u/s 24
(a) Standard deduction @ 30% 30,000 NIL
(b) Interest on borrowed Capital NIL (30,000)
Income from House Property 70,000 30,000

Income from House Property


H-1 H-2 Total
Option -A NIL 54,060 54,060
Option - B 70,000 (30,000) 40,000
Since option B results in a lower income from house property, it would also
result in lower tax burden. The assessee should therefore treat the first
house as let out and the second house as self occupied.
Space to write important points for revision

2018 - June [2] (b) Two brothers Rama and Shankar are co-owners of a
house property with equal shares. The property was constructed during the
Financial year 1999-2000. The property consists of 8 identical units and is
7.128 O Scanner CMA Inter Gr. I Paper 7 (2016 Syllabus)

situated at Salem. During the Financial Year 2018-2019 each owner


occupied 1 unit for residence and balance 6 units were let out at a rent of
` 14,000 per unit per month. The municipal value of property is ` 9,00,000
and municipal tax are 10% of municipal value, paid during the year. The
other expenses are as follows:
(i) Repairs ` 90,000
(ii) Insurance premium paid ` 15,000
(iii) Interest payable on loan taken ` 3,50,000
One of the let out remained vacant for 4 months during the year. Rama could
not occupy his unit for 6 months as he was transferred to Bangalore. He
does not own any other house. The other income of Rama and Shankar are
` 3,50,000 and ` 1,80,000 respectively for the Financial Year 2018-2019.
The co-owners received during the year ` 1,40,000 as unrealized rent for
2015-2016 and ` 50,000 as arrears of rent.
Compute the income under the head “Income from House Property” and total
income of the two brothers for the Assessment Year 2019-2020.
(7 marks)
Answer:
Computation of Income from house property of Mr. Nitin for the
Assessment Year 2018-19
` `
Income from self occupied Unit
Net Annual Value (NAV) Nil
Less: Deduction u/s 24
(a) Standard deduction i.e. @ 30% of NAV Nil
(b) Interest on loan taken 87,500
Net Loss from self occupied unit (87,500)
Computation of income from let out unit
Gross annual value (GAV) 9,52,000
Less: Municipal tax 67,500
Net Annual Value (NAV) 8,84,500
[Chapter  6] Income from House Property O 7.129

Less: Deduction u/s 24


(a) Standard deduction i.e. 30% of NAV
(8,84,500 × 30%) 2,65,350
(b) Interest on capital borrowed 3,50,000 ×
2,62,500 (5,27,850)
3,56,650
Add: Arrear Rent 50,000
Less: Standard deduction 30% of arrears of
rent received (15,000) 35,000
3,91,650
Add: Unrealized rent for 2015-16 1,40,000
Less: Standard deduction 30% of unrealized
rent (42,000) 98,000
Total income from let out portion of house 4,89,650
Income of both brother from house property Ram Shankar
and other source
Income from house property
Let out portion 2,44,825 2,44,825
Less: Self-occupied portion (Restricted (30,000) (30,000)
` 30,000)
Net Income from house property 2,14,825 2,14,825
Other income 3,50,000 1,80,000
Total income 5,64,825 3,94,825
Space to write important points for revision
7.130 O Scanner CMA Inter Gr. I Paper 7 (2016 Syllabus)

2018 - Dec [2] (b) Mr. Chaturvedi, Delhi has 3 house properties in various
parts of India. The details are given below:
Location of property Delhi Chandigarh Kolkata
Usage Self occupied Let out Let out
Amount ` Amount ` Amount `
Rent received NIL 3,60,000 1,80,000
Fair rent 2,40,000 3,00,000 1,50,000
Municipal value 2,10,000 2,40,000 1,20,000
Standard rent 1,80,000 2,10,000 90,000
Municipal tax - Due 20,000 40,000 30,000
Municipal tax - paid by the NIL NIL 20,000
assessee
Interest on moneys borrowed 2,80,000 1,40,000 1,50,000
Note: All the properties were acquired/constructed after 01.04.2011.
You are required to compute the income of Mr. Chaturvedi chargeable under
the head “Income from house property” for the assessment year 2019-20.
(6 marks)
7 PROFITS AND GAINS OF
BUSINESS OR PROFESSION
THIS CHAPTER INCLUDES
 Meaning of Business and  Expenditures allowed on cash
Profession basis [Sec. 43B]
 Expenditures allowed as  Maintenance of books of
deduction account [Sec. 44AA & Rule 6F]
 Depreciation [Sec. 32]  Tax Audit [Sec. 44AB]
 Scientific Research [Sec. 35]  Incomes not taxable under the
 General deductions [Sec. head profits and gains of
37(1)] business or profession
 Disallowed Expenditure [Sec.
40]
Marks of Objective, Short Notes, Distinguish Between, Descriptive & Practical Questions
Legend
Objective Short Notes Distinguish Descriptive Practical

For detailed analysis Login at www.scannerclasses.com


for registration and password see first page of this book.

7.131
7.132 O Scanner CMA Inter Gr. I Paper 7 (2016 Syllabus)

SHORT NOTES
2009 - June [7] Write a short note on the following:
(b) Reverse mortgage and its income-tax implications; (5 marks)
Answer:
Reverse Mortgage: The Reverse Mortgage scheme is for the benefit of
senior citizens, who own a residential house property. In order to supplement
their existing income, they can mortgage their house property with a
scheduled bank or housing finance company, in return for a lump-sum
amount or for a regular monthly/quarterly/annual income. The senior citizen
can continue to live in the house and receive regular income, without the
botheration of having to pay back the loan.
The loan will be given up it say, 60% of the value of residential house
property mortgaged. Also, the bank/housing finance company would
undertake a revaluation of the property once every 5 years. The borrower
can use the loan amount for renovation and extension of residential property,
family’s medical and emergency expenditure etc. amongst others. However,
he cannot use the amount for speculative or trading purposes.
The bank will recover the loan along with the accumulated interest by
selling the house after the death of the borrower. The excess amount will be
given to the legal heir. However, before resorting to sale of the house,
preference will be given to the legal heirs to repay the loan and interest and
get the mortgaged property released.
Not regarded as “transfer”
The Finance Act, 2008 has inserted Clause (xvi) in Section 47 to
clarify that any transfer of a capital asset in a transaction of reverse
mortgage under a scheme made and notified by the Central Government
would not amount to a transfer for the purpose of capital gains.
Exemption for moneys received in reverse mortgage
Further, Clause (43) has been inserted in Section 10 to provided that
the amount received by the senior citizen as a loan, either in lump sum or in
installments, in a transaction of reverse mortgage would be exempt from
income-tax.
Space to write important points for revision
[Chapter  7 ] Profits and Gains of Business ... O 7.133

DESCRIPTIVE QUESTIONS
2008 - Dec [5] (f) State any ten circumstances when payment exceeding
` 10,000 relating to an expenditure, made otherwise than by a crossed
cheque or by a crossed demand draft, will not be disallowed under section
40A(3) of the Income-tax Act, 1961. (5 marks)
Answer:
In rule 6DD, expenditure even exceeding ` 10,000/` 35,000 shall be allowed
as deduction, even though the payment or aggregate of payments made to
a person in a day is not made by an account payee cheque/draft.
These circumstances are:-
(a) where the payment is made to the Government and, under the rules
framed by it, such payment is required to be made in legal tender;
(b) where the payment is made by way of adjustment against the amount of
any liability incurred by the payee for any goods supplied or services
rendered by the assessee to such payee;
(c) where the payment is made for the purchase of the products
manufacturing or processed without the aid of power in a cottage
industry, to the producer of such products;
(d) where the payment is made in a village or town, which on the date of
such payment is not served by any bank, to any person who ordinarily
resides, or is carrying on any business, profession or vocation, in any
such village or town;
(e) where any payment is made to an employee of the assessee or the heir
of any such employee, in connection with the retirement, retrenchment,
resignation, discharge or death of such employee, on account of gratuity,
retrenchment compensation or similar terminal benefit and the aggregate
of such sums payable to the employee or his heir does not exceed fifty
thousand rupees;
(f) where the payment was required to be made on a day on which the
banks were closed either on account of holiday or strike;
(g) where the payment is made by any person to his agent who is required
to make payment in cash for goods or services on behalf of such person;
7.134 O Scanner CMA Inter Gr. I Paper 7 (2016 Syllabus)

(h) where the payment is made by an authorized dealer or a money changer


against purchase of foreign currency or travelers cheques in the normal
course of his business;
(i) where the payment is made by an assessee by way of salary to his
employee after deducting the income tax from salary in accordance with
the provisions of Section 192 of the Act, and when such employee-
(a) is temporarily posted for a continuous period of fifteen days or more
in a place other than his normal place of duty or on a ship; and
(b) does not maintain any account in any bank at such place or ship.
(j) where the payment is made for the purchase of –
(a) agriculture or forest produce; or
(b) the produce of animal husbandry (including livestock, meat, hides
and skins) or dairy or poultry farming; or
(c) fish or fish produce; or
(d) the products of horticulture or apiculture, to the cultivator, grower or
producer of such articles, produce or products.
Space to write important points for revision

2009 - June [2] (c) Is commodities transaction tax a permissible business


expenditure? Explain. (4 marks)
Answer:
Deduction of commodities transaction tax paid [Section 36(1)(xvi)]
Commodities Transaction Tax (CTT) would be levied on taxable commodities
transaction entered in a recognized association.
Where an assessee has paid commodities transaction tax in a Taxable
commodities transaction, the same is a permissible deduction while
computing the business income from such taxable commodities transaction.
Taxable commodities transaction' means a transaction of purchase or sale
in a recognized association of.
(i) option in goods; or
(ii) option in commodity derivation; or
(iii) any other commodity derivative.
Space to write important points for revision
[Chapter  7 ] Profits and Gains of Business ... O 7.135

2009 - Dec [1] {C} (d) (iii) Whether profit arising on the valuation of closing
stock is a source of income? Discuss. (2 marks)
Answer:
No profit arises on the valuation of closing stock. It is a misconception to
think that any profit “arises out of the valuation of closing stock”. Valuation
of unsold stock at the close of an accounting period is a necessary part of
the process of determining the trading result of that period and can in no
sense be regarded as the “source” of such profits, nor can the place where
such valuation is made be regarded as the situs of their accrual - Charnrup
Sampatram vs. CIT [1953] 24 ITR 481 (SC).
Space to write important points for revision

2010 - June [3] (b) Discuss the allowability of the following expenditure while
computing income under the head "Profit and gains of business or
profession”.
(i) Expenses incurred on partly convertible debentures, and (2 marks)
(ii) Expenditure incurred on MS Office software. (2 marks)
Answer:
(i) The expenditure incurred on partly - convertible debentures is revenue
expenditure eligible for deduction while computing business income.
The Court/Tribunal observed that issue of shares is a future event,
which may or may not happen. Since, at present, it is an expenditure
incurred on the issue of debentures only, it is revenue expenditure
eligible for deduction under section 37(1).
(ii) The Delhi High Court has, in CIT vs. GE Capital Services Ltd. (2007)
held that only customized software can have enduring value. The
Court/Tribunal observed that MS Office software is not a customized
software. Due to technological changes and the need to upgrade the
software on a regular basis, it could not be said that the software is of
an enduring nature. Therefore, the expenditure incurred on MS Office
software is allowable business expenditure.
Space to write important points for revision
7.136 O Scanner CMA Inter Gr. I Paper 7 (2016 Syllabus)

2010 - Dec [2] (c) Discuss the conditions to be fulfilled for the applicability of
Section 35AD of the Income-tax Act, 1961. (5 marks)
Answer:
Sec. 35AD - Deduction in respect of expenditure on specified business.
Conditions for the applicability of Section 35AD of the Income Tax Act,
1961:
The specified business must not be set up
(i) by splitting up, or the reconstruction, of a business already in
existence,
(ii) by transfer of plant and machinery previously used for any purpose.
However there is no violation of the conditions given under section
35AD
(a) If the value of second hand plant and machinery do not exceed
20% of total plant & machinery used in that business or
(b) Such second hand machinery is imported by the assessee on
which no depreciation was allowed or allowable under this Act in
any other earlier year.
(iii) Where the nature of business referred in Section 8(c)(iii) such
business-
(a) is owned by a company formed and registered in India under the
Companies Act, 2013 or by a consortium of such companies or
by an authority or board or a corporation established or
constituted under any Central or State Act;
(b) has been approved by the Petroleum and Natural Gas Regulatory
Board established under section 3(1) of the Petroleum and
Natural Gas Regulatory Board Act, 2006 (19 of 2006) and
notified by CG in the official Gazette in this behalf
(c) has made not less than such proportion of its total pipeline
capacity as specified by regulations made by the Petroleum and
Natural Gas Regulatory Board established under section3(1) of
the Petroleum and Natural Gas Regulatory Board Act, 2006
available for use on common carrier basis by any person other
than the assessee or an associated person; and
(d) fulfils any other condition as may be prescribed.
Space to write important points for revision
[Chapter  7 ] Profits and Gains of Business ... O 7.137

2012 - Dec [4] (b) Name any four specified businesses covered under
section 35AD and state the fiscal incentives available to such businesses.
(5 marks)
Answer:
Specified businesses covered by Section 35AD are:
(i) Setting up and operating cold chain facilities for specified products.
(ii) Setting up and operating warehousing facilities for storing agricultural
produce.
(iii) Laying and operating cross – country natural gas or crude or
petroleum oil pipe line network for distribution, including storage
facilities being as integral part of such network.
(iv) Building and operating, anywhere in India, a hotel of two star or above
category as classified by CG.
(v) Building and operating, anywhere in India, a hospital with atleast one
hundred beds for patients.
(vi) Developing and building a housing project under a scheme for slum
redevelopment or rehabilitation framed by the Central Government or
State Government, as the case may be, and notified by the CBDT in
this behalf in accordance with the guidelines as may be prescribed.
(vii) Developing and building a housing project under a scheme for
affordable housing framed by the Central Government or State
Government, as the case may be, and notified by the CBDT in this
behalf in accordance with the guidelines as may be prescribed.
(viii) Production of fertilizer in India in new plant or in a newly installed
capacity in an existing plant. (w.e.f. 1-4-2012)
Availability of Fiscal Incentives:
100% of the capital expenditure incurred, wholly and exclusively, for the
purpose of any specified business carried on by the assessee shall be
allowed as deduction during the previous year in which such expenditure is
incurred by him. However, expenditure towards acquisition of land, goodwill
or financial instrument would not be eligible for deduction.
The loss of an assessee on account of a “specified business” claiming
deduction u/s 35 would be allowed to be set-off against the profits of any
other specified business under section 73A, whether or not the latter is
eligible for deduction u/s 35AD.
Space to write important points for revision
7.138 O Scanner CMA Inter Gr. I Paper 7 (2016 Syllabus)

2012 - Dec [6] (b) Discuss whether it is required on the part of an assessee
desiring to claim deduction of bad debt written off, to prove that the debt
written off in the books as irrecoverable or bad debt, is incapable of recovery
or that the said debt had really turned bad. (4 marks)
Answer:
Write-off of bad debt
Bad debts to be allowed as deduction only in the year in which they
become irrecoverable on the basis of recently notified income
computation and disclosure standards without recording the same in
the accounts [Section 36(1)(vii)] [W.e.f. A.Y. 2016-17]
Where the amount of debt or part thereof which has been taken into account
in computing the income of the assessee of the previous year in which the
amount of such debt or part thereof becomes irrecoverable or of an earlier
previous year on the basis of income computation and disclosure standards
notified under section 145(2) without recording the same in the accounts,
then, such debt or part thereof shall be allowed in the previous year in which
such debt or part thereof becomes irrecoverable and it shall be deemed that
such debt or part thereof has been written of as irrecoverable in the accounts
for the purposes of this clause.
Space to write important points for revision

2012 - Dec [8] (a) Please state the deductibility of the following expenses:
(i) Litigation expenses for restraining another company from using
assessee’s trade mark.
(ii) Brokerage paid for raising loan to finance assessee-company’s
business.
(iii) Compensation paid to cancel purchase order of a machine due to
abnormal rise in its price. Assessee claims it as a trading loss.
(6 marks)
Answer:
(i) Litigation expenses for restraining another company from using
assessee’s trade mark, is one expended out of commercial
expediency, wholly and exclusively for the purchase of the business;
being revenue expenditure, the same is an allowable expenditure u/s
37.
[Chapter  7 ] Profits and Gains of Business ... O 7.139

(ii) As per Section 37 deduction of expenditure is allowed if it is done to


expend wholly and exclusively for the purpose of such business or
profession. In this case brokerage paid for raising loan to finance
business is a deductible expenditure u/s 37 as it is incurred wholly and
exclusively for the purpose of the business.
If any tax is deductible at source u/s 194-H, the same has to be done;
the provisions of Section 40(a)(ia) should not be contravened.
(iii) As per Section 37 deduction of expenditure is not allowed if it’s nature
of capital expenditure. In this case compensation paid to cancel
purchase order of a machine, is a capital expenditure. It cannot be
deducted u/s 37. As there is no ‘transfer’ of any capital asset,
compensation paid cannot be claimed as a Capital loss also.
Space to write important points for revision

2013 - June [6] (d) (i) State who are the persons not eligible to avail any
benefit u/s 44 AD. (2 marks)
(ii) An assessee owns a light commercial vehicle for 8 months and 3 days,
a medium goods vehicle for 11 months and another medium goods
vehicle for 12 months during the previous year.
Compute his profits from the three trucks in terms of Section 44 AE.
(2 marks)
Answer:
Sec. 44AD: Special provision for computing profit and gains of business of
civil construction.
(i) The following persons are not eligible u/s 44AD.
(a) Person carrying on profession as referred to in Section 44AA(1)
(b) A person earning income in the nature of commission or
brokerage;
(c) A person carrying on any agency business; or
(d) A person who is in the business of plying, hiring or leasing goods
carriages.

Note: Section 44AA - maintenance of books of accounts by certain


person carrying on profession or business.
7.140 O Scanner CMA Inter Gr. I Paper 7 (2016 Syllabus)

(ii) Provision of Section 44AE for transporters:


A taxpayer who own not more than 10 goods carriage and is engaged
in the business of plying, hiring and leasing of goods carriage, his
income would be deemed to be ` 7,500 per good carriage per month.
So, profit and gain of the assessee u/s 44AE would be:
(7,500 x 9) + (7,500 x 11) + (7,500 x 12) = ` 2,40,000
Amendment to [Section-44AE]
Section 44AE(2) has been substituted (with effect from the assessment year
2019-20) so as to provide that for a heavy goods vehicle, the profits and
gains shall be an amount equal to ` 1,000 per ton of gross vehicle weight (or
unladen weight) for every month (or part of a month) during which the heavy
goods vehicle is owned by the assessee in the previous year or an amount
claimed to have been actually earned from such vehicle, whichever is higher.
In the case of a goods carriage other than heavy vehicle, the profits and
gains shall be an amount equal to ` 7,500 for every month (or part of a
month) during which the goods carriage is owned by the assessee in the
previous year or an amount claimed to have been actually earned from such
goods carriage, whichever is higher.
For this purpose, “heavy goods vehicle” means any goods carriage the gross
vehicle weight of which exceeds 12,000 kilograms.
Space to write important points for revision

2013 - June [8] (e) State the deductibility of the following expenses while
computing the business income:
(i) Anticipated hedging loss under a contract to purchase raw material;
(ii) Consultation fees paid to tax advisor;
(iii) Advertisement expenses incurred outside India in foreign currency.
RBI permission has not been obtained;
(iv) 500 VIP briefcases costing ` 2,000 each presented to customers;
(v) Travelling expenses incurred to explore the feasibility of new line of
business;
(vi) The assessee claims the setoff of unabsorbed depreciation of a
discontinued business against the profits of another business.
(6 × 1= 6 marks)
[Chapter  7 ] Profits and Gains of Business ... O 7.141

Answer:
The deductibility of the following expenses while computing the business
income:
(i) Anticipated hedging loss under a forward contract is not allowed to be
deducted
(ii) Consultation fees paid to tax advisor is allowed under section 37(1).
(iii) Advertisement expenses incurred in India or outside India is allowed
to be deducted under section 37(1), provided it is not of a capital
nature and it is incurred wholly and exclusively for the purpose of
business. Permission of RBI is not relevant.
(iv) Presentation of VIP bags to customers is allowed as expenditure on
advertisement under section 37(1). There is no ceiling limit for gift
articles.
(v) Travelling expenditure for exploring new line of business is a capital
expenditure. It is not allowed under sec. 37(1). It may be capitalized
for the purposes of Sec. 35D.
(vi) Unabsorbed depreciation of a discounted business now can be set off
against the profits of any other business and thereafter against income
of any other head.
Space to write important points for revision

2013 - Dec [6] (a) State the conditions to be fulfilled for deduction in respect
of write off of bad debt. (3 marks)
Answer:
Please refer 2012 - Dec {6} (b) on page no. 138
Space to write important points for revision

2014 - June [2] (b) Briefly explain the deduction allowable u/s 37 of the
Income Tax Act, 1961 in respect of income from business or profession.
(4 marks)
Answer:
General Deduction [Section 37(1)]
Section 37(1) is a residuary section. In order to claim deduction under this
Section, the following conditions should be satisfied:
7.142 O Scanner CMA Inter Gr. I Paper 7 (2016 Syllabus)

1. The expenditure should not be of the nature described under section


30 to 36.
2. It should not be in the nature of capital expenditure.
3. It should not be personal expenditure of the assessee.
4. It should have been incurred in the previous year.
5. It should be in respect of business carried on by the assessee.
6. It should not have been incurred for any purpose which is an offence or
is prohibited by any law. Thus, penalty paid under any law is not
deductible.
Space to write important points for revision

2015 - June [1] {C} (c) Attempt the question below:


(iii) What is the rate of tax applicable to income of a non-resident
sportsman who is a foreign national by way of participation in any sport
in India? (1 mark)
Answer:
(iii) 20%
Space to write important points for revision

2015 - June [2] (c) Name any four businesses which are eligible for
deduction under section 35AD of the Income-tax Act, 1961. (4 marks)
Answer:
Section 35AD: Deduction in respect of expenditure on specified business.
Eligible businesses u/s 35AD:
The following businesses are specified business as per Section 35AD(8):
• Setting up and operating a cold chain facility
• Setting up and operating a warehouse facility for storage of agricultural
produce.
• Laying and operating a cross country natural gas or crude or petroleum
oil pipe line network for distribution, including storage facilities being an
integral part of such network.
• Building and operating, anywhere in India, a hotel of two star or above
category as classified by the Central Government.
• Building and operating, anywhere in India, a hospital with at least one
hundred beds for patients.
[Chapter  7 ] Profits and Gains of Business ... O 7.143

• Developing and building a housing project under a scheme for slum


redevelopment or rehabilitation framed by the Central Government or
State Government, as the case may be and notified in this behalf.
• Developing and building a housing project under a scheme for affordable
housing framed by the Central Government or State Government, as the
case may be and notified in this behalf.
• Production of fertilizers in India.
• Setting up and operating an inland container depot or a container freight
station notified or approved under the Customs Act, 1962.
• Bee-keeping and production of honey and beeswax.
• Setting up and operating a warehouse facility for storage of sugar.
• Laying and operating a slurry pipeline for the transportation of iron ore;
• Setting and operating a semi-conductor wafer fabrication manufacturing
unit notified by the Board in accordance with such guidelines as may be
prescribed.
Space to write important points for revision

2015 - June [4] (b) When are the books of account of a person engaged in
notified profession liable for audit under section 44AB? What is the penalty
is leviable for failure to get the accounts audited? (2 marks)
Answer:
Every person carrying on profession shall, if his gross receipts in profession
exceed ` 50 lakhs in any previous year must get his accounts audited under
section 44AB.
For failure to get the accounts audited the penalty leviable is 0.5% of the
gross receipt or ` 1,50,000 – whichever is less (Section 271B).
Space to write important points for revision

2015 - Dec [1] {C} Answer the following question with brief reason/working:
(b) Are profits on transfer of shares and securities held by a Foreign
Institutional Investor chargeable to tax under the head “profits and gains
of business or profession” or “capital gains”? (2 marks)
7.144 O Scanner CMA Inter Gr. I Paper 7 (2016 Syllabus)

Answer:
Profit/gain on transfer of share and securities held by a foreign institutional
investor shall be chargeable to tax under the head ‘capital gain’.
Space to write important points for revision

PRACTICAL QUESTIONS
2008 - Dec [2] (a) Answer the following question briefly:
(v) Vasudha Plastics Ltd., engaged in manufacture of PVC pipes,
purchased a machinery on 25.09.2018 for ` 2,00,000 and put it to use
after two weeks. There is no other asset in the block. What is the WDV
of the block as on 31.03.2019. (3 marks)
Answer:
As per the provision of Sec. 32:
The machine was put to use for less than 180 days, only half of normal
depreciation of 15% and additional depreciation of 20% (i.e., 7.5 plus 10%)
will be available.
Total depreciation available ` 35,000
WDV of the block is ` 1,65,000.
Note:- Computation of Depreciation:
7.5% on ` 2,00,000 = 15,000
Additional Dep. 10% on ` 2,00,000 = 20,000
Total = 35,000
Space to write important points for revision

2008 - Dec [6] (c) Vasudevan purchased a motor car in December, 2018,
which was used by him for business purposes during rest of the year.
However, the relevant transfer in the records of the Regional Transport
Office was made only in April, 2019. Can the assessee claim depreciation
on motor car for the assessment year 2019-20. (2 marks)
[Chapter  7 ] Profits and Gains of Business ... O 7.145

Answer:
Assessee (Vasudevan) becomes the owner of the motor car, a movable
asset, when he takes delivery of the motor car i.e., in December, 2018. He
has used the car in his business during the year 2018-19. Non-registration
in RTO records during the year is not relevant.
The car, being a plant, will form part of the relevant block of assets and
he is entitled to claim depreciation allowance under section 32 at the
applicable rate.
Space to write important points for revision

2009 - June [8] (c) Discuss whether the following payments are permissible
as business expenditure;
(i) For an existing business, on 10.05.2018, an assessee had borrowed
` 25 lakhs for acquiring a machinery. Interest paid ` 1,10,000/-, The
machinery was not put to use during the year ended 31.03.2019.
(ii) Payment of ` 60,000/- was made to a Don for ensuring that the
employees will not indulge in strike. The Don had threatened initiating
a labour strike. (2 × 2 = 4 marks)
Answer:
(i) This is a case of borrowing for an existing business and not extension
of the existing business or starting of new line of business. The
borrowing is for the purpose of the business and hence the interest on
such borrowing is allowable u/s. 36(1)(iii). It is not essential that the
asset acquired using the borrowing funds should have been put to use
it previous year or should have yielded some income. This is a
deductible expenditure. The proviso to Section 36(1)(iii) allowing
interest on borrowing for a new asset only for the date on which the
asset is first put to use, applies only where the borrowing is for
extension of the existing business.
(ii) As per Explanation to Section 37(1), since the payment to Don is for
an unlawful purpose, the same is not permissible business
expenditure.
Space to write important points for revision
7.146 O Scanner CMA Inter Gr. I Paper 7 (2016 Syllabus)

2009 - Dec [4] (a) X, a retail trader of Mumbai gives the following Trading
and Profit and Loss Account for the year ended 31st March, 2019
Trading and Profit and Loss Account for
the year ended 31.03.2019
` `

To Opening Stock 86,000 By Sales 12,11,500


" Purchases 10,00,000 " Other business receipts 6,500
" Gross profits 3,06,000 " Income from UTI 2,400
" Closing Stock 1,71,600
13,92,000 13,92,000
To Salaries 60,000 By Gross Profit B/D 3,06,000
" Rent and rates 40,000
" Interest on Loan 10,000
" Depreciation 1,02,000
" Postage and Telegram 1,640
" Printing and Stationary 23,200
" Loss on sale of Shares 8,100
(Short term)
" General Expenses 11,060
" Net Profit 50,000
3,06,000 3,06,000
Additional Information:
(i) Some stocks were found to be not included in both the opening and
Closing Stock, the value of which were: Opening Stock ` 12,000,
Closing stock ` 21,000.
(ii) Salary include ` 18,000 paid to his brother, which is unreasonable to
the extent of ` 2,000.
(iii) The whole amount of Printing and Stationary was paid in cash.
(iv) Depreciation as per Income Tax rules is ` 66,000.
(v) Rent and Rates includes Sales Tax Liability of ` 5,000 paid on April, 4, 2019.
(vi) Other Business Receipts include `3,000 received as refund of Sales
Tax relating to the year, 2017-18.
(vii) General expenses include ` 1,500 paid as donation to A Public
Charitable Trust.
You are required to Advise X whether he can offer his Business
Income under sec. 44 AD, i.e. presumptive Taxation. (10 marks)
[Chapter  7 ] Profits and Gains of Business ... O 7.147

Answer:
Computation of Income from Business of Mr. X for the A.Y. 2019-20

` `
Net profit as per P/L A/c 50,000
Add: Item to be added back:
Loss on sale of shares 8,100
Closing stock not included 21,000
Depreciation 1,02,000
Unreasonable Salary 2,000 1,33,100
[Section40(a) (2)] 1,83,100
Less: Item to be deducted:
Income from UTI 2,400
Opening stock not included 12,000
Depreciation allowed 66,000 (80,400)
Income from business 1,02,700
As per Section 44AD, in the case of an assessee engaged in any
business (except playing, hiring or leasing of goods carriage as per section
44AE) a sum equal to 8% of the turnover in previous year on account of such
business shall be deemed to be profit and gains of such business
chargeable under the head profits and gains of business or profession
provided total turnover of such retail trades does not exceed ` 2 crore.
Income under presumptive taxation scheme 12,11,500 × 8% = ` 96,920
Advise: Total turnover is less than ` 2 crore so, he can offer his business
income under section 44AD, i.e., presumptive taxation.
Since, the income from business in more than 8% of turnover, so it is
advisable to Mr. X to opt for Section 44AD.
Space to write important points for revision

2009 - Dec [7] (c) Simplex Ltd. a company providing telecommunication


service, obtain a Telecom Licence on April 20, 2018 for a period of 10 years
which ends on March 31, 2018 (Licence Fee being ` 18 lakh). Find out the
amount of deduction under section 35ABB if—
7.148 O Scanner CMA Inter Gr. I Paper 7 (2016 Syllabus)

(i) The entire amount is paid on May 6, 2018; or


(ii) The entire amount is paid on April 1, 2019; or
(iii) The entire amount is paid in 3 equal instalments on April 30, 2018,
April 30, 2019 and April 30, 2020. (5 marks)
Answer:
Situation (a) The payment is ` 18 lakh is deductible in 10 instalments over
a period of 10 years from the previous year's 2018-19 to 2027-28 (the
amount deductible each year being ` 1.8 lakh).
Situation (b) The payment is deductible in 9 years starting from the year of
payment, i.e., the previous year 2019-20 and ending with the previous year
2027-28 (the amount deductible each year being ` 2 lakhs).
Situation (c) The entire payment is made in three installments. Deduction
under section 35ABB is available as under-
First Second Third Total
installment installment installment
Date of payment April April April
Period during which 30, 2018 30, 2019 30, 2020
deduction is available 10 years 9 years 8 years
Amount of payment (2018-19 to (2019-20 to (2020-21 to
2027-28) 2027-28) 2027-28)
` 6 lakhs ` 6 lakhs ` 6 lakhs
Amount deductible in ` ` ` `
previous year
2018-19 60,000 -- -- 60,000
2019-20 60,000 66,667 -- 1,26,667
2020-21 to 2027-28 60,000 66,667 75,000 2,01,667
Space to write important points for revision

2009 - Dec [8] (a) Indicate, with reasons whether the following expenses are
deductible in computing income from business for the Assessment Year
2019-20.
[Chapter  7 ] Profits and Gains of Business ... O 7.149

(i) ` 10,000 paid to a Lawyer to examine the Title Deed which the
Assessee intends to purchase.
(ii) Compensation of ` 20,000 paid to an employee who has been
retrenched for continuously committing some fraudulent Acts.
(iii) ` 5,000 paid for shifting the factory from one place to another for
easier supply of raw materials.
(iv) Penalties of ` 10,000 paid to the West Bengal Government for the
non-fulfilment of contract entered into with the said Government.
(4 marks)
Answer:
(i) It's a capital expenditure since it is made for purchasing capital assets.
(ii) It is allowable expenditure, because its for business purpose.
(iii) Expenses paid for shifting the factory from one place to another for
easier supply of raw material is trading expenditure and allowable
expenditure.
(iv) As per case law “Hoshiyari Lal Keval Krishna (2008)” it was held by
High Court that if any penalties are compensatory in nature it will be
allowed as business expenditure, but if they are penal in nature it will
be not allowed as business expenditure, this contention was held true
also by apex (Supreme) Court in Prakash Mills case. So, penalty for
non fulfilling of contract is compensatory in nature and allowable.
Space to write important points for revision

2010 - June [2] (b) Ramesh an individual, is engaged in manufacture of


fertilizers . He is following mercantile system of accounting. He borrowed
loans from Tamil Nadu Industrial Development Corporation and Indian Bank
and has not paid interest as detailed below:
`
(i) Tamil Nadu Industrial Development Corporation 15,00,000
(P.Y. 2017-18 & 2018-19)
(ii) Indian Bank (P.Y. 2018-19) 30,00,000
45,00,000
7.150 O Scanner CMA Inter Gr. I Paper 7 (2016 Syllabus)

Both Tamil Nadu Industrial Development Corporation and Indian Bank,


while re-phasing the loan facilities of Ramesh during the year ended
31.3.2019, converted the above outstanding interest due to them from
Ramesh as a loan repayable in 30 equal instalments. During the year ended
31.03.2019, Ramesh paid 4 instalments to Tamil Nadu Industrial
Development Corporation and 3 instalments to Indian Bank.
Ramesh claimed the entire interest of ` 45,00,000 as an expenditure
while computing the income from fertilizer business. Discuss whether his
claim is valid and if not what amount of interest, if any, allowable, while
computing the business income. (6 marks)
Answer:
Deduction under section 43B
According to Section 43B, any interest payable on the term loan to specified
financial institutions and any interest payable on any loans and advance to
scheduled banks shall be allowed only in the year of payment of such
interest. If any sum payable by an assessee as interest on any loan or
borrowing or advance is converted by the bank or financial institution into a
fresh loan or borrowing or advance, the interest so converted and not
'actually paid', shall not be deemed as 'actual payment' and not allowed as
deduction in the computation of income under section 43B.
In the given case of Ramesh, the unpaid interest of ` 15,00,000 due to
Tamil Nadu Industrial Development Corporation (TIDCO) and ` 30,00,000
due to Indian Bank was converted into loan. Such conversion would not
amount to payment of interest and would not, therefore, be eligible for
deduction in the year of such conversion. Hence, claim of Ramesh that the
entire interest of ` 45,00,000 is to be allowed as deduction in the year of
conversion is not tenable. The deduction shall be allowed only to the extent
of repayment made during the financial year. Accordingly, the amount of
interest eligible for deduction while computing the business income for the
F.Y.2018-19 shall be calculated as follows:
[Chapter  7 ] Profits and Gains of Business ... O 7.151

Interest Number of Amount per Instal- Interest


Outstanding Instalments Instalment ments allowable
paid (`)
TIDCO 15 Lakh 30 50,000 4 2,00,000
Indian 30 Lakh 30 1,00,000 3 3,00,000
Bank
Total amount eligible for deduction 5,00,000
Space to write important points for revision

2010 - Dec [8] Answer the following:


(d) Narang Textiles Ltd. purchased a machinery from Germany for Euro
1,00,000 on 3.9.2017 through a term loan from Fortune Bank Ltd. The
exchange rate on the date of acquisition was ` 65. The assessee took
a forward exchange rate on 05.10.2018 when the rate specified in the
contract was ` 67 per Euro. Compute depreciation for the assessment
years 2018-19 and 2019-20. Ignore additional depreciation. (5 marks)
Answer:
Computation of Depreciation u/s 32
Particulars Amount (`)

Cost of the Asset (Euro1,00,000 x `65.00) 65,00,000


Less: Depreciation @ 15% (Put to use for more than 180 (9,75,000)
days)
(` 65,00,000 × 15%) for Financial year 2017-2018
WDV as on 31.03.2018 55,25,000
Add: exchange Rate Difference (` 2 × Euro 1,00,000) 2,00,000
WDV for Claiming depreciation 57,25,000
Less: Depreciation @15%for FY 2018-2019 (8,58,750)
(` 57,25,000×15%)
WDV as on 31.03.2019 48,66,250
Space to write important points for revision
7.152 O Scanner CMA Inter Gr. I Paper 7 (2016 Syllabus)

2011 - June [5] (a) Miss Vivitha, who carried on wholesale cum retail trade
during the year 2018-19, reports a turnover of ` 52,50,000.
WDV of plant as on 01.04.2018 was ` 3,10,000.
On 01.12.2018, she acquired a plant for ` 50,000.
Net profit as per profit and loss account (before depreciation) was
` 4,35,000.
Compute business income both under regular provisions and applicable
presumptive provisions of the Act. State what would be the income
chargeable to tax with the conditions to be complied with in this regard.
(6 marks)
Answer:
Particulars Presumption Regular
Income at 8% on ` 52,50,000 4,20,000
Net profit as per P&L Account 4,35,000
Depreciation on machinery
` 3,10,000 x 15% Not eligible 46,500
` 50,000 x 15% x ½ Not eligible 3,750
Income from business 4,20,000 3,84,750

Note: Assessee has two options as follows-


Case I: If assessee wants to offer income under section 44AD at 8%,
there is no need of audit under section 44AB of the Act.
Case II: If assessee wants to offer income as per regular provisions then
her books of account have to be audited u/s 44AB of the Act, since the
income admitted falls below the presumption limit 8% of the turnover.
Space to write important points for revision

2011 - Dec [2] (a) Answer the following with reference to the provisions of
the Income-tax Act, 1961:
(i) Bad debt claim disallowed in an earlier assessment year, recovered
subsequently. Is the sum recovered, chargeable to tax? (1.5 marks)
(iii) Tax deducted at source on salary paid to employees not remitted till
the 'due date' for filing the return prescribed in Section 139. Is the
expenditure to be disallowed under section 40(a)(ia)? (1.5 marks)
[Chapter  7 ] Profits and Gains of Business ... O 7.153

(iv) X Co. Ltd. paid ` 120 lakhs as compensation as per approved Voluntary
Retirement Scheme (VRS) during the financial year 2018-19.
How much is deductible under section 35DDA for the assessment year 2019-
20? (1.5 marks)
Answer:
(i) As per Section 41(4), Bad Debt, earlier claimed as deduction, if
recovered subsequently, then such recovery shall be taxable as
business income. Recovery of a bad debt claim disallowed in the
earlier year cannot be brought to tax under section 41(4).
(iii) Disallowance of any sum paid to a resident at any time during previous
year without TDS u/s 40 (a)(ia) [Circular No. 10/2013, dated
16.12.2013]
In order to rectify inconsistency and improve TDS compliance in
respect of all payments to residents, disallowance u/s 40 (a)(ia) has
been extended to all expenditure on which tax has not been deducted
or after deduction not deposited on or before due date specified in
Section 139(1). Disallowance is restricted to 30% of sum payable to
resident & not 100% in order to prevent undue hardship on assessee.
Hence, the expenditure is not allowed as deduction.
(iv) It is deductible in 5 equal annual instalments commencing from the
previous year of payment ` 24 lakhs, being 1/5th of ` 120 lakhs, is
deductible under section 35DDA for the A. Y. 2019-20.
Space to write important points for revision

2011 - Dec [2] (d) TUSHAR had 4 heavy goods vehicles as on 1.4.2018. He
acquired 7 heavy goods vehicles on 27.6.2018. He sold 2 heavy goods
vehicles on 31.5.2018.
He has brought forward business loss of ` 50,000 relating to assessment
year 2015-16 of a discontinued business. Assuming that he opts for
presumptive taxation of income as per Section 44AE, compute his total
income chargeable to tax for the assessment year 2019-20. (3 marks)
7.154 O Scanner CMA Inter Gr. I Paper 7 (2016 Syllabus)

Answer:
Computation of total income of Mr. TUSHAR for A. Y. 2019-20 `
Presumptive business income under section 44AE
4 heavy goods vehicles for 2 months (4 × ` 12,000 × 2) 96,000
Balance 2 heavy goods vehicles for 10 months (2 × ` 12,000 × 10) 2,40,000
7 heavy goods vehicles for 10 months (7 × ` 12,000 × 10) 8,40,000
Business Income 11,76,000
Less: Brought forward business loss of discontinued business 50,000
Total Income 11,26,000
Note: The assessee is eligible for computing the income from goods
carriages applying the presumptive provisions of Section 44AE, since he
did not own more than 10 goods carriages at any time during the previous
year.
Amendment to [Section-44AE]
Section 44AE(2)has been substituted (with effect from the assessment
year 2019-20) so as to provide that for a heavy goods vehicle, the profits
and gains shall be an amount equal to ` 1,000 per ton of gross vehicle
weight (or unladen weight) for every month (or part of a month) during
which the heavy goods vehicle is owned by the assessee in the previous
year or an amount claimed to have been actually earned form such
vehicle, whichever is higher.
In the case of a goods carriage other than heavy vehicle, the profits and
gains shall be an amount equal to ` 7,500 for every month (or part of a
month) during which the goods carriage is owned by the assessee in the
previous year or an amount claimed to have been actually earned from
such goods carriage, whichever is higher.
For this purpose, “heavy goods vehicle” means any goods carriage the
gross vehicle weight of which exceeds 12,000 kilograms.
Space to write important points for revision
[Chapter  7 ] Profits and Gains of Business ... O 7.155

2011 - Dec [3] (a) Mr. PADAM engaged in retail trade, reports a turnover of
` 58,50,000 for the financial year 2018-19. His income from the said
business as per books of account is computed at ` 2,90,000. Retail trade is
the only source of his income:
(i) Is Mr. PADAM eligible to opt for presumptive determination of his
income chargeable to tax for the assessment year 2019-20?
(ii) If so, determine his income from retail trade as per the applicable
presumptive provision.
(iii) In case he does not opt for presumptive taxation of income from retail
trade, what are his obligations under the Income-tax Act, 1961?
(iv) What is the due date for filing his return of income under both the
options? (5 marks)
Answer:
(i) Yes. Since his total turnover for the F. Y. 2018-19 is below ` 2 crore,
he is eligible to opt for presumptive taxation scheme under section
44AD in respect of his retail trade business.
(ii) His income from retail trade, applying the presumptive tax provisions
under section 44AD, would be ` 4,68,000, being 8% of ` 58,50,000.
(iii) In case he does not opt for the presumptive taxation scheme under
section 44AD, and claims that his income is ` 2,90,000 (which is
lower than the presumptive business income of ` 4,68,000), he has to
maintain books of account as required under section 44AA(2) and
also get them audited and furnish a report of such audit under section
44AB, since his total income exceeds the basic exemption limit of
` 2,50,000.
(iv) In case he opts for the presumptive taxation scheme under section
44AD, the due date would be 31st July, 2019.
In case he does not opt for the presumptive taxation scheme and
claims that his income is ` 2,90,000 as per books of account, get his
books audited u/s 44AB then due date is 30th September, 2019.
Space to write important points for revision
7.156 O Scanner CMA Inter Gr. I Paper 7 (2016 Syllabus)

2011 - Dec [3] (c) CHERRY Limited commenced the business of operating
a three star hotel in Tirupati on 1.4.2018.
It furnishes you the following information:
`
(i) Cost of land (acquired in June 2014) 60 lakhs
(ii) Cost of construction of hotel building
Financial year 2016-17 30 lakhs
Financial year 2017-18 150 lakhs
(iii) Plant and Machineries (all new) acquired during financial
year 2017-18 30 lakhs
[All the above expenditures were capitalized in the books of the
company]
Net profit before depreciation for the financial year 2018-19 80 lakhs
Determine the amount eligible for deduction under section 35AD of the
Income-tax Act, 1961, for the assessment year 2019-20. (5 marks)
Answer:
Under Section 35AD, 100% of the capital expenditure incurred during the
previous year, wholly and exclusively for the specified business, which
includes the business of building and operating a hotel of two star or above
category anywhere in India, would be allowed as deduction from the
business income. However, expenditure incurred on acquisition of any land,
goodwill or financial instrument would not be eligible for deduction.
Further, the expenditure incurred, wholly and exclusively, for the purpose
of specified business prior to commencement of operation would be allowed
as deduction during the previous year in which the assessee commences
operation of his specified business. A condition has been inserted that such
amount incurred prior to commencement should be capitalized in the books
of account of the assessee on the date of commencement of its operations.
Accordingly, the deduction under section 35AD for the A. Y. 2019-20
in the case of Cherry Ltd. would be calculated as follows, assuming that the
expenditures were capitalized in the books of the company on 1.4.2018,
being the date of commencement of operations —
[Chapter  7 ] Profits and Gains of Business ... O 7.157

Particulars ` (in lakhs)


Cost of land (not eligible for deduction under section 35AD) Nil
Cost of construction of hotel building (` 30 Iakhs + ` 150 lakhs) 180
Cost of plant and machinery 30
Deduction under section 35AD 210

Note: For A. Y. 2019-20, the loss from specified business of operating a


three star hotel would be ` 130 lakhs (i.e. ` 210 lakhs - 80 lakhs). As per
Section 73A, any loss computed in respect of the specified business
referred to in Section 35AD shall be set off only against profits and gains,
if any, of any other specified business. The unabsorbed loss, if any, will be
carried forward for set off against profits and gains of any specified
business in the following assessment year.
Space to write important points for revision

2011 - Dec [4] (c) During the financial year 2018-19, the following payments/
expenditures were made/incurred by Mr. ATUL GUPTA, a resident individual
(whose turnover during the year ended 31.3.2019 was ` 39 lakhs).
(i) Interest of ` 12,000 was paid to P C T & Co., a resident partnership
firm, without deduction of tax at source;
(ii) Interest of ` 4,000 was paid as interest to Mr. PULKIT a non-resident,
without deduction of tax at source;
(iii) ` 3,00,000 was paid as salary to a resident individual without
deduction of tax at source;
(iv) He had sold goods worth ` 5 lakhs to Mr. SANCHIT. He gave Mr.
SANCHIT a cash discount of ` 12,000 later. Commission of ` 15,000
was paid to Mr. Vidyasagar on 2.7.2018. In none of these transactions,
tax was deducted at source.
Briefly discuss whether any disallowance arises under the provisions of
Section 40(a)(i)/40(a)(ia) of the Income-tax Act, 1961. (5 marks)
Answer:
Disallowance under section 40(a)(i)/40(a)(ia) of the Income-tax Act, 1961
is attracted where the assessee fails to deduct tax at source as is required
under the Act, or having deducted tax at source, fails to remit the same to
the credit of the Central Government within the stipulated time limit.
7.158 O Scanner CMA Inter Gr. I Paper 7 (2016 Syllabus)

The assessee is a resident individual, who was not subject to tax audit during
the immediately preceding previous year i.e., P. Y. 2018-19 (as his turnover
is less than ` 100 lakh in that year) and the TDS obligations have to be
considered bearing this in mind.
(i) The obligation to deduct tax source from interest paid to a resident
arises under section 194A in the case of an individual, only where he
was subject to tax audit under section 44AB in the immediately
preceding previous year, i.e.. P. Y. 2018-19. From the data given. it is
clear that he was not subject to tax audit under- Section 44AB in the
P. Y. 2018-19. Hence, disallowance under section 40(a)(ia) is not
attracted in this case.
(ii) In the case of interest paid to a non-resident, there is obligation to
deduct tax at source under section 195, hence non-deduction of tax
at source will attract disallowance under section 40(a)(i).
(iii) Disallowance under section 40(a)(ia) is not attracted for failure to
deduct tax at source under section 192 from salaries.
(iv) The obligation to deduct tax at source under section 194-H from
commission paid in excess of ` 5,000 to a resident arises in the case
of an individual, only where he was subject to tax audit under section
44AB in the immediately preceding previous year. From the data
given, it is clear that he was not subject to tax audit under section
44AB in the P. Y. 2018-19 Hence, there is no obligation to deduct tax
at source under section 194H during the P. Y. 2019-20. Therefore,
disallowance under section 40(a)(ia) is not attracted in this case.
Space to write important points for revision

2012 - June [3] (b) Mr. Rajan doing business, donated ` 50,000 to Madras
University on 10.01.2019, to be used for scientific research programme
approved by Principal Scientific Advisor to the Government of India.
(i) How much of such donation is deductible in computing business
income of Mr. Rajan for the assessment year 2019-20?
(ii) If Mr. Rajan was an employee having only salary income, what would
be your answer? (3 marks)
[Chapter  7 ] Profits and Gains of Business ... O 7.159

Answer:
(i) As per Section 35(2AA), where an assessee pays any sum to a
National Laboratory or a University or Indian Institute of Technology
or a specified person with a specific direction that the sum shall be
used for scientific research under a programme approved by the
prescribed authority then he shall be allowed a deduction of 150% the
sum so paid.
As per said section, Mr. Rajan is eligible for a deduction of ` 75,000
(i.e. 150% of 50,000, donation given to the University).
(ii) If Mr. Rajan was an employee only, then answer would be
different as under-
If Mr. Rajan is a salaried person only then it is deductible under
section 80 GGA if the donation is meant to be used for scientific
research by the university. It is deductible at 100% of the amount of
donation.
Hence, the deduction in such case would be equal to the amount of
donation i.e. ` 50,000. Any payment made exceeding ` 10,000 shall
only be allowed if such sum is paid in any mode other than cash.
Space to write important points for revision

2012 - June [5] (a) Vaibhav Solvents Ltd. is an existing Indian Company
which sets up a new industrial unit. It incurs the following expenditure in
connection with the setting up of a new industrial unit:
`
Preparation of project report 4,00,000
Feasibility report expenses 6,00,000
Expenses for raising additional capital required for the new unit 3,00,000
The following additional data are given:
Factory construction cost 12,00,000
Cost of project 40,00,000
Capital employed in the new unit 30,00,000
Compute the deduction admissible to the company under section 35D of the
Income-tax Act, 1961 for the assessment year 2019-20. (5 marks)
7.160 O Scanner CMA Inter Gr. I Paper 7 (2016 Syllabus)

Answer:
As per Section 35D, the deduction is admissible one-fifth of the expenditure
incurred for the project.
Hence, admissible deduction ` 2,60,000 [ i.e. one-fifth of (` 4,00,000
+ ` 6,00,000 + ` 3,00,000)]: factory construction cost is to be ignored here]
However, such expenditure should not exceed the following limits as
prescribed in sub-Section (3):
(a) 5% of cost of the project or
(b) 5% of the capital employed in the new industrial undertaking (being a
company) — Whichever is higher.
In this case
(a) 5% of the project cost is ` 2,00,000 and
(b) 5% of the capital employed is ` 1,50,000.
Hence, the expenditure eligible for amortization under section 35D would
be ` 2,00,000.
And the admissible deduction for the current assessment year is ` 2,00,000
× 1/5 = ` 40,000.
Space to write important points for revision

2012 - Dec [4] (a) Mr. Gopi carrying on business as proprietor converted the
same into a limited company by name Gopi Pipes (P) Ltd. from 01.07.2018.
The details of the assets are given below:
Block—I WDV of plant & machinery (rate of depreciation @15%) ` 12,00,000
Block—II WDV of building (rate of depreciation @10%) ` 25,00,000
The company Gopi Pipes (P) Ltd. acquired plant and machinery in December
2018 for ` 10,00,000. It has been doing the business from 01.07.2019.
Compute the quantum of depreciation to be claimed by Mr. Gopi and
successor Gopi Pipes (P) Ltd. for the assessment year 2019-20.
Note : Ignore additional depreciation. (5 marks)
Answer:
Computation of depreciation for the Assessment Year 2019-20 u/s 32
Block-I WDV of plant & machinery @ 15% on ` 12,00,000 1,80,000
Block-II WDV of building @ 10% on ` 25,00,000 2,50,000
Total 4,30,000
[Chapter  7 ] Profits and Gains of Business ... O 7.161

Depreciation for Gopi (individual) for 91 days


= 4,30,000 × 91/365 = 1,07,205
Depreciation for Gopi Pipes (P) Ltd. for 274 days
= 4,30,000 × 274/365 = 3,22,795
Additions during the year ` 10,00,000 × 15% × ½ = 75,000
Depreciation claim for the company 3,97,795
The asset acquired during the year is used for less than 180 days,
hence 50% of the depreciation will be allowed.
Space to write important points for revision

2012 - Dec [7] (a) ‘A’ Ltd. owns two plant : Plant X and Plant Y, as on April
1, 2018 (rate of depreciation: 15% depreciated value on 01.04.2018:
` 2,37,000).
The company purchases Plant Z on May 31, 2018 for ` 20,000.
It sells Plant X (on April 10, 2018) and Plant Y (on December 11, 2018) and
Plant Z (on March 1, 2019) for ` 40,000; ` 36,000 and ` 24,000 respectively.
Find out the WDV of the block of assets on 31.03.2019. Explain the
treatment relating to depreciation and allied issues relating to the above.
What is the opening WDV of the block as on 01.04.2019? (5 marks)
Answer:
Written down value of the block of assets will be determined as below:
Amount `

Opening WDV of the block consisting of Plants X & Y 2,37,000

Add: Cost of Plant Z (Because Put to use > 180 days) 20,000

Total 2,57,000

Less: Sale of Plants X, Y and Z 1,00,000

Short Term Capital Loss 1,57,000


7.162 O Scanner CMA Inter Gr. I Paper 7 (2016 Syllabus)

Note:
(i) No depreciation is allowable as the block of assets ceases to exist
on the last day of the previous year, since all plants are sold.
(ii) As per Section 50(2) ` 1,57,000 will be treated as short-term capital
loss on sale of the assets,
(iii) On 1st of April, 2018 the WDV will be taken as ‘NIL’.
Space to write important points for revision

2013 - June [7] (c) Please advise regarding admissibility of the following
items of expenditure:
(i) Payment of interest of ` 40,000 on monies borrowed from bank for
payment of dividends to share holders.
(ii) ` 12,000 has been expended for shifting of business from the original
site to another place which is more advantageously located.
(iii) Lump sum paid to acquire a licence regarding technical information to
reduce production cost.
(iv) Expenses for registration of trademarks.
(v) Theft of stock-in-trade assuming
(a) it is insured (b) it was uninsured (10 marks)
Answer:
(i) Loan utilized for payment of dividend. This is allowable u/s. 36(1)(iii).
(ii) Shifting expenses of business premises results in an expenditure of
enduring benefit. It is a capital expenditure and hence it is not
allowable.
(iii) Payment made to acquire license regarding technical information is a
capital expenditure. Only depreciation is allowed on such cost u/s 32.
(iv) Expenditure incurred for registration of trademark is a revenue
expenditure. It is allowable u/s 38(1).
(v) Loss of stock in trade due to theft is allowed as incidental to business.
However if it is insured, insurance compensation received will be a
trading receipt.
Space to write important points for revision
[Chapter  7 ] Profits and Gains of Business ... O 7.163

2013 - Dec [1] {C} (a) Answer the following sub-divisions briefly in the light
of the provisions of the Income-tax Act, 1961:
(i) Interest on bank term loan paid during financial year 2018-19 was
` 1,21,000. Outstanding as on 31.03.2019 was ` 28,000. The
assessee paid ` 15,000 before the “due date” for filing the return of
income under section 139(1). Is the amount paid after the end of the
year be eligible for deduction?
(ii) A nationalized bank gave interest reduction of ` 1,70,000 in a cash
credit account of a trader relating to earlier years. Is the interest
reduction chargeable to income-tax? Said interest had not been paid
to the bank.
(iii) A wind mill was installed in June, 2018 for ` 200 lakhs. What is the
rate of normal depreciation applicable on such wind mill?
(vi) A company owned chain of star hotels in India. It has been availing
deduction under section 35AD. In December, 2018, it transferred the
operation of hotels (all above two star category) to another group
company. Is it eligible to avail the benefit of Section 35AD even after
the transfer of operating the hotels? (1 x 4 = 4 marks)
Answer:
(i) The amount of interest paid before the due date for filing the return is
eligible for deduction under section 43B. Thus the amount paid after
the end of the account year but before the due date for filing the return
of income is deductible.
(ii) Since the interest payment would not have been allowed earlier in
view of Section 43B, the waiver is also not chargeable to tax by
invoking Section 41 (1) of the Act.
(iii) CBDT has amended the Income Tax Rules to restrict the depreciation
allowed on windmills installed on or after 1 April 2012 to 15%.
(Notification No. 15/2012).
(iv) As per Section 35AD (6A) transfer of operation of hotels will not deny
the assessee from availing or continuing to avail the benefits of
Section 35AD.
Space to write important points for revision
7.164 O Scanner CMA Inter Gr. I Paper 7 (2016 Syllabus)

2013 - Dec [3] (b) State, with reasons, the deductibility or taxability of the
following items in computation of income under the head “Profits and gains
of business or profession”:
(i) Profit of ` 10 lacs on sale of import entitlement.
(ii) A sum of ` 15 lacs was spent for acquiring one equipment used for in-
house scientific research project which was approved by the
prescribed authority.
(iii) Share of profit of ` 12 lacs as partner of a partnership firm.
(iv) Expenditure on purchase of raw materials amounting to ` 5 lacs from
a concern owned by son of the Managing Director of the assessee
company.
(v) Interest of ` 10 lacs on loan taken from a bank for acquiring a machine
in connection with expansion project of the assessee. Loan was taken
on 1st April, 2018 and the machine was put to use on 1st July, 2018.
(8 marks)
Answer:
(i) Profit on sale of import entitlement is chargeable to tax under the
head “Profits or gains from business or profession” as per charging
Section 28.
(ii) The assessee is entitled to claim a weighted deduction at 150% of
the amount of capital expenditure on in-house scientific research
programme approved by the prescribed authority [Section 35
(2AB)]: Amount of Deduction = 15 lacs × 150% = 22.5 Lacs.
(iii) Partner’s share in profits of firm is specifically exempted under
section 10 (2A) in the hands of the partner.
(iv) Such expenditure is allowed unless it is found that the payment is
excessive or unreasonable having regard to the fair market value of
raw materials received. In such case the excess amount, if any, shall
be disallowed under section 40A (2).
(v) Interest on loan for the period from 1st April 2018 to 1st July 2018 i.e.
` 2.50 lacs (` 10 lacs X 3 /12) shall not be allowed under section 36
(1) (iii). The balance interest i.e. ` 7.50 lacs can be claimed under
section 36 (1) (iii). Interest to the extent of ` 2.50 lacs shall be
added to the cost of machine and depreciation can be claimed.
Space to write important points for revision
[Chapter  7 ] Profits and Gains of Business ... O 7.165

2013 - Dec [6] (d) Zoom Ltd. acquired a machinery from Japan on
17.08.2017 for $ 2,50,000.
The eligible rate of depreciation is 15% and it is used regularly from
10.09.2017. The exchange rate at the time of acquisition was ` 50 per dollar
and the company paid $ 1,50,000.
The balance payment to the supplier i.e. $ 1,00,000 was paid in September,
2018 when the exchange rate was ` 54 per dollar.
Compute depreciation for the assessment year 2019-20. Ignore additional
depreciation. (4 marks)
Answer:
Exchange fluctuation and depreciation
Particulars `
17.08.2017: Cost of acquisition $ 2,50,000 x ` 50 1,25,00,000
Less: Depreciation for the financial year 2017 - 18 @ 15% 18,75,000
WDV as on 01-04-2018 1,06,25,000
Add: Exchange fluctuation in September 2018
$ 1,00,000 x ` 4 each 4,00,000
Block value 1,10,25,000
Depreciation @ 15% for the financial year 2018 - 19 16,53,750
Space to write important points for revision

2014 - Dec [5] (b) State, with brief reason, whether disallowance is attracted
under any provision of the Income Tax Act, 1961 in the following cases, while
computing income under the head ‘Profits and gains of business or
profession’:
(i) Salary of ` 3,00,000 to each working partner by a firm without
deduction of tax at source.
(ii) Interest to a nationalized bank on term loan ` 72,000 of which the
amount actually paid during the year was ` 40,000 and ` 15,000
was paid before the ‘due date’ for filing the return of income.
7.166 O Scanner CMA Inter Gr. I Paper 7 (2016 Syllabus)

(iii) Demerger expenses of ` 7,00,000 wholly debited to profit and loss


account.
(iv) Expenditure incurred towards issue of bonus shares ` 2,00,000.
(4 marks)
Answer:
(i) Salary paid to each working partner by a firm ` 3,00,000 without
deduction of tax at source will not attract disallowance under
section 40a(ia).
However, based on book profit of the firm and subject to the limits
in Section 40(b), disallowance may apply.
(ii) Interest paid to a nationalized bank on term loan during the year
` 40,000 and the amount paid before the ‘due date’ for filing the
return ` 15,000 are eligible for deduction as there will be no
disallowance u/s 43B.
The balance of ` 17,000 will be disallowed under section 43B.
(iii) Demerger expenses applicable for companies contained in Section
35DD of ` 7,00,000 is deductible in five equal annual installments.
Therefore, ` 1,40,000 is allowable and ` 5,60,000 would be
disallowed.
(iv) Expenditure incurred towards issue of bonus shares ` 2,00,000 is
deductible and would not attract any disallowance. Issue of bonus
shares would not bring any fresh inflow of funds or increase in
capital employed, hence, the expenditure is fully deductible. CIT vs.
General Insurance Corporation (2006) 286 ITR 232 (SC).
Space to write important points for revision

2015 - June [3] (a) Sarath Ltd., engaged in manufacturing activity, acquired
new plant for ` 30 crores in August 2018 which was put to use from
15.09.2018 Compute depreciation, additional depreciation under section
32(1)(iia) and investment allowance under section 32AC of the Income-tax
Act, 1961 for the plant. Quantify the written down value of asset as it would
be on 31.03.2019. (4 marks)
[Chapter  7 ] Profits and Gains of Business ... O 7.167

(b) Vivitha Pipes Ltd., set up a new unit for extension of its manufacturing
activity. It incurred ` 45 lakhs towards preliminary expenses. The cost of the
project is ` 600 lakhs and the amount of capital employed is ` 700 lakhs.
Determine the amount eligible for amortization under section 35D and the
period of amortization. (6 marks)
Answer:
(a)
Particulars ` in lakhs
Normal depreciation at 15% on ` 30 crores A 450
Additional depreciation at 20% on ` 30 crores B 600
Investment allowance under section 32AC at 15% on C
` 30 crores 450
Closing WDV = `30 crores – Depreciation – Additional
depreciation [i.e. ` 30 crores - A – B] = ` (30 crores –
4.50 crores – 6 crores) 1950

Note: Investment allowance under section 32AC will not reduce the block
value of assets.

Answer:
(b) Amortization of preliminary expenses:
Preliminary expenses are eligible for amortization in 5 equal annual
installments.
In the case of company, 5% of the cost of the project or the capital
employed shall be considered for computing the amount eligible for
amortization.
5% of the cost of the project = `600 lakhs × 5% = `30 lakhs
5% of the capital employed = `700 lakhs × 5% = `35 lakhs
Actual preliminary expenditure incurred = `45 lakhs.
The assessee hence is eligible to amortise ` 35 lakhs under section
35D.
Amount eligible for amortization for 5 years = ` 7 lakhs for each year.
Space to write important points for revision
7.168 O Scanner CMA Inter Gr. I Paper 7 (2016 Syllabus)

2015 - June [6] (a) Vimala Pharma Ltd. informs that it has net profit of ` 60
lakhs for the year ended 31st March, 2019. It gives you the following further
information:
(i) Depreciation as per books ` 3,50,000.
(ii) Bad debts written off in the books ` 5,00,000, which includes ` 1 lakh
due from one customer who has disputed the liability to pay but
continues to have business relationship with the company.
(iii) Proposed dividend debited to Profit and Loss Account ` 6 lakhs.
(iv) One machinery which has became useless has been written off in
P & L Account, the amount debited being ` 90,000.
(v) Provident Fund collections from employees for the year ` 1,50,000 and
company’s own contribution of ` 1,10,000 for the year have not been
remitted. These amounts are shown as Sundry Liability in the books.
Assume it will be remitted after 31st December, 2019.
(vi) Income from agricultural lands surrounding the factory ` 50,000
credited to Profit and Loss Account.
(vii) Bank term loan for purchase of machinery waived ` 2 lakhs is credited
to capital reserve account.
(viii) The opening WDV of plant and machinery was ` 15,90,000. One
machinery for ` 4,10,000 was acquired on 01.06.2018 and was put to
use immediately.
(ix) Provisions for taxation debited in the Profit and Loss Account amounts
to ` 15 lakhs.
You are requested to compute the income of the company under the head
‘Profits and gains of business or profession’ for the assessment year 2019-
20. (9 marks)
Answer:
Computation of business income of Vimala Pharma Ltd. for the
assessment year 2019-20
Particulars ` `
Net Profit as per Profit and Loss account 60,00,000
Add:
Depreciation debited to P&L A/c 3,50,000
[Chapter  7 ] Profits and Gains of Business ... O 7.169

Bad debts write off is deductible if it is written off in the books.


Even if the customer has relationship with the assessee the
write off is deductible without any attendant conditions. TRF
Ltd. vs. CIT 323 ITR 397 Nil
Proposed dividend debited to P&L Account 6,00,000
Machinery discarded debited to P&L Account – disallowed 90,000
Provident fund collections and deductions out of
salary not remitted to be treated as income 2,60,000
Provision for taxation debited to P & L Account 15,00,000
88,00,000
Less:
Agricultural income credited to P&L Account 50,000
Bank term loan waiver is not liable to tax as Section
41(1) will not apply Nil
Depreciation on plant and machinery
Opening WDV 15,90,000
Add: New additions 4,10,000
20,00,000
Depreciation @ 15% on ` 20 lakhs 3,00,000
3,50,000
Income from Business 84,50,000
Space to write important points for revision

2015 - Dec [1] {C} Answer the following questions with brief reas-
ons/workings:
(g) The Statement of Profit and Loss of KCL Limited is debited by an
amount of ` 1,20,000 in respect of an advertisement of company’s
product in a newspaper owned by a political party. Is such expense
allowable in computation of income from business? (2 marks)
7.170 O Scanner CMA Inter Gr. I Paper 7 (2016 Syllabus)

(i) The Statement of Profit and Loss of a company includes interest of


` 5,00,000 on a loan taken for financing its expansion scheme. The
machineries purchased with the borrowed amount were in transit at the
end of the year. Is such interest allowable as deduction in computation
of the company’s business income? (2 marks)
Answer:
(g) Advertisement expenses is allowed to be deducted u/s 37(1); provided
it is not of a capital nature and it is incurred wholly and exclusively for the
purpose of business.
Hence, the amount of ` 1,20,000 is allowable as deduction u/s 37(1).
(i) Section 36(1)(ii) provided that any amount of the interest paid in respect
of an asset for extension of existing business or profession (whether
capitalized in the books of accounts or not) for any period beginning from
the date on which the capital was borrowed for acquisition of the asset
till the date on which such asset was first put to use, shall not be allowed
as deduction.
So, the amount of interest of ` 5,00,000 is not allowed as deduction
because it is the interest on borrowing of machine for expansion
scheme.
Space to write important points for revision

2015 - Dec [4] (b) The Statement of Profit and Loss of XYZ Limited for the
previous year 2018-19 shows a net profit of ` 8,50,390 after
debiting/crediting the following items:
(i) Purchase of goods for ` 42,000 (market value ` 35,000) from one of
the directors of the company.
(ii) Interest of ` 1,00,000 paid on loan taken from Mr. Ron of USA without
deducting tax at source.
(iii) Advance of ` 90,000 paid in earlier year for purchase of machinery
written off.
(iv) Income tax on perquisites of employees paid by the company
` 20,000.
[Chapter  7 ] Profits and Gains of Business ... O 7.171

(v) Recovery of bad debt of ` 30,000 which was disallowed in previous


assessment of the company.
Compute income of XYZ Limited under the head “profits and gains of
business or profession” for Assessment Year 2019-20 indicating
reasons for treatment of each item. (6 marks)
Answer:
Calculation of profit and gain of XYZ Limited. Amount
Net profit as per books 8,50,390
Add:
- Excess amount paid to director 7,000
- Interest paid to Mr. Ron (USA) 1,00,000
- Advance payment for purchase of Machinery 90,000
- Disallowance of tax on perquisites 20,000 2,17,000
10,67,390
Less:
- Recovery of Bed Debts disallowed in earlier year 30,000
Net Profit taxable under head profit from business 10,37,390

Notes:
(i) If the payment to director is excessive or unreasonable having regard
to the fair market value of the material. In such case the excess
amount shall be disallowed under section 40(a)(2).
(ii) In the case of interest paid to non resident, there is a obligation to
deduct tax at source u/s 195, hence non deduction of tax at source
will attract disallowance u/s 40(a)(i).
(iii) Advance paid for purchase of machine is a capital expenditure.
Hence, its not allowed as deduction.
(iv) Income tax on perquisites is allowable expense.
(v) As per Section 41(4), bad debts, earlier claimed as deduction 8
disallowed, if recovered subsequently, then such recovery shall not
be taxable as business gain/income.
Space to write important points for revision
7.172 O Scanner CMA Inter Gr. I Paper 7 (2016 Syllabus)

2016 - June [5] (a) Mr. Sivasankar carrying on trading business could not
recover ` 1,00,000 due from a customer. He deems that the amount is not
recoverable and hence has to be written off as bad debt. State the conditions
that are to be satisfied for allowance of bad debt claim. (3 marks)
(b) Mr. Ghosh established an undertaking in a notified backward area in the
State of West Bengal. He invested ` 150 lakhs in June, 2018 towards
acquisition of plant and machinery and was engaged in manufacturing
activity. The regular rate of depreciation on such machineries is 15%.
Advise the maximum amount of deduction that Mr. Ghosh could avail by
way of depreciation, additional depreciation and deduction under section
32AD of the Income-tax Act, 1961. State the closing WDV of the plant
and machinery after such claim. (4 marks)
Answer:
(a) Bad debt write-off is eligible for deduction in computing income under the
head “Profits and Gains of business or profession” if the stipulated
conditions are satisfied:
(i) The debt should be incidental to the business.
(ii) The debt should have been taken into account in computing the
income of the assessee or it should represent money lent in the
ordinary course of banking or money lending business; and
(iii) It should be written off in the books of account.
If the assess is able to satisfy all the three conditions given above, he
can claim the same as bad debt.
(b) Depreciation, additional depreciation and WDV (` in Lacs)
Particulars `
Normal depreciation u/s 32 at 15% of the actual cost of plant 22.5
and machinery (15% of ` 150 lakhs).
(Assumed that the new machinery has been put of use for more
than 180 days)
Additional depreciation under section 32 (iia) (20% of ` 150 30
lakhs)
Closing WDV of the plant and machinery (150 – 52.5) 97.5
[Chapter  7 ] Profits and Gains of Business ... O 7.173

Besides depreciation and additional depreciation, the assessee can


claim 15% deduction under section 32AD for the investment in new
plant and machinery in setting up an undertaking in notified backward
area in the State of West Bengal. Thus he can claim 15% of the actual
cost of plant and machinery being ` 22.50 lakhs. This is not deductible
while computing the WDV.
However, this amount will not go to reduce the written down value of the
assets.
Space to write important points for revision

2016 - Dec [4] (c) State, with reasons, whether the following items are
allowable as deduction in computation of income of Tyagi Aluminium Limited
under the head ‘profits and gains from business or profession’ or otherwise:
(i) Contribution of ` 1,20,000 to political party on the occasion of its silver
jubilee.
(ii) Interest of ` 80,000 paid to bank on loan taken and utilised for
payment of dividend.
(iii) Fees of ` 75,000 paid to independent director for attending board
meetings without deduction of tax at source under Section 194J.
(iv) Interest of ` 40,000 on bank overdraft utilised for payment of dividend.
(v) Few customers are irregular in payment of dues against sale
proceeds, for which provision for bad and doubtful debts has been
created by debiting Statement of Profit & Loss.
(1 × 5 = 5 marks)
Answer:
(i) U/s 80 GGB: Deduction in respect of contribution given by companies
to Political Parties is allowed as deduction if paid by Account Payee
Cheque.
It is assumed that contribution made by Company to Political Party by
Cheque. Therefore, deduction U/s 80 GGB will be allowed.
(ii) Interest of ` 80,000 paid to Bank on loan taken and utilised for
payment of dividend will be allowed u/s 36 (1) (iii).
7.174 O Scanner CMA Inter Gr. I Paper 7 (2016 Syllabus)

(iii) 30% of ` 75,000 i.e. ` 22,500 will be disallowed due to non deduction
of TDS u/s 194 J (ba) @ 10%.
(iv) Interest of ` 40,000 on Bank Overdraft utilised for payment of dividend
will be allowed u/s 37.
(v) Provision for Bad & Doubtful Debts will not be allowed as deduction.
Space to write important points for revision

2016 - Dec [5] (a) Under Section 43B of the Income-tax Act, certain items
are allowed only on actual payment basis, regardless of the method of
accounting followed by the assessee. Name four such items and the due
date by which they can be paid to claim deduction in the current year itself.
(5 marks)
Answer:
Expenses allowed on actual payment basis u/s 43 B:
(i) Bonus & commission.
(ii) Interest on loan or advances taken from Schedule Banks.
(iii) Any sum payable by the assessee to the Indian Railways for the use
of Railway Assets.
(iv) Any sum payable by the assessee by way of tax, duty, cess or fee
whatever name called, under any law for time being in force.
(v) Any Interest on any loan or borrowings from public financial institutions
or state financial corporation.
(vi) Any sum payable by an employer in lieu of any leave at the credit to
his employee.
Due date for payment is on or before the due date for furnishing of
Return of Income u/s 139(1).
Space to write important points for revision

2016 - Dec [6] (c) Express Shipping Inc., a foreign company operating its
ships in Indian ports during the previous year 2018-19 had collected the
revenue as follows:
[Chapter  7 ] Profits and Gains of Business ... O 7.175

Freight (including ` 40 lakhs collected in US dollar for the ` 200 lakhs


cargo booked for Pradeep Port from UK)
Demurrages ` 40 lakhs
Handling charges ` 20 lakhs
The expenses of operating its fleet during the year for the Indian ports were
` 110 lakhs which includes an expense of ` 0.40 lakhs paid in cash to an
agency. Compute income of the company under the head ‘Profits and Gains
from business or profession’ for Assessment Year 2019-20. (5 marks)
Answer:
Computation of Income from Business of Express Shipping Inc.
Freight ` 200 lakhs
Demurrages ` 40 lakhs
Handling charges ` 20 lakhs
` 260 lakhs
Deemed Income 7.5% of ` 260 lakhs ` 19.50 lakhs
Expenses are not allowed u/s 44 B
Space to write important points for revision

2017 - June [3] (a) Gopi Industries furnishes you the following details:
Particulars Machinery Computers Furnitures
WDV as on 01.04.2018 20,00,000 6,00,000 2,00,000
Purchased during the 4,00,000 1,00,000 40,000
year and used for more
than 180 days
Purchased and used 1,00,000 2,00,000 20,000
w.e.f. 01.01.2019
Sold a group of assets on 2,00,000 1,00,000 50,000
01.03.2019
Compute depreciation allowable for the assessment year 2019-20. lgnore
additional depreciation. (6 marks)
7.176 O Scanner CMA Inter Gr. I Paper 7 (2016 Syllabus)

Answer:
Computation of Depreciation Allowable
to Gopi Industries
for A.Y. 2019-20
Particulars Machinery Computers Furnitures
WDV as on 01/04/2018 20,00,000 6,00,000 2,00,000
Add: Purchased during the year 4,00,000 1,00,000 40,000
and used for more than 180
days
Purchased and used w.e.f. 1,00,000 2,00,000 20,000
01/01/2019
Total 25,00,000 9,00,000 2,60,000
Less: Sold during the year on 2,00,000 1,00,000 50,000
01/03/2019
Balance 23,00,000 8,00,000 2,10,000
Less: Depreciation See W. Note 3,37,500 2,80,000 20,000
WDV as on 31/03/2019 19,62,500 5,20,000 1,90,000
Working Note:
Computation of Depreciation
Machinery Computers Furnitures
WDV as on 31/03/2019 before
Depreciation 23,00,000 8,00,000 2,10,000
Depreciation Rate 15% 40% 10%
Machinery:
On ` 22,00,000 @ 15% for full
year 3,30,000 — —
On ` 1,00,000 @ 7.5% used for
less than 180 days 7,500 — —
[Chapter  7 ] Profits and Gains of Business ... O 7.177

Computers:
On ` 6,00,000 @ 40% — 2,40,000 —
On ` 2,00,000 @ 20% — 40,000 —
Furnitures:
On ` 1,90,000 @ 10% — — 19,000
On ` 20,000 @ 5% — — 1,000
Total Depreciation 3,37,500 2,80,000 20,000
Space to write important points for revision

2018 - June [5] (a) Ahuja Industries Ltd. engaged in manufacturing activity
and generation of power, gives you the following information for the year
ended 31st March, 2019:
Description Op. WDV Acquisition/ New acquisition Sold during
(01.04.2018) Date used from the year
Plant ` 5,00,000 ` 60,000 01.11.2018 ` 80,000
(01.05.2018) (01.01.2019)
Windmill – ` 60,00,000 01.09.2018 –
(01.06.2018)
Computer ` 3,00,000 ` 90,000 01.11.2018 ` 40,000
(01.10.2018) (Office use) (01.03.2019)
Patent – ` 4,00,000 01.12.2018 –
(01.12.2018)
Compute the depreciation and additional depreciation for the assessment
year 2019-20. The computation must be such that the same is most
beneficial to the assessee. (9 marks)
Answer:
Assessee in the business of generation or generation and distribution of
power, have the option to claim depreciation on:
(i) Straight line method on each assets or
(ii) Written down value method or block of assets.
7.178 O Scanner CMA Inter Gr. I Paper 7 (2016 Syllabus)

Computation of depreciation allowance under section 32 and 32 (1) (IIA)


for the Assessment Year 2019-20
Detail Normal Add:
Depreciation Depreciation
(A) Plant and Machinery:
Opening WDV as on
01/04/2018 5,00,000
Acquire during the year 60,000
Less: Sale during the year 80,000
WDV as on 31/03/2019 4,80,000
Depreciation on Plant and
Machinery used for more
than 180 days (4,80,000 –
60,000)15% 63,000 Nil
Depreciation on Plant and
Machinery used for less
than 180 days (60,000 ×
7.5%) 4,500
Additional depreciation on
new Plant purchased and
used on 01/11/2018
(60,000 × 10%) 6,000
(B) Windmill:
Opening WDV as on
01/04/2018 Nil
Acquire during the year 60,00,000
Sale during the year Nil
WDV as on 31/03/2019 60,00,000
[Chapter  7 ] Profits and Gains of Business ... O 7.179

Depreciation windmill used


for more than 180 days
(60,00,000 × 40%) 24,00,000
Additional Depreciation on new
Plant purchased
(60,00,000 × 20%) 12,00,000`
(C) Computer:
Opening WDV as on
01/04/2018 3,00,000
Acquire during the year 90,000
Sale during the year 40,000
WDV as on 31/03/2019 3,50,000
Depreciation on computer
used for more than 180
days (3,50,000 – 90,000)
40% 1,04,000
Depreciation on computer
used for less than 180
days (90,000 × 20%) 18,000
(D) Patent:
Opening WDV as on Nil
01/04/2018
Acquire during the year 4,00,000
Sale during the year Nil
WDV as on 31/03/2019 4,00,000
Depreciation of Patent
used for less than 180
days @ 12.5% 50,000
Total Depreciation and
additional Depreciation 26,39,500 12,06,000
7.180 O Scanner CMA Inter Gr. I Paper 7 (2016 Syllabus)

Where any assets is acquired by the assessee during the previous year and
is put to use for the purpose of business for a period of less than 180 days
in that previous year, the depreciation allowance in respect of such asset
shall be restricted to 50% of the amount calculated at the precribed
percentage.
Space to write important points for revision

2018 - Dec [5] (a) Mr. Bhushan, engaged in manufacture of chemicals,


furnishes his Manufacturing, Trading and Profit & Loss Account for the year
ended 31st March, 2019 as under:
Particulars ` Particulars `
To Opening stock 3,40,000 By Sales 1,14,00,000
To Purchases 1,00,20,000 By Closing stock 19,00,000
To Manufacturing 10,40,000
Expenses
To Gross Profit 19,00,000
1,33,00,000 1,33,00,000
To Salary 4,30,000 By Gross Profit 19,00,000
To Bonus 80,000 By Discount 25,000
To Bank term loan 90,000 By Agricultural Income 1,50,000
interest
To Factory rent 1,20,000 B y D i v i d e n d f r o m 75,000
Indian Companies
To Office rent 2,10,000
To Administration 3,30,000
Expenses
To Net Profit 8,90,000
21,50,000 21,50,000
[Chapter  7 ] Profits and Gains of Business ... O 7.181

Additional Information:
(i) The total turnover of Mr. Bhushan for the Financial Year 2017-18 was
` 132 lakhs.
(ii) Salary includes ` 1,80,000 paid to his daughter. The excess payment
considering her qualification and experience is ascertained as
` 40,000.
(iii) Factory rent was paid to his brother. Similar portions are let out to
others by him for a rent of ` 96,000 per annum.
(iv) No tax was deducted at source from the office rent paid during the
year.
(v) Purchases include ` 70,000 paid by cash to an agriculturist for
purchase of grains (being raw material).
(vi) Depreciation allowable under section 32 of the Income-tax Act, 1961
amounts to ` 45,000 for assets held as on 01.04.2018. During the
year, a machinery costing ` 5,00,000 was acquired on 01.07.2018 and
was put to use from 15.10.2018.
(vii) Administration expenses include commission paid to a purchase agent
of ` 12,000 for which no tax was deducted at source.
(viii) The following expenses debited above were not paid till 31.03.2019
and up to the ‘due date’ for filling the return specified in section 139(1)
(I) Term loan interest of ` 35,000;
(II) Demurrages to Indian Railways for using their clearing yard
beyond stipulated hours (disputed by the assessee) forming part
of manufacturing expenses ` 30,000.
Compute the income of Mr. Bhushan chargeable under the head “Profits and
gains of business or profession” for the Assessment Year 2019-20:
(10 marks)
7.182 O Scanner CMA Inter Gr. I Paper 7 (2016 Syllabus)

Table Showing Marks of Compulsory Questions

Year 14 14 15 15 16 16 17 17 18 18
J D J D J D J D J D

Descriptive 1 2

Practical 4

Total 1 6
8 CAPITAL GAINS
THIS CHAPTER INCLUDES
 Meaning of Capital Asset  Computation of Capital Gains
 Classification of Capital Assets  Capital Gains Exempt from tax
 Transfer of Capital Assets
Marks of Objective, Short Notes, Distinguish Between, Descriptive & Practical Questions
Legend
Objective Short Notes Distinguish Descriptive Practical

For detailed analysis Login at www.scannerclasses.com


for registration and password see first page of this book.

7.183
7.184 O Scanner CMA Inter Gr. I Paper 7 (2016 Syllabus)

SHORT NOTES
2010 - June [7] Write short note on the following:
(a) Capital gains on buy back of shares (5 marks)
Answer:
Capital gains on buy back of shares [Section 46A]
Any consideration received by a shareholders or a holder of other specified
securities from any company on purchases of its own shares or other
specified securities held by such shareholder or holder of other specified
securities shall be chargeable to tax on the difference between the cost of
acquisition and the value of consideration received by the holder of securities
or by the shareholders, as the case may be, as capital gain. The computation
of capital gain shall be made in accordance with the provisions of Section
48. Such capital gain shall be chargeable in the year which such
shares/securities were purchased by the company for this purpose.
Space to write important points for revision

DESCRIPTIVE QUESTIONS

2008 - Dec [1] {C} (c) Provide very brief answer to the following :
(ii) What is the cost of acquisition of sweat equity shares in the hands of
the employee ? (2 marks)
Answer:
As per Section 49(AA), where the capital gain arises from the transfer of
specified security or sweat equity, which has already been taxed under the
head salary as perquisite, the cost of acquisition of such security or share
shall be the fair market value which has been taken into account for the
purpose of valuation of perquisite.
Space to write important points for revision
[Chapter  8] Capital Gains O 7.185

2008 - Dec [6] (a) State briefly the requisites of a charge income-tax of
capital gains under section 45 (1) of the Income-tax Act, 1961. (3 marks)
Answer:
Any profits or gain arising from the transfer of a capital asset effected in the
previous year, shall be chargeable to income-tax under the head ‘Capital
Gains’ and shall be deemed to be the income of the previous year in which
the transfer took place unless such capital gain is exempt u/s 54, 54B, 54D,
54EC, 54F, 54G, 54GA or 54 GB.
The following are the essential conditions for taxing capital gains:
(a) There must be a capital asset;
(b) The capital asset must be transferred;
(c) There must be profit or gain on such transfer, which will be known as
capital gain;
(d) Such capital gain should not be exempt u/s 54, 54B, 54D, 54EC, 54F,
54G, 54GA or 54 GB.
Space to write important points for revision

2010 - June [1] {C} (b) (i) “Capital Gains arises on sale of Capital Assets”.—
Comment. (2 marks)
Answer:
Capital gain arises on transfer of Capital Assets. The term 'transfer' has been
defined in Section 2 (47) of the I.T. Act, which includes sale apart from other
transactions of transfer. Hence the statement is not correct.
Space to write important points for revision

2010 - June [2] (a) Discuss whether the expenditure incurred by an


assessee to remove an encumbrance be claimed as a deduction under
section 48, while computing the capital gains, in the following cases:
(i) Where the mortgage was created by the assessee himself;
(ii) Where the mortgage was created by the previous owner. (5 marks)
Answer:
Deduction claimable while computing capital gains
Capital gain is computed as
7.186 O Scanner CMA Inter Gr. I Paper 7 (2016 Syllabus)

Full value of the consideration


Less: actual cost of acquisition of the asset,
Less: cost of improvement
Less: expenses incurred on transfer.
Where the capital asset acquired by the assessee is subject to mortgage
created by the previous owner, he acquired absolute interest in that property
only after the discharge of mortgage debt. In such a case, the expenditure
incurred by the assessee to discharge of mortgage debt, created by the
previous owner, to acquire absolute interest in the property is treated as
"cost of acquisition" and is deductible from the full value of consideration
received by the assessee on transfer of the property.
However, where the assessee acquires a property which is unencumbered,
he gets absolute interest in that property on acquisition. Where the assessee
transfer that property, he is liable for capital gains tax on the full value
realized, even if he has himself created an encumbrance on that property.
The assessee is under an obligation to remove that encumbrance for
effectively transferring the property. In other words, the expenditure incurred
by the assessee to remove an encumbrance created by himself on the
property, acquired by him without any encumbrance, is not an allowable
deduction under section 48.
In CIT Vs. Roshanbabu Mohammed Hussein Merchant (2005) (Bom). The
Bombay High Court opined that there is a difference between the obligation
to discharge the mortgage debt prevailing on the capital asset created by the
previous owner and the obligation to discharge mortgage debt created by the
assessee himself.
Space to write important points for revision

2010 - June [3] (c) (ii) Where an urban agricultural land owned by an
individual, continuously used by him for agricultural purposes for a period of
two years prior to the date of transfer, is compulsorily acquired under law and
the compensation is fixed by the State Government, is the resultant capital
gain chargeable to tax? (2 marks)
[Chapter  8] Capital Gains O 7.187

Answer:
The exemption can be claimed u/s 10(37) if the following conditions are
satisfied:
(i) The assessee is an individual or a Hindu Undivided Family (HUF)
(ii) He owns an urban agricultural land situated in urban area mentioned
in Section 2(14) (iii)(a)(b)
(iii) There is transfer of the agricultural land by way of compulsory
acquisition under any law, or
The consideration for transfer is approved or determined by the
Central Government, or Reserve Bank of India (RBI).
(iv) The agricultural land was used by the assessee for agricultural
purposes during 2 years immediately prior to the date of transfer.
As the acquisition can be under any law, even if the compensation is fixed
by the State Government the resultant capital gain of the compulsory
acquisition will be exempt from tax.
Space to write important points for revision

2010 - June [5] (b) Is it correct to say that Section 50C can be invoked only
in situations in which transfer of land or building or both takes place through
a registered deed of conveyance and that the section cannot be invoked
where there is no registered deed of conveyance, like in power of attorney
transactions and agreements to sell? -Discuss. (4 marks)
Answer:
Scope of Section 50C
Special provision for full value of consideration in certain cases.
This statement is not correct.
Till the recent amendment, the scope of Section 50C did not include within
its ambit, transactions which were not registered with stamp duty valuation
authority and executed through an agreement to sell or power of attorney.
Therefore, in order to prevent tax evasion on this account, Section 50C has
been amended by the Finance (No.2) Act, 2009, to provide that where the
consideration received or accruing as a result of transfer of a capital asset,
being land or building or both, is less than the value adopted or assessed or
7.188 O Scanner CMA Inter Gr. I Paper 7 (2016 Syllabus)

assessable by an authority of a State Government for the purpose of


payment of stamp duty in respect of such transfer, the value so adopted or
assessed or assessable shall be deemed to be the full value of the
consideration received or accruing as a result of such transfer for computing
capital gain. The term “assessable’ has been added to cover transfers
executed through an agreement to sell or power of attorney transactions.
Explanation 2 has been inserted after Section 50C(2) to define the term
‘assessable’ to mean the price which the stamp valuation authority would
have, notwithstanding anything to the contrary contained in any other law for
the time being in force, adopted or assessed, if it were referred to such
authority for the purposes of the payment of stamp duty.
As per amendment made by Finance Act, 2018 w.e.f. 1st April, 2018
where the value adopted or assessed or assessable by the stamp valuation
authority does not exceed 105% of the consideration received or accruing as
a result of the transfer, the consideration so received or accruing as a result
of the transfer shall, for the purpose of section 48, be deemed to be the full
value of the consideration.
And where the value adopted or assessed or assessable by the stamp
valuation authority is exceeds 105% of the consideration received or
accruing as a result of the transfer then Stamp Duty Value shall, for the
purpose of section 48, be deemed to the full value of the consideration.
Space to write important points for revision

2010 - Dec [3] (c) What is the income-tax treatment of consequence of


repurchase or buy back of shares or specified securities by a company?
(5 marks)
Answer:
As per Section 46A of the Income-tax Act, 1961, any consideration
received by a holder of shares or other specified securities from any
company under a scheme of buy back shall constitute transfer and the
difference between such consideration and the cost/indexed cost of
acquisition shall be chargeable to tax as capital gains in the previous year in
which such buy back takes place. Payment made by a company on buy back
doesn’t constitute dividend u/s 2(22)(d).
[Chapter  8] Capital Gains O 7.189

In case the shares are treated as stock in trade and are exchanged for
shares of any other company, then difference between market value of
shares exchanged and book value of original shares is considered as
business profits.[Orient Trading Co. Ltd.(SC)]
Space to write important points for revision

2010 - Dec [8] Answer the following:


(a) Discuss the tax issues including cost of acquisition and period of holding,
determined in the hands of the shareholder determined after demerger,
covering deemed dividend and capital gains. (5 marks)
Answer:
Cost of acquisition and period of holding in the hands of the share-
holder determined after demerger- A shareholder of de-merged company
gets some shares of resulting company after de-merger. Such transfer is not
treated as dividend distributed by the de-merged company [Section 2(22)(v)
of Income Tax Act].
Issue of share of resulting company to shareholders of de-merged
company will not be treated as “transfer” and hence will not attract any
capital gains.[Section 47(vid) of Income Tax Act]
The shareholders will have shares of original i.e. company which has
demerged. If, he sales such shares, capital gain may arise. In such case, the
cost of acquisition of original shares will be bifurcated between the resulting
company and the original company in proportion to net book value of assets
transferred in de-merger to the net worth of original company before de-
merger [Section 49(2C) and 45(2D) of Income Tax Act]. E.g. assume that
net worth of original i.e. demerged company was ` one hundred lakhs and
net book value of assets transferred to resulting company was ` 25 lakhs. If
the cost of acquisition of original shares at the hands of shareholder was
` 1,000 cost of acquisition of resulting company will be ` 250 and cost of
acquisition of original shares will be ` 75. Thus, total cost of acquisition of
original shares is bifurcated between resulting company and original
company in ratio of books value of assets transferred to the net worth of
original company.
7.190 O Scanner CMA Inter Gr. I Paper 7 (2016 Syllabus)

Capital gains will arise only when the shares of the resulting company is
sold by the shareholder. If the shareholder sells the shares of resulting
company, the period for which original shares were held will also be
considered to determine whether the gain is a short term or a long term gain.
[Section 2(42A)(g)]
Space to write important points for revision

2012 - June [6] (a) What are the circumstances the Assessing Officer may
refer the valuation of the capital asset to the Valuation Officer under section
55A of the Income-tax Act, 1961? (5 marks)
Answer:
Reference to Valuation Officer [Section 55A]
Under the following circumstances the Assessing Officer may refer the
valuation of the capital asset to the Valuation Officer and his valuation report
shall be binding on the Assessing Officer —
1. Where the value of the asset is estimated by the registered valuer but
the Assessing Officer is of the opinion that the value so determined is
at variance from its fair market value.
2. In any other case, the Assessing Officer is of the opinion that
(i) The fair market value of the asset exceeds the value of the assets
declared by the assessee either by more than 15% or by ` 25,000
(Rule 111AA); or
(ii) The nature of the asset and other relevant circumstances are such
that, it is necessary to do so.
Space to write important points for revision

2012 - Dec [8] (c) Explain the provisions in regard to a reference to Valuation
Officer for valuation of a capital asset under the provision of the Income-tax
Act, 1961. (3 marks)
Answer:
Please refer 2012 - June [6] (a) on page no. 190
Space to write important points for revision
[Chapter  8] Capital Gains O 7.191

2013 - June [2] (c) Write a note on how interest received by an assessee on
delayed compensation or enhanced compensation is taxed. (5 marks)
Answer:
As per Section 145A (b) irrespective of method of accounting followed by
the assessee interest received on compensation or on enhanced
compensation shall be deemed to be the income of the year in which it is
received.
As per Section 56(2) (viii) income by way of interest on compensation or
enhanced compensation shall be chargeable to income tax under the head
“income from other sources”.
Under Section 57 (iv) in computation of above Income of the assessee, a
deduction for a sum equal to 50% of such income shall be allowed to the
assessee.
Space to write important points for revision

2014 - June [6] (b) State the provisions relating to claiming of exemption in
order to reduce tax liability on short-term capital gains. (5 marks)
Answer:
Provisions relating to claiming of exemption in order to reduce tax liability on
short term capital gains
Applicable for Exemption
U/S
(i) Transfer of Agricultural land 54B
(ii) Transfer by way of compulsory acquisition by Government 54D
(iii) Shifting of Industrial undertaking from Urban to Rural area 54G
(iv) Compulsory acquisition of Agricultural land by Central
Government / RBI 10(37)
(v) Transfer by companies engaged in power sector business 10(41)
Space to write important points for revision
7.192 O Scanner CMA Inter Gr. I Paper 7 (2016 Syllabus)

2014 - Dec [3] (b) State the provisions of the Income Tax Act, 1961 relating
to interest on compensation or enhanced compensation on compulsory
acquisition of property. (4 marks)
Answer:
As per Section 145A (b), interest received by an assessee on compensation
or enhanced compensation, as the case may be, shall be deemed to be the
income of the year in which it is received irrespective of the method of
accounting followed by the assessee. Such interest is taxable under the
head “income from other sources” as per the provision of Section 56.
In respect of such interest, the assessee shall be entitled under section 57
to claim deduction of a sum equal to 50% of such interest. No other
deduction shall be allowed under any other provision of the Income-tax Act.
Key Points:
(i) Regardless of the method of accounting;
(ii) Interest on compensation/enhanced compensation is taxed in the year
of receipt;
(iii) It is taxed as income from other sources;
(iv) 50% of such interest is allowable as expenditure.
Space to write important points for revision

2015 - June [2] (a) Can Mr. Ajit who has long-term capital gain from sale of
vacant site in India buy a residential house outside India to claim exemption
under section 54F? Assume that he has no residential property in India.
(3 marks)
Answer:
Exemption u/s 54F:
• When the assessee has no residential house property or has not more
than one residential house property, he is eligible to claim exemption
under section 54F.
• When a long-term capital asset other than a residential house is
transferred, the exemption by way of investment in residential house
could be obtained by deploying the net consideration.
[Chapter  8] Capital Gains O 7.193

• The exemption is subject to the condition that the assessee has within
a period of one year before or two years after the date of transfer of
long-term capital asset acquires a residential house or three years after
transfer constructs a new residential house in India for the purpose of
availing the exemption under section 54F.
• A person transferring a long-term capital asset in India cannot acquire
a residential house outside India and be eligible for exemption under
section 54F.
• Thus, the person cannot claim exemption in respect of long-term capital
gain on sale of vacant site in India by acquiring a residential house
outside India.
Space to write important points for revision

2015 - June [3] (c) When does advance money received for transfer of
capital asset become chargeable to tax? Under what head of income is it
chargeable? (3 marks)
Answer:
As per Section 56(2)(ix) any sum of money received as advance in the
course of transfer of capital asset is chargeable.
(i) If such sum is forfeited; and
(ii) The negotiations did not result in transfer of such capital asset.
Thus, the advance amount forfeited will be taxed as income from other
sources.
Space to write important points for revision

2015 - June [4] (c) State the conditions to be satisfied when a sole
proprietary concern is succeeded by a company, to avail tax exemption in
respect of capital gains. (4 marks)
Answer:
Conversion of sole proprietary concern into a company:
Where a sole proprietary concern is succeeded by the company the capital
gain on transfer of any capital asset or intangible asset will be tax-free.
7.194 O Scanner CMA Inter Gr. I Paper 7 (2016 Syllabus)

The following conditions are to be satisfied in this regard:


(i) All the assets and liabilities of the sole proprietary concern relating to
the business immediately before the succession become the assets
and liabilities of the company;
(ii) The shareholding of the sole proprietor in the company is not less than
50% of the total voting power in the company and
(iii) The continues to remain so for a period of 5 years from the date of
succession; and
(iv) The sole proprietor does not receive any consideration or benefit,
directly or indirectly, in any form or manner, otherwise than by way of
allotment of shares in the company.
Space to write important points for revision

2015 - Dec [4] (a) State the circumstances under which the Assessing
Officer may refer the valuation of capital asset to the Valuation Officer.
(4 marks)
Answer:
Please refer 2012 - June [6] (a) on page no. 190
Space to write important points for revision

PRACTICAL QUESTIONS
2008 - Dec [3] (b) Are the profits arising from sales of agricultural lands
situated in a village 10 kms. from Vishakhapatnam Corporation limits and
another situated in city limits of Vishakhapatnam liable to Income-tax?
(2 marks)
Answer:
Agricultural land situated in a village is not a “Capital asset” as envisaged by
Section 2(14), while the agricultural land within the city limit of
Vishakhapatnam is capital asset. Profit arising from sale of the latter gives
rise to capital gains. Depending on the period of holding of the land, it will be
short-time or long –term.
Space to write important points for revision
[Chapter  8] Capital Gains O 7.195

2009 - June [3] (c) The urban lands of Mr. A were acquired by the State
Government 10 years back and compensation was paid. Mr. A took up the
matter before the Court/Tribunal. Enhanced compensation of ` 10 lakhs was
awarded by the Court/Tribunal in February, 2018 and the same was received
in May, 2018.
State the consequences under the Income-tax Act, 1961, showing
clearly the year of taxability. What will happen if Mr. A dies and Mr. L, his
legal heir receives the enhanced compensation? (4 marks)
Answer:
Enhanced Compensation: Many a time, person whose capital assets have
been taken over or acquired by the Government and who get compensation
from the Government, go to the Court/Tribunal of law, seeking enhancement
of compensation. If the Court/Tribunal awards a compensation which is
higher than the original compensation, the difference (in other words, the
enhanced compensation) thereof will be chargeable to capital gains in the
year in which the same is received from the government.
For this purpose, the cost of acquisition and cost of improvement shall
be taken to be nil.
So, in the given case, ` 10 lakhs will be charged to tax as capital gains
u/s 45(5) in the assessment year 2019-20.
Death of the transferor: It is possible that the transferor may die before
he receives the enhanced compensation. In that case, the enhanced
compensation will be chargeable to tax in the hands of Mr. L, i.e., the person
who receives the same.
Space to write important points for revision

2009 - Dec [2] (b) Miss. Rama sells a residential building at Jodhpur for
` 15,00,000 on July 1, 2017 The building was acquired for ` 1,50,000 on
June 1, 2007.
She pays brokerage @ 2% at the time of sale of the Building. She
invests ` 7 lakh in purchase a residential building on December, 2018 and
deposit ` 2 lakh under section 54EC in bonds of NHAI (Redeemable after 3
years) on March, 2019.
Compute the capital gain Chargeable to tax for the assessment year
2019-20. (3 marks)
7.196 O Scanner CMA Inter Gr. I Paper 7 (2016 Syllabus)

Answer:
Computation of taxable capital gain of Miss Rama for the
A.Y. 2019 -20
Particulars (`)
Sales consideration 15,00,000
Less: sales Expenses (2% Brokerage) 30,000
14,70,000
Less: Cost of acquisition 1,50,000 × 3,25,581
11,44,419
Long term Capital Gain
Less: Exemption
Section 54 7,00,000 7,00,000
Taxable long term Capital Gain 4,44,419
Space to write important points for revision

2010 - Dec [5] (a) Manish owned a land located in Chennai-Bangalore


highway in Thambaram Municipal Corporation limits, which was acquired by
NHAI in the financial year 2018-19 for ` 10,00,000. The land had been
purchased by Manish on 2-4-2001 for ` 10,000. The fair market value of the
land as on 1-4-2002 was ` 19,000.
Yet another piece of urban land located in Chennai purchased in April,
2010 for ` 25 lakhs was sold by him in February, 2019 for ` 38 lakhs, but the
sale deed thereof, was not registered till 31-3-2019. The possession was
given to the buyer on 31.1.2019 and the sale deed was finally registered on
16-4-2019. The value adopted by the Stamp Valuation Authority was ` 42
lakhs. Manish paid 2% of the sale consideration towards brokerage. Manish
deposited ` 10 lakhs in Capital Gain Deposit Account of SBI on 20-11-2019
in order to avail exemption under section 54F of the Income-tax Act, 1961
subsequently by constructing a residential house.
Cost inflation indices are 280 for the FY 2018-19, 167 for the FY 2010-11
and 105 for the FY 2002-03.
Compute the capital gain chargeable to tax arising as a result of these
transactions. (10 marks)
[Chapter  8] Capital Gains O 7.197

Answer:
Particulars Property 1 Property 2

Sale Consideration 10,00,000 42,00,000


Less: Brokerage @2% Nil 76,000
Net Sale consideration 10,00,000 41,24,000

Less: Indexed cost of 50,667 41,91,617


acquisition (19,000 ×280/105) (25,00,000 ×280/167)
Long term Capital Gain 9,49,333 (67,617)

Total Long term Capital Gain 8,81,716


Less: Exemption u/s 54F Nil
Taxable Long term Capital Gain 8,81,716
NOTE:
(i) As per Sec. 50C, value adopted by Stamp Valuation Authority should
be considered as sale consideration if sold for less than that amount
stamp duty value exceeds 105% of consideration.
(ii) As per Sec. 2(47), if the consideration has been received and
possession has been given, then it will be considered as transfer.
Registration of conveyance deed is immaterial.
(iii) Amount deposited in capital gains deposit scheme is not eligible for
deduction as the deposit was made after the due date specified in
Section139(1).
As per amendment made by Finance Act, 2018 w.e.f. 1st April 2018
where the value adopted or assessed or assessable by the stamp valuation
authority does not exceed 105% of the consideration received or accruing
as a result of the transfer the consideration so received or accruing as a
result of the transfer shall, for the purpose of section 48, be deemed to be
the full value of the consideration.
And where the value adopted or assessed or assessable by the stamp
valuation authority is exceeds 105% of the consideration received or
accruing as a result of the transfer then Stamp Duty Value shall, for the
purpose of section 48, be deemed to be the full value of the consideration.
Space to write important points for revision
7.198 O Scanner CMA Inter Gr. I Paper 7 (2016 Syllabus)

2011 - June [2] (b) Scindia furnishes the following particulars for the
previous year 2018-19:
A plot of land was sold on 19-7-2018 for ` 35,00,000. He paid brokerage on
its sales at one percent. He had purchased this plot on 20-12-2002 for
` 4,20,000. On 1-2-2019, he had purchased a residential house for
` 18,00,000. He owns another residential house, purchased on 8-7-2014.
The cost of inflation index for financial years 2002-03 and 2018-19 are
105 and 280 respectively.
Find out the amount of capital gains chargeable to income tax.
Suppose Scindia sells the new residential house on 1-1-2021, what will
be the taxable amount of capital gains and in which year it will be charged
to tax ?
If he purchases any other residential house on 1-6-2020, what will be
taxable amount of capital gains and in which year it will be charged to tax?
Is the same short-term or long term in nature ? (6 marks)
Answer:
Name of the Assessee: Mr. Scindia
Financial Year: 2018-19
Assessment Year: 2019-20
Residential Status: Resident and Ordinarily Resident
Statement of computation of Capital Gains
Particulars Amount Amount
(`) (`)
Sale Consideration 35,00,000
Less: Brokerage @1% 35,000 34,65,000
Net Sale Consideration
Less: Indexed cost of
acquisition [4,20,000 × 280/105] 11,20,000
Long Term Capital Gains 23,45,000
Less: Exemption u/s 54 F [ ( 1 8 , 0 0 , 0 0 0 / 3 4 , 6 5 , 0 0 0 ) × 12,18,182
Long Term Capital Gain 23,45,600] 11,26,818
[Chapter  8] Capital Gains O 7.199

As per Section 54F assessee should not sell new house within 3 years of its
purchase otherwise the amount of capital gains which was earlier exempt will
be taxed in the year in which the house is sold. If Mr. Scindia sells his house
on 1-1-2021 i.e. before expiry of 3 years from date of purchase, the
exemption given in the assessment year 2019-20 will be withdrawn and
amount of ` 11,26,818 will be taxed in assessment year 2021-22.
If Mr. Scindia purchases another residential house within 2 years of sale of
original house, then exemption will be withdrawn and amount earlier
exempted u/s 54F will be taxable as Long Term Capital Gain in the
assessment year 2021-22.
Space to write important points for revision

2011 - June [6] (c) Vasudevan converted jewellery/ornaments owned by him


into stock in trade on 1.9.2018, when he commenced a sole proprietary
business by name Vaasu Jewellers. The jewellery/ornaments were acquired
on 1.4.1976 for ` 2,00,000. The FMV on 1.4.2002 was ` 4 lakhs. The market
value on 1.9.2018 was ` 35 lakhs.
He recorded the value at ` 25 lakhs in his books of account.
Compute taxable capital gain chargeable to tax.
If the entire jewellery stock so converted was sold for ` 32 lakhs before
31.03.2019, how much is taxable and under what head it is taxable?
Clearly indicate the assessment years in which the above are taxable.
Cost inflation indices for financial years 2002-03 and 2018-19 are 105 and
280 respectively. (4 marks)
Answer:
Computation of Capital Gain/Loss
Sale Consideration [Being the FMV on the date of conversion) 35,00,000
(Section 45(2)]
Less: Indexed Cost of Acquisition (FMV on 1/4/2001) 10,66,667
(4,00,000 × 280/105)
Long Term Capital Loss 24,33,333
7.200 O Scanner CMA Inter Gr. I Paper 7 (2016 Syllabus)

Computation of Business Income/Loss


Sale Consideration 32,00,000
Less: FMV on date of conversion 35,00,000
Business Loss 3,00,000
The Long term capital loss as well as business loss will be eligible for set off
and carry forward taxed in the assessment year 2019-20.
Space to write important points for revision

2011 - Dec [6] (a) VERTIKA Limited has two units - one engaged in
manufacture of computer hardware and the other involved in developing
software. As a restructuring drive, the company has decided to sell its
software unit as a going concern by way of slump sale for ` 385 lakh to a
new company called SUMEDHA Limited, in which it holds 74% equity
shares. The balance sheet of VERTIKA Limited as on 31st March, 2019 being
the date on which software unit has been transferred, is given hereunder
Balance Sheet as on 31.3.2019
Liabilities ` in Lakh Assets ` in lakh
Paid up Share Capital 300 Fixed Assets
General Reserve 150 Hardware unit 170
Share Premium 50 Software unit 200
Revaluation Reserve 120 Debtors
Current Liabilities Hardware unit 140
Hardware unit 40 Software unit 110
Software unit 90 Inventories
Hardware unit 95
Software unit 35
750 750
Following additional information are furnished by the management.
(i) The Software unit is in existence since May, 2011.
(ii) Fixed assets of software unit includes land which was purchased at
` 40 lakh in the year 2008 and revalued at ` 60 lakh as on March, 31,
2019.
[Chapter  8] Capital Gains O 7.201

(iii) Fixed assets of software unit mirrored at ` 140 lakh ( ` 200 lakh minus
land value ` 60 lakh) is written down value of depreciable assets as
per books of account. However, the written down value of these assets
under section 43 (6) of the Income Tax Act is ` 90 lakh.
Ascertain the tax liability, which would arise from slump sale to SUMEDHA
Limited. (8 marks)
Answer:
As per Section 50B, any profits and gains arising from the slump sale
affected in the previous year shall be chargeable to income-tax as capital
gains arising from the transfer of capital assets and shall be deemed to be
the income of the previous year in which the transfer took place.
If the assessee owned and held the undertaking transferred under slump
sale for more than 36 months before slump sale, the capital gain shall be
deemed to be long-term capital gain. Indexation benefit is not available in
case of slump sale as per Section 50B(2).
Computation of tax liability from slump sale of software unit
(` in lakh)
Sale consideration for slump sale of Software Unit 385
Less: Cost of acquisition being the net worth of Software Unit 185
Long term capital gains arising on slump sale 200
(The capital gains is long-term as the Software Unit is held for more than
36 months)
Tax liability on LTCG
Under Section 112 @ 20% on ` 200 lakhs 40.00
Add: Surcharge @7% 2.80
42.80
Add: Health and Education cess @ 4% 1.712
44.51
Working Note:
Computation of net worth of Software Unit (` in lakh)
(1) Book value of non-depreciable assets
(i) Land (Revaluation not to be considered) 40
(ii) Debtors 110
(iii) Inventories 35
7.202 O Scanner CMA Inter Gr. I Paper 7 (2016 Syllabus)

(2) Written down value of depreciable assets under section 43(6)(Note 1) 90


Aggregate value of total assets 275
Less: Current liabilities of software unit 90
Net worth of software unit 185
Note: For computing net worth, the aggregate value of total assets in the
case of depreciable assets shall be the written down value of the block of
assets as per Section 43(6).
Space to write important points for revision

2011 - Dec [6] (b) Mr. AGGARWAL purchased a house property on 14th
April, 1980 for ` 1,05,000. He entered into an agreement with Mr. B for the
sale of house on 15th September, 2003 and received an advance of
` 25,000. However, since Mr. B did not remit the balance amount, Mr.
AGGARWAL forfeited the advance.
Later on, he gifted the house property to his friend Mr. A on 15th June, 2007.
Following renovations were carried out by Mr. AGGARWAL and Mr. A to the
house property:
Amount (`)
By Mr. AGGARWAL during FY 1980-81 10,000
By Mr. AGGARWAL during FY 2004-05 50,000
By Mr. A during FY 2005-06 1,90,000
The fair market value of the property as on 1.4.2001 is ` 1,50,000.
Mr. A entered into an agreement with Mr. C for sale of the house on 1st June,
2006 and received an advance of ` 80,000. The said amount was forfeited
by Mr. A since Mr. C could not fulfil the terms of the agreement.
Finally, the house was sold by Mr. A to Mr. Sanjay on 2nd January, 2019 for
a consideration of `12,00,000. Compute the capital gains chargeable to tax
in the hands of Mr. A for the assessment year 2019-20. Cost inflation indices
are as under:
Financial Year Cost Inflation Index
2001-02 100
2004-05 113
2007-08 129
[Chapter  8] Capital Gains O 7.203

2005-06 117
2012-13 200
2018-19 280 (7 marks)
Answer:
Computation of capital gains chargeable to tax in the hands of Mr. A
`
Sale consideration 12,00,000
Less: Indexed cost of acquisition
(1,50,000 – 80,000 = 70,000 × 280/129) 1,51,938
Less: Indexed cost of improvement
(50,000 × 280/113) ` 1,23,894
(1,90,000 × 280/117) ` 4,54,700 5,78,594
Long term capital gain (4,69,468)

Note:
1. Cost to the previous owner will be the cost for the assessee.
2. Amount forfeited by the previous owner will not be deducted.
3. Cost of improvement before 1/4/2001 will not be considered.
Space to write important points for revision

2013 - Dec [1] {C} (a) Answer the following sub-divisions briefly in the light
of the provisions of the Income-tax Act, 1961:
(iv) Is the right of management in an Indian company a capital asset ? On
relinquishment directly or indirectly, is it liable to tax? (1 mark)
Answer:
Yes, it is a capital asset in view of the Explanation to Section 2(14). It is a
‘transfer’ in view of Explanation 2 to Section 2(47). Hence the relinquishment
is liable to capital gains tax.
Space to write important points for revision

2013 - Dec [3] (a) Chirag, an individual, purchased 5,000 shares of X.


Limited at ` 50 per share and 4,000 shares of Y. Limited at ` 60 per share
in the previous year 2012-13 and held them as capital assets.
7.204 O Scanner CMA Inter Gr. I Paper 7 (2016 Syllabus)

In the previous year 2017-18, he converted the shares into his stock-in-trade.
The fair market value of the shares of both the companies on the date of
conversion was ` 300 per share.
In the previous year 2018-19, he sold the shares of the two companies at
` 380 per share. Shares were sold by way of private sale and hence
securities transactions tax was not payable.
Ascertain chargeable capital gain and business income from the above-
noted transactions in the hands of Chirag.
Cost Inflation Index:
Financial Year 2012-13 : 200
Financial Year 2017-18 : 272
Financial Year 2018-19 : 280 (5 marks)
Answer:
Computation of Capital gain for Assessment Year 2019-20
Particulars `
Fair Market value of Y Ltd. on the date of conversion
(deemed consideration) 9,000 X ` 300 27,00,000
Less: Indexed cost of Acquisition (` 2,50,000 + ` 2,40,000)
× 272/ 200 6,66,400
Long term capital gain 20,33,600

Computation of Business Income for Assessment Year 2019 - 2020


Particulars `
Sale proceeds of Shares (9,000 X ` 380) 34,20,000
Less: Fair market value of Shares on the date of conversion 27,00,000
Business Income 7,20,000

Note: As securities transaction tax was not paid, exemption under


section 10(38) in respect of long term capital gain is not available.
Space to write important points for revision
[Chapter  8] Capital Gains O 7.205

2014 - Dec [4] (a) Mr. Ahuja purchased one plot of land in 2006-07 at a cost
of ` 1,00,000 in Delhi. The land was held by him as capital asset. He
converted the plot into his stock in trade on 1st April, 2017, on which date the
fair market value of the plot was ` 15,50,000.
He started constructing a building consisting of eight flats of equal size and
dimension on the plot on 1st April, 2017. Cost of construction of each flat is
` 6,00,000. Construction was completed in June 2018. He sold five flats at
` 12,00,000 per flat from June, 2018 to March, 2019. The remaining three
flats were held as stock on 31st March, 2019.
Compute Capital Gain and Business Income arising from above transactions
for Assessment year 2019-20.
(Cost Inflation Index:
2006-07: 122
2017-18: 272
2018-19: 280) (9 marks)
Answer:
Computation of Capital Gain for Assessment Year 2019-20
Particulars `
Proportionate Fair market value of land-deemed consideration 9,68,750
under section 45(2): ` 15,50,000 x 5 / 8 (Note 1)
Less: Proportionate indexed cost of acquisition 1,43,443
(` 1,00,000 x 280 / 122 x 5 /8 (Note 2))
Long term capital gain 8,25,307

Computation of Business Income for Assessment Year 2019-20


Particulars ` `
Sale proceeds of 5 flats : ` 12,00,000 x 5 60,00,000
Less: Proportionate fair market value of land 9,68,750
Cost of construction (` 6,00,000 x 5) 30,00,000 39,68,750
Business Income 20,31,250
7.206 O Scanner CMA Inter Gr. I Paper 7 (2016 Syllabus)

Notes:
1. Capital gain arising on conversion of capital asset into stock-in-trade is
taxed in the year in which the converted asset is transferred. Therefore,
capital gain in respect of proportionate land for 5 flats is chargeable to
tax in assessment year 2019-20.
2. Although tax liability for capital gain is taxed in the year of actual
transfer, cost inflation index of the year in which capital asset is
converted is to be used for determining capital gain, as transfer under
section 2(47) is recognized in the year of conversion.
Space to write important points for revision

2014 - Dec [4] (b) Ajay purchased a house for ` 6 lacs from Anup Properties,
a property dealer on 1st June, 2017, for his residential purpose. He paid
stamp duty on value of ` 10 lacs assessed by Stamp Valuation Authority.
Examine the tax implications in the hands of Ajay and Anup Properties.
(4 marks)
Answer:
In the hands of Ajay:
As per Section 56(2)(vii), where an individual receives any immovable
property, being his capital asset for a consideration, which is less than the
stamp duty value of the property by an amount exceeding ` 50,000, then the
excess of stamp duty value over such consideration shall be taxed in the
hands of the individual under the head “income from other sources”.
Therefore, the sum of ` 4 lacs (i.e., ` 10 lacs – ` 6 lacs) shall be taxed in the
hands of Ajay under the head “income from other sources”.
In the hands of Anup Properties:
Anup Properties is a property dealer. So the property sold constituted his
stock-in-trade. As per Section 43CA, where the consideration on transfer by
assessee of an asset (other than capital asset), being land or building is less
than the stamp duty value assessed or assessable, such stamp duty value
shall be deemed to be the full value of consideration for the purpose of
computing profits and gains from transfer of such asset.
[Chapter  8] Capital Gains O 7.207

Therefore, sum of ` 10 lacs shall be taken to be the consideration on sale of


the building to Ajay and the said sum shall be considered for determination
of business income of Anup Properties.
Space to write important points for revision

2014 - Dec [6] (a) From the following information, compute the income
taxable under the head ‘Capital gains’ and ‘Income from other sources’ in the
hands of Sachin:
(i) Sehwag gifted a vacant site to his friend Sachin on 23.05.2018 on the
occasion of latter’s birthday.
(ii) Sehwag had acquired the said vacant site in May, 2012 for
` 30,00,000.
(iii) The fair market value of the site for stamp duty purposes on the date
of gift i.e. on 23.05.2018 was ` 60,00,000.
(iv) Sachin sold the vacant site on 15.03.2019 for a consideration of ` 70
lakhs when its stamp duty value on the date of sale was ` 90 lakhs.
For capital gains, state with reason, whether it is short-term or long-
term.
Also compute the capital gains chargeable to tax in the hands of
Sehwag. (4 marks)
Answer:
Computation of total income in the hands of Sachin A.Y. 2019-20
Particulars `
Capital gains:
Sale consideration - Stamp duty valuation, as per Section 50C 90,00,000
(i.e. more than 105% of sale consideration)
Less: Cost of acquisition 60,00,000
Short term capital gain 30,00,000
Income from other sources:
Stamp duty value of the property on the date of receipt of gift
i.e., 23.05.18 60,00,000
Total income 90,00,000
7.208 O Scanner CMA Inter Gr. I Paper 7 (2016 Syllabus)

Note:
Where a property is received without consideration, the FMV i.e., stamp
duty valuation is the value taxable under the head ‘income from other
sources’ in the hands of Sachin.
The holding period in the hands of Sachin shall be from the date of receipt
of property and not from the date of acquisition by the previous owner.
Since it was received on 23.05.2018 and sold on 15.03.2019 it is a
short-term capital asset and the resultant gain is short-term capital gain.
The donor Sehwag is not subject to capital gains as the property was not
transferred but was gifted which is not regarded as transfer under Section
47.
Space to write important points for revision

2015 - June [5] (a) Mr. Dhoni sold a residential building at Cochin for ` 65
lakhs in December, 2018. The stamp valuation authority determined the
value at ` 80 lakhs which was not contested by Mr. Dhoni. The property was
acquired in April, 2007 for ` 6 lakhs. He acquired a residential flat at Ranchi
for ` 55 lakhs and an other residential house at Cuttack for ` 25 lakhs before
March, 2019. Compute the capital gain of Mr. Dhoni for the assessment year
2019-20.
Note:
Financial year Cost inflation index
2007-08 129
2018-19 280
You are required to plan in such a way that the incidence of tax is the least.
(5 marks)
Answer:
Computation of capital gain of Mr. Dhoni for the assessment year 2019-20
Particulars `
Sale consideration of residential building at Cochin 65,00,000
Stamp valuation authority has determined the value at ` 80 lakhs.
The higher of the two is to be adopted as deemed sale 80,00,000
consideration U/s 50C (See Note - 1)
[Chapter  8] Capital Gains O 7.209

Less: Indexed cost of acquisition


` 6,00,000 × 280 ÷ 129 13,02,326
Capital Gains before exemption 66,97,674
Less: Exemption U/s 54
In respect of residential property acquired at Ranchi 55,00,000
Taxable long-term Capital Gain 11,97,674

Note:
1. As per amendment made by Finance Act, 2018 w.e.f. 1st April, 2018
where the value adopted or assessed or assessable by the stamp
valuation authority does not exceed 105% of the consideration received
or accruing as a result of the transfer, the consideration so received or
accruing as a result of the transfer shall, for the purpose of section 48,
be deemed to be the full value of the consideration.
And where the value adopted or assessed or assessable by the stamp
valuation authority is exceeds 105% of the consideration received or
accruing as a result or the transfer then Stamp Duty Value shall, for the
purpose of section 48, be deemed to be the full value of the
consideration.
2. From the assessment year 2018-19 onwards, the exemption under
Section 54 is limited to one residential house in India. Out of the two
residential properties acquired by him, he can option for exemption
under Section 54 in respect of one property only. The higher of the
values has been adopted for maximizing the benefit to the assessee.
Space to write important points for revision

2015 - Dec [1] {C} Answer the following questions with brief
reasons/workings:
(a) Mr. Vishnu received ` 2,00,000 on 10th April, 2018 as advance from Mr.
Ram in pursuance of an agreement to transfer a vacant land held as
capital asset. The agreement was cancelled later. Mr. Vishnu retained
50% of the advance and refunded the balance to Mr. Ram. What is the
tax implication in the hands of Mr. Vishnu? (2 marks)
7.210 O Scanner CMA Inter Gr. I Paper 7 (2016 Syllabus)

(d) Mr. G gifted a house property to his son, Mr. H (age 20 years) on 5th
June, 2018. The property was acquired by Mr. G on 10th October, 2015.
Mr. H held the property as capital asset and transferred the property on
20th October, 2018 for ` 20 lakhs. Is the capital gain short-term or long-
term? (2 marks)
Answer:
(a) As per Section 56(2)(ix) any sum of money received as advance in the
course of transfer of capital asset is chargeable:
(i) If such sum is forfeited; and
(ii) The negotiations did not result in transfer of such capital asset.
Thus the amount of ` 1,00,000 (50% of 2,00,000) will be taxable under
the head “Income from other sources” in hands of Mr. Vishnu.
(d) Where the capital asset became the property of the assessee under gift
or will, the cost of acquisition of the asset shall be deemed to be the cost
which the previous owner of the property acquired it; as increase by the
cost of improvement incurred by the previous owner/assessee. (Section
49) and
Where an asset received by the way of gift has been sold, the period of
holding of the previous owner should be considered.
Therefore capital gain rising on sale of house property by Mr. H. will be
long term capital gain because the period of holding is more than 24
months.
Space to write important points for revision

2015 - Dec [2] (c) ABC Private Limited allotted 10,000 shares of ` 10 each
to Mr. A at ` 20 per share. The fair market value of the shares on the date of
allotment was determined at ` 15 per share. Will such allotment have any tax
implication in the hands of ABC (P) Ltd? What would be your answer in case
the company is a listed company in Bombay Stock Exchange? If Mr. A is a
non-resident, state the implication. (4 marks)
Answer:
Amendment to Section 56(2) of Income Tax Act, 1961. When a company
(public unlisted or private company) issues its share to an India resident at
a value more than fair market value of those shares, will be subjected to tax
in hands of the company under head “Income from other sources”.
[Chapter  8] Capital Gains O 7.211

Thus share premium of ` 50,000 (i.e. 1,000 share at premium of ` 5 each)


is taxable in hands of ABC private limited under head “Income from other
sources”.
IInd part of question/Answer:
This clause is applicable to companies in which public are not substantially
interested. The clause is not applicable to companies listed in any
recognised stock exchange.
This clause would not be applicable if ABC private limited is listed in BSE.
IIIrd part of question/Answer:
Any consideration paid by a non resident to a closely held company for the
share in it, the amount of share premium exceeds the fair market value of the
share, company may not be liable to tax.
If Mr. A is a non resident, company would not be liable to tax for share
premium (value in excess of fair market value of share) by Mr. A.
Space to write important points for revision

2016 - June [2] (b) Ms. Sanvitha sold a residential house at Salem for a
consideration of ` 9.5 crores on 10-01-2019. The buyer is an unrelated
person. The stamp duty valuation of the house is ` 10.20 crores. Brokerage
on sale paid at 2%.
In April, 2005 she had bought land for ` 1.20 crores. Registration and other
expenses incurred were 10% of the same. The construction of the house
was completed in March, 2008 for ` 1 crore.
She purchased the following two residential houses in March, 2019:
(i) House at Chennai for ` 1.1 crores
(ii) House at Mumbai for ` 3 crores.
She also purchased bonds of National Highway Authority of India on the
following dates:
23-02-2019 ` 40 lakhs
12-04-2019 ` 50 lakhs
Compute income of Ms. Sanvitha under the head “capital gains” for
Assessment Year 2019-20. Cost inflation indices are:
FY 2006-07 122;
FY 2007-08 129;
FY 2018-19 280 (7 marks)
7.212 O Scanner CMA Inter Gr. I Paper 7 (2016 Syllabus)

Answer:
(` in crores)
Full value of consideration Note 1 10.2
Less: Brokerage 2% of actual sale consideration 0.19
Net sale consideration 10.01
Less: Cost of acquisition
Land Portion: Original cost = 3.03

House construction Indexed COA 2.17 5.20

LTCG from sale of residential house at Salem 4.81


Less: Exemption u/s 54 Note 2 3
Exemption u/s 54EC Note 3 0.5 3.5
LTCG chargeable to tax 1.31

Notes:
1. As per Section 50C, where the stamp duty value is more than actual
sale consideration, the former is to be adopted for computing capital
gains.
As per amendment made by Finance Act, 2018 w.e.f. 1st April,
2018 where the value adopted or assessed or assessable by the
stamp valuation authority does not exceed 105% of the consideration
received or accruing as a result of the transfer, the consideration so
received or accruing as a result of the transfer shall, for the purpose
of section 48, be deemed to be the full value of the consideration.
And where the value adopted or assessed or assessable by the stamp
valuation authority is exceeds 105% of the consideration received or
accruing as a result of the transfer then Stamp Duty Value shall, for
the purpose of section 48, be deemed to be the full value of the
consideration.
[Chapter  8] Capital Gains O 7.213

2. U/s 54, exemption is available only in respect of only one residential


house acquired within the eligible period. Here it will be beneficial to
claim for the house property bought at Mumbai.
3. U/s 54EC, maximum permissible deduction is ` 50 lacs.
Space to write important points for revision

2016 - Dec [1] {C} (d) Answer the following question with brief
reason/working:
(i) Mr. Rao sold a vacant site to Mr. Jain in August, 2017 for ` 5 lakhs.
The stamp duty valuation of the site at the time of sale was ` 8 lakhs.
The difference of ` 3 lakhs was taxed as income in the hands of Mr.
Jain under the head ‘other sources’. Now in March, 2019 Mr. Jain sold
the vacant site for ` 12 lakhs. What is the cost of acquisition of site to
be adopted by Mr. Jain? (2 marks)
Answer:
Cost of acquisition of site to be adopted by Mr. Jain will be ` 8 lakhs as per
Section 49 (4).
Space to write important points for revision

2016 - Dec [3] (b) Mrs. Kiruthika sold a residential house at Mumbai for a
consideration of ` 70,00,000 on 14.02.2019. She had bought land in May,
2008 for ` 8,00,000. Registration and other expenses incurred were 10% of
the same. The construction of the house was completed in November, 2010
at a cost of ` 10,00,000.
The sale was made to a stranger. She had taken an advance of ` 2,00,000
in cash on 10.04.2018, for which clear evidence is available. At that time
stamp duty valuation was ` 70,00,000. At the time of registration in February,
2019, there was 10% increase in stamp duty valuation. She received the
balance consideration of ` 68,00,000 in February, 2019 by cheque.
Brokerage paid for sale was 1% on sale consideration received.
She purchased the following residential houses in March, 2019:
(i) House at Delhi for ` 11,00,000
(ii) House at London for ` 19,00,000
7.214 O Scanner CMA Inter Gr. I Paper 7 (2016 Syllabus)

He also purchased bonds of National Highway Authority of India approved


for the purpose of Section 54EC on the following dates:

29/03/2018 ` 4,50,000
22/05/2018 ` 5,00,000
Compute income chargeable under the head ‘Capital Gains’ in the hands of
Mrs. Kiruthika for Assessment Year 2019-20.
Cost Inflation Indices (CII) are as under:
Financial Year CII
2008-09 137
2010-11 167
2018-19 280
(7 marks)
Answer:
Computation of Capital Gain of Mrs. Kirithika for A.Y. 2019 -20:
Sale Consideration 77,00,000
Less: Brokerage @ 1% 70,000
Net consideration 76,30,000
Less Index Cost of acquisition:
Land May 2008 8,00,000
Registration Exp. 80,000
(8,80,000 × 280/137) 17,98,540
Cost of Construction:
(10,00,000 × 280/167) 16,76,647
Long Term Capital Gain 41,54,813
Less: Exemption u/s 54 11,00,000
u/s 54 EC 9,50,000 20,50,000
Taxable LTCG 21,04,813
Space to write important points for revision
[Chapter  8] Capital Gains O 7.215

2017 - Dec [5] (a) State the taxability of the following transactions for the
assessment year 2019-20:
(i) Mr. Ashok acquired a vacant site from Mr. Brijesh (non relative) for ` 6
lakhs when the stamp duty valuation of the vacant site on the date of
registration of document was ` 10 lakhs.
(ii) Rosy & Co. a partnership firm engaged in trading of vacant lands. It
sold vacant land for ` 40 lakhs when the stamp duty valuation of the
lands was ` 55 lakhs.
(v) Mr. Jayaram retired from a nationalized bank on 30.11.2018, sold his
motor car for ` 5 lakhs. The Car was used by him for the last 5 years
and was received as gift from his brother who acquired the car for ` 10
lakhs on 10.01.2012.
(vi) Mr. Vasu acquired an agricultural land situated in a village (rural area)
for ` 10 lakhs from Mr. Sundar (non relative) when the stamp duty
valuation on the date of registration of document was ` 12 lakhs.
(2 × 4 = 8 marks)
Answer:
(i) Under section 56(2)(vii): Income from other sources: Where an
Individual or HUF receives, in any previous year from any person or
persons an immovable property being land or building with
consideration (if stamp duty value exceeds the purchase price by more
than ` 50,000) then value of gift = stamp duty value – purchase price
i.e. ` 10 lakhs – ` 6 lakhs = ` 4 lakhs being taken as income from
other sources and liable to tax.
(ii) Section 43CA: Special provisions for full value of consideration for
transfer of assets other than capital assets in certain cases.
As per provisions of Section 43CA of the Income Tax Act, 1961 full
value of consideration for sale of vacant land by Rosy & Co. will be
` 55 lakhs and difference between full value of consideration and cost
of acquisition will be treated as income from business.
(v) Difference between sale consideration i.e. ` 5,00,000 and WDV of Car
as on 01/04/2018 will Not be treated as short term capital gain in the
hands of Mr. Jayaram.
7.216 O Scanner CMA Inter Gr. I Paper 7 (2016 Syllabus)

(vi) Rural agricultural land will not be treated as capital asset and
difference between stamp duty value and cost of acquisition will not be
treated as income from other sources, hence not liable to tax.
Space to write important points for revision

2018 - Dec [4] (a) The summarised financial position of Purva India (P) Ltd.
as on 31/12/2018 is as under:
Liabilities Amount ` Assets Amount `
Equity share capital of 8,00,000 Land 6,00,000
` 10 each
Preference share capital 1,00,000 Building (WDV as per 3,00,000
Income tax Act)
Reserves 2,00,000 Machinery (WDV as per 4,00,000
Income tax Act)
Loan Creditors 6,00,000 Current Assets 10,40,715
Creditors 6,00,000
Provision of Dividend 40,715
Distribution Tax
23,40,715 23,40,715
Additional Information:
The Company went into liquidation on the balance sheet date; all current
assets and building realized at book value. The realized money was applied
towards payment of outside liabilities including Dividend Distribution Tax, and
there after the preference shareholders.
Mr. Utkarsh is a holder of 10% equity shares and 20% preference shares of
the company. Equity shares were originally acquired by him 16.08.2003 at
face value. However, he had subscribed to preference shares on
01.04.2018, which were issued at par. He received a part of land (MV
` 5,00,000) and cash (for preference share) ` 20,000.
Compute the capital gain in hands of the company and Mr. Utkarsh.
(8 marks)
[Chapter  8] Capital Gains O 7.217

Table Showing Marks of Compulsory Questions


Year 14 14 15 15 16 16 17 17 18 18
J D J D J D J D J D
Practical 4 2
Total 4 2
9 INCOME FROM OTHER SOURCES
THIS CHAPTER INCLUDES
 Method of Accounting  Interest on Securities
 Income from Dividend  Family Pension
 Winnings from Lotteries,  Other Incomes under IFOS
Crossword puzzles, Horse
racing etc.
Marks of Objective, Short Notes, Distinguish Between, Descriptive & Practical Questions
Legend
Objective Short Notes Distinguish Descriptive Practical

For detailed analysis Login at www.scannerclasses.com


for registration and password see first page of this book.

7.218
[Chapter  9] Income from other Sources O 7.219

DESCRIPTIVE QUESTIONS
2009 - Dec [4] (b) State briefly with regard to deduction allowable under
Section 57 of the Income Tax Act, 1961 in respect of income from other
sources. (5 marks)
Answer:
Deductions allowable under Section 57 of the Income tax act, 1961 in
respect of Income from other sources are as follows;
(i) [Section 57(i)] Deduction in case of dividends or interest on
Securities:
Assessee can claim deduction of expenses of collection charges and
interest on loan etc. from gross dividend. If the expenditure has been
laid out for the purpose of earning the dividend income then whether
income is actually earned or not is immaterial and deduction on
account of interest can be claimed.
(ii) [Section 57(ia)] Deduction in case of employee's contribution to
welfare fund:
If contribution received by employer from the employee is deposited
under the welfare scheme is deposited, on or before the due date
mentioned in the scheme, the amount so deposited is allowed as
deduction.
(iii) [Section, 57(ii)] Deduction permissible for letting out of
machinery, plant, furniture and buildings:
Expenses such as current repairs, insurance premium, depreciation
etc. are permissible in respect of letting out machinery, plant, furniture
and buildings.
(iv) [Section 57(iia)] Deduction from family pension received by legal
heirs of deceased employee:
After the death of the employee, if there is any family pension received
by the legal heirs of the deceased, it will deemed to be the income of
the legal heir and will be taxable under the head `Income from other
sources'. On such pension a standard deduction shall be allowed to
the legal heir @ 33.33% of such pension, or ` 15,000, whichever is less.
7.220 O Scanner CMA Inter Gr. I Paper 7 (2016 Syllabus)

(v) [Section 57(iii)] Deduction for any other expenditure:


Any other expenditure, not being a expenditure of a capital nature,
expended wholly and exclusively in respect of other income, can be
claimed as a deduction.
(vi) [Section 57(iv)] Deduction from compensation and enhanced
compensation:
Any interest received by an assessee on compensation or enhanced
compensation, as the case may be, shall be deemed to be the income
of the year in which it is received under the head `Income from other
sources'. A deduction of a sum equal to 50% of such income(interest)
shall be allowed.
Space to write important points for revision

2012 - June [4] (a) Under what head of income is family pension assessed?
Is any deduction available for such amount received? (3 marks)
Answer:
Family pension:
Family pension is chargeable to tax under the head “Income from other
sources”.
Under Section 57, of Income Tax Act, 1961, the least of the following is
allowed as deduction:
(i) 33 % of gross pension

(ii) `15,000.
Space to write important points for revision

2015 - June [1] {C} (c) Attempt the question below:


(i) At what rate of interest on debentures of an Indian public limited
company subscribed in foreign exchange by a non-resident Indian
taxed? (1 mark)
Answer:
(i) 20%
Space to write important points for revision
[Chapter  9] Income from other Sources O 7.221

PRACTICAL QUESTIONS
2008 - Dec [2] (a) Answer the following question briefly:
(iii) RR Ltd., engaged in manufacture of paper, has taken a building on
lease, sub-leased portion of it along with furniture and fixtures from
1.4.2018. It received rent ` 1,20,000 and non-refundable deposit of
` 10,000 for the financial year 2018-19. It also received an amount of
` 70,000 as arrears of rent. What are the heads of income under which
the various receipts will be taxed, if found eligible to tax? (2 marks)
Answer:
Rental income in hands of owner is chargeable under head “Income from
house property” but income received by tenant or the person other than
owner shall be charged to tax under head “Income from other sources”.
Space to write important points for revision

2009 - June [5] (c) Mr. A, is engaged in retail trade and is a distributor for the
products of Mr. X. In appreciation of high turnover effected by Mr. A, Mr. X
presented him a new car worth ` 4 lakhs. Discuss the eligibility to tax of the
above receipt in the hands of Mr. A. (2 marks)
Answer:
As per the provision of Section 28(v), the value of any benefit or perquisite,
whether convertible into money or not, arising from business or in exercise
of a profession shall be taxable as business income. The value of car of
` 4,00,000 is chargeable to tax as business income.
Space to write important points for revision

2010 - June [6] (c) In the following cases, state the head of income under
which the relevant receipt is to be assessed, along with reasons:
(i) Vibhisana let out his property to Bharath. Bharath sublets it. How is
subletting receipt to be assessed in the hands of Bharath?
(2 marks)
7.222 O Scanner CMA Inter Gr. I Paper 7 (2016 Syllabus)

Answer:
Sub-letting receipt is to be assessed as “Income from Other Sources” or as
“Profits and gains of business or profession” in hands of Mr. Bharath,
depending upon the facts and circumstances of each case. It is not
assessable as income from house property, since one of the conditions for
assessing an income under this head is that the assessee should be the
owner of the property. In this case, since Mr. Bharath is not the owner of the
house property, sub-letting receipt cannot be assessed under the head
“Income from House Property”.
Space to write important points for revision

2010 - Dec [2] (a) Janak received the following gifts during the financial year
2018-19:
Cash gift from father-in-law ` 25,000
Cash gift from friends on the occasion of marriage ` 37,000
Gift by way of wrist watch from a friend.
Value of the watch ` 12,000
Shares received as gift from wife’s friend. Market value as
on the date of gift ` 51,000
State the taxability of each item above. Calculation of total amount liable to
tax is not required. (5 marks)
Answer:
(i) Cash gift from father- in-law Exempt
(ii) Gifts received at the time of marriage Exempt
(iii) Gift received from a friend Exempt
(not covered by the term ‘Property’)
(iv) Shares received from wife’s friend Taxable
(As per explanation to Section 56(2)(vii), shares
are given in the definition of Property)
Space to write important points for revision
[Chapter  9] Income from other Sources O 7.223

2011 - June [4] (b) Mr. Janak receives the following gifts during the previous
year 2018-19:
Date of gift Details of gift and donor Amount of gift (`)
1. 01.07.2018 Gift from Raju, a friend, by cheque 50,000
2. 01.09.2018 Cash gift from sister-in-law 1,00,000
3. 01.12.2018 Gift of diamond ring on his birthday, by
his friend living in Dubai 75,000
4. 15.12.2018 Cash gifts of ` 31,000 each made by four
friends on the occasion of his son's marriage 1,24,000
5. 01.12.2018 Gift of a rosewood cot made by friend on
house opening ceremony 51,000
Discuss the taxability of each of the above as income from other sources.
(5 marks)
Answer:
Date of gift Name of donor Taxable
amount
(`)
01-7-2018 Gift from Raju, a friend, by cheque is taxable 50,000
01-9-2018 Cash gift from sister-in-law (relative)exempt u/s 56 Nil
01-12-2018 Gift of diamond ring on his birthday, by his friend
living in Dubai (Since, it has been received from
friend taxable) 75,000
15-12-2018 Cash gift received on occasion of his son’s
marriage from four friends is taxable. Since, it is
not marriage of assessee. 1,24,000
01-12-2018 Gift of rosewood cot made by friend on house
opening ceremony.( rosewood cot is not ‘property,
hence it is not taxable) Nil
Amount chargeable as income from other
sources 2,49,000
Space to write important points for revision
7.224 O Scanner CMA Inter Gr. I Paper 7 (2016 Syllabus)

2011 - June [8] (a) X held 18% shares in a private limited company. He
gifted all the shares to his wife Y on 1st Nov., 2018. Y obtained a loan of
` 8,00,000 from the company when the company's accumulated profit was
` 6,00,000. What are the tax implications of the above transaction?
(2 marks)
Answer:
Loan given by a closely held company to any person who has substantial
interest is chargeable to tax as deemed dividend. In this case, loan is given
to Y who holds 18% equity shares of the company having accumulated profit
of ` 6,00,000. Y is a shareholder in the company and she has substantial
interest in the company. Loan made to Y is considered as dividend to the
extent of accumulated profit i.e. ` 6, 00,000. Hence, ` 6, 00,000/ shall be
deemed as dividend under Section 2(22)(e) in the hands of Mrs. Y but w.e.f.
A.Y. 2019-20 Deemed Dividend is exempt u/s 10(34)
Note 1:
Amendment to section 115-O and 2(22) (e): With effect from 1st April 2018,
loan or advance given by a company (on or after April 1, 2018) which is
deemed as dividend under section 2(22)(e) will be subject to the following
provisions–
1. On such deemed dividend, the company which gives loan or advance
will have to pay dividend tax under section 115-O at the rate of 30%
(without grossing up) [+12% of such tax as surcharge + 4% of tax and
surcharge as health and education cess, effective rate: 34.944%.]
2. Such deemed dividend will be exempt in the hands of recipient by virtue
of section 10(34).
Space to write important points for revision

2013 - Dec [4] (a) Discuss whether the following receipts are chargeable to
tax in the hands of the recipients:
(i) Prize money of ` 10,000 awarded to Arjun for participating in a motor
car rally.
(ii) Madhu received ` 10,000 each from six friends on his 50th birth day.
(iii) Ramesh received a plot of land from his father-in-law as gift on his
marriage anniversary. The value assessed by the Stamp Valuation
Officer for the purpose of stamp duty was ` 5 lacs.
[Chapter  9] Income from other Sources O 7.225

(iv) Pramod purchased shares of Z. Limited at ` 1 lac from his friend. The
fair market value of the shares on the date of purchase was ` 1.70
lacs. (4 marks)
(b) Sanjay holds 16% shares in XYZ Private Limited. The company has
given him a loan of ` 2 lacs on 1st February, 2018. Accumulated profit of
the company on that date was ` 1.75 lacs. Sanjay repaid the loan on 31st
March, 2018. Examine the tax implication, if any, of the above
transactions in the hands of Sanjay. (4 marks)
Answer:
(a) (i) Prize money is an income under Section 2(24). It was received by
Arjun as a result of application of his skill and experience.
(ii) Any amount received without consideration from any person or
persons other than specified relatives, is not an income assessable
under the head “income from other sources” under Section
56(2)(vii), if such amount does not exceeds ` 50,000. As the
aggregate amount received as gift from six friends exceeds ` 50,000,
the amount ` 60,000 is taxable under Section 56(2)(vii).
(iii) As per Section 56(2)(vii) any immovable property received form
relative as gift not taxable in the hands recipient, irrespective of
amount of property gifted.
(iv) As per Section 56(2)(vii) even if asset is received for inadequate
consideration it is taxable in the hands of the recipient, if the
difference between fair market value and the amount of consideration
exceeds ` 50,000. Hence ` 70,000 is taxable in the hands of Pramod
under the head “income from other sources”.
Space to write important points for revision

Answer:
(b) As per Section 2(22)(e) where a shareholder of a closely held company
having 10% or more voting rights in the company, obtains loan or
advance from such company, the amount of loan or advance to the
extent of accumulated profit of the company shall be deemed to be
dividend. Therefore, the amount of loan to the extent of ` 1.75 lacs shall
be deemed to be dividend in the hands of Sanjay.
7.226 O Scanner CMA Inter Gr. I Paper 7 (2016 Syllabus)

Sanjay can claim exemption under Section 10(34), as the company is


required to pay dividend distribution tax on the amount of deemed
dividend.
Note:
Amendment to Sec. 115-0 and 2(22)(e):
With effect from 1st April 2018, loan or advance given by a company(on
or after April 1, 2018) which is deemed as dividend under section 2(22)
(e) will be subject to the following provisions–
1. On such deemed dividend, the company which gives loan or advance
will have to pay dividend tax under section 115-O at the rate of 30%
(without grossing up)[+ 12% of such tax as surcharge+4% of tax and
surcharge as health and education cess, effective rate: 34.944%].
2. Such deemed dividend will be exempt in the hands of recipient by virtue
of section 10(34).
Space to write important points for revision

2014 - June [3] (b) Mr. Somu placed a deposit of ` 20,00,000 in LMN Bank
on which he is eligible for interest of ` 1,90,000 for the year. He also
borrowed ` 7,00,000 from the bank on the security of the deposit. The
amount borrowed was used for his medical expenses and daughter’s
education. Interest on the amount borrowed was ` 63,000. He offered net
interest income of ` 1,27,000 under the head ‘income from other sources’.
Is he correct ? (3 marks)
Answer:
Mr. Somu is not correct as the amount borrowed was not used for making
deposit with LMN Bank. Interest income under the head ‘Income from other
Sources’ would be ` 1,90,000.
Space to write important points for revision

2014 - June [6] (c) Discuss the taxability or otherwise of the following gifts
received by Arun, an individual during the financial year 2018-19 :
(i) ` 50,000 each from his four friends on the occasion of his birthday.
(ii) Wrist watch valued at ` 20,000 from his friend. (2 marks)
[Chapter  9] Income from other Sources O 7.227

Answer:
(i) Gift in cash exceeding ` 50,000 in aggregate in a financial year is
taxable. Therefore, gift of ` 2,00,000 received from friends is taxable
under head income from other sources.
(ii) Wrist Watch is not covered under the definition of gift. Therefore, wrist
watch valued at ` 20,000 from his friend is not taxable.
(iii) As per section 56(2) (vii) any immovable property received from
relative as gift is not taxable in the hands of recipient, irrespective of
amount of property gifted.
Space to write important points for revision

2015 - Dec [5] (c) Ms. Priya has kept a fixed deposit of ` 10 lakhs with State
Bank of India on which she received interest of ` 80,000. Subsequently she
borrowed ` 5 lakhs from the same bank on the security of the fixed deposit.
Interest paid on such loan is ` 50,000. She offered interest income of
` 30,000 (after adjustment of interest paid ` 50,000). Is she correct?
(2 marks)
Answer:
Ms. Priya is not correct, there is no provision in the act to allow the interest
paid against the interest received by the assessee. Interest liability can not
be claimed as deduction u/s 57(III).
Hence, ` 80,000 will be taxed under head “Income from other sources”.
Space to write important points for revision

2017 - Dec [5] (a) State the taxability of the following transactions for the
assessment year 2019-20:
(iii) Ms. Janaki received family pension of ` 84,000. (2 marks)
Answer:
Family pension will be treated as income from other sources but a deduction
of ` 15,000 will be allowed from family pension. Therefore, income from other
sources in the hands of Ms. Janaki will be ` 84,000 less ` 15,000 = ` 69,000.
Space to write important points for revision
7.228 O Scanner CMA Inter Gr. I Paper 7 (2016 Syllabus)

2017 - Dec [5] (b) Mr. Vidyasagar received following gifts during the
financial year 2018-19:
(i) Gift on the occasion of marriage from friends ` 70,000.
(ii) Gift on the occasion of birthday from friends ` 55,000.
(iii) Gift from maternal uncle on birthday ` 35,000.
(iv) Gift of motor car by grandfather’s younger brother. Fair market value
of the car on the date of gift ` 3,50,000.
Compute the amount of gifts includible in the total income of Mr. Vidyasagar
for the financial year 2018-19. (3 marks)
Answer:
(i) Gift on the occasion of marriage from friends of ` 70,000 is exempt
from tax.
(ii) Gift on the occasion of birthday from friends of ` 55,000 is treated as
income from other sources u/s 56(2)(vii) because the amount of gift is
more than ` 50,000.
(iii) Gift from Material Uncle on birthday of ` 35,000 is exempt because the
amount of gift is less than ` 50,000.
(iv) Gift received from relatives are not includible in total income.
Therefore, gift of Motor Car by grandfather’s younger brother is not
includible in total income of Mr. Vidyasagar.
Space to write important points for revision

2018 - June [7] (a) Discuss the taxability or otherwise in the hands of the
recipients:
(i) PQR Private Limited issued 15,000 shares at ` 150 per share (face
value ` 100 per share). The fair market value of the share is ` 130 per share.
(ii) Mr. Sakshitha received a sum of ` 92,000 being proceeds at the time
of maturity of a life insurance policy (taken 5 years back) and
` 1,10,000 being proceeds of maturity value of a Key-man insurance policy.
(iv) Rashmi received a cell phone worth ` 60,000 as gift from her friend on
the occasion of her birthday.
(v) On the occasion of her marriage Tripti received cash gifts of
` 1,30,000, which includes ` 60,000 from her friends.
(2 marks each)
[Chapter  9] Income from other Sources O 7.229

Answer:
(i) Section 56 (2) (viib) provides that the following shall be taxable
under the head Income from other sources:
Where a company, not being a company in which the public are
substantially interested, receives in any previous year, from any
person being a resident, any consideration for issue of shares that
exceeds the face value of such shares, the agreegate consideration
received for such shares as exceeds the fair market value of the
shares.
Therefore, income from other sources in the hands of PQR Pvt. Ltd.
will be:
15,000 shares × (` 150 – ` 130)
= ` 3,00,000
(ii) proceeds of maturity of a life insurance policy ` 92,000 is exempt in
the hands of Mr. Sakshitha and proceeds of Maturity value of a key-
man insurance policy is also exempt in the hands of Mr. Sakshitha.
(iv) Sec. 56(2)(x)(c): Gift Received by any person any property other than
immovable property without consideration, the Agreegate fair market
value of which exceeds ` 50,000, the whole of the agreegate fair
market value of such property is taxable under the head Income from
other sources. But cell phone is not included in the definition of
property.
Therefore, Rashmi Received a cell phone worth ` 60,000 as gift from
her friend on the occasion of her birthday is not taxable under the
head Income from other sources.
(v) Gift Received on the occasion of the marriage of the individual is not
taxable under the head Income from other sources. Therefore, on the
occasion of her marriage Tripti received cash gifts of ` 1,30,000 is
exempt from tax.
Space to write important points for revision
7.230 O Scanner CMA Inter Gr. I Paper 7 (2016 Syllabus)

2018 - Dec [7] (a) Explain with reasons, the taxability of the following
transactions under the head “Income from other sources”:
(i) Veena received interest of ` 5,00,000 on additional compensation on
account of compulsory acquisition of land acquired few years back.
Year-wise break up of interest received:
` 1,20,000 for the Financial Year 2016-17, ` 2,40,000 for the Financial
Year 2017 -18 and ` 1,40,000 for the financial Year 2018-19.
(3 marks)
(ii) Gopal has shareholding (with voting rights) of 12% in Krishna Pvt. Ltd.,
a closely held company. He received loan of ` 2,50,000 from the
company on 1st May, 2018, for which he furnished adequate security
to the company. The accumulated profit of the company at that time
was ` 1,75,000. Gopal repaid the loan on 30th Sept., 2018.
(3 marks)
(iii) Family pension of ` 60,000 received by Sreelekha, window of Late
Vikram. (1 marks)
(iv) Vasant, whose salary income is ` 4,00,000 has received a cash gift
of ` 60,000 from a charitable trust registered under section 12AA for
meeting his medical expenses. (1marks)

Table Showing Marks of Compulsory Questions

Year 14 14 15 15 16 16 17 17 18 18
J D J D J D J D J D

Descriptive 1

Total 1
10 CLUBBING OF INCOME
THIS CHAPTER INCLUDES
 Provisions Related to Clubbing  Income from Assets
of Income Transferred To: Spouse, Minor
 Income of Spouse Child, Son’s Wife
 Income of Minor Child
Marks of Objective, Short Notes, Distinguish Between, Descriptive & Practical
Questions
Legend
Objective Short Notes Distinguish Descriptive Practical

For detailed analysis Login at www.scannerclasses.com


for registration and password see first page of this book.

7.231
7.232 O Scanner CMA Inter Gr. I Paper 7 (2016 Syllabus)

DESCRIPTIVE QUESTIONS
2015 - Dec [2] (d) Explain the tax incidence in the case of a transfer of a let
out property, which is not revocable during the life time of the transferee.
(3 marks)
Answer:
Section 62 of Income Tax Act, 1961 states that any asset transfer to the
transferee:
(i) By way of trust which is not revocable during the life time of the
beneficiary and in the case of any other transfer, which is not
revocable during the life time of the transferee; or
(ii) Made before the Ist day of April 1961 which is not revocable for a
period exceeding 6 years;
Any income from such transfer shall be chargeable to tax as the income of
the transferor as and when the power to revoke the transfer arises, and shall
then be included in his total income.
Space to write important points for revision

PRACTICAL QUESTIONS
2008 - Dec [7] (c) Mr. Daga is a trader. Particulars of his income and those
of the members of his family are given below (These incomes relate to the
year ended 31st March, 2019) :
`
(i) Income from Mr. Daga's business 90,000
(ii) Salary derived from an educational institution by
Mrs. Daga ; she is the principal of the institution 50,000
(iii) Interest on company deposits derived Master Deep Daga
(Minor son). These deposits were made in the name of
Deep Daga by his father's father about 6 years ago 12,000
(iv) Receipts from sale of paintings and drawings made
by minor Dipali Daga (Minor Daughter of Mr. and
Mrs. Daga and a noted child artist) 60,000
[Chapter  10] Clubbing of Income O 7.233

(v) Income by way of lottery earning by Master


Dipendar Daga (Minor son of Mr. and Mrs. Daga) 26,000
Discuss whether the above will form part of the total income of any
individual, duly considering applicability of clubbing provisions in each case,
and also compute the total income of Mr. Daga. (7 marks)
Answer:
(i) Mr. Daga’s income from business is assessable in his individual
hands.
(ii) Mrs. Daga’s salary is assessable in her individual hands. The clubbing
provisions do not apply to salary income unless it is received from a
concern in which the spouse has substantial interest.
(iii) The income of minor sons Deep and Dipender will be clubbed with
income of the parent whose total income before clubbing is greater. Mr
Daga’s income (` 90,000) is greater than the total income of Mrs.
Daga (` 50,000). Hence the income of minor children will be clubbed
in the hands of Mr. Daga. Sec. 64 (1A)
(iv) The income of minor daughter Dipali from painting is chargeable in her
individual hands. The clubbing provisions do not apply to minor child’s
income which is earned by her from manual work or activity involving
application of specialized knowledge, talent, skill or experience. Sec.
64 (1A)
Computation of Total Income of Mr. Daga
For the Assessment Year 2019 – 20
` ` `
Profits and gains of business or Profession 90,000
Income from other sources:
Interest on company deposit derived by
Master Deep
Daga clubbed under Section 64(1A) 12,000
Less: Exemption under Section 10(32) 1,500 10,500
Lottery earning of Master Dipender 26,000
Less: Exemption under Section 10(32) 1,500 24,500 35,000
Gross Total Income/ Total Income 1,25,000
Space to write important points for revision
7.234 O Scanner CMA Inter Gr. I Paper 7 (2016 Syllabus)

2009 - June [6] (a) The following details of income of Mr. X and his wife, for
the assessment year 2019-20 are made available to you:
Mr. X Mrs. X
` `
Income from own business/profession 1,20,000 90,000
Income from other sources 2,10,000 1,10,000
Interest received from Z & Co. 20,000 4,10,000
Salary received from Z & Co. 96,000 84,000
Mr. X and Mrs. X are partners in Z & Co. each having 10% share in profits.
Determine the total income of Mr. X and Mrs. X.
Will your answer be different, (a) If each one of them hold 8% shares in
profits of Z & Co.? (b) If Mr. X and Mrs. X both possess professional
qualifications. (11 marks)
Answer:
Salary arising to spouse of the individual from a concern in which an
individual has substantial interest will be clubbed u/s 64(1).
An individual is deemed to have substantial interest in a partnership firm,
where the individual himself and together with his relatives is entitled to 20%
share in the profits.
In this case, this stands satisfied. So clubbing provisions will be attracted.
Where both husband and wife derive salary from the firm, to determine
in whose hands the income of the other has to be clubbed, the total income
excluding the salary has to be computed.
Clubbing has to be made in the hands of that person whose such income
is higher.
Hence, ignoring salary, the income of each individual is to be computed
as:
Income of Mr. X = 1,20,000 + 2,10,000 + 20,000 = 3,50,000
Income of Mrs. X = 90,000 + 1,10,000 + 4,10,000 = 6,10,000
So, in the hands of Mrs. X., salary of Mr. X will be clubbed.
Now the total income will be –
Mr. X Mrs. X
Own Business income 1,20,000 90,000
Business: Interest from Z & Co., 20,000 4,10,000
Remuneration from of Mrs. X 84,000
[Chapter  10] Clubbing of Income O 7.235

Income from other sources: 2,10,000 1,10,000


As given
Remuneration of Mr X clubbed u/s. 64(1) 96,000
Gross total income/Total Income 3,50,000 7,90,000
If the total share profits in Z & Co. of Mr. X and Mrs. X are less than 20%,
clubbing provisions will not apply. Remuneration from Z & Co. will be
assessed in their respective hands. Clubbing provisions are also not
attracted when the individual’s spouse possesses technical or professional
qualification and the salary is on account of such qualification.
Space to write important points for revision

2009 - Dec [7] (a) C is the minor child of Mr. and Mrs. X, Mr. X has business
income of ` 2,80,000. Mrs. X has salary income of ` 2,20,000. C earns
income of ` 60,000 from a dancing programme. C also earns interest of
` 20,000 from a fixed deposit with a Bank. Compute total income of Mr. X,
Mrs. X and C. (3 marks)
Answer:
The income of a minor child is to be clubbed in the hands of either of his
parents. The income shall be clubbed in the hands of that parent whose total
income (excluding the income of the minor) is greater.
Where the income of a minor child has been included in the total income
of a parent, such parent shall be entitled to an exemption to the extent of
such income or ` 1,500 whichever is less, in respect of each minor child
whose income is so included.
Income which accrues or arises to the minor child on account of any
manual work done by him, such income is taxable in the hands of minor child
only.
Computation of total income of Mr. X
Business Income 2,80,000
Interest income under clubbing provision 20,000
Less: Deduction 1,500 18,500
Total taxable income 2,98,500
7.236 O Scanner CMA Inter Gr. I Paper 7 (2016 Syllabus)

Computation of total income of Mrs. X


Income from salary 2,20,000
Other income _____Nil
Total income 2,20,000
Computation of total income of C
Income from a dancing programme 60,000
Total income 60,000
Space to write important points for revision

2010 - Dec [8] Answer the following:


(c) Mrs. Sukanya is a qualified cost accountant. She is a salaried employee
in a firm of cost accountants in which Mr. Ashok (her husband) is a
partner. Mr. Ashok’s share in the firm is 10%. His younger brother holds
10% share in this firm. Mrs. Sukanya draws a salary of ` 18,000 per
month from the firm. This is however paid in kind and not in cash. Mr.
Ashok’s income by way of sitting fees from the various boards of the
companies in which he is an independent director is ` 3,50,000. Will Mrs.
Sukanya’s income be clubbed with that of Mr. Ashok under Section 64
of the Income-tax Act, 1961? (5 marks)
Answer:
Clubbing of Income of Spouse:
Section 64 (1)(ii) provides for clubbing of income arising to spouse of
individual by way of salary, fee or remuneration, whether in cash or kind,
from a concern in which such individual has a substantial interest.
As per explanation 2 to Sec. 64(1), for the purpose of clause (ii) an
individual shall be deemed to have a substantial interest in a partnership
concern, if such person is entitled, or such person and one or more of his
relatives are entitled in the aggregate, at any time during the previous year,
to not less than twenty per cent of the profits of such concern. Here, Mr.
Ashok and his brother hold 20% share in the firm and hence prima facie, the
provisions of Section 64(1)(ii) apply
[Chapter  10] Clubbing of Income O 7.237

As per explanation 1 to Sec. 64(1), for the purpose of clause (ii) an


individual in computing whose total income referred to in that clause is to be
included, shall be the husband or wife whose total income (excluding the
income referred to in that clause) is greater.
However, the proviso to Section 64(1)(ii) says that such the provision
of Section 64(1)(ii) will not apply to any income earned by spouse if the
spouse possesses technical qualifications and the income is solely
attributable to the application of his or her technical knowledge or
professional knowledge or experience.
Therefore, the case appears to fall squarely within the aforesaid proviso
and the clubbing provisions of Section 64(1)(ii) cannot be invoked to club
the salary income of Mrs. Sukanya in Mr. Ashok’s hands.
Space to write important points for revision

2011 - June [6] (a) Mr. Rajiv commenced business with a capital of ` 2 lakhs
in the financial year 2016-17. His capital as on 1.4.2017 was ` 5 lakhs. His
wife gifted ` 1 lakh on 10.04.2017 which was also invested in the business.
His Net profit for the year 2017-18 = ` 2 lakhs
His Net profit for the year 2018-19 = ` 4 lakhs
Compute the income from business to be clubbed in the hands of Mrs.
Rajiv and the income from business taxable in the hands of Mr. Rajiv for the
assessment year 2019-20.
Mr. Rajiv did not withdraw any money from the business from 01.04.2018 to
31.03.2019. (6 marks)
Answer:
Particulars Rajiv’s Capital Total
Capital contribu- `
Contribu- tion out
tion of gift
from wife
Capital as on 01-04-2017 5,00,000 Nil 5,00,000
Investment as on 10-04-2017 out of gift
from his wife Nil 1,00,000 1,00,000
7.238 O Scanner CMA Inter Gr. I Paper 7 (2016 Syllabus)

Total capital invested till 31-3-2018 5,00,000 1,00,000 6,00,000


Profit for the F.Y. 2016-18 2,00,000 Nil 2,00,000
Capital employed on 1-04-2018 7,00,000 1,00,000 8,00,000
Profit for the F.Y. 2018-19 to be
apportioned on the basis of capital
employed on 01-04-2018 (i.e. 7:1 ratio) 3,50,000 50,000 4,00,000
Hence, the income assessable in the hands of Mr. Rajiv for A.Y.
2019-20 is ` 3,50,000 and the income assessable in the hands of Mrs.
Rajiv for A.Y. 2019-20 would be ` 50,000.
Space to write important points for revision

2011 - Dec [8] (b) On 21.3.2018, Mr. PIYUSH gifted to his wife Mrs.
PRARTHANA 200 listed shares, which had been bought by him on
19.4.2017 at ` 2,000 per share. On 1.6.2018 bonus shares were allotted in
the ratio of 1:1. All these shares were sold by Mrs. PRARTHANA as under.
Date of sale Manner of sale No. of Net sales
shares value (`)
21.5.2018 Sold in recognized stock exchange, 100 2,20,000
STT paid
21.7.2018 Private sale to an outsider All bonus 1,25,000
shares
28.2.2019 Private sale to her friend Mrs. NIKITA 100 1,70,000
(Market value on this date was ` 2,10,000)
Briefly state the income-tax consequences in respect of the sale of the
shares by Mrs. PRARTHANA showing clearly the person in whose hands the
same is chargeable, the quantum and the head of income in respect of the
above transactions. Detailed computation of total income is NOT required.
Net sales value represents the amount credited after all taxes, levies,
brokerage, etc., and the same may be adopted for computing the capital gains.
Cost inflation index for the FY 2017-18 is 272 and for the FY 2018-19 is 280.
(7 marks)
[Chapter  10] Clubbing of Income O 7.239

Answer:
Where an asset has been transferred by an individual to his spouse
otherwise than for adequate consideration, the income arising from the sale
of the said asset by the spouse will be clubbed in the hands of the individual.
Where there is any accretion to the asset transferred, income arising to
the transferee from such accretion will not be clubbed.
Therefore, the profit from sale of bonus shares allotted to Mrs.
PRARTHANA will be chargeable to tax in the hands of Mrs. PRARTHANA.
Therefore, the capital gains arising from the sale of the original shares
has to be included in the hands of Mr. Piyush.
The capital gains arising from the sale of bonus shares would be taxable
in the hands of Mrs. PRARTHANA.
Where an asset received by way of gift has been sold, the period of
holding of the previous owner should be considered. The cost to the previous
owner has to be taken as the cost of acquisition.
Income/loss to be clubbed in the hands of Mr. PIYUSH
Long-term capital gains/loss
(i) 100 shares sold on 21.5.2018 in a recognized stock exchange, STT
paid. Long-term capital gains on sale of such shares is exempt under
Section10(38)
(ii) Shares sold to a friend on 28.2.2019
Sale consideration 1,70,000
Less: Indexed cost of acquisition of 100 shares
(` 2,000 × 100 ×280 /272) 2,05,882
Long term capital loss to be included in the hands
of Mr. PIYUSH (35,882)
Income taxable in the hands of Mrs. PRARTHANA
Short-term capital gains (on sale of 100 bonus shares)
`
Sale consideration 1,25,000
Less: Cost of acquisition of bonus shares Nil
Short-term capital gains 1,25,000
Space to write important points for revision
7.240 O Scanner CMA Inter Gr. I Paper 7 (2016 Syllabus)

2012 - Dec [3] (a) On 01.05.2018, Mr. Rama transferred the right to receive
rental income arising from a factory godown owned by him, to his major son
Mr. Lava, for a period of 10 years. The rental income derived is ` 10,000 per
month.
On 12.03.2014, he gifted 2000 shares of face value of ` 100 each, in Janak
Granites Ltd., a listed company, to his wife Mrs. Seetha. Mr. Rama had
purchased them on 19.02.2012 at ` 110 each.
Janak Granites Ltd. allotted bonus shares in the ratio of 1:1 on 12.04.2015.
Mrs. Seetha sold all shares of the above company on 15.01.2019 in the
National Stock Exchange for a net consideration of ` 180 per share, paying
the applicable STT thereon.
Discuss how the above items will be treated in the hands of Mr. Rama and
Mrs. Seetha for the assessment year 2019-20 (Computation of income is not
required). (5 marks)
Answer:
As per Section 60, in case of transfer of income, whether recoverable or not,
without transfer of assets from the income arises, such transferred income
shall be included in total income of the transferor, not the transferee. Hence,
in this case the rental income from the godown will be assessed in the hands
of Mr. Rama.
As per Sec. 64(1)(iv), any income arising from assets transferred without
adequate consideration to the spouse shall be included in the total income
of the transferor –individual. Where such transferred asset is subsequently
sold by the spouse, the resultant tax will have to be considered in the hands
of the transferor only i.e., Mr. Rama.
Thus, capital gains on transfer of the original 2000 shares will have to be
considered in the hands of Mr. Rama. Since the shares are long-term in
nature and are listed shares sold in recognized stock exchange, STT also
being paid, the ensuing capital gain is exempt under Section 10(38).
However, accretion from the transferred asset is not clubbed in the hands of
Mrs. Rama. Bonus shares are accretions and hence capital gains arising on
transfer of the bonus shares will be assessed in the hands of Mrs. Seetha.
Thus, capital gain on transfer of bonus shares will be taxed in the hands of
M Seetha. The bonus shares have been held for less than12 months and
hence is a short-term capital asset. The cost of acquisition of bonus share
[Chapter  10] Clubbing of Income O 7.241

is taken as nil. The profit from sale of the bonus shares being the net sale
proceeds will be assessed as short-term capital gains and may be
chargeable to tax at 15%.
Space to write important points for revision

2012 - Dec [4] (c) Mr. Ashwin started a proprietary business on 20.04.2018
with a capital of ` 5,50,000. His wife Smt. Padma gifted ` 2,00,000 on the
occasion of his birthday on 28.07.2017, out of which he introduced `1,00,000
into his proprietary business.
Details of his income from business are given below :
Financial year (Loss) Income
2017-18 ` (1,50,000)
2018-19 ` 4,00,000
He did not withdraw any amount from the business for his personal use.
Determine the amount chargeable to tax in the hands of Ashwin and the
amount liable for clubbing in the hands of his wife Smt. Padma. (5 marks)
Answer:
Computation of Income from business chargeable in the hands of Mr.
Ashwin and clubbing in the hands of Smt. Padma for the Previous Year
2017-18. A.Y. 2018-19
`

Profit/ loss (1,50,000)

Gifted amount considering the investing part in the business 1,00,000

Capital 5,50,000

Clubbed amount Nil

Income chargeable in the hands of Ashwin Nil


Computation of Income from business chargeable in the hands of Mr.
Ashwin and clubbing in the hands of Smt. Padma for the Previous Year
2018-19. A.Y. 2019-20
7.242 O Scanner CMA Inter Gr. I Paper 7 (2016 Syllabus)

`
Profit/ loss 4,00,000

Gifted amount considering the investing part in the business ---

Capital 5,00,000

Clubbed amount 80,000

(Previous year capital + Gifted amount - loss of business)


(i.e. 4,00,000 x 1,00,000 ÷ 5,00,000)

Income chargeable in the hands of Ashwin 3,20,000

(Profit earned – clubbed amount = 4,00,000- 80,000)

Hence, Clubbed amount in the hands of Padma = ` 80,000 and


Income chargeable in the hands of Ashwin = `3,20,000

Note:
Based on cases-R. Ganesan vs. CIT [1965] 58 ITR 411 (Mad.). Asset
acquired by transferee out of gift from spouse is also covered. Smt.
Mohini Thapar vs. CIT [1972] 83 ITR 208 (SC ) Indirect transfers are
also covered.
i. The amount of profit to the extent of gifted to total capital on the first
day of the previous year must be clubbed in the hands of Smt.
Padma.
ii. The gift was made on 28.07.2017 therefore, the clubbing provisions
shall not apply as the gift was made after the 1st day of the previous
year.
iii. As per question, Ashwin has not withdrawn any amount for his
personal use. So, closing capital of 2017-18 plus profit for that year
is taken as the capital for Financial Year 2018-19.
Space to write important points for revision
[Chapter  10] Clubbing of Income O 7.243

2013 - June [8] (c) A, a mentally retarded minor has a total income of
` 2,40,000 for the assessment year 2019-20. The total income of his father
B and of his mother C for the relevant assessment year is ` 4,00,000 and
` 3,00,000 respectively. Discuss the treatment to be accorded to the total
income of A for the relevant assessment year. (3 marks)
Answer:
All income accruing or arising to a minor child has to be included in the
income of that parent whose total income is greater. However the income of
a minor child suffering from any disability of the nature specified in Section
80U shall not be included in the income of the parents but shall be assessed
in the hands of the child. (Sec. 64)
Thus the total income of A had to be assessed in his hands and cannot be
included in the total income of either his father or his mother.
Space to write important points for revision

2013 - Dec [6] (b) Manoj gifted ` 3,00,000 to his wife on 01.07.2018, which
she invested in her beauty parlour business. The capital of Mrs. Manoj as on
01.04.2018 was ` 6,00,000. The profit for the year ended 31.03.2019
(computed) from business amounts to ` 2,40,000.
The total income of Manoj (before clubbing) who is employed in a company
amounts to ` 4,50,000 (computed).
Determine the income liable for clubbing in the hands of Mr. Manoj out of the
incomes earned by Mrs. Manoj during the financial year 2018-19.
(3 marks)
Answer:
The gift was given by Manoj to his wife on 01.07.2018. Reference to
Explanation 3 to Section 64 must be made. As the opening capital as on
01.04.2018 belongs to Mrs. Manoj, Explanation 3 to Section 64 cannot be
invoked for the financial year 2018-19.
The opening capital as on 01.04.2018 wholly belonged to Mrs. Manoj and
therefore in spite of the gift and its investment in business, no portion of such
income would be liable for clubbing.
The total income of Mr. Manoj will not change in any manner by applying the
clubbing provisions.
Space to write important points for revision
7.244 O Scanner CMA Inter Gr. I Paper 7 (2016 Syllabus)

2014 - Dec [5] (c) Mr. Vishal gifted a sum of ` 3 lacs to Miss Mrinal on
01.04.2018. Miss Mrinal got married to Mr. Vishal’s son on 01.06.2018.
Mrinal earned an interest of ` 22,000 from this gifted amount, for the year
ended 31.03.2019. Can the interest income of ` 22,000 be clubbed in the
hands of Mr. Vishal? (4 marks)
Answer:
Clubbing of income u/s 64
In computing the total income of an individual, the income arising from assets
transferred by an individual, directly or indirectly, otherwise than for adequate
consideration, to the son’s wife, will be clubbed u/s 64(1).
As on the date of such transfer for inadequate consideration (i.e., gift), the
relationship of father-in-law/daughter-in-law should exist.
In the instant case, Mr. Vishal gave the gift, will before the marriage of Mrinal
with Mr. Vishal’s son.
Hence, the interest income of ` 22,000 earned from such gifted amount
cannot be clubbed in the hands of Mr. Vishal.
Space to write important points for revision

2015 - Dec [1] {C} Answer the following question with brief reason/working:
(h) Interest of ` 15,000 on bank fixed deposits is received by the minor son
of Ms. Santi. These fixed deposits were made by Ms. Santi out of her
son’s earnings from stage acting. What is the tax treatment of interest?
(2 marks)
Answer:
The interest received `15,000 will be taxed in hands of Mrs. Shanti.
The income of minor son of Mrs. Shanti is chargeable under her individual
hands. The clubbing provision do not apply to minor child’s income which is
earned by his specialized skills but interest received is not the income
earned by his manual work or activity involving application of specialized
knowledge, talent, skill or experience.
Space to write important points for revision
[Chapter  10] Clubbing of Income O 7.245

2015 - Dec [6] (b) Mr. Vignesh and his wife Smt. Buddhi furnish the following
information for the year ended 31.03.2019:
Particulars `

(i) Salary income (computed) of Smt. Buddhi 5,50,000

(ii) Income of minor son Brijesh who suffers from disability 1,50,000
specified in Section 80U

(iii) Income of minor daughter Chitra from singing 85,000

(iv) Income from business (computed) of Mr. Vignesh 4,00,000

(v) Rental income from property earned by Smt. Buddhi


during the year ` 4,80,000. The property was gifted by
Vignesh 3 years ago out of love and affection.

(vi) Income of minor daughter Chitra from company deposit. 20,000

Compute the total income of Mr. Vignesh and Smt. Buddhi for the Assess-
ment Year 2019-20. (5 marks)
Answer:
Computation of Total Income of Mr. Vignesh and Mrs. Buddhi for
assessment year 2019-20.
Income of Income of
Mr. Vignesh Mrs. Buddhi
(i) Salary income of Mrs. Buddhi 5,50,000
(ii) Income of minor son Brijesh, who suffer
from disability specified in Section 80U — —
(iii) Income of minor daughter Chitra from
singing — —
(iv) Income from business of Mr. Vignesh 4,00,000
7.246 O Scanner CMA Inter Gr. I Paper 7 (2016 Syllabus)

(v) Rental income from property earned by


Smt. Buddhi during the year ` 4,80,000.
The property was gifted by Vignesh 3
year ago out of love and affection. 4,80,000
(vi) Income of minor daughter Chitra from
company deposit. 20,000
9,00,000 5,50,000

Notes:
(i) All income occurring or arising to a minor child has to be included in
the income of that parent whose total income is greater. However, the
income of a minor child suffering from any disability of the nature
specified in Sec. 80U shall not be included in the income of the
parents but shall be taxed in the hands of the child.
Thus income of Brijesh (who is suffering from disability specified in
Section 80U) will be taxable in his own hands.
(ii) Income of Chitra (minor child of Mr. Vignesh and Smt. Buddhi) will be
assessed in her own hands because it is income earned by minor
child by her specialized skill and talent.
(iii) Income from business of Mr. Vignesh will be taxable in Mr. Vignesh
hand under head “Profit and gain from business”.
(iv) As per Sec. 64(1)(iv) any income arising from asset transferred
without adequate consideration to the spouse shall be included in the
total income of the transferor.
Thus the income from house property will be clubbed in Mr. Vignesh’s
income.
(v) Income of minor daughter Chitra from company deposit will be
clubbed in total income of Mr. Vignesh.

Space to write important points for revision


[Chapter  10] Clubbing of Income O 7.247

2017 - Dec [5] (a) State the taxability of the following transactions for the
assessment year 2019-20:
(iv) Ms. Jency got gift of 500 listed equity shares of a company from her
husband when the market value of the share was ` 150 per share.
After a month, the company issued bonus shares in 1:1 ratio. The
original shares were acquired by her husband 4 months before the
date of gift for ` 50,000. All the 1000 shares were sold for ` 1,50,000
through off-market transaction. How much is taxable and in whose
hands it is taxable as income? (2 marks)
Answer:
Short-term capital gain arising from sale of original shares gifted i.e.,
` 25,000 (` 75,000 - ` 50,000) shall be taxed in the hands of husband of Ms.
Jency under section 64.
Capital gain attributable to bonus shares will not be liable for clubbing under
section 64 since it is an accretion to the original shares. Therefore,` 75,000
being the sale consideration from sale of bonus shares whose cost of
acquisition is Nil is taxable in the hands of Ms. Jency as short-term capital
gain.
Space to write important points for revision

Table Showing Marks of Compulsory Questions


Year 14 14 15 15 16 16 17 17 18 18
J D J D J D J D J D
Practical 2
Total 2
11 SET OFF AND CARRY
FORWARD OF LOSSES
THIS CHAPTER INCLUDES
 Inter-source adjustments  Carry Forward and Set Off of
 Inter-head adjustments Loss in Subsequent Year
 Set Off of Loss in the Same  Submission of Return of Loss
Year
Marks of Objective, Short Notes, Distinguish Between, Descriptive & Practical Questions
Legend
Objective Short Notes Distinguish Descriptive Practical

For detailed analysis Login at www.scannerclasses.com


for registration and password see first page of this book.

7.248
[Chapter  11] Set Off and Carry Forward of Losses O 7.249

SHORT NOTES

2010 - June [7] Write a short note on the following:


(b) Restriction under Section 79 of the Income Tax Act, 1961 for carry
forward and set off of losses in case of certain companies; (5 marks)
Answer:
Carry forward and set off of losses in case of certain companies
[Section 79]
Where in any previous year there has been a change in the shareholding of
company (other than the one in which the public are substantially interested)
any unabsorbed loss shall be allowed to be carried forward and set off
against the income of the previous year only if the beneficial shareholder of
at least 51% of the voting power on the last day of previous year remained
the same as on the last day of the year or year in which the loss was incurred.
However, this restriction shall not apply in the following two cases:
(i) Where change in the voting power is consequent upon death of a
shareholder or on account of transfer of shares by way of gift by a
shareholder to his relative; and
(ii) Where the change in shareholding takes place in an Indian company,
being subsidiary of a foreign company, as result of amalgamation or
demerger of the foreign company. However, this is subject to the
condition that 51% of the shareholders of the amalgamating/
demerged company continue to be shareholders of the
amalgamated/resulting company.
(iii) Where a change in the shareholding takes place in a previous year
pursuant to a resolution plan approved under the Insolvency and
Bankruptcy Code, 2016, after affording a reasonable opportunity of
being head to the jurisdictional principal Commissioner or
Commissioner.
Space to write important points for revision
7.250 O Scanner CMA Inter Gr. I Paper 7 (2016 Syllabus)

DISTINGUISH BETWEEN

2011 - June [3] (d) Distinguish between inter-source adjustment and inter-
head adjustment. (2 marks)
Answer:
Inter source adjustments and inter-head adjustments.
Inter source adjustments [Section 70] - Loss arising from one source of
income under a head can be adjusted against income arising from other
source under the same head, except in the following.
• LTC loss can be adjusted against LTC gains;
• Speculation business loss can be adjusted only against speculation
business income;
• Loss from a source, the income from which is exempt, cannot be
adjusted against any income;
• Loss from lottery, card gambling, betting, etc. cannot be adjusted against
any income.
Inter head adjustment [Section 71]: Loss arising from one head of income
can be adjusted against income under any other head, except in the
following:
• Loss arising under the head capital gain cannot be adjusted from income
under any other head
• Losses under the head “Profit and Gain of business or profession”
cannot be adjusted against income under the head ‘Salaries’.
Space to write important points for revision

DESCRIPTIVE QUESTIONS
2012 - June [7] (a) Explain the tax treatment for set off and carry forward of
loss from house property. (5 marks)
[Chapter  11] Set Off and Carry Forward of Losses O 7.251

Answer:
Carry forwarded set off of loss from house property [Section 71B]:
Loss under house property, if could not be set off or was not wholly set off
in the same assessment year from any other heads of income in accordance
of Section 71, will be allowed to be carry forward for 8 assessment years to
claim it as a set off in the subsequent year under the head “ Income from
House Property” e.g. If the loss of house property of the previous year
2017-18 which could not be set off, because of absence or inadequacy of
income of previous year 2017-18 can be carried forwarded for 8 assessment
year starting from the AY succeeding the AY 2018-19.

Note:
1. Loss under the head House Property can be carried forwarded even
if belated return is filed i.e. not filed within the due date of Section
139(1).
2. Although loss is allowed to be carry forward for 8 AYs, If income from
house property is available in any year, set off of the loss must be
availed in subsequent year and balance in the next succeeding
subsequent years and so on.
Space to write important points for revision

2012 - Dec [8] (b) Explain the provisions for carry forward and set off of
business losses under Section 72. Explain the order of priority amongst
business loss, current depreciation and brought forward unabsorbed
depreciation. (6 marks)
Answer:
The right of carry forward and set off of loss arising in a business or
profession is subject to the following restrictions:
1. Loss can be set off only against Business Income : A loss to the
assessee under the head “Profits and gains of business or profession”,
and such loss cannot be or is not wholly set off against income under
any head of income and he has no income under any other head, the
whole loss shall be carried forward to the following assessment year,
and:
7.252 O Scanner CMA Inter Gr. I Paper 7 (2016 Syllabus)

(i) it shall be set off against the profits and gains, if any, of any
business or profession carried on by him ;
(ii) if the loss cannot be wholly so set off, the amount of loss not so set
off shall be carried forward to 4the following assessment year and
so on :
2. Loss can be carried forward for 8 Years: No loss shall be carried
forward under this Section for more than eight assessment years
immediately succeeding the assessment year for which the loss was first
computed.
3. Return of Loss should be submitted in Time : A Loss cannot be
carried forward unless it is determined in pursuance of a Return Filed
within the time allowed:
(i) Current year depreciation u/s 32
(ii) Brought forward business loss
(iii) Brought forward unabsorbed depreciation
(iv) Priority of set off.
Space to write important points for revision

2014 - June [3] (c) What is reverse merger? Explain the reasons for
resorting to the same by the assessees. (3 marks)
Answer:
A reverse merger refers to an arrangement where private company acquires
a public company, usually a shell company, in order to acquire the status of
a public company. Also known as a reverse takeover, it is an alternative to
the traditional initial public offering (IPO) method of floating a public
company. It is an easier way that allows private companies to change their
type while avoiding the complex regulations and formalities associated with
an IPO. Also, the degree of ownership and control of the private
stakeholders increases in the public company. It also leads to combining of
resources thereby giving greater liquidity to the private company.
Space to write important points for revision

2015 - June [5] (b) Explain the provisions relating to carry forward and set
off of losses by closely held companies. (3 marks)
[Chapter  11] Set Off and Carry Forward of Losses O 7.253

Answer:
Losses of closely-held companies:
Section 79 deals with carry forward and set off of business loss in the case
of a company in which public are not substantially interested.
Where on the last day of the previous year in which the loss is sought to be
carried forward and set off, the shares of the company carrying not less than
51% of the voting power must continue to be held by the persons who held
51% of the total voting power on the last day in which the loss was incurred.
This restriction will not apply if the change in shareholding takes place due
to death of a shareholder or transfer of shares is by way of gift by a
shareholder to his relative.
Also, it is not applicable where there is a change in shareholding of an Indian
company which is a subsidiary of a foreign company as a result of
amalgamation or demerger of a foreign company, provided 51% of the
shareholders of the amalgamating or demerged foreign company continue
to be shareholders of the amalgamated or the resulting foreign company.
Space to write important points for revision

PRACTICAL QUESTIONS

2009 - Dec [3] (c) From the following information compute the total income
of Mr. Ramesh for the assessment year 2019-20:
`
Income from salary 2,60,000
Income from House Property 1,00,000
Business loss (non speculative) 3,20,000
Short-term capital gain 1,40,000
Long-term capital gain 2,80,000 (5 marks)
Answer:
Computation of Total Income of Mr. Ramesh for Assessment year 2019-20.
Income from salary 2,60,000
Income from House property 1,00,000
7.254 O Scanner CMA Inter Gr. I Paper 7 (2016 Syllabus)

Business loss (3,20,000)


Short term Capital gain 1,40,000
Long term capital gain 2,80,000
Total income 4,60,000

Note: Business loss cannot be set off against income under salary.
Therefore, the business loss of ` 3,20,000 will be adjusted from house
property (1,00,000) and long term capital gain (2,20,000).
Space to write important points for revision

2010 - June [3] (a) During the year ended 31.3.2019, Mr. Subramani has
following income and the brought forward losses:
Particulars `
Short-term capital gain on sale of shares 2,60,000
Long-term capital loss of A.Y. 2017-18 90,000
Short-term capital loss of A.Y. 2018-19 80,000
Long-term capital gain 78,000
Income from lotteries 3,10,000
Cost of lottery tickets purchased 2,000
Loss from betting 1,20,000
Income from card games 80,000
Briefly compute the gross total income and loss eligible for carry forward
in the hands of Mr. Subramani for the A.Y. 2019-20. (7 marks)
Answer:
Particulars ` `
Capital Gains
(a) Short-term capital gain on sale of shares 2,60,000
Less: Brought forward short term capital
loss of the A.Y. 2018-19 (80,000) 1,80,000
(b) Long-term capital gain 78,000
Less: Brought forward long term capital
loss of A.Y. 2017-18 [see note 1 below] 78,000 nil
Income chargeable under the head Capital gain 1,80,000
[Chapter  11] Set Off and Carry Forward of Losses O 7.255

Income from other sources


(a) Income from lotteries 3,10,000
(Cost of lottery tickets is not deductible)
(b) Income from card games 80,000 3,90,000
Gross total income 5,70,000
Note:
1. Long-term capital loss cannot be set off against short-term capital
gain. Hence, the remaining unadjusted long term capital loss of A.Y.
2017-18 of ` 12,000 (i.e. ` 90,000 - ` 78,000) has to be carried
forward to the next year to be set-off against long-term capital gain of
that year.
This is subject to the return of income for the current year having
been filed within the time allowed u/s 139(1).
2. Loss from betting can neither be set off against any other income, nor
can it be carried forward to subsequent year.
3. As per Section 58(4), cost of lottery tickets cannot be set off against
income from lotteries.
Space to write important points for revision

2011 - Dec [5] (a) The following are the details relating to Mr. SUNIL a
resident Indian, aged 57, relating to the year ended 31.3.2019 :
`
Income from salaries 2,20,000
Loss from house property 1,90,000
Loss from cloth business 2,40,000
Income from speculation business 30,000
Loss from specified business covered by Section 35 AD 20,000
Long-term capital gains from sale of urban land 2,50,000
Long-term capital loss from sale of listed shares in recognized stock 1,10,000
exchange (STT paid)
Loss from card games 32,000
Income from betting 45,000
Life Insurance Premium paid 1,20,000
Compute the total income and show the items eligible for carry forward.
(8 marks)
7.256 O Scanner CMA Inter Gr. I Paper 7 (2016 Syllabus)

Answer:
Computation of total income of Mr. Sunil for the A. Y. 2019-20
` `
Income from Salaries
Income from salaries 2,20,000
Less: Loss from house property 1,90,000 30,000
Income from Profits and gains of business or
profession
Income from speculation business 30,000
Less: Loss from cloth business set off 30,000 Nil
Income from Capital gains
Long-term capital gains from sale of urban land 2,50,000
Less: Loss from cloth business set off 2,10,000 40,000
Income from other sources
Income from betting 45 ,000
Gross total income 1,15,000
Less: Deduction under Section 80C (life insurance premium paid) 30,000
Total income 85,000
Losses to be carried forward `
(1) Loss from cloth business (2,40,000-30,000 - 2,10,000) Nil
(2) Loss from specified business covered by Section 35AD 20,000

Notes:
(i) Long-term capital gains from sale of listed shares in a recognized
stock exchange is exempt under Section 10(38). Loss from an
exempt source cannot be set off against profits from a taxable
source.
Therefore, long-term capital loss on sale of listed shares cannot be
set-off against long-term capital gains from sale of urban land.
(ii) Loss from specified business covered by Section 35AD can be
set-off only against profits and gains of any other specified business.
Therefore, such loss cannot be set off against any other income. The
unabsorbed loss has to be carried forward for set-off against profits
and gains of any specified business in the following year.
[Chapter  11] Set Off and Carry Forward of Losses O 7.257

(iii) Business loss cannot be set off against salary income. However, the
balance business loss of ` 2, 10,000 (` 2, 40,000 - 30,000 set-off
against income from speculation business) can be set-off against
long term capital gains of ` 2,50,000 from sale of urban land.
Consequently, the taxable long-term capital gains would be ` 40,000.
(iv) Loss from card games can neither be set off against any other
income, nor it can be carried forward.
(v) For providing deduction under Chapter VIA, gross total income has
to be reduced by the amount of long-term capital gains and casual
income. Therefore, the deduction under Section 80C in respect of
life insurance premium paid has to be restricted to ` 30,000 [i.e.,
Gross Total Income of ` 1,15,000 - ` 40,000 (LTCG) - ` 45,000
(Casual income)].
(vi) Income from betting is chargeable at a flat rate of 30% under
Section 115BB and no expenditure or allowance can be allowed as
deduction from such income, nor can any loss be set-off against such
income.
Space to write important points for revision

2013 - Dec [4] (c) Ameet furnishes the following particulars of income/loss
pertaining to previous year 2018-19:
(` in lacs)
(i) Profit from trading business 6
(ii) Loss from manufacturing business 1.50
(iii) Loss from profession 2.50
(iv) Profit from speculation in shares 2.50
(v) Loss from speculation in commodities 3
He has no other income during the year. Determine total income of Ameet
for the Assessment Year 2019-20. Also state the loss to be carried forward.
The manner of set off must be clearly shown in your answer. (5 marks)
7.258 O Scanner CMA Inter Gr. I Paper 7 (2016 Syllabus)

Answer:
Computation of Total Income of Ameet for Assessment Year 2019- 20
Particulars ` in lacs
Profit from trading business 6
Less: Set- off of loss from manufacturing business and loss
from profession under Section 70 4
2
Profit from speculation in shares 2.5
Less: Set- off of loss from speculation in commodities to the
extent possible under Section 73 2.5
Nil
Total Income 2
Unabsorbed loss of ` 0.50 lacs from speculation in commodities can be
carried forward by Ameet for four assessment years.
Space to write important points for revision

2014 - Dec [6] (b) (ii) X Co. Ltd. filed its return for the assessment year 2019-
20 on 10.12.2019, declaring a business loss of ` 12,00,000 and unabsorbed
depreciation of ` 6,00,000. How much of loss and/or depreciation is eligible
for carry forward? (3 marks)
Answer:
The assessee, in order to carry forward business loss, has to file the return
of income before the ‘due date’ prescribed in Section 139(1). Section 80
debars carry forward of business loss to subsequent assessment years
unless the return of income in which it has loss, is filed within due date
specified in Section 139(1).
However, the embargo contained in Section 80 will not apply to carry forward
of depreciation, both current year and that brought forward of preceding years.
In view of the above, X Co. Ltd. cannot carry forward business loss of
` 12,00,000 but it can carry forward unabsorbed depreciation to future years
for set off.
Space to write important points for revision
[Chapter  11] Set Off and Carry Forward of Losses O 7.259

2016 - June [4] (c) Mr. Anurag, an individual engaged in the business,
having turnover of `1.50 crores and no international transaction or specified
domestic transaction incurred loss from business during the previous year
2017-18. Such business loss could not be set off against any other income
during the year. He filed return of loss for Assessment Year 2018-19 on
31st March,2019.
(i) Can Mr. Anurag carry forward such loss for set off against income from
business of the assessment year 2019-20?
(ii) Is there any difference if Mr. Anurag has unabsorbed depreciation
instead of loss from business in the previous year 2017-18 for carry
forward to assessment year 2019-20 for set off? (5 marks)
Answer:
(i) As per Section 80 read with Section 139(3), business loss cannot be
carried forward unless such loss is determined in pursuance of return
filed within the time allowed under section 139(1).
It is essential such return is filled within the due date laid down in
Section 139(1).
In other words, if the assessee fails to file his return of loss on or
before the due date of furnishing of return as prescribed by
Section 139(1) i.e. 30th September, 2018, then the business loss of
the assessment year 2018-19 cannot be carried forward for set off in
the subsequent eligible assessment years.
(ii) The answer will be different as regards unabsorbed depreciation. The
above provision is not applicable in case of carry forward of
unabsorbed depreciation and set off in the next years. Carry forward
of unabsorbed depreciation and set off in the next assessment year is
governed by Section 32(2). Section 80 does not restrict the carry
forward of unabsorbed depreciation in case of delayed submission of
return of income.
Space to write important points for revision

2016 - Dec [5] (c) For the assessment year 2019-20, an individual assessee
filed return of income on 30.11.2019. He has unabsorbed depreciation of
` 2 lakhs and business loss of ` 3 lakhs. Can these items be carried forward
to subsequent assessment year? Can this return be revised upon discovery
of any error? (5 marks)
7.260 O Scanner CMA Inter Gr. I Paper 7 (2016 Syllabus)

Answer:
Only Depreciation can be carried forward due to belated filing of Return
business loss cannot be carried forward with effect from A.Y. 2019-20
belated return filed u/s 139(4) can be revised.
Space to write important points for revision

2017 - June [5] (b) The following details have been furnished by Parikshit
relating to previous year 2018-19
Particulars `
(i) Income from business (non-speculation) 6,00,000
(ii) Interest on fixed deposit (net of TDS) 63,000
(iii) Long-term capital gain on sale of a residential house 1,00,000
(iv) Unabsorbed short-term capital loss carried forward from
Assessment Year 2018-19 1,10,000
(v) Loss in non-speculative business carried on by his wife,
Prerana. The business was started with the amount
gifted by Parikshit during the year 45,000
You are required to compute the total income of Parikshit for Assessment
Year 2019-20. (6 marks)
Answer:
Computation of Total Income
of Mr. Parikshit
for the A.Y. 2019-20
Particulars Amount (`) Amount (`)
Profits & gains of Business or profession
Income from Business 6,00,000
(Non speculation)
Less: Set off of loss in non-speculative
business carried on by his wife, Prerna.
The business was started with the
amount gifted by Parikshit during the
year 45,000 5,55,000
[Chapter  11] Set Off and Carry Forward of Losses O 7.261

Income from capital gains


Long Term capital gain on sale of a
residential house 1,00,000
Less: Set off of unabsorbed Short Term
capital loss carried forward from A.Y.
2018-19 1,00,000 NIL
Short term capital loss to be carried forward 10,000
Income from other sources
Interest on fixed deposit to be grossup 70,000

Gross Total Income 6,25,000


Less: Deductions under chapter VI-A NIL
Total Income 6,25,000
Space to write important points for revision

2017 - Dec [6] (a) Mr. Rahman furnishes you the following information for the
financial year 2018-19:

Particulars `
Loss from speculation business - A 80,000
Profit from speculation business - B 40,000
Loss from self occupied house property 1,80,000
Income from let out house property 4,00,000
Income from trading and manufacturing business @ 8% 2,00,000
Salary income 3,70,000
Interest on PPF deposit 65,000
Long term capital gain on sale of vacant site 1,10,000
7.262 O Scanner CMA Inter Gr. I Paper 7 (2016 Syllabus)

Short term capital loss on sale of jewellery 50,000


Investment in tax saver deposit on 31.03.2019 60,000
Brought forward loss of business of assessment year 2013-14 1,00,000
Donation to a charitable trust recognized under section 12AA
and approved under section 80G 1,40,000
Enhancement compensation received from Government for
compulsory acquisition of lands in the year 2006. 3,00,000
You are requested to compute the total income of Mr. Rahman for the
financial year 2018-19 and any loss eligible for carry forward. (8 marks)
Answer:
Computation of Total Income of Mr. Rahman for the A.Y. 2019-20
Income from Salary 3,70,000
Income from House Property
Income from Let-out House Property 40,000
Less: Loss from Self-occupied Property 1,80,000 2,20,000
Income from Business or Profession
Profit from Speculation Business 40,000
Less: Loss from Speculation Business 40,000 NIL
Loss from Speculation Business Carried Forward
(80,000 – 40,000) (40,000)
Income from Trading & Manufacturing Business 2,00,000
Less: Brought forward loss of A.Y. 2013-14 1,00,000 1,00,000
Income from Capital Gains
Long Term Capital Gain on Sale of Vacant Land 1,10,000
Less: Short Term Capital Loss (Jewellery) 50,000 60,000
Enhanced compensation received from Government for 3,00,000
compulsory acquisition of land in the year 2006
[Chapter  11] Set Off and Carry Forward of Losses O 7.263

Income from Other Sources


Interest on PPF Deposit 65,000 Exempt —
Gross Total Income 10,50,000
Less: Deduction under chapter VI-A
U/s 80C Investment in tax saver deposit 60,000
U/s 80G — Donation to Charitable Trust
Qualifying Amount
GTI 10,50,000
Less: Deduction U/s 80C 60,000
LTCGT 3,60,000
Adjusted Total Income 6,30,000
10% of Adjusted Total Income i.e. 63,000
Or
50% of Donation (1,40,000 × 50%) 70,000
Whichever is less 63,000 1,23,000
Total Income 9,27,000
Space to write important points for revision

2018 - June [6] (b) Aswini’s accounts are not required to be audited under
section 44AB. He furnished his return of income for Assessment Year 2019-
20 on 1st August, 2019. He has the following losses during the previous year
2018-19:
Loss from house property let out: ` 12,000
Loss from business: ` 60,000
Unabsorbed depreciation: ` 15,000
Short-term capital loss from sale of shares: ` 8,000
State, with reason, whether Aswini is entitled to carry forward above losses
and unabsorbed depreciation. (5 marks)
7.264 O Scanner CMA Inter Gr. I Paper 7 (2016 Syllabus)

Answer:
Where for any assessment year the net result of computation under the head
“Income from house property”, “Profit and gains of business or profession”,
income from the head “Capital Gains” is a loss to the assessee and such
loss is not wholly set off then the loss, which has not been set off wholly,
shall be carried froward to the following assessment year, not being more
than eight assessment year immediately succeeding the assessment year
for which the loss was first computed.
Unabsorbed depreciation can be set off and carried forward to the following
assessment year not being more then eight assessment years immediately
succeeding the assessment year for which the loss was first computed.
However, loss from house property can be set off only against the income
from house property, and short term capital loss can be set off only against
the income from LTCG or STCG. On the basis of above discussion we can
say that Aswani is entitled to carry forward all these losses.
Space to write important points for revision

2018 - Dec [4] (b) Ms. Pinky submits the following particulars for the year
ended 31st March, 2019:
Sl. Particulars `
No.
(i) Loss from let out residential building-computed 3,00,000
(ii) Arrear rent from a commercial building received during
the year (commercial property had been sold in June,
2016) 10,000
(iii) Textile business discontinued from 31st October
2017–Brought forward business loss of Assment Year
2015-16 60,000
(iv) Profit from chemical business of current year (computed) 5,50,000
(v) Bad debt written off in the Assessment Year 2014-15
relating to textile business recovered during the year
consequent to Court decree 1,00,000
[Chapter  11] Set Off and Carry Forward of Losses O 7.265

(vi) Long-term capital gain on sale of shares (STT paid) in


recognized stock exchange on 23.05.2018 90,000
(vii) Speculation business in oil seeds–profit 3,00,000
(viii) Winning from lottery (Gross) 11,00,000
(ix) Loss from the activity of owning and maintaining race
horses 2,10,000
You are required to compute the total income of Ms. Pinky and also ascertain
the amount of losses that can be carried forward. (7 marks)
12 DEDUCTIONS IN COMPUTING
TOTAL INCOME
THIS CHAPTER INCLUDES
 Deductions from Gross Total  Section 80C to Section 80U
Income under Chapter VI-A of
the Income Tax Act, 1961
Marks of Objective, Short Notes, Distinguish Between, Descriptive & Practical Questions
Legend
Objective Short Notes Distinguish Descriptive Practical

For detailed analysis Login at www.scannerclasses.com


for registration and password see first page of this book.

7.266
[Chapter  12] Deductions in Computing....... O 7.267

SHORT NOTES
2010 - June [7] Write a short note the following:
(c) Deduction under Section 80RRB of the Income Tax Act, 1961 towards
royalty of a patent; (5 marks)
Answer:
Deduction in respect of royalty on patents [Section 80RRB]
(i) This Section allows deduction to a resident individual in respect of
income by way of royalty of a patent registered on or after 01-04-2003
upto an amount ` 3 lakhs.
(ii) This deduction shall be available only to a resident individual who is
registered as the true and first inventor in respect of an invention
under the Patents Act, 1970, including the co-owner of the patent.
(iii) This exemption shall be restricted to the royalty income including
consideration for transfer of rights in the patent or for providing
information for working or use thereof in India.
(iv) The exemption shall not be available on any consideration for sale of
product manufactured with the use of the patented process or
patented article for commercial use.
(v) In respect of any such income which is earned from sources outside
India. The deduction shall be restricted to such sum as is brought to
India in convertible foreign exchange within a period of 6 months or
extended period as is allowed by the competent authority (Reserve
Bank of India). For claiming this deduction the assessee shall be
required to furnish a certificate in the prescribed form signed by the
prescribed authority, alongwith the return of income.
(vi) No deduction in respect of such income will be allowed under any
other provision of the Income-Tax Act.
(vii) Where the patent is subsequently revoked or the name of the
assessee was excluded from the patents register as patentee in
respect of that patent, the deduction allowed during the period shall be
deemed to have been wrongly allowed and the assessment shall be
rectified under the provisions of the Section 155.
(viii) A corresponding amendment has been made by inserting
sub-Section (117).
Space to write important points for revision
7.268 O Scanner CMA Inter Gr. I Paper 7 (2016 Syllabus)

2014 - June [5] (a) Write short notes on deduction under Section 80TTA in
respect of interest from banks. (4 marks)
Answer:
80TTA
Individual/ HUF - Eligible Assessee
Interest on saving account with a Scheduled Bank, or a Co-operative Bank
or Post Office Upto ` 10,000.
Section - 80 TTB
(Deduction in respect of Interest on Deposits in case of Senior Citizens)
Deduction under section 80TTB is available (from the assessment year
2019-20), if the following conditions are satisfied–
1. The assessee is a senior citizen (i.e., a resident individual who is atleast
60 years of age at any time during the previous year).
2. His income includes interest on deposits with a bank/co-operative
bank/post office (it may be interest on fixed deposits, interest on savings
account or any other interest).
 Amount of deduction – If these conditions are satisfied, the assessee
can claim deduction under section 80TTB which is equal to ` 50,000 or
the amount of aforesaid interest, whichever is lower.
Space to write important points for revision

DESCRIPTIVE QUESTIONS
2009 - June [3] (a) Write a brief note on deduction available under Section
8ODD of the Income-tax Act, 1961. (6 marks)
Answer:
Deduction in respect of maintenance including medical treatment of a
dependant who is a person with disability (Section 80 DD)
Essential conditions for claiming deduction under this Section:
(1) Deduction is available to a person who is:
(a) resident in India, and
(b) is either an individual or a HUF
[Chapter  12] Deductions in Computing....... O 7.269

(2) Deduction is available if the assessee has during the previous year;
(a) incurred any expenditure for the medical treatment (including
nursing), training and rehabilitation of a dependant, being a person
with disability; or
(b) paid or deposited any amount, under any scheme framed by the LIC
or any other insurer or UTI and which is approved by the CBDT. The
scheme should provide form the payment of annuity or a lump sum
amount for the death of the dependant being a person with disability
in the event of the death of the individual or the member of the HUF,
in whose name subscription to the scheme has been made by the
HUF for the benefit of the handicapped member. The assessee must
nominate either the dependant being a person with disability or the
dependant being a person with disability.
(3) The assessee, claiming a deduction under this section, shall furnish a
copy of the certificate issued by the medical authority in the prescribed
form and manner, along with the return of income under Section 139 in
respect of the assessment year for which deduction is claimed. Further
where the condition of disability requires reassessment of its extent after
a period stipulated in the aforesaid certificate, no deduction under this
section shall be allowed for any assessment year relating to any
previous year beginning after the expiry of the previous year during
which the aforesaid certificate of disability had expired, unless a new
certificate is obtained from the medical authority in the form and manner,
as may be prescribed, and a copy thereof is furnished along with the
return of income.
(4) A fix deduction of ` 75,000 is available. A higher deduction of ` 1,25,000
is available if such dependent relative is suffering from a severe
disability. Deduction under this section is available regardless of actual
expenditure.
Space to write important points for revision

2009 - Dec [3] (b) State briefly the deduction available under Section 80E in
respect of interest on loan taken for Higher Education. (5 marks)
7.270 O Scanner CMA Inter Gr. I Paper 7 (2016 Syllabus)

Answer:
Deduction in Respect of Interest on Loan Taken for Higher Education
[Section 80E]
1. Eligible Assessee: Individual;
2. Deduction in respect of: Deduction is available in respect of sum paid
by the assessee in the previous year, out of his income chargeable to
tax, by way of interest on loan taken:
(a) For his higher education, or
(b) For the higher education of his relative.
“Higher education” means any course of study pursued after passing the
Senior Secondary Examination or its equivalent from any school, Board
or University recognized by the Central Government or State
Government or local authority or by other authority authorizes by the
Central Government or State Government or local authority to do so.
3. Quantum and period of deduction: Deduction = 100% of the amount
of interest on such loan. Deduction will be admissible :
(a) For 8 assessment years starting from assessment year in which
interest was first repaid; or
4. Nature of loan: the loan referred to above must be taken:
(a) From any bank or other financial institution approved by prescribed
authority or any charitable institution notified by under Section
10(23C) or Section 80G(2)(a); and
(b) For pursuing higher education.
Space to write important points for revision

2013 - June [8] (b) What is the effect of contribution made by an individual
to electoral trust on his taxable income? (2 marks)
Answer:
The scope of Section 80GGC has been widened so as to enable an
individual to claim deduction from gross total income in respect of amount of
contribution made by him to an electoral trust during the year.
Space to write important points for revision
[Chapter  12] Deductions in Computing....... O 7.271

2014 - June [4] (b) Write a brief note on the deduction available under
Section 80DDB. (4 marks)
Answer:
80DDB
Resident Individual/ HUF - Eligible Assessee
Medical Treatment of Specified disease of Self, Spouse, Parent, Children,
Brother, Sister. Amount received from Insurance Company shall be deducted
from ` 40,000 or ` 1,00,000 if incurred in respect of Senior Citizen or in
respect of very senior citizen. The Assessee must furnish a certificate in form
No. 10-I along with the return of Income.
Space to write important points for revision

2014 - June [6] (a) Who are not ‘Regular Workmen, u/s 80JJAA of the
Income Tax Act, 1961? (3 marks)
Answer:
Following are not regular workmen u/s 80JJAA —
1. Casual workmen and workmen employed through contract labour.
2. Other workmen if employed for less than 150 days during the previous
year.
Space to write important points for revision

PRACTICAL QUESTIONS
2008 - Dec [2] (a) Answer the following question briefly:
(ii) ABC Company derived income from two separate industrial units
during the year ending March 31, 2019, eligible for deduction under
Section 80-IB. The profit of one unit was ` 4,50,000 while the other
unit suffered a loss of ` 2,50,000. What is the deduction available
under Section 80-IB of the Income-tax Act, 1961? (2 marks)
Answer:
The given information in incomplete. Hence we assume that Unit I is
engaged the production a refining of mineral oil and Unit II is engaged in
operating and maintaining a hospital located anywhere in India other than
7.272 O Scanner CMA Inter Gr. I Paper 7 (2016 Syllabus)

excluded area. Since 60th businesses come in the preview of Section 35 AD


i.e. specified business, inter source adjustment can be made. The treatment
will be as follows:
`
Business income from unit I 4,50,000
Business loss from unit II (2,50,000)
Business income (after set-off) 2,00,000
Since Unit I is engaged in production a refining of mineral oil, the entire
profits of ` 2,00,000 shall be eligible for 100% deduction u/s 80 - IB(a).
Space to write important points for revision

2008 - Dec [3] (a) Thilagam has given donation of ` 30,000 in cash and
cement bags worth ` 20,000 to an approved charitable institution. What is
the deduction under Section 80G of the Income-tax Act, 1961 available to
the assessee whose gross total income is ` 6,00,000 ? (3 marks)
Answer:
Donations given in cash alone are eligible for deduction u/s. 80G. Donations
in kind are not.
Maximum qualifying amount is 10% of GTI i.e., 10% of 6,00,000 i.e.,
` 60,000. Actual donation of ` 30,000 being lower is taken.
Eligible deduction u/s. 80G is 50% of ` 30,000…….` 15,000.
Space to write important points for revision

2010 - June [4] (a) Mr. Srinivasan, aged 66 years, furnishes the following
particulars for the year ending 31.3.2019:
(i) Life Insurance premium paid ` 40,000, actual capital sum of the policy
assured for ` 1,50,000. Policy was taken in year 2010.
(ii) Contribution to Public Provident Fund ` 50,000 in the name of father;
(iii) Tuition fees payment ` 5,000 each for 3 sons pursuing full time
graduation course in Mumbai; Tuition fee paid for daughter pursuing
Ph.D. in Melbourne University, Australia ` 3.50 lakhs;
(iv) Housing loan principal repayment ` 30,000 to HDFC Bank. This
property is under construction at Bangalore as on 31.03.2019.
(v) Principal repayment of housing loan taken from a relative ` 60,000.
The property is self-occupied and situated at Chennai;
[Chapter  12] Deductions in Computing....... O 7.273

(vi) Deposit under Senior Citizens Savings Scheme ` 15,000;


(vii) Five-year deposits in an account under Post Office Time Deposit
Scheme ` 20,000;
(viii) Investment in National Saving Certificate ` 25,000;
(ix) Subscription to bonds issued by NABARD ` 30,000.
Compute the quantum of eligible deduction under Section 80C of the Income
Tax Act, 1961 for A.Y. 2019-20. (7 marks)
Answer:
Computation of deduction under Section 80C for A.Y. 2019-20
Particulars Amount eligible for
Deduction u/s 80C
`
Life Insurance premium (See Note 1) 30,000
Contribution to Public Provident Fund (See Note 2) Nil
Tuition fee of 2 children for graduation course (See Note 3) 10,000
Housing Loan Principal Repayment (See Note 4 & 5) Nil
Senior Citizen saving scheme deposit 15,000
Post Office Time Deposit Scheme 20,000
Investment in National Saving Certificate 25,000
Subscription to Bond issued by NABARD 30,000
Gross amount eligible for deduction under Section 80C 1,30,000
Notes:
1. Any amount of life insurance premium paid in excess of 10% of capital
sum assured shall be ignored for deduction under Section 80C in the
given case, 10% of actual capital sum assured is ` 15,000. Whereas,
the premium paid during the year is ` 40,000. Therefore, the excess
premium of ` 25,000 is not qualify for deduction.
2. In the case of an individual, contribution to PPF can be made in
individual's name or in the name of spouse or children to qualify for
deduction under Section 80C. As the contribution was made in the
name of his father, deduction is not allowable.
3. Tuition fees paid are eligible for deduction under Section 80C for a
maximum of two children. Therefore, ` 10,000 shall be allowed as
deduction.
Tuition fee paid to an educational institution situated outside India is
not eligible for deduction.
7.274 O Scanner CMA Inter Gr. I Paper 7 (2016 Syllabus)

4. In order to claim the principal repayment on loan borrowed for house


property as deduction, the construction of such property should have
been completed and should be chargeable to tax under the head
"income from house property". In the given case, since the property is
under construction, principal repayment does not qualify for deduction.
5. Repayment of principal on housing loan is not allowed as deduction
in case the loan is borrowed from friends, relatives, etc. In order to
qualify for deduction, the loan should have been obtained from
specified employer/institution.
Space to write important points for revision

2012 - Dec [3] (c) Determine the eligibility and quantum of deduction under
Chapter VI-A in the following cases:
(ii) Contribution to notified pension scheme (referred to Section 80 CCD)
by the employer ` 40,000 for an employee whose basic salary plus
dearness allowance was ` 3,00,000 for the year. (4 marks)
Answer:
An assessee who is an individual employed by central Govt. (or any other
employer) make any contribution to his account under a pension scheme
notified (or as may be notified by Central Govt.) the assessee shall be
allowed a deduction in computation of his total income, the deduction is
subjected to amount of 10% of his salary in previous year. (Sec. 80CCD)
Contribution to Notified Pension Scheme
Amount of deduction u/s 80CCD(2)
(least of the following)
1. Actual Amount Paid – ` 40,000
2. 10% of Salary 3,00,000 × 10% – ` 30,000
Hence, Amount of Deduction is ` 30,000.
Space to write important points for revision
[Chapter  12] Deductions in Computing....... O 7.275

2013 - Dec [1] {C} (a) Answer the following sub-divisions briefly in the light
of the provisions of the Income-tax Act, 1961:
(v) A life insurance policy was taken in April, 2018 for a capital sum
assured of ` 8 lakhs. The annual premium amounts to ` 1,10,000 for
10 years. How much is deductible under Section 80C? (1 mark)
Answer:
Only 10% of the premium on the capital sum assured is eligible for
deduction. Amount Deductible u/s 80C = 8,00,000 X 10% = ` 80,000.
Space to write important points for revision

2013 - Dec [6] (c) Compute the quantum of deduction under Section 80C for
Mr. Niraj for the assessment year 2019-20.
`
Life Insurance premium
- Own – Capital sum assured 2,00,000 (being the first premium paid) 25,000
- Brother’s life – dependent on Niraj 10,000
- Major son – doing business 5,000
Contribution to recognized provident fund 15,000
Repayment of bank loan for purchase of residential apartment-let out 60,000
Tuition fees for M.Com (part-time) pursued by wife 12,000
(3 marks)
Answer:
Computation of deduction u/s 80c
Particulars `
Life insurance premium
-Own- capital sum assured ` 2,00,000 (being the first premium
paid) - limited to 10% i.e. ` 2,00,000 x 10% 20,000
- Major son - doing business - allowed 5,000
Contribution to recognized provident fund - allowed 15,000
Repayment of loan for purchase of residential apartment -let out -
allowed 60,000
Total amount eligible for deduction u/s 80 C 1,00,000
7.276 O Scanner CMA Inter Gr. I Paper 7 (2016 Syllabus)

Note: Brother’s Life - Dependent on Niraj & Tuition fees for M.Com (part
time) pursued by wife will not be allowed as Deduction u /s 80 C.
Space to write important points for revision

2016 - Dec [4] (b) Mr. Praveen Kumar (aged 59 years) having gross total
income of ` 27,50,000 during previous year 2018-19 incurred expenditure
of ` 1,30,000 during the said year on medical treatment of his dependant
father (aged 80 years) who is suffering from chronic disease specified in
Section 80DDB of the Income-tax Act and also paid medical insurance
premium of ` 26,000. Determine total income of Mr. Praveen Kumar for
Assessment Year 2019-20. (4 marks)
Answer:
Computation of Total Income of Mr. Praveen Kumar for A.Y. 2019-20:
Gross Total Income 27,50,000

Less: Deductions u/s 80 D

For medical insurance premium for self under section 80D-


maximum allowable is ` 25,000 25,000

u/s 80 DD B 1,00,000

Total Income 26,25,000

Note: Amendment to [Section-80DDB]


Section 80DDB has been amended (with effect from the assessment year
2019-20) so as to raise the above monetary limit of deduction to
` 1,00,000 for both senior citizens and super senior citizens.
Space to write important points for revision
[Chapter  12] Deductions in Computing....... O 7.277

2017 - Dec [6] (b) Mr. Gangai Amaran (age 50) incurred following
expenditures during the financial year 2018-19:
S. Particulars `
No.

(i) Medical expenditure on the treatment of his non- 30,000


dependent father (age 82)

(ii) Medical expenditure on treatment of his non-dependent 25,000


mother (age 73)

(iii) Medical expenditure for a surgery undergone by himself 50,000

(iv) Medical insurance premium for non-dependent mother 35,000


(age 73)

(v) Medical insurance premium for self (paid by cheque) 27,000

(vi) Preventive medical health check-up paid in cash for 7,000


himself.
Computer the amount eligible for deduction under section 80-D for the
financial year 2018-19. (7 marks)
Answer:
Computation of deduction under section 80-D for the Assessment Year
2019-20
Particulars Reason `

(i) Medical expenditure on the Eligible for deduction as he 30,000


treatment of his non-dependent is a senior citizen. Also
father (age 82) because no amount was
paid towards his health
insurance.
7.278 O Scanner CMA Inter Gr. I Paper 7 (2016 Syllabus)

(ii) Medical expenditure on deductible as she is a 20,000


treatment of his non-dependent senior citizen. But limited to
mother (age 73) ` 50,000 since ` 30,000
exhausted in (i) above only
` 20,000 will be eligible for
deduction out of ` 25,000.

(iii) Medical expenditure for a Not deductible as he is not a Nil


surgery undergone by himself senior citizen.
(age 50)

(iv) Medical Insurance premium for It is deductible even though Nil


non-dependent mother (age 73) she is not dependent on
him. But limited to ` 50,000
but since the limit has been
exhausted in (i) & (ii) above,
no further deduction is
allowable.

(v) Medical Insurance premium for Eligible, but limited to 25,000


himself paid by cheque. ` 25,000

(vi) Preventive medical health check Eligible but limited to Nil


up paid in cash for himself ` 5,000. But as the entire
amount of ` 25,000 has
already been exhausted in
(v) above, no further
deduction is allowed.

Total deduction 75,000


[Chapter  12] Deductions in Computing....... O 7.279

Note: Amendment to [Section - 80D]


The following amendments have been made to the scheme of section
80D with effect from the assessment year 2019-20 as follows:
 Section 80D, inter alia, provides that for medical insurance (or
preventive health ‘ check-up of a senior citizen), deduction of ` 30,000
shall be allowed. Further, in the case of super senior citizens, the said
section also provides for a deduction of medical expenditure within the
overall limits of ` 30,000.
The above monetary limits have been extended so as to provide that
the deduction of ` 50,000 in aggregate shall be allowed to senior
citizens in respect of medical insurance or preventive health check -
up or medical expenditure.
 In case of single premium health insurance policies having cover of
more than one year, deduction under section 80D shall be allowed on
proportionate basis for the number of years for which health insurance
cover is provided, subject to the specified monetary limit.
Space to write important points for revision
13 REBATE AND RELIEFS
THIS CHAPTER INCLUDES
 Rebate  Reliefs (Section 89)

Marks of Objective, Short Notes, Distinguish Between, Descriptive & Practical Questions

Legend
Objective Short Notes Distinguish Descriptive Practical

For detailed analysis Login at www.scannerclasses.com


for registration and password see first page of this book.

7.280
[Chapter  13] Rebate and Reliefs O 7.281

DESCRIPTIVE QUESTIONS
2010 - June [4] (b) Briefly explain marginal relief allowable while computing
tax payable by certain assessee. (3 marks)
Answer:
Marginal Relief: Marginal relief shall be allowed in all the cases to ensure
that the additional amount of income-tax payable including surcharge, on the
excess of income over ` 1 crore is limited to the amount by which the income
is more than ` 1 crore.
Applicable in case of assessee only having a Total Income > ` 1 Crore
• Marginal Relief = Increase in Tax - Increase in Income [If it is Nil or
Negative, No relief is allowed]
• Education Cess & Secondary Higher Education Cess shall be applied
only after permitting Marginal Relief.
Space to write important points for revision
14 ASSESSMENT OF VARIOUS
PERSONS
THIS CHAPTER INCLUDES
 Provisions Related to  Essential requirements of an
Assessment of Income of HUF
Individual Persons  Computation of income of
 Alternate Minimum Tax (AMT) Association of person (AOP)/
[Sec. 115JC] Body of individual (BOI)
 HUF Meaning  Computation of Total Income
 Computation of Tax Liability
Marks of Objective, Short Notes, Distinguish Between, Descriptive & Practical Questions
Legend
Objective Short Notes Distinguish Descriptive Practical

For detailed analysis Login at www.scannerclasses.com


for registration and password see first page of this book.
7.282
[Chapter  14] Assessment of Various Persons O 7.283

DISTINGUISH BETWEEN
2009 - June [8] (d) Distinguish between Association of persons and Body of
Individuals as envisaged in the income-tax Act, 1961. (2 marks)
Answer:
Difference between AOP & BOI: Charging Section 4 provides for charging
to tax the income of every person. The term ‘person’ includes association of
person and Body Of Individuals (BOI). In the case of AOP and BOI the
provision relating to computation and taxability of income are the same. But
there is a conceptual difference between both.
Simply put, AOP implies a voluntary getting together for a definite
purpose; a BOI would be just a body without an intention to get together.
That is to say there is common will and desire among member of an
association of person and they, on their own, joint together to carry on the
activities. In the case of a body of individual, such common will and desire
are lacking and it is form by operation of law.
Further, members of BOI can be individuals only whereas member of
‘association of person’ can be two or more firm or Hindu undivided family etc.
Space to write important points for revision

DESCRIPTIVE QUESTIONS
2010 - June [3] (c) (I) If two or more persons jointly own a property and if
their shares are definite and ascertainable, can the income from such
property be taxed as income of an association of persons? (2 marks)
Answer:
No. If two or more persons jointly own a property and if their shares are
definite and ascertainable, then the income from such property cannot be
taxed as income of an association of persons. In such case the property
owned by co-owners are assessable individually.
Space to write important points for revision
7.284 O Scanner CMA Inter Gr. I Paper 7 (2016 Syllabus)

PRACTICAL QUESTIONS
2008 - Dec [3] (c) A, B and C are three partners of a firm ; the firm provides
the following information:
(i) Total income of the firm is () ` 1,40,000, out of which unabsorbed
depreciation is ` 50,000. The return of income of the firm was filed in
time.
(ii) On March 31, 2019, C retires from the firm and the other partners
continue the same business;
(iii) The firm's income for the assessment year 2019-20 before adjusting
the aforesaid loss and depreciation is ` 1,08,000 ;
(iv) A, B and C share Profit and Loss in ratio of 1: 1: 2.
Compute firm's total income for the assessment year 2019-20,
assuming that salary and interest are not paid to any partner.
(5 marks)
Answer:
Computation of Total Income of a Firm (ABC)
For Assessment Year 2019 – 20
Brought forward loss excluding depreciation 90,000
(1,40,000 – 50,000)
C’s share therein (2/4 of ` 90,000) 45,000
Balance loss eligible for set off 45,000
Computation of income of the firm:
Income before adjustment 1,08,000
Less: Brought forward loss (eligible portion) 45,000
Unabsorbed depreciation 50,000 95,000
Total Income 13,000

Note: As per Section 78 of the Income-tax Act, 1961, share of the


retiring partner in losses shall not be available for set off in the hands of
the firm. This however does not apply for depreciation.
Space to write important points for revision
[Chapter  14] Assessment of Various Persons O 7.285

2008 - Dec [4] (a) The income and related particulars of Mr. Coorg, aged 56,
for the year ended 31.03.2019 are given below:
(i) Salary ` 24,000 per month;
(ii) He was provided with a rent-free accommodation in Hyderabad for
which rent of ` 6,000 per month was paid by his employer;
(iii) His wife was sick and treatment was taken from a private hospital, for
which an amount of ` 32,000 paid towards medical expenses by his
employer in December, 2018.
(iv) An allowance of ` 13,200 was paid by his employer towards his son's
education;
(v) The employer paid DA 10,000 per month (considered for retirement
benefits), professional tax of ` 2,400 and income-tax liability of
` 15,000.
(vi) He encashed earned leave to his credit to the tune of ` 10,000;
(vii) Loss from speculative business ` 20,000;
(viii) Loss from sale of shares in ABC Pvt. Ltd. Held for ten months ` 8,000;
(ix) Profit on sale of long-term capital assets ` 10,000.
Compute the total Income and Tax liability of Mr. Coorg for the
assessment year 2019-20. (10 marks)
Answer:
Name of Assessee: MR. COORG
Status: Resident and Ordinarily resident (Individual)
Assessment Year: 2019 – 20
Previous year: 2018 – 19
COMPUTATION OF TOTAL INCOME
Salaries:
(a) Basic salary 2,88,000
(b) Earned leave encashment 10,000
(c) DA (` 10,000 × 12) 1,20,000
(d) Perquisites:
(i) Rent-free accommodation* 62,700
(ii) Medical expenses 32,000
(iii) Education fee paid by employer 13,200
Less: Exempt u/s 10(14) 1,200 12,000
7.286 O Scanner CMA Inter Gr. I Paper 7 (2016 Syllabus)

(iv) Income- tax paid by employer 15,000


(v) Professional tax paid by the employer 2,400
Gross salary 5,42,100
Less: Standard Deduction u/s 16(i) 40,000
Less: Professional tax u/s 16(iii) 2,400
Net salary 4,99,700
Add: Capital gains (Long-term) 2,000
[Long-term gain ` 10,000) – (Short-term loss ` 8,000)]
Total Income 5,01,700
Tax liability
Tax on long –term capital gains (` 2,000 at 20%) 400
Tax on other incomes of ` 4,99,700 12,485
12,885
Add: Health and Education Cess 4% 515
Total Tax Liability 13,400
Working Note:
*Rent – free accommodation:
Rent paid by employer ` 72,000
15% of salary (basic salary, leave encashment and DA)
i.e., (2,88,000 + 10,000 + 1,20,000 = 4,18,000) ` 62,700
Lower of the two is taken
Note: Speculation business loss is to be set off against income from
speculation business income only. In the given case, speculative business
loss cannot be set off as there is no income from speculative business.
The same is to be carried forward.
Space to write important points for revision

2008 - Dec [6] (b) Vatsan has the following incomes for the financial year
2018-19:
`
Business Income () 40,000
Short-term capital gains 16,000
Long-term capital gains 1,90,000
He deposits ` 10,000 in public provident fund account. You are required
to find out his tax liability for the assessment year 2019-20. (4 marks)
[Chapter  14] Assessment of Various Persons O 7.287

Answer:
Computation on income of Mr. VATSAN
for the assessment year 2019-20
` `
Business income (-) 40,000
Capital gains:
Short term 16,000
Long term 1,90,000 2,06,000
Total Income (Taxable): 1,66,000

Note:
1. Business loss can be set off against capital gain. It is beneficial to first
set it off against short-term capital gain and the balance against long-
term capital gain. The whole of total income would consist of long-term
capital gain.
2. Deduction u/s 80C is not available in respect of PPF, since the whole
of gross total income consists of income by way of long term capital
gains.
Computation of tax liability
`
Net Income (Long-term Capital gain) 1,66,000
Tax thereon: Nil
(Maximum exemption limit ` 2,50,000)
Space to write important points for revision

2009 - June [4] (b) M, N and O are partners in the firm Upadhyay & Co.,
sharing profit and losses in the ratio of 2: 1: 1 respectively. The summarized
profit and loss account for the year ended 31.03.2019 is given below:
` `
Office salaries 5,680 Gross profit 60,570
Interest on loan from M 2,000 Rent received 6,000
Telephone, telegram, etc., 2,000 Interest on securities 4,000
Local taxes (let-out property) 1,000
Salary to working partner N 3,000
7.288 O Scanner CMA Inter Gr. I Paper 7 (2016 Syllabus)

Commission to partners:
M 4,000
N 5,000
O 6,000 15,000
Collection charges of interest on securities 50
Provision for bad debts 1,000
Net profit to partners:
M 20,420
N 10,210
O 10,210 40,840
70,570 70,570
Compute total income of the firm for the assessment year 2019-20 and tax
payable thereon. (10 marks)
Answer:
Computation of Book Profit
Net profit as per profit and loss account 40,840
Add: Inadmissible expenses:
Local taxes (on let-out property) 1,000
Salary to partner N 3,000
Commission to partners 15,000
Collection charges 50
Bad debt provision 1,000 20,050
60,890
Less: Other incomes
Rent received 6,000
Interest on securities 4,000 10,000
Books profits as per Section 40(b) 50,890
Note:
(1) it is assumed that interest on loan taken from M is at 12%
(2) In this case, ` 50,000 could be allowed as maximum remuneration, but
the partners have drawn only ` 18,000 by way of salary and
commission. The entire amount is hence allowed.
[Chapter  14] Assessment of Various Persons O 7.289

Computation of total income of firm for the assessment year 2019 – 20


`
Profits and gains of business or profession:
Book profits as computed above 50,890
Less: Salary & Commission to partners 18,000
Taxable business income 32,890
Income from House property 25 February
`
Annual value (Rent received is taken) 6,000
Less: Municipal Tax 1,000
Net annual value 5,000
Less: Statutory allowance @ 30% 1,500
Chargeable income from house property 3,500
Income from other sources
Interest on securities 4,000
Less: Collection charges 50 3,950
Total Income 40,340
Tax on above @ 30% 12,102
Health & Education cess @ 4% 484
Total tax 12,586
Rounded off 12,590
Space to write important points for revision

2009 - June [5] (a) Mr. Y submits the following particulars for the year ended
31.03.2019:
(i) On 30.04.2018, when he attained the age of 60, his friends gave him
a new flat at Surat, each contributing ` 40,000 in cash. The cost of the
flat was ` 6.4 lakhs.
(ii) Another friend sent cash gift of ` 75,000 for the occasion.
(iii) Mr. Y sold the flat on 30.01.2019 for ` 8.9 lakhs. The Registrar's
valuation for stamp duty purposes was ` 9.2 lakhs. Neither the buyer,
nor Mr. Y questioned this value.
(iv) He had purchased equity shares in X Ltd. on 05.02.2018 for ` 3.5
lakhs. These were sold on 15.03.2019 for ` 2.8 lakhs.
7.290 O Scanner CMA Inter Gr. I Paper 7 (2016 Syllabus)

(v) He has paid life insurance premium of ` 90,000 for his major son who
is not dependent on him.
You are required to calculate the total income of Mr. Y for the
assessment year 2019-20, Cost inflation indices 272 for financial year 2017-
18 and 280 for financial year 2018-19. (7 marks)
Answer:
Total income of Mr. Y for the assessment year 2019–20
`
Capital gains as worked out below (W. Note 1) 2,50,000
Income form other sources (W. Note 2) 75,000
Gross total income 3,25,000
Less: Deduction under Chapter VI – A (W. Note 3) 90,000
Total Income 2,35,000
Working Note 1: Working of capital gains
Sale of flat at Surat: Short-term capital asset
Sale consideration: As per sec 50C
Unagitated stamp value or
Actual sale price whichever is higher Diff. is not
more than 5% 8,90,000
Less: Cost of acquisition 6,40,000
Short-term capital gain 2,50,000
Sale of equity share – long term capital asset
Sale consideration 2,80,000
Less: Indexed cost of acquisition
3,50,000 × 280/272 3,60,294
Long term capital loss (80,294)
Chargeable capital gains
(LTC loss cannot be set-off against STCG) 2,50,000
Working Note 2: Section 56(2) (vi) cash gifts and gifts in kind. Mr. Y’s
friends’ contributed ` 40,000/- each but the gift received by Mr. Y is only the
flat. As it is in the nature of gift in kind, it is covered by this section.
The gift of ` 75,000/- received from another friend (non-relative) is in
cash and is taxable u/s. 56 as ‘Income from other sources’.
[Chapter  14] Assessment of Various Persons O 7.291

Note: w.e.f. 1-10-2009 value of immovable property shall be the stamp


duty value of the property. Hence, stamp duty value of flat is taxable for
Mr. Y.
As per amendment made by Finance Act, 2018 w.e.f 1st April 2018
where the value adopted or assessed or assessable by the stamp
valuation authority does not exceed 105% of the consideration received
or accruing as a result of the transfer, the consideration so received or
accruing as a result of the transfer shall, for the purpose of section 48, be
deemed to be the full value of the consideration.
And where the value adopted or assessed or assessable by the stamp
valuation authority is exceeds 105% of the consideration received or
accruing as a result of the transfer then Stamp Duty Value shall, for the
purpose of section 48, be deemed to be the full value of the consideration.
Working Note 3: Deduction u/s 80C can be claimed in respect of LIP paid
for any child. It is immaterial that the child is not dependent on the assessee.
Space to write important points for revision

2009 - Dec [5] (a) P.K. retired as General manager of XYZ Co. Ltd. on
November 30, 2018 after rendering service for 20 years and 10 months. He
received ` 3,00,000 as gratuity from the employer. He is not covered by the
Payment of Gratuity Act, 1972.
His salary particulars are given below
Basic pay ` 10,000 p.m. up to June 30, 2018
Basic pay ` 12,000 p.m. from July 1, 2018
Dearness Allowance 50% of basic pay
(Eligible for Retirement Benefits)
Transport Allowance ` 1,500 p.m.
He resides his own house. Interest on moneys borrowed for the self
occupied house is ` 20,000 for the year ended March 31, 2019.
From a fixed deposit with a Bank he earned interest income of ` 14,000
for the year ended 31.03.19.
Compute taxable income of P.K. for the year ended March 31, 2019.
(10 marks)
7.292 O Scanner CMA Inter Gr. I Paper 7 (2016 Syllabus)

Answer:
Computation of Total Income and tax payable
Name of the Assessee: Mr. P.K.
Particulars Amount `
I. Salary As per Working Note 2,45,000
II. Income from House Property
Net Annual Valueself occupied property Nil
Less: Deduction u/s 24(b)Interest paid 20,000 (20,000)
III. Income from Other Sources
- Interest on fixed deposits 14,000
Gross Total Income 2,39,000
Less: Deduction under Chapter VI A Nil
Taxable Income 2,39,000

Working NoteSalary `
(a) Basic PayApril to June (10,000 × 3) 30,000 30,000
- July to November (12,000 × 5) 60,000 60,000 90,000
(b) Dearness Allowance (50% of Basic pay) 45,000
(c) Transport Allowance (1,500×8) 12,000
(d) Gratuity received 3,00,000
Less: Exempt u/s 10(10) Least of the following:
(i) Statutory limit ` 10,00,000
(ii) ` 8,100 (Refer Note) × 20 yrs = 1,62,000
(iii) Actual gratuity received ` 3,20,000 1,62,000 1,38,000
Gross Salary 2,85,000
Less: Deduction u/s 16 Standard deduction 40,000
Taxable Salary 2,45,000

Note: Gratuity for employee not covered under payment of Gratuity Act
shall be allowed based on the average salary of 10 months preceding the
month of retirement. Therefore, average salary is calculated as follows:
[Chapter  14] Assessment of Various Persons O 7.293

Particulars `
Basic: (10,000 × 6) 60,000
(12,000 × 4) 48,000
1,08,000
Add: Dearness Allowance (50% of 1,08,000) 54,000
1,62,000
Average Salary (1,62,000/10) 16,200
½ Month average salary (16,200/2) 8,100
Space to write important points for revision

2009 - Dec [6] (a) P, Q and R are partners in a firm sharing profits and
losses in the ratio of 1: 1: 2 provide the following information. Find firm’s Net
Income assuming that salary and interest are not paid to partners.
(i) Net income of the firm is () ` 1,20,000, out of which unadjusted
depreciation is ` 40,000.
(ii) On 30.04.18 R retires from the firm and the other partners carry on the
same business.
(iii) The firm’s income for the Assessment Year 2019-20 before adjusting
the aforesaid loss and depreciation is ` 1,20,000. (5 marks)
Answer:
Brought forward loss from the assessment year 2018-19 1,20,000
Out of which depreciation 40,000
Brought forward loss (excluding depreciation) 80,000
Share of R, the retiring partner (2/4 of ` 80,000) 40,000
Income of the firm of the assessment year 2019-20 1,20,000
(before adjustment of loss)
Out of which share of R, the retiring partner
[1/12×2/ 4 × ` 1,20,000]= 5,000
Amount of brought forward loss which cannot be set off
[`40,000 -5,000] 35,000
7.294 O Scanner CMA Inter Gr. I Paper 7 (2016 Syllabus)

Computation of income of the firm for the assessment


year 2019-20
Income (before adjustment) 1,20,000
Less:
Brought forward loss (i.e., ` 80,000  35,000) 45,000
Unadjusted depreciation 40,000
Net income 35,000
Space to write important points for revision

2010 - June [5] (a) Compute the total income of Mr. Pankaj Dhimani from
the information given below:
Particulars `
Net income for house property 1,75,000
Income from business (before providing for depreciation) 2,25,000
Short-term capital gain on sale of shares 80,000
Long-term capital loss from sale of property (70,000)
(brought forward A.Y. 2016-17)
Income from integrated activities of growing tea crops
and manufacturing tea 1,50,000
Dividends from Indian companies carrying on agricultural operations 70,000
Current year depreciation 35,000
Brought forward business loss(loss incurred six years ago) (65,000)
Expenditure incurred on medical treatment of dependant 1,20,000
with severe disability (7 marks)
Answer:
Name of Assessee - Mr. Pankaj Dhimani
Assessment Year - 2019-20
Financial Year - 2018-19
Computation of Total Income
Particulars ` `
Income from House property 1,75,000
Income from Business
Profit before depreciation 2,25,000
[Chapter  14] Assessment of Various Persons O 7.295

Less: Current year depreciation 35,000


Less: Brought forward business loss 65,000
1,25,000
Income from Tea business (40%) 60,000 1,85,000
Income from Capital Gain
Short term Capital gain 80,000
Long term capital loss from property
(setoff not allowed) Nil 80,000
Gross Total Income 4,40,000
Less: Deduction u/s 80DD 1,25,000
Total Income 3,15,000
Space to write important points for revision

2010 - Dec [6] (a) Devender, aged 55, resident in India, furnishes the
following information for the previous year ended 31-3-2019:
`
House property income (Net) 18,500
Business income 5,000
Capital gains (short-term) 22,000
Capital gains (long-term) 2,500
Income from horse race 15,000
Income from card games 16,000
Additional information are as follows:
Brought forward business loss for AY 2015-16 12,000
Unabsorbed depreciation for AY 2016-17 6,000
Long-term capital loss for AY 2015-16 12,000
Loss from horse race suffered in AY 2015-16 8,000
Speculative loss for AY 2014-15 10,000
Devender has taken a life insurance policy for his major son working in
a software company for a salary of ` 5 lacs per annum. He has paid premium
of ` 60,000 in cash for a capital sum assured of ` 4,00,000/-
He has paid PPF of ` 70,000 by raising a hand loan from his friend.
Calculate total income and tax liability. State the items to be carried forward.
(10 marks)
7.296 O Scanner CMA Inter Gr. I Paper 7 (2016 Syllabus)

Answer:
Name of Assessee: Mr. Devender
Residential Status: Resident & Ordinarily Resident
Financial Year: 2018-19
Assessment Year: 2019-20
Statement of Computation of Total Income
Particulars (`) (`)
Income from Salary nil
Income from House Property 18,500
Profits & Gains of Business & Profession 5,000
Less: B/F business loss (12,000) nil
Capital Gains/ Losses
Long Term Capital Gain 2,500
Less: B/F long term capital loss (12,000) nil
Short Term Capital Gain 22,000
Less: Unabsorbed Depreciation for 2015-2016 (6,000) 16,000
Income from Other Sources
Income from Horse race (15,000 - 8,000 set off) 7,000
Income from Card Games 16,000 23,000
Gross Total Income 57,500
Less: Deductions under chapter VI-A
U/S 80 C
LIP (maximum 10% of sum assured) 40,000
PPF 70,000
(deductions can’t exceed) 34,500
Taxable Income 23,000
Tax payable @30% on Income from horse race &
Income from Card game 6,900
Less: Rebate 2,500
4,400
Health & Education cess @ 4% 176
Item to be c/f 4,576
Long term capital loss A.Y. 2015-16 9,500
[Chapter  14] Assessment of Various Persons O 7.297

Note: In respect of B/F loss for A.Y. 2010-11 & speculative loss 2014-15,-
15 time limit for C/F has expired.
Space to write important points for revision

2011 - June [2] (a) Mrs. Sneha, aged 64, has carried on business during the
year ended March 31, 2019. The particulars of Profit and Loss Account are
given below:
Profit and Loss Account for the year ended 31-3-2019
` `
To Office salaries 22,000 By Gross profit 3,78,150
To Proprietor's salary 12,000 By profit on sale of residence 33,500
To General expenses 8,500 By disallowed bad debts recovered 62,000
To Bad debts 7,500 By Interest from Govt.
To Fire insurance premium 5,500 securities (net) 12,600
To Depreciation 2,500 By Dividend from JB Agro Ltd. 4,000
To Motor car expenses 7,500 By Income from Horse race (gross) 16,000
To Donation to Goa university 60,000 By Sundry receipts 1,500
To Income tax 2016-17 8,000
To Life Insurance premium 10,000
To Reserve for future loss 2,000
To Advertisement 6,000
To Net profit 3,56,250
5,07,750 5,07,750
Additional information:
(i) General expenses include ` 1,500 paid as compensation to an old
employee whose services were terminated. His service was
considered detrimental to business interest. A sum of ` 6,000 being
cost of small machine is also included in general expenses.
(ii) One-third of motor car expenses is for personal use.
(iii) Reserve for future loss represents a demand of sales tax under
dispute.
(iv) Depreciation is found to be in excess by ` 500 as per Income-tax
Rules.
(v) Actual income tax for 2018-19 ` 12,000.
7.298 O Scanner CMA Inter Gr. I Paper 7 (2016 Syllabus)

(vi) Profit on sale of residence represents long-term capital gains


computed in the prescribed manner.
(vii) Tax has been deducted at source from the Govt. Securities at 10%.
(viii) JB Agro Ltd. is a listed Indian company.
(ix) She received interest of ` 4,000 on moneys lent to her friends, which
has not been reflected in her books.
Compute her total income and tax liability. (9 marks)
Answer:
Particulars Amt. Amt.
(`) (`)
Net profit as per P&L Account 3,56,250
Add: Inadmissible Expenses
Proprietor’s salary 12,000
Motor Car Expenses 2,500
Donation to Goa University 60,000
Income Tax 8,000
Insurance Premium 10,000
Reserve for future loss 2,000
Excess depreciation 500
Cost of small machine (capital exp.) 6,000 1,01,000
Less: Admissible Exp. & Income of other heads credited. 4,57,250
Profits of House Property 33,500
Bad debts not allowed 62,000
Interest on Securities 12,600
Dividend income 4,000
Horse race income 16,000 1,28,100
Business Income 3,29,150
Income from House property
Long Term Capital Gain on sale of House Property 33,500
Income from Other Sources
Interest on Securities (12,600 × 100/90 = 14,000) 14,000
Income from horse race 16,000
Interest from friends 4,000
[Chapter  14] Assessment of Various Persons O 7.299

Dividend from domestic company Exempt 34,000


Gross Total Income 3,96,650
Less: Deduction under chapter VI A
LIP 10,000
3,86,650
Tax on Income other than Race Horse ` 3,70,650 6,033
Tax on Horse Race Income
16,000 x 30% 4,800
10,833
Less: Rebate u/s 87A NIL
10,833
Health & Education cess @ 4% 433
11,266
Less: TDS 1,400
Tax Payable 9,866
Space to write important points for revision

2011 - June [5] (b) Mrs. Arundhati aged 70 years furnishes, the following
details for the assessment year 2019-20: `
Family pension (gross) 1,32,000
Income from house property (computed) (62,000)
Income from horse racing (gross)(TDS deducted @ 30%) 55,000
Bank deposit interest 17,000
Short term capital loss from shares (1,00,000)
Agricultural income in India 75,000
Compute total income of Mrs. Arundhati and her tax liability. (6 marks)
Answer:
Particulars Amount (`)
Income from house property (A) (62,000)
Short term capital loss from shares ` 1,00,000 eligible for set
off only against capital gain. Nil
Income from other sources:
Family pension (gross) 1,32,000
7.300 O Scanner CMA Inter Gr. I Paper 7 (2016 Syllabus)

Bank deposit interest 17,000


Total income chargeable at normal rates under this head
other than income from horse races (B) 1,49,000
Income after set off [of (A against (B)] 87,000
Income from horse racing 55,000
Agriculture income exempt Nil
Total income 1,42,000
Tax on normal income (Agricultural income not to be
considered even for rate purposes) Nil
Tax payable @30% on horse racing as per Section 115BB 16,500
Less: Rebate u/s 87A 2,500
14,000
Add: Health and Education cess @ 4% 560
Tax Liability 14,560
Space to write important points for revision

2011 - June [8] (b) Mr. Ramachandran submits details of his income/savings
for the previous year 2018-19 as below: `
(i) Interest on capital from partnership firm 1,04,000
(ii) Other interest receipts:
(a) Interest on loan from Y 20,000
(b) Interest from Bank FDR 10,000
(iii) Long term capital gain 46,000
(iv) Contribution to PPF 50,000
Compute the tax liability of Mr. Ramachandran. (7 marks)
Answer:
Computation of total Income for assessment year 2019-20
Particulars ` `
Profit and gains of business or profession:
Income on capital from firm at 12% 1,04,000
Income from capital gain(long-term) 46,000
(i) Interest on loan from Y 20,000
[Chapter  14] Assessment of Various Persons O 7.301

(ii) Interest on bank FD 10,000 30,000


Gross total income 1,80,000
Less: Deduction u/s 80C PPF 50,000
Total Income 1,30,000
Tax liability Nil
Space to write important points for revision

2011 - Dec [3] (b) MITTAL (P) Ltd., converted into a Limited Liability
Partnership (LLP) by name All Trade LLP, with effect from 01.04.2018:
The following details are given to you:
`
Asst. year 2009-10: Business loss brought forward 2,00,000
Asst. year 2018-19: Business loss brought forward 5,00,000
(These are related to erstwhile MITTAL (P) Ltd.
Total income of All Trade LLP, for the financial year
2018-19 (Before set off of brought forward business
losses of erstwhile company i.e. MITTAL (P) Ltd.) 6,00,000
Assume that all the conditions prescribed in Section 47(xiiib) were satisfied
by (P) Ltd. at the time of conversion into LLP: (P)
(i) Explain whether All Trade LLP can set off and carry forward the
business loss of its predecessor i.e. MITTAL (P) Ltd.?
(ii) State whether any change in partners of All Trade LLP at later date
would have any tax consequence. (5 marks)
Answer:
(i) Sub-Section (6A), inserted by the Finance Act, 2010 in Section 72A,
provides that where a private company is succeeded by a LLP fulfilling
the conditions laid down in the proviso to Section 47(xiiib), then,
notwithstanding anything contained in any other provision of the
Income-tax Act, 1961, the accumulated loss and unabsorbed
depreciation of the predecessor company shall be deemed to be the
loss or allowance for depreciation of the successor LLP for the
purpose of the previous year in which the business reorganization was
effected and other provisions of the Act relating to set-off and carry
forward of losses and depreciation allowance shall apply accordingly.
7.302 O Scanner CMA Inter Gr. I Paper 7 (2016 Syllabus)

Therefore, All Trade LLP can carry forward and set-off the business
loss of ` 7 lakh of erstwhile. Mittal (P) Ltd. against its business income
for the F. Y. 2018-19. The unabsorbed business loss will be carried
forward to the next year.
(ii) Section 47(xiiib) requires that the shareholders of the company
become partners of the LLP in the same proportion as their
shareholding in the company. Further, the erstwhile shareholders of
the company should continue to be entitled to receive at least 50% of
the profits of the LLP for a period of 5 years from the date of
conversion. If the entity fails to fulfill this condition, the benefit of set-off
of business loss availed by the LLP would be deemed to be the profits
and gains of the LLP chargeable to tax in the previous year in which
the LLP fails to fulfill the condition.
Space to write important points for revision

2011 - Dec [5] (b) NIKITA Pvt. Ltd. is converted into NIKITA LLP on
1.1.2019. The following particulars are available to you:
`
(i) WDV of land as on 1.4.2018 5,00,000
(ii) WDV of machinery as on 1.4.2018 3,30,000
(iii) Patents acquired on 1.6.2018 3,00,000
(iv) Building acquired on 12.3.2017 for which deduction was
allowed under Section 35AD 7,00,000
(v) Above building was revalued as on the date of conversion
into LLP as 12,00,000
(vi) Unabsorbed business loss as on 1.4.2018 (A.Y. 2019-20) 9, 00,000
Though the conversion into LLP took place on 1.1.2018, there was disruption
of business and the assets were put in to use by the LLP only from 1st March,
2019 onwards.
The company earned profits of ` 8 lakhs prior to computation of depreciation.
Assuming that the necessary conditions laid down in Section 47 (xiiib)
of the Income-tax Act, 1961 have been complied with, explain the tax
treatment of the above in the hands of the LLP.
Note  “WDV of land as on 1.4.2018" may be read as “Cost of land”.
(7 marks)
[Chapter  14] Assessment of Various Persons O 7.303

Answer:
Tax treatment of depreciation and unabsorbed business loss of a private
company on its conversion into a LLP.
The LLP would be allowed to carry forward and set-off the business loss
and unabsorbed depreciation of the predecessor company [Section
72A(6A)].
1. Depreciation: The aggregate depreciation allowable to the predecessor
company and successor LLP shall not exceed, in any previous year, the
depreciation calculated at the prescribed rates as if the conversion had
not taken place. Such depreciation shall be apportioned between the
predecessor company and the successor LLP in the ratio of the number
of days for which the assets were used by them [Section 32(1)].
Therefore, depreciation has to be first calculated as if the conversion
had not taken place and then apportioned between the company and the
LLP in the ratio of the number of days for which the assets were used by
them.
Amount Dep. Rate Dep. Amount
` ` `
Block I Machinery 3,30,000 15% 49,500
Block II Patents 3,00,000 25% 75,000
1,24,500
Allocation of depreciation
Depreciation on machinery and patents have to be apportioned between the
company and the LLP in the ratio of the number of days for which the assets
were used by them. Since patents were acquired only on 1.6.2018, it could
have been used by the company for 214 days only. Therefore, the
depreciation on assets has to be allocated between the company and LLP
as follows:
Company LLP
Asset Total depreciation No. of days Depreciation No. of days Depreciation
for the year of usage of usage
7.304 O Scanner CMA Inter Gr. I Paper 7 (2016 Syllabus)

Company LLP
Assets Total Dep. No. of the Depreciation No. of Depreciation
For the year days of days of
usage usage
Machinery 49,000 275 44,485 31 5,015
Patents 75,000 214 65,510 31 9,490
1,24,500 1,09,995 14,505
Therefore, depreciation to be allowed in the hands of the company is
` 1,09,995 and depreciation to be allowed in the hands of the LLP is
` 14,505.
2. Unabsorbed business loss to be carried forward by the LLP.
Particulars `
Profits of the company before depreciation 8,00,000
Less: Current year depreciation 1,09,995
Business income of the company after depreciation 6,90,005
Brought forward business loss 9,00,000
Unabsorbed business loss as on 31.12.2018 to be carried
forward by the LLP 2,09,995
3. Actual cost of assets to the LLP
1. The actual cost of the block of assets in case of the LLP shall be the
WDV of the block of assets as in the case of the company on the
date of conversion. The WDV as on 1.1.2019 for Machinery and
Patents are ` 2,85,515 and ` 2,34,490, respectively, which would be
the actual cost in the case of the LLP.
WDV of Machinery as on 1.1.2019 = ` 3,30,000 - 44,485 = ` 2,85,515
WDV of Patents as on 1.1.2019 = ` 3,00,000 – 65,510 = ` 2,34,490
2. Land is not a depreciable asset. The cost of acquisition of land to
the LLP would be the cost for which the company acquired it, as
increased by the cost of improvement.
In respect of the building, deduction had been allowed in the earlier
year under Section 35AD, there is no question of depreciation
during the current financial year.
Space to write important points for revision
[Chapter  14] Assessment of Various Persons O 7.305

2012 - June [4] (d) The following are the income details of Padmaja Textiles
LLP, for the assessment year 2019-20:
`
Book profits of the LLP (before considering interest and
remuneration to partners) 15,00,000
Interest to partners at 15% 2,10,000
Remuneration to working partners as per books 4,50,000
Permissible deduction u/s 80-ID 6,20,000
Compute the tax payable by the LLP for the assessment year 2019-20.
(5 marks)
Answer:
Padmaja Textiles
Computation of Taxable income
Particulars `
Book profits of the LLP 15,00,000
Less: Interest on capital at 12% 1,68,000
13,32,000
Less: Remuneration to working partners u/s 40(b) 4,50,000
Gross profit 8,82,000
Less: Deduction u/s 80-ID 6,20,000
Total income as per normal provisions 2,62,000
Tax as per normal provisions at 30% 78,600
Add: Health and Education cess @4% 3,144
Total tax payable 81,744
Alternate minimum tax
Adjusted income (2,62,000 + 6,20,000) 8,82,000
MAT at 18.5% 1,63,170
Add: Health and Education Cess @ 4% 6,527
Total tax payable 1,69,697
7.306 O Scanner CMA Inter Gr. I Paper 7 (2016 Syllabus)

Since MAT is more than normal tax, the same has to be paid.
Tax payable by the LLP is 1,69,697
Working Note:
Remuneration of working partners u/s 40(b)
First 3,00,000 at 90% 2,70,000
Balance at 60% 6,19,200
8,89,200
Remuneration as per books being lower, the same should be taken.
Space to write important points for revision

2012 - June [6] (b) Mr. Ram, the karta of a Hindu Undivided Family (HUF)
invested family funds of ` 5 lakhs in the shares of Hanumaan Couriers P.
Ltd. He was appointed as the Managing Director of the company and was
paid a remuneration of ` 3.6 lakhs for the year ended 31.3.2019. Discuss
whether the said remuneration will be assessed in the individual hands of Mr.
Ram or in the hands of the HUF, for the assessment year 2019-20.
(4 marks)
Answer:
Remuneration is Individual income or income of HUF, if Karta is appointed
as MD of the company.
Mr. Ram as Managing Director in the company was on account of investment
of family funds in the company, the payment of remuneration involves a
consideration for services rendered by Mr. Ram, in his personal capacity, to
the company.
In the recent case decided by Supreme Court it was held the remuneration
and commission received by the Karta of HUF on account of his personal
qualification and exertions and not on account of investments of the family
funds in the company be treated as income of HUF.
Hence, in this case the remuneration received as Managing Director is
assessable as the individual income of Mr. Ram and not as that of the HUF.
Space to write important points for revision
[Chapter  14] Assessment of Various Persons O 7.307

2012 - June [8] (b) Compute total income of Mr. Malik for the assessment
year 2019-20 from the following particulars:
`
Income from house property (computed) (1,40,000)
Income from salary (computed) 4,60,000
Long-term capital gain from sale of vacant site (computed) 2,00,000
Business loss—computed (automobile business) 80,000
Income from growing and manufacturing coffee
(cured and roasted) 1,00,000
Tuition fees for higher education pursued by Mrs. Malik 25,000
Tax saver deposit in the name of Mrs. Malik 10,000
Public provident fund deposit in the name of minor son Pervez 15,000
(8 marks)
Answer:
Computation of total income of Mr. Malik for the assessment year 2019-20
Particulars ` `
Income from salary - computed 4,60,000
Income from house property - set off (1,40,000)
3,20,000
Long term capital gain from sale of vacant site
(computed) 2,00,000
Business loss computed (automobile business)* 55,000
Income from growing and manufacturing coffee 1,00,000 1,45,000
(cured and roasted)
Less: Income eligible for exemption under rule 7B 75,000
@75%
Business income 25,000
Less: Automobile business loss 25,000 Nil
Gross total income 4,65,000
Tax saver deposit in the name of Mrs. Malik
` 10,000 not eligible for deduction Nil
Public provident fund deposit in the name of minor 15,000 15,000
son Pervez ` 15,000 eligible for deduction
Total income 4,50,000
7.308 O Scanner CMA Inter Gr. I Paper 7 (2016 Syllabus)

Tax on regular income ` 3,05,000 ( 5% above of


` 2,50,000) 2,750
On long term capital gain ` 1,45,000 @ 20% 29,000
31,750
Less: Rebate u/s 87 A -
31,750
Add: Health and Education Cess @ 4% 1,270
Tax Liability 33,020
Tax Liability (R/O U/S 288 B) 33,020

* Business loss ` 80,000 of which ` 25,000 set off against income from
growing and manufacturing coffee and the balance is set off against capital
gain.
Space to write important points for revision

2014 - June [5] (b) ABC LLP furnishes you the following details pertaining
to the financial year 2018-19 :
`
Net Profit as per Profit and Loss Account 90,00,000
Depreciation debited in Profit and Loss Account 7,00,000
Depreciation allowable under Section 32 9,00,000
Inadmissible/disallowable expenses 5,00,000
Deduction under Section 10AA (computed) 12,00,000
Deduction under Section 80IA (computed) 60,00,000
Compute total income, adjusted total income under Section 115JC and tax
liability of ABC LLP for the assessment year 2019-20. (6 marks)
Answer:
Computation of Total Income
Net profit as per P/L 90,00,000
+ Depreciation debited in P/L 7,00,000
 Depreciation allowable u/s 32 9,00,000
+ Inadmissible/disallowable expenses 5,00,000
93,00,000
[Chapter  14] Assessment of Various Persons O 7.309

 Deduction u/s 10AA 12,00,000


 Deduction u/s 80 -IA 60,00,000
21,00,000
Adjusted Total Income u/s 115JC
Total Income 21,00,000
+ Deduction u/s 10AA 12,00,000
+ Deduction u/s 80 -IA 60,00,000
93,00,000
Tax Liability 93,00,000×18.5% 17,20,500
Health and Education cess @ 4% 68,820
Tax Liability 17,89,320
Space to write important points for revision

2014 - Dec [3] (a) The Profit & Loss Account of M/s. Saxena & Chaturvedi,
Cost Accountants, shows a net profit of ` 25,50,000 after debiting/crediting
the following items:
(i) Interest of ` 1,40,000 on capital of Mr. Saxena, partner calculated at
14% per annum.
(ii) Remuneration ` 35,00,000 to Mr. Saxena and Mr. Chaturvedi, who are
working partners.
(iii) Salary of ` 40,000 to Mr. Chatterjee, Manager for February, 2019 was
paid by bearer cheque on 1st March, 2019.
(iv) Depreciation ` 1,20,000.
(v) Professional fee of ` 45,000 was paid to a lawyer for obtaining a legal
opinion. No tax was deducted at source.
(vi) A sum of ` 30,000 was paid to a trainee as a special award for ranking
first in Final Examination of the Institute of Cost Accountants of India.
(vii) Refund of penalty ` 35,000 paid in the financial year 2016-17 relating
to delayed payment of service tax, after decision of the appellate
authority in favour of the assessee.
(viii) Interest on fixed deposit ` 5,00,000.
The firm is entitled to depreciation of ` 1,45,000 under Section 32.
Compute total income of the firm for the assessment year 2019-20.
(9 marks)
7.310 O Scanner CMA Inter Gr. I Paper 7 (2016 Syllabus)

Answer:
Computation of total income of M/s. Saxena & Chaturvedi for
Assessment Year 2019 -20:

Particulars ` `

Profit & gains from business or profession

Net profit as per Profit & Loss Account 25,50,000

Add: Interest to Mr. Saxena in excess of 12%


p.a., disallowed under section 40(b)
(` 1,40,000 x 2/14) 20,000

Remuneration to partners considered


separately 35,00,000

Disallowance under section 40A(3) for


payment of salary to the manager otherwise
than by account payee cheque or account
payee bank draft 40,000

Depreciation as per books 1,20,000

Disallowance under section 40(a)(ia) for


payment of lawyer’s fee without deduction of
tax at source 15,000 36,95,000

62,45,000

Less: Refund of penalty not taxable (Note 1) 35,000

Interest on fixed deposit considered under


other head 5,00,000

Depreciation under section 32 1,45,000 6,80,000

Book Profit under section 40(b) 55,65,000


[Chapter  14] Assessment of Various Persons O 7.311

Remuneration admissible under section


40(b):

On first ` 3,00,000: 90% of book profit or


` 1,50,000, whichever is higher 2,70,000

On balance of book profit (i.e., 55,65,000 –


3,00,000) at 60% 31,59,000 34,29,000

Business Income 21,36,000

Income from other sources

Interest on fixed deposit 5,00,000

Total income 26,36,000

Notes:
1. Penalty for infraction of law is never allowed as deduction under
section 37(1). Therefore, penalty for delayed payment of service tax
was disallowed in financial year 2016-17. As penalty was disallowed,
refund of such penalty as a result of favourable appeal order does not
attract deeming provision of Section 41(1). Hence, it is not taxable.
2. Special award was given to trainee for excelling in final examination
of the institute to motivate him and in recognition of his caliber.
Payment was made to boost his morale so that he could contribute
more to the assessee firm. Thus, it has a nexus with the profession
carried on by the firm and hence, the same is allowable is deduction
under section 37(1). As it is debited to profit & loss account, no
further adjustment is required.
Space to write important points for revision

2015 - June [5] (c) Mohit Sharma gives you the following information for the
year ended 31.03.2019:
Owns 3 goods carriages throughout the financial year 2018-19
Retail trade turnover ` 36,00,000
7.312 O Scanner CMA Inter Gr. I Paper 7 (2016 Syllabus)

Has eligible brought forward depreciation of the assessment year 2016-17


` 60,000 relating to retail trade.
Deposited ` 80,000 in PPF account and ` 90,000 in tax saver deposit.
Assume that he wants to offer income by opting for Sections 44AD and
44AE.
Compute his total income for the assessment year 2019-20 (5 marks)
Answer:
Computation of total income of Mr. Mohit Sharma for the Assessment
Year 2019-20
Particulars ` `
Income U/s 44AD from retail trade @ 8% on 2,88,000
` 36,00,000
Income U/s 44AE in respect of plying of goods carriages ` (7,500 2,70,000
× 3 × 12)(It is assumed small vehicles)
Business income prior to set off 5,58,000
Less: Brought depreciation not eligible for set off as the b/f ----
depreciation is deemed to be current year depreciation and has
been deducted while computing income under section 44AD
Gross Total Income 5,58,000
Less: Deduction U/s 80C
In respect of PPF contribution 80,000
In respect of Tax saver deposit 90,000
Maximum amount deductible 1,50,000
Total Income 4,08,000
Note:
Amendment to [Section-44AE]
Section 44AE(2) has been substituted (with effect from the assessment year
2019-20) so as to provide that for a heavy goods vehicle, the profits and
gains shall be an amount equal to ` 1,000 per ton of gross vehicle weight (or
unladen weight) for every month (or part of a month) during which the heavy
goods vehicle is owned by the assessee in the previous year or an amount
claimed to have been actually earned from such vehicle, whichever is higher.
[Chapter  14] Assessment of Various Persons O 7.313

In the case of a goods carriage other than heavy vehicle, the profits an gains
shall be an amount equal to ` 7,500 for every month (or part of a month)
during which the goods carriage is owned by the assessee in the previous
year or an amount claimed to have been actually earned from such goods
carriage, whichever is higher.
For this purpose, “heavy goods vehicle” means any goods carriage the gross
vehicle weight of which exceeds 12,000 kilograms.
Space to write important points for revision

2015 - Dec [3] (a) Mr. Shiva (age 62) having 4 heavy goods vehicles and 3
non-heavy goods vehicles wants to show the income chargeable to tax on
presumptive basis. During the year 2018-19, he received interest on
enhanced compensation of ` 5 lakhs relating to compulsory acquisition of
land (made in the year 1991). He borrowed ` 3 lakhs for renovation of a let
out property from which he earned rental income of ` 10,000 per month.
Interest on borrowal for house renovation for the year is ` 42,000. Compute
his total income and income tax liability for the assessment year 2019-20.
(5 marks)
Answer:
Computation of total Income of Mr. Shiva
Profit from business 8,46,000
Income from other sources 2,50,000
Income from house property 1,54,000
Total income 12,50,000
Computation of tax liability:
0 to 3,00,000 — Nil
3,00,001 to 5,00,000 @ 5% — 10,000
5,00,001 to 10,00,000 @ 20% — 1,00,000
10,00,001 to 12,50,000@30% 75,000
Tax Payable 1,85,000
Add: Health & Education cess @ 4% 7,400
Total Tax 1,92,400
7.314 O Scanner CMA Inter Gr. I Paper 7 (2016 Syllabus)

Working Notes:
1. Income from business:
Income = 7 truck 4 heavy & 3 Non heavy goods vehicles
= (3 × 7,500 × 12) + (4 × 12,000 × 12)
= 8,46,000
2. Income from other sources:
Interest on enhanced compensation received – 5,00,000
Less: 50% as deduction 2,50,000
2,50,000
3. Income from house property:
Rent (10,000 × 12) 1,20,000
— Standard deduction @ 30% 36,000
— Interest on loan 30,000
54,000

Note: Amendment to [Section-44AE]


1. Section 44AE (2) has been substituted (with effect from the
assessment year 2019-20) so as to provide that for a heavy goods
vehicle, the profits and gains shall be an amount equal to ` 1,000 per
ton of gross vehicle weight (or unladen weight) for every month (or
part of a month) during which the heavy goods vehicle is owned by the
assessee in the previous year or an amount claimed to have been
actually earned from such vehicle, whichever is higher.
In the case of a goods carriage other than heavy vehicle, the profits
and gains shall be an amount equal to ` 7,500 for every month (or part
of a month) during which the goods carriage is owned by the assessee
in the previous year or an amount claimed to have been actually
earned from such goods carriage, whichever is higher.
For this purpose, “heavy goods vehicle” means any goods carriage the
gross vehicle weight of which exceeds 12,000 kilograms.
Space to write important points for revision
[Chapter  14] Assessment of Various Persons O 7.315

2015 - Dec [4] (c) Mr. Suresh Raina sold a residential building for
` 75,00,000 on 1st July, 2018. It was acquired for ` 22,04,000 on 1st June,
2009. The stamp duty valuation of the property at the time of acquisition was
` 25,00,000 and at the time of transfer was ` 85,00,000. He paid brokerage
at 1% at the time of transfer. He deposited ` 40,00,000 in bonds of Rural
Electrification Corporation Ltd. in March, 2019 and deployed the balance in
a business commenced by him. He has business loss of ` 5,00,000 for the
year ended 31st March, 2019. Compute total income of Mr. Suresh Raina and
advise by what date he must file his return of income.
Cost inflation index - F. Y. 2009-10: 148; F. Y. 2018-19: 280 (5 marks)
Answer:
Computation of income from capital gain
Sales consideration 85,00,000
 Expense on sale 75,000
84,25,000
 Cost of acquisition 22,04,000 × 41,69,730

Long term capital gain 42,55,270


Computation of total income of Mr. Raina
Long term capital gain 40,49,175
Income from other sources 2,96,000
Loss from business (5,00,000)
Total income of Mr. Raina 38,45,175
Mr. Raina must file his return of income on or before 31 July 2019.
Calculation of income from other sources:
Stamp duty valuation at the time of acquisition 25,00,000
— Cost of acquisition 22,04,000
2,96,000
As per Sec. 56 (2)(x) the difference of stamp duty valuation and cost of
acquisition is more than ` 50,000 (positive) the excess of stamp duty
valuation over such cost of acquisition shall be taxed under head “Income
from other sources”.
Space to write important points for revision
7.316 O Scanner CMA Inter Gr. I Paper 7 (2016 Syllabus)

2016 - June [3] (b) From the following particulars compute total income of
Mr. Chatterjee for the assessment year 2019 - 20:
(i) Basic salary ` 50,000 per month.
(ii) Bonus for the year 2017-18 received in October 2018 ` 30,000.
(iii) Commission for the year 2018 -19 but not received till 31.03.2019
` 70,000.
(iv) Reimbursement of medical expenses on production of bills ` 20,000.
(v) Education allowance for 2 children paid by the employer ` 24,000.
(vi) Maid servant’s salary reimbursed by the employer ` 30,000.
(vii) Income from cultivation and manufacture of rubber in the rubber estate
owned in Kerala ` 1,00,000.
(viii) Tax on non-monetary perquisites paid by the employer ` 35,000.
(9 marks)
Answer:
Computation of total income of Mr. Chatterjee for the assessment year
2019-20
` `
Basic Salary 6,00,000
Bonus of year 2017-18 is taxable on due basis and Nil
hence it is not taxable on actual receipt.
Note: It is also possible to take the view that the bonus
for the year 2017-18 had not been ascertained then,
hence had not been taxed in the said year and
consequently is taxable in the current year of receipt.
Commission due for the year– taxable on due basis 70,000
Medical reimbursement liable to tax w.e.f. A.Y. 2019-20 20,000
Educational allowance 24,000
Less: Exempt U/s. 10 (14)@ ` 100 per child for two 2,400
children
21,600
Maid’s salary reimbursed–taxable 30,000
[Chapter  14] Assessment of Various Persons O 7.317

Tax on non–monetary perquisite paid by employer – Nil


exempt under section 10(10CC)
Gross Salary 7,41,600
Less: Deduction u/s 16(i)
Standard deduction income from salary 40,000
Income from business 7,01,600
Income from rubber estate at 35% 35,000
Income from Rubber estate from manufacture of rubber
is exempt at 65% as per Rule 7A
Total Income 7,36,600
Space to write important points for revision

2016 - June [4] (b) Mr. Anjan, an individual, aged 40 years having gross total
income of ` 5,00,000 (including long-term capital gain from sale of land
amounting to ` 3,90,000) during previous year 2018-19. He is entitled to get
deduction under Chapter V1-A for ` 1,79,000 consisting of deductions
permissible under sections 80C, 80D and 80DD for ` 95,000, ` 4,000 and
` 80,000 respectively.
Compute income tax payable by Mr. Anjan for assessment year 2019-20.
(5 marks)
Answer:
Computation of income tax payable by Mr. Anjan for assessment year
2019-20
Particulars `
Gross Total Income 5,00,000
Less: Deduction under chapter VI-A
Eligible deduction ` 1,79,000
Deduction under chapter VI-A cannot be claimed from long-term
capital gain. Hence deduction is restricted to other income i.e.
` 5,00,000 – ` 3,90,000= ` 1,10,000 1,10,000
Total Income (representative long–term capital gain only) 3,90,000
7.318 O Scanner CMA Inter Gr. I Paper 7 (2016 Syllabus)

Tax on other income Nil


On long–term capital gain @ 20% of (` 3,90,000–`2,50,000) 28,000
Less: Rebate under section 87A Nil
28,000
Add: Health and Education Cess @ 4% 1,120
Total Tax Liability 29,120
Space to write important points for revision

2016 - June [6] (a) Mr. Manoj gives you the following particulars of his
income for the year ended 31st March, 2019:
Serial Particulars `
No.
1. Interest on loan given to a friend (non-relative) 80,000
2. Interest on public provident fund 21,500
3. Winning from crossword puzzle (net) 70,000
4. Directors fee from a company 25,000
5. Royalty on a book written by him 1,50,000
6. Expenses for typing the manuscript of the book 15,000
7. Cash gift from father in law 2,00,000
Compute the income of Mr. Manoj for the assessment year 2019-20.
(8 marks)
Answer:
Computation of Total Income of Mr. Manoj for the Assessment year
2019-20
Particulars ` `
Income from Other sources:
Interest on loan 80,000
Interest on public provident fund 21,500
Less: Exemption U/s.10(15) 21,500
Nil
[Chapter  14] Assessment of Various Persons O 7.319

Winning from crossword puzzle ` 70,000 × 100/70 1,00,000


Directors fee from a company 25,000
Royalty on book 1,50,000
Less: Expenses allowable U/s. 57 15,000
1,35,000
Cash gift father-in-law-relative hence not taxable Nil
Total Income 3,40,000
Space to write important points for revision

2016 - Dec [6] (b) Determine the total income of Mr. Raman from the
following details for the Assessment Year 2019-20:
Particulars ` In lakhs
Loss from house property 3.20
Income from salary 2.20
Income from non-speculation business 1.60
Loss from speculation business 5.40
Long-tem capital gain from sale of land 2.40
Long-tem capital gain from sale of listed shares in recognised 1.80
stock exchange
Short-term capital loss from sale of listed shares in recognised 2.80
stock exchange
Family pension received 2.20
Lottery winnings (net of TDS) 2.10
Lottery tickets purchased 0.30
Show clearly the items to be carried forward and those which cannot be
carried forward. (6 marks)
7.320 O Scanner CMA Inter Gr. I Paper 7 (2016 Syllabus)

Answer:
Computation of total Income of Mr. Raman: for the A.Y. 2019-20
Income from Salary: 2,20,000
Income from House Property (Loss): (3,20,000)
Income from Business:
Non Speculation Business 1,60,000 1,60,000
Loss from Speculation Business (5,40,000)
Capital Gain:
ST CG on Listed from Listed Securities (2,80,000)
LT CG on Sale of Land 2,40,000
LT CG on Sale of Listed Securities exempt
u/s 10(38) Exempt —
Income from Other Sources:
Family pension 2,20,000
Less: Deduction u/s 57 15,000 2,05,000
Income from Lottery 2,10,000
Add: TDS 90,000 3,00,000 5,05,000
Inter head set off (Adjustment) u/s 71:
House Property loss set off from Salary Income (2,20,000)
House Property loss set off from Business
Income (1,00,000)
Gross Total Income 5,65,000
Being Total Income
Losses to be carried forward
(1) Loss from Speculation Business 5,40,000
(2) ST CL on Sale of Listed Securities 40,000
Space to write important points for revision
[Chapter  14] Assessment of Various Persons O 7.321

2017 - June [3] (b) Rao and Jain is a partnership firm, consisting of 5
partners, with turnover of ` 1,20,00,000 for the year ended 31.03.2019. The
partnership deed provides for interest on capital at 14% per annum on the
capital contribution of ` 5 lakhs each made by all the partners. All the
partners are eligible for monthly working partner salary of ` 10,000 each. The
firm provides you the following additional information:
Depreciation eligible under Income-tax Rules, 1962 ` 2,00,000
Interest paid on unsecured loans for which no tax was ` 5,00,000
deducted at source during the year or before the due date for
filing return of income u/s 139(1)
Contract payments made during the year for which tax was ` 4,00,000
deducted but remitted in financial year 2019-20 and before
‘due date’ for filing the return of income specified in Section
139(1).
Rent paid to a partner Mr. Jain for premises occupied by the firm ` 1,20,000
on which no tax was deducted at source. The reasonable rent for similar
premises was determined at ` 1,80,000.
The Net Profit of the firm before charging interest on capital and working
partner salary as per books was ` 8,40,000. Depreciation notionally
computed and provided in the books amounts to ` 1,60,000.
The partners of the firm want you to compute income under section 44AD
and also as per regular provisions, and suggest which option would be
beneficial to them. (9 marks)
Answer:
Computation of Total Income
of Partnership firm Rao & Jain
for the A.Y. 2019-20
under Section 44AD
Particulars Amount (`)
Total Turnover 1,20,00,000
Deemed Income u/s 44AD 8% of Gross Turnover 9,60,000
7.322 O Scanner CMA Inter Gr. I Paper 7 (2016 Syllabus)

Tax on Deemed Total Income @ 30% 2,88,000


Add: Heath and Education cess @ 4% 11,520
Total Tax Payable 2,99,520
Note:
 Deduction of Remuneration to partners and Interest on capital to
partners will not allowed from Deemed Income u/s 44AD w.e.f. A.Y.
2019-20.
 Computation of Total Income of partnership firm Rao & Jain for the A.Y.
2019-20 (under Normal Provisions of the Income Tax Act.)
Particulars Amount Amount
(`) (`)
Net Profit as per Profit & Loss Account 8,40,000
Add: Expenses disallowed
Depreciation 1,60,000
30% of Interest paid on unsecured Loans 1,50,000 3,10,000
11,50,000
Less: Expenses Allowed but not debited
Depreciation as per Income Tax Rules 2,00,000
Less: Interest on capital to working partners not
more than 12% (25,00,000 × 12%) 3,00,000 5,00,000
Book Profit 6,50,000
Less: Remuneration to working partners.
Actual Remuneration
(10,000 × 12 × 5) = 6,00,000
or
[Chapter  14] Assessment of Various Persons O 7.323

Statutory Limit an book profit


on first ` 3,00,000 @ 90% 2,70,000
on Bal ` 3,50,000 @ 60% 2,10,000
4,80,000 4,80,000
whichever is Less Income from Business 1,70,000
Tax payable on ` 1,70,000 @ 30% 51,000
Add: Health and Education cess @ 4% 2,040
Tax Payable 53,040
Note: It is in the interest of the firm to maintains the Books of Accounts and
get done tax Audit because the Tax liability in case of computation of Total
Income and tax under regular provisions of Income Tax Act is less.
Space to write important points for revision

2017 - June [4] (b) Ms. Vidya residing in Chennai acquired a residential
house for ` 15,25,000 on 28th May, 2007. It was sold for ` 100 lakhs in July
2018. The stamp duty valuation on the date of sale was ` 110 lakhs. She
paid brokerage @2% of sale consideration and on which no tax was
deducted at source.
She deposited ` 40 lakhs in REC Capital Gain Bonds in September, 2018
and ` 20 lakhs in NHAI Capital Gain bonds in February 2019. She acquired
a residential property in Colombo for ` 50 lakhs and left for Colombo in
August 2018 and occupied the said property. She returned to India in
September 2018 leaving the property vacant till the date of sale.
Her other incomes include (i) ` 1,50,000 by way of interest on capital @ 15%
from a firm at Salem; and (ii) income from rubber estates and manufacture
of rubber in Kerala amounting to ` 3 lakhs., where rubber is grown and
processed by her.
She paid ` 35,000 towards health insurance of her parents who are senior
citizens through credit card and paid in cash ` 8,000 towards master health
checkup for herself.
Cost Inflation Index: F.Y. 2007-08= 129; F.Y.2018-19:280
Compute the total income of Ms. Vidya for the Assessment Year 2019-20
under proper heads of income. Ignore DTAA provisions. (8 marks)
7.324 O Scanner CMA Inter Gr. I Paper 7 (2016 Syllabus)

Answer:
Computation of Total Income
of Mr. Vidya
for the A.Y. 2019-20
Particulars Amount (`) Amount (`)
Income from Business
Interest on capital from firm upto 12% 1,20,000
Income from Rubber Estate 3,00,000
Less: Agricultural Income 65% 1,95,000 1,05,000 2,25,000
Income from capital gains
Full value of consideration:
Actual sale consideration 1,00,00,000
or
Stamp Duty valuation
(which ever is more) 1,10,00,000
1,10,00,000
Less: Brokerage @ 2% 2,00,000
Net sale consideration 1,08,00,000
Less: Index Cost of Acquisition

33,10,078
Long Term capital gain 74,89,922
Less: Exemption u/s 54
purchase of Residential
property (out of India) NIL
U/s 54EC Investment
in REC capital gain
Bonds 40,00,000
Subject to Maximum upto Long Term
capital gain 40,00,000 34,89,922
Gross Total Income 37,14,922
[Chapter  14] Assessment of Various Persons O 7.325

Less: Deduction u/s 80D


In respect of health insurance of
parents limited to ` 50,000 35,000
In respect of self preventive health
checkup 5,000 40,000
Total Income 36,74,922
Notes:
1. Deduction under Section 80D: Maximum upto ` 30,000 including
preventive health checkup.
2. Exemption U/s 54EC: Maximum upto 50 lakhs subject to investment
made with in 6 months prom the date of transfer of Capital Asset.
Space to write important points for revision

2017 - June [5] (a) Mr. Chirag has given the following details relating to
financial year 2018-19:
(i) Received ` 56,000 by way of gift from his friends on the occasion of
his marriage.
(ii) Purchased a land at Kanpur for ` 12,50,000 for construction of a
residential house from a friend. The stamp duty value of the land on
the date of purchase was ` 15,00,000.
(iii) Interest amounting to ` 1,80,000 relating to earlier years, on enhanced
compensation received during the year. Legal expenses incurred
` 25,000.
(iv) Received loan of ` 3,50,000 from CNK Private Limited in which Mr.
Chirag holds 12% voting power. Accumulated profit in the hands of the
company at the time receipt of loan was ` 2,90,000.
Briefly narrate the tax consequences of the aforesaid items, sharing clearly
the amount to be taxed in each case. (9 marks)
Answer:
(i) Gift Received from friends of ` 56,000 on the occasion of his marriage
is not taxable U/s 56(2)(vii).
7.326 O Scanner CMA Inter Gr. I Paper 7 (2016 Syllabus)

(ii) Since the Difference between stamp duty value and actual purchase
price is more than ` 50,000 i.e. (15,00,000 - 12,50,000 = 2,50,000) is
liable to tax under the head Income from other sources U/s 56(2)(vii).
(iii) Interest on enhanced compensation received ` 1,80,000 is taxable
under the head Income from other sources under section 56(2)(viii)
and 50% of such interest is eligible for deduction u/s 57(iv).
(iv) Loan Received by Mr. Chirag from CNK Private Limited will be treated
as dividend U/s 2(22)(e) upto the amount of Accumulated profit in the
hands of the company on the date of loan given by the company if Mr.
Chirag holds 10% or more voting power in the company.
In the given case Amount of ` 2,90,000 out of ` 3,50,000 will be
treated as dividend U/s 2(22)(e).
Note:
Amendment to Section 115-O and 2(22)(e):
With effect from 1st April 2018, loan or advance given by a company (on
or after April 1, 2018) which is deemed as dividend under section 2(22)(e)
will be subject to the following provisions–
1. On such deemed dividend, the company which given loan or advance
will have to pay dividend tax under section 115-O at the rate of 30%
(without grossing up)[+12% of such tax as surcharge + 4% of tax and
surcharge as health and education cess, effective rate: 34.944%.]
2. Such deemed dividend will be exempt in the hands of recipient by
virtue of section 10(34).
Space to write important points for revision

2017 - June [6] (a) Mr. Sen G. gupta (aged 65 years) is a retired person
drawing a monthly pension of ` 6,000. His taxable long-term capital gain
from sale of painting during the previous year 2018-19 is ` 2,75,000. He has
no other income during the year.
Compute his tax liability for Assessment Year 2019-20 (i) if he is resident and
(ii) if he is non-resident. (8 marks)
[Chapter  14] Assessment of Various Persons O 7.327

Answer:
Computation of Total Income
and Tax Liability
of Mr. Sengupta (Aged 65 years)
(i) When he is resident
Income from salary pension (6,000 × 12) 72,000
Less: Standard Deduction 40,000 32,000
Long Term capital gain 2,75,000
Gross Total Income 3,07,000
Less: Deductions under chapter VI-A NIL
Total Income 3,07,000
Tax on Total Income 1,400
Less: Relief under section 87A 1,400
Net Tax Payable NIL
Add: Health and Education cess @ 4% NIL
Tax Payable NIL
Tax Payable Rounded off NIL
(ii) When he is non resident
Tax on income other than long-term capital gain Nil
Tax on long-term capital gain: 20% of ` 2,75,000 (Note) 55,000
Heath and Education cess @ 4% 2,200
Tax liability 57,200
Note:
Where the individual assessee is non-resident, he is not entitled to deduct
the ex cess of basic exemption limit over other income from long-term capital
gain for computing tax liability (Section 112).
Space to write important points for revision
7.328 O Scanner CMA Inter Gr. I Paper 7 (2016 Syllabus)

2017 - Dec [2] (b) State the implications of the following transactions carried
out by Kalai & Co. a partnership firm (Whose turnover always exceeded
` 500 lakhs) with reverence to the provisions applicable for the assessment
year 2019-20:
(i) Audit fees of ` 35,000 paid by electronic transfer but no tax was
deducted at source.
(ii) Arrear salary of ` 60,000 paid in cash to an employee who was posted
in a ship for 10 days continuously. Tax was deducted at source on the
total salary paid to the employee during the year.
(iii) Lorry freight paid by cash ` 30,000.
(iv) ` 3 lakhs freight paid to Indian Railways without deduction of tax at
source.
(v) Salary paid to a son of a partner ` 20,000 per month. The market rate
of such salary for similar qualification is found to be ` 15,000 per
month.
(vi) Income tax paid in cash ` 22,000.
(vii) Interest on term loan paid to Canara Bank ` 18,000 without deduction
of tax at source.
(viii) Interest on capital paid to partners at 15% in accordance with the
condition contained in the partnership deed.
(ix) Keyman insurance policy premium paid ` 40,000. (9 marks)
Answer:
(i) 30% of Audit fees of ` 35,000. i.e. ` 10,500 will be disallowed.
(ii) Arrear of salary of ` 60,000 paid in cash will be disallowed, there is no
need of adjustment.
(iii) Lorry freight paid by cash upto ` 35,000 allowed therefore no
adjustment required.
(iv) TDS provisions are not applicable to payment made in respect of
Railway freight.
(v) Excess salary paid to a son of a partner disallowed i.e. ` 5,000 per
month will be disallowed.
(vi) Income tax is a personal liability of partnership firm, not an expense.
Therefore not allowed as expense.
[Chapter  14] Assessment of Various Persons O 7.329

(vii) Interest on term loan paid to Canara Bank of ` 18,000 is an allowed


expense. There is no need of TDS.
(viii) Interest on capital paid to partners allowed at 12%, therefore 3%
interest will be disallowed and added to profit.
(ix) Keyman insurance policy premium paid ` 40,000 is an allowed
expense. There is no need of adjustment.
Space to write important points for revision

2017 - Dec [3] (a) Mr. Rajiv, a resident individual, engaged in a wholesale
business of health products. He is also a partner in XYZ & Co., a partnership
firm. The following details are made available for the year ended 31.03.2019:
Sl. Particulars ` `
No.
(i) Interest on capital received from XYZ & 1,50,000
Co., at 15%
(ii) Interest form bank on fixed deposit 13,500
(Net of TDS ` 1,500)
(iii) Income - tax refund received relating 34,500
to assessment year 2016-17 including
interest of ` 2,300
(iv) Net Profit from wholesale business 5,60,000
Amounts debited include the following:
Depreciation as per books
Motor car expenses 34,000
Municipal taxes for the shop 40,000
(For two half years: payment for 7,000
one half year made on 12.07.2019
and for the other on 31.12.2019)
Salary to manager by way of a
single cash payment 21,000
7.330 O Scanner CMA Inter Gr. I Paper 7 (2016 Syllabus)

(v) The WDV of the assets (as on


01.04.2018) used in above wholesale
business is as under:
Computers 1,20,000
Motor Car (20% used for personal use) 3,20,000
(vi) LIP paid for independent son 60,000
PPF of his wife 70,000
You are required to compute the total income of the Mr. Rajiv for the
assessment year 2019-20 and the closing WDV of each block of assets.
(9 marks)
Answer:
Computation of total income of Mr. Rajeev

Income from Business


Net profit from whole sale business 5,60,000
Add: Adjustment for expenses debited to P&L
A/c
Depreciation as per books 34,000
Motor car expenses 20% 8,000
Municipal taxes for the shop 50% 3,500
Salary to manager paid in cash 21,000 66,500
Less: Expenses Allowed 6,26,500
Depreciation
Computers @ 40% 48,000
Motor car @ 15%
38,400
86,400
5,16,000
Interest on capital upto 12% 5,40,100
[Chapter  14] Assessment of Various Persons O 7.331

Income from other Sources 6,60,100


(i) Interest from Bank on FD (13,500 + 1,500) 15,000
(ii) Interest on Income Tax Refund 2,300 17,300
Gross Total Income 6,77,400
Less: Deduction under chapter VI-A
LIP paid for independent son 60,000
PPF of his wife 70,000 1,30,000
Total Income 5,47,400
Closing WDV of Assets
Computers (1,20,000 - 48,000) 72,000
Motor car (3,20,000 – 48,000) 2,72,000
Space to write important points for revision

2017 - Dec [4] (a) Following is the Profit and Loss Account of Mr. Abdul for
the year ended 31.03.2019:
Particulars ` Particulars `
To Staff Salary 4,85,000 By Gross Profit 14,48,800
To Shop rent 1,20,000 By Post office SB A/c 14,200
To Admin. Expenses 2,96,000 interest
To Drawings 96,000 By Dividend from listed 43,000
To Depreciation 2,22,000 Indian Companies
To Medical Expenses 37,000 By Bank SB interest 12,000
To Net Profit 2,62,000
15,18,000 15,18,000
Additional information:
(i) Shop rent was paid to wife of Mr. Abdul and ` 60,000 is found to be
excessive payment considering its size and location.
(ii) Depreciation allowable under the income-tax rules works out to
` 1,81,000.
(iii) Medical expenses include expenditure for family members of ` 16,000.
7.332 O Scanner CMA Inter Gr. I Paper 7 (2016 Syllabus)

Balance relate to staff medical expenses.


(iv) Drawings denote personal expenses of the proprietor.
(v) During the year he acquired a residential house for ` 20 lakhs which
included stamp duty and registration fee of ` 1,60,000.
You are requested to compute the total income of Mr. Abdul for the
assessment year 2019-20. (7 marks)
Answer:
Computation of Total Income of Mr. Abdul
Gross Profit 14,48,800
Less: Expenses Allowed
Staff Salary 4,85,000
Shop Rent (1,20,000 - 60,000) 60,000
Admin. Expenses 2,96,000
Depreciation 1,81,000
Medical Expenses (37,000 – 16,000) 21,000 10,43,000
Income from Business 4,05,800
Income from other sources
Post office SB A/c Interest (14,200 – 3,500) 10,700
Dividend firm Listed Indian Companies Exempt
Bank SB Interest 12,000 22,700
Gross Total Income 4,28,500
Less: Deduction under chapter VI-A
Under Sec. 80C in respect of stamp duty on
residential house ` 1,50,000
u/s 80 TTA (Max upto 10,000) 1,60,000
Total Income 2,68,500
Space to write important points for revision
[Chapter  14] Assessment of Various Persons O 7.333

2018 - June [2] (a) (i) Compute the tax liability of Sri A. Harichandraprakash
whose total income is
(a) ` 49,62,500
(b) ` 51,00,000
Note : (Source of income is Salary only) (5 marks)
Answer:
Computation of tax liability of Sri A. Harichandraprakash
(A)
`
Total income (Income from Salary) 49,62,500
Tax on first 2,50,000 Nil
Tax on 2,50,001 to 5,00,000 12,500
i.e. (2,50,000 × 5/100)
Tax on 5,00,001 to 10,00,000 (i.e. 5,00,000 × 20/100) 1,00,000
Tax on Rest 10,00,001 to 49,62,500 (i.e. 39,62,500 × 30/100) 11,88,750
Total 13,01,250
Add: Health & Education Cess @ 4% 52,050
Total tax Liability 13,53,300

(B)
`
Total income (Income from Salary) 51,00,000
Tax on first 2,50,000 Nil
Tax on 2,50,001 to 5,00,000 i.e. (2,50,000 × 5/100) 12,500
Tax on 5,00,001 to 10,00,000 (i.e. 5,00,000 × 20/100) 1,00,000
Tax on 10,00,001 to 50,00,000 (i.e. 40,00,000 × 30/100) 12,00,000
Total tax before Surcharge and Cess 13,12,500
7.334 O Scanner CMA Inter Gr. I Paper 7 (2016 Syllabus)

Add: Surcharge @10% 1,00,000


(See Note below) 14,12,500
Add: Health & Education Cess @ 4% (i.e. 14,12,500 × 4/100) 56,500
Total tax liability 14,69,000
Note: In case of individual and HUF, where total income exceeds ` 50 lakhs
but does not exceed ` 1 crore, the aggregate of income tax and surcharge
shall be restricted to:
Tax on 50 lakhs + (Total income – 50 lakhs)
Space to write important points for revision

2018 - June [3] (a) Mr. Ashwin of Chennai sold a vacant site for ` 30 lakhs
to Mr. Raina on 01.05.2018. The value of land for stamp duty purposes was
` 25 lakhs. The vacant site was acquired in April, 2001 for ` 3 lakhs. The fair
market value of the vacant site on 01.04.2002 was ` 4 lakhs. The entire sale
consideration plus a housing loan of ` 38 lakhs from a nationalized bank was
availed for acquiring a residential building for ` 68 lakhs in Pune on
01.07.2018. The stamp duty paid for the purpose of acquisition was
` 2,90,000. The property was let out for a monthly rent of ` 10,000 from
01.07.2018. Interest on housing loan during the year till its closure,
amounted to ` 2,80,000.
Mr. Ashwin sold yet another vacant site for ` 28 lakhs on 21.01.2019. This
vacant site was acquired in October, 2016 for ` 20 lakhs. He utilized the
entire sale proceeds realized in January 2019 for repaying the housing loan.
His other incomes are (i) Income from business (computed) ` 3,90,000 and
(ii) Bank interest of ` 60,000 from term deposits and ` 15,000 from SB
account.
Compute the total income of Mr. Ashwin for the assessment year 2019-20.
Cost inflation index : F.Y. 2002-03 = 105; F.Y. 2016-17 = 264; F.Y. 2018-19
= 280. (7 marks)
[Chapter  14] Assessment of Various Persons O 7.335

Answer:
Computation of total income of Mr. Ashwin
for assessment year 2019-20
` `
Calculation of income under the “head
Income from house property”
Gross Annual value (10,000 × 9) 90,000
Less: Municipal Taxes paid Nil
Net annual value 90,000
Less: Deduction u/s 24
(a) 30% of NAV as standard deduction 27,000
(90,000 × 30/100)
(b) Interest on loan 2,00,000
(Sub to Max. of `)
Loss from house property 1,37,000 (1,37,000)
Income under the head “Profits and Gains of
business or profession (compiled) 3,90,000
Calculation of income under the head of
“Capital Gain”
Capital gain on sale of first vacant site
Sale Price 30,00,000
Less: Index cost of acquisition
4,00,000 × 280/105 10,66,667
Long term capital gain 19,33,333
Capital gain on sale of second vacant site
Period of holding 01/10/2016 to 21/01/2019
(Long term capital assets)
7.336 O Scanner CMA Inter Gr. I Paper 7 (2016 Syllabus)

Sale Consideration 28,00,000


Less: Index cost of acquisition
20 lakh × 280/264 21,21,212
long term capital gain 6,78,788 6,78,788
Total taxable long term capital gain 26,12,121
Entire sale consideration used for
purchases of new house property u/s 54F
Less: Exemption u/s 54F 26,12,121 NIL
Income under the head “Income from other
Sources”
Bank interest from term loan 60,000
Bank interest from S.B. account 15,000
Total income from the head IFOS 75,000
Gross total income of Mr. Ashwin 3,28,000
Less: Deduction under Chapter VI-A
Deduction u/s 80C
(i) Repayment of housing loan 3,80,000
(ii) Stamp duty paid as the time of acquisition
of house 2,90,000
Subject to maximum of ` 1,50,000 1,50,000
Deduction u/s 80 TTA
Interest on S.B. A/c 15,000
Subject to maximum of ` 10,000 10,000
Total taxable income of Mr. Ashwin 1,68,000
[Chapter  14] Assessment of Various Persons O 7.337

Note No 1 :
a. As per Section 50C where the consideration received or accruing as a
result of the transfer by an assessee of a capital assets, being land or
building or both, is less than the value adopted or assessed or
assessable by any authority of a State Government (hereafter in this
section referred to as the “Stamp valuation authority”) for the purpose of
payment of stamp duty in respect of such transfer, the value so adopted
or assessed or assessable shall, for the purpose of Section 48, be
deemed to be the full value of the consideration received or accruing as
a result of such transfer.
b. As amended by Finance Act, 2018, that where the value adopted or
assessed or assessable by the stamp valuation authority does not
exceed 105% of the consideration received or accruing as a result of the
transfer, the consideration so received or accruing as a result of the
transfer shall, for the purpose of section 48 be deemed to be the full
value of the consideration.
And where the value adopted or assessed or assessable by the stamp
valuation authority is exceed 105% of the consideration received or
accruing as a result of the transfer then Stamp Duty Value shall, for the
purpose of section 48, be deemed to be the full value of the
consideration.
Note No 2:
Where the capital asset become the property of the assessee by any of the
modes specified in Section 49(1), and the capital assets become the
property of the previous owner before 1.04.2001, means the cost of the
capital asset to the previous owner or the fair market value of the assets as
on 1.04.2001, at the option of the assessee.
7.338 O Scanner CMA Inter Gr. I Paper 7 (2016 Syllabus)

Note No : 3
According to Section 54F subject to sub-section (4), where in the case of an
assessee being an individual or HUF, the capital gain arises from the transfer
of any long-term capital assets, not being a residential house (hereinafter in
this section referred to as the original assets) and the assessee has, within
a period of one year before or two years after the date on which the transfer
took place purchased, or has within a period of three year after that date
constructed, one residential house in India and cost of new asset is more
than the net consideration in respect of the original asset, the whole of such
capital gain shall not be charged under Section 45.
Space to write important points for revision

2018 - June [4] (a) Mr. Kamal employed in Rajini Mfg. Co. Ltd. Mumbai as
General Manager furnishes the following information for the year ended
31.03.2019:
Particulars Amount (`)
Basic salary (per month) 50,000
Dearness Allowance (eligible for retirement benefits) 80% of basic salary
House Rent Allowance (per month) 10,000
Rent paid by him ` 15,000 per month for 6 months and ` 20,000 per
month for balance 6 months (at Mumbai)
City Compensatory Allowance (per month) 2,500
Medical reimbursements (annual) 13,000
Gymkhana club annual membership fee reimbursed by employer 20,000
Mobile phone bill reimbursed by the employer 37,500
(Used for both official and personal use)
Motor car (cubic capacity of engine 2.2 litres ) owned by the 85,800
employee but the maintenance expenses fully met by the employer
(Motor car was used both for personal and official use)
Cash gift paid by the employer in appreciation of performance on 30,000
01.01.2019
[Chapter  14] Assessment of Various Persons O 7.339

Contribution to recognized provident fund :


Employee 1,20,000
Employer 90,000
Contribution to National Pension Trust:
Employee 60,000
Employer 55,000
Medical insurance premium paid by means of uncrossed cheque 18,000
You are requested compute the total income of Mr. Kamal for the
assessment year 2019-20. (10 marks)
Answer:
Computation of total income of Mr. Kamal
for the Assessment Year 2019-20
`
Basic Salary (50,000 × 12) 6,00,000
Dearness allowance (80% of 6,00,000) 4,80,000
House rent allowance (Note 1) 18,000
City compensatory allowance 30,000
Medical reimbursement (Annual) 13,000
Annual membership fee 20,000
Mobile phone bill reimbursed 37,500
Motor Car (2,400 × 12 month) (85,800 - 28,800) 57,000
Cash gift paid by employer 30,000
Contribution to RPF (Note 2) Nil
Contribution to Nation Pension trust (Note 3) Nil
Employers Contribution to NPS 55,000
Gross Income from salary/ Gross Total Income 13,40,500
7.340 O Scanner CMA Inter Gr. I Paper 7 (2016 Syllabus)

Less: Deduction u/s 16 40,000


Standard Deduction
13,00,500
Less: Deduction under chapter VI-A
(i) u/s 80C in respect of PPF 1,20,000
(ii) u/s 80CCD
Employees contribution
below 10% hence allowed 50,000
(iii) u/s 80D Medical Insurance
Premium paid by cheque 18,000 1,88,000
Total Income 11,12,500

Working Note : 1
House rent allowance received 1,20,000
Less: Exemption u/s 10(13A)
(a) Actual house rent allowance received 1,20,000
(b) Rent paid over 10% of salary
2,10,000 – 1,08,000 = 1,02,000
(c) 50% of salary = 5,40,000
Least of the above exempt i.e. = 1,02,000
Taxable H.A. = Actual – Exempt
= 1,20,000 – 1,02,000 = 18,000
Working Note : 2
Any amount contributed to recognized provident fund by employee shall be
allowed as deduction u/s 80C from gross total income subject to the limit
specified therein and contribution of employer shall be exempted upto 12%
of salary.
Working Note : 3
Any contribution by an individual to any pension fund set up by any mutual
Fund referred U/S 10(23D) shall be allowed as deduction U/S 80C.
Space to write important points for revision
[Chapter  14] Assessment of Various Persons O 7.341

2018 - June [6] (a) The Profit & Loss Account of ABC & Associates, a
partnership firm for the previous year 2018-19 is given below :
Particulars ` ` Particulars `

Establishment and 96,00,000 Gross Profit 1,56,40,000


other expenses

Interest to partners Profit on sale 2,80,000


@15% of equity
shares (Sold
after 2 years
through recog-
nized stock
exchange)

A 1,80,000

B 2,40,000

C 1,20,000 5,40,000

Salary to working Rent from 1,20,000


partners house
property

A 4,80,000

B 3,60,000 8,40,000

I n t e r e st o n 20,000
bank deposit
7.342 O Scanner CMA Inter Gr. I Paper 7 (2016 Syllabus)

Profit on sale 2,40,000


of equity
shares (after
10 months
through recog-
nized stock
exchange)

Net Profit 53,20,000

1,63,00,000 1,63,00,000
Additional information:
(i) Establishment expenses include bonus ` 2,40,000 which was paid on
30-12-2019.
(ii) The firm is eligible for deduction under section 80-IC.
(iii) Establishment expenses also included securities transaction tax of
` 2,000.
Compute the tax liability of the firm for the assessment year 2019-20.
Assume that no extension of time has been granted u/s 139 (1) for filing the
return of income. (10 marks)
Answer:
Computation to total income and tax liability of the firm for the
assessment year 2019-20
` `
(A) Income under the head “Income from
house Property”
GAV 1,20,000
Less: Municipal tax Nil
NAV 1,20,000
Less: Deduction u/s 24
(a) 30% of NAV i.e. 1,20,000 36,000 84,000
[Chapter  14] Assessment of Various Persons O 7.343

(B) Computation of income from Business


Gross profit as per P&L a/c 1,56,40,000
Less: Allowable Expenses
Establishment and other expenses 96,00,000
Less: Bonus paid on 30/12/2019 (2,40,000)
Securities transaction tax (2,000) 93,58,000
Interest to working partners
A 1,80,000 × 1,44,000

B 2,40,000 × 1,92,000

C 1,20,000 ×
96,000 4,32,000
58,50,000
Book Profit
Less: Remuneration to working partners
(A) For the first ` 3,00,000 of BP @ 90%
2,70,000
(B) On rest of BP ` 55,50,000 @ 6% 33,30,000
36,00,000
or
Actual Remuneration paid to working partners 8,40,000
Which ever is less 8,40,000
Total income from Business 50,10,000
7.344 O Scanner CMA Inter Gr. I Paper 7 (2016 Syllabus)

(C) Income under head “Capital Gain”


Long term Capital Gain:
(i) profit on sale of equity share (Sale
after 2 year through recognized
stock exchange) Nil
(ii) Short term Capital Gain;
Profit on Sale of equity share (after 10
months through recognized stock
exchange) 2,40,000 2,40,000
(D) Income from other Sources
Interest on bank deposit 20,0000
Gross total income (A+B+C+D) 53,54,000
Less: Deduction u/s 80IC 100% of business
income 50,10,000
Total taxable income of ABC & Associates 3,44,000
Tax on total income other than short term
capital gain (i.e. 3,44,000 – 2,40,000 × 30%) 31,200
Tax on short term capital gain (2,40,000 × 15%) 36,000
Total tax before surcharge and Cess: 67,200
Add: Surcharge Nil
Add: Total tax before Cess 67,200
Add: Health & Education Cess @4% (i.e. 2,688 69,888
67,200 × 4/100)
Total Tax payable by ABC and Associates 69,890
(Rounded off)
[Chapter  14] Assessment of Various Persons O 7.345

Note : 1
Interest and remuneration paid to the partners by a firm are not deductible.
However, the interest and remuneration paid to partners by a firm are
deductible if all the following conditions are satisfied:
(i) Payment of salary, bonus, commission or remuneration, by whatever
name called (hereinafter referred as remuneration) is to a working
partner. If it is paid to a non-working partner, the same shall be
disallowed.
Note : 2
The payment of interest to a partner should not exceed the amount
calculated at the rate of 12% per annum simple interest (any amount in
excess will be disallowed).
Note : 3
According to Section 10(38) any income arising from the transfer of a long
term capital assets being an equity share of a company, on or after
1.10.2004 will be exempt.
Note : 4
Bonus received by an employee is charged to tax in the year of receipt relief
under section 89 can be claimed in respect of arrears of bonus received
during the year.
Space to write important points for revision

2018 - June [7] (a) Discuss the taxability or otherwise in the hand of the
recipient:
(iii) Nilay, a member of his father’s HUF, gifted a house property to the
HUF. The stamp duty value of the house is ` 8 lakhs. (2 marks)
(b) ABC & Co., a partnership firm, consisted of 4 equal partners up to
31.03.2018. It had accumulated business losses of ` 8 lakhs and
unabsorbed depreciation of ` 6 lakhs relating to assessment year 2017-
18. On 01.04.2018 one partner retired. The firm, for the previous year
ended 31st March, 2019, made a turnover of ` 150 lakhs. The firm
wishes to opt for presumptive taxation.
The entire sale proceeds were realized through banking channel.
Compute the total income of the firm for the assessment year 2019-20.
(5 marks)
7.346 O Scanner CMA Inter Gr. I Paper 7 (2016 Syllabus)

Answer:
(a) Since the HUF is covered under the definition of relative, therefore, gift
of house property to the HUF by its member is exempt from Tax.
(b) Computation of total income of the firm ABC & Company
for the Assessment Year 2019-20
Total turnover of ABC & Co. A partnership firm 150 lakhs
Profit and gains of Business or Profession 9 lakhs
150 lakhs × 6% (u/s 44AD)
Less: B/F Business Loss and unabsorbed
Depreciation
Accumulated Business loss 3/4 6 Lakhs
*Subject to max upto income from Business 6 Lakhs
 Total income of the firm 3 Lakhs
Note: An individual HUF or partnership firm, who is a resident, whose total
turnover or gross receipt in the previous year does not exceed on amount of
two crore rupees are eligible for opting presemptive taxation scheme.
Space to write important points for revision

2018 - Dec [5] (b) Mr. Raghavan, aged 57, is a person with disability. He
furnishes you the following information for the year ended 31.03.2019.
(i) Income from business (computed) ` 7,00,000
(ii) Dividend from an Indian company ` 10,50,000
(iii) Interest on Saving bank account with a nationalized bank ` 17,000
(iv) Medical insurance premium paid by account payee cheque
For self ` 20,000
For brother, wholly dependent on him ` 15,000
Compute his total income for the Assessment Year 2019-20. (5 marks)
[Chapter  14] Assessment of Various Persons O 7.347

2018 - Dec [6] CMA Anup Banerjee is in practice as Cost Accountant. He


follows mercantile basis of accounting. His income & expenditure account for
the year ended 31st March, 2019 is given below:
Expenditure ` Receipts `

Salary and stipends 10,50,000 Professional fees 45,00,000

Bonus to staff 1,00,000 Share of profit from a 2,00,000


partnership firm

Meeting Conference and 2,50,000 Interest on fixed 27,000


seminars deposit in a bank (Net
of TDS)

Fees to consultants 1,50,000 H o n o r a r i u m f o r 54,000


valuation of answer
papers of various
institutes (Net of TDS)

Travelling and conveyance 4,60,000

Rent for office premises 6,00,000

Provision for bad debts 40,000

Depreciation 1,45,000

Provision for income tax 7,02,000

Excess of income over 12,84,000


expenditure

47,79,000 47,79,000
Other information:
(i) Depreciation as per the Income - tax Act ` 2,00,000.
(ii) Salary and stipends include ` 40,000 paid to one trainee for passing
CMA final examination with rank.
7.348 O Scanner CMA Inter Gr. I Paper 7 (2016 Syllabus)

(iii) Bonus to staff was paid in November 2019.


(iv) In the financial year 2017-18, a sum of ` 15,000 was due to a
consultant, which was allowed. The said amount was paid on 14th May,
2018 in cash.
Compute the total income of CMA Anup Banerjee for the assessment Year
2019 -20. He has not opted for presumptive taxation scheme under section
44ADA. The due date for furnishing the return of income under section
139(1) may be taken as 31st Oct, 2019. (15 marks)
Section - C
Tax Management,
Administrative
Procedure and ICDS
15 TAX DEDUCTED AT SOURCE
THIS CHAPTER INCLUDES
 Meaning  TDS on Interest to
 TDS on Salary [Sec.192] non-resident [Sec. 194LC]
 TDS on payment to Contractor  TDS on income from units
[Sec. 194C] [Sec. 196B]
 TDS on Insurance Commission  Tax deduction and collection
[194D] account number [Sec. 203A]
 TDS on Rent [Sec. 194-I]  TDS is to be deducted on
 Fees for Professional or amount excluding GST
Technical Services [Sec. 194J] component
Marks of Objective, Short Notes, Distinguish Between, Descriptive & Practical Questions
Legend
Objective Short Notes Distinguish Descriptive Practical

For detailed analysis Login at www.scannerclasses.com


for registration and password see first page of this book.

7.351
7.352 O Scanner CMA Inter Gr. I Paper 7 (2016 Syllabus)

DESCRIPTIVE QUESTIONS
2009 - June [2] (b) Discuss the consequence of non-compliance of TDS
provisions of the Income-tax Act, 1961. (5 marks)
Answer:
Consequence of non compliance of TDS provisions:
Section Consequence
Section Effect
(a) 197A Delay, non 272A (2)(i) ` 100 per day/max. tax
submission of amount on Declaration.
TDS declarations
(b) Chapter Fails to deduct 271C (1)(a) Penalty of sum equal to
XVII-B whole or any part amount of TDS not so
of TDS deducted
(c) 200 Delay in payment 201(1A) Interest at 1% p.m.
of TDS
(d) 200 Delay, non 272A(2)(k) ` 100 per day/max. tax
submission of amount of TDS in
quarterly TDS Quarterly statement
Statement
203A Default in the 272BB Penalty of ` 10,000
matter of TAN
Space to write important points for revision

2009 - Dec [7] (d) Is there any option for an employer, not to deduct Tax at
Source, in respect of perquisites portion of salaries paid to an employees?
Discuss. (5 marks)
Answer:
An employer has option to pay tax in respect of non-monetary perquisites on
behalf of employee. In this case such option is exercised, then to that extent
tax need not be deducted at source from the salary of the employee on a
monthly average basis to the Government. [Section 192(1A)]
Space to write important points for revision
[Chapter  15] Tax Deducted at Source O 7.353

2010 - Dec [6] (b) Discuss the provisions of the Income-tax Act, 1961
requiring quoting of PAN mandatory in the context of TDS rates and filing of
declarations by the deductees. Also discuss the related TDS areas relating
to mandatory quoting of PAN. (5 marks)
Answer:
As per Section 206AA of Income-tax Act, 1961, quoting of PAN is
mandatory in the context of TDS.
(i) Notwithstanding anything contained in any other provisions of the Act,
any person entitled to receive any sum on which tax is deductible
shall furnish his permanent Account Number to the person responsible
for deducting such tax. If not given, tax shall be deducted at the
higher of the following rates;
(a) At the rate specified in the relevant provision of this Act; or
(b) At the rate or rates in force; or
(c) At the rate of 20%
(ii) No declaration under Section 197A(1) or 197A(1A) or 197A(1C) shall
be valid unless the person furnishes his PAN in such declaration.
(iii) No certificate under Section 197 shall be granted unless the PAN is
written in the application.
(iv) Deductor and deductee has to furnish the same for all correspondence
like bills, vouchers and other documents sent to each other.
(v) If the PAN provided is invalid, it shall be deemed that the deductee
has not furnished his PAN to the deductor and the provisions of
sub-Section (1) shall apply.
Space to write important points for revision

2013 - June [6] (c) A Company pays in Aug., 2017, a remuneration of


` 50,000 to its Director, which is not in the nature of salary. State whether tax
has to be deducted from the payment and if so the amount to be deducted.
(3 marks)
Answer:
Section 194J has been amended with effect from July 12, 2017 to provide
that tax will be deducted u/s 194J on the remuneration paid/payable to a
Director, which is not in the nature of salary, at the rate of 10% of such
remuneration. The amount of tax to be deducted would therefore come to `
5,000.
Space to write important points for revision
7.354 O Scanner CMA Inter Gr. I Paper 7 (2016 Syllabus)

PRACTICAL QUESTIONS
2009 - Dec [2] (a) Raja who was born on 1.06.1945 is a retired Government
Officer. Approximately he earns ` 1,80,000 as interest on company deposits.
Besides, he gets ` 1,00,000 pension. He invested ` 1,00,000 in securities
and other investments qualified for deduction u/s 80C. He also paid ` 20,000
as Medical Claim Insurance Premium. Can he submit the declaration in Form
No. 15H to the Company which will pay interest on company deposits so that
tax is not deducted by the payer company u/s 194A read with Section 197A.
(3 marks)
Answer:
As per Section 197(1B), no deduction of tax shall be made from the incomes
in the case of an individual resident in India who is of the age of 60 year or
more at any time during the previous year, if such individual furnishes to the
payer, a declaration in writing in duplicate in Form no. 15H to the effect that
the tax on his estimated total income of the previous year in which interest
other than interest on securities is to be included in computing his total
income will be nil.
In the case of such senior citizen, the income from the interest other than
interest on securities can be more than the maximum amount which is not
chargeable to tax but the tax on his estimated total income, inclusive of such
incomes, should be nil.
Computation of total income of the assessee:
Interest other than interest
On securities 1,80,000
Income from salary:
Pension 1,00,000
Gross Total Income 2,80,000
Less: Deduction u/s 80C & 80D 1,20,000
Net Income 1,60,000
Maximum exemption for senior citizen ` 3,00,000. So, taxable income of Mr.
Raja is Nil. Mr. Raja can submit the declaration in Form No. 15H.
Space to write important points for revision
[Chapter  15] Tax Deducted at Source O 7.355

2009 - Dec [6] (c) Mr. D.K. is employed with X Ltd. drawing salary of
` 75,000 per month (Eligible deduction under Section 80C ` 1,00,000). He
furnishes the following particulars of income and losses from other sources
as follows:
Loss from House property ` 25,400
Bank interest (after deduction of tax ` 3,090) ` 26,910
Dividend from Reliance Industries ` 30,000
Compute the liability of X Ltd. to deduct Tax at source. (4 marks)
Answer:
On receipt of the above information, X Ltd. will make the following two
computations.
Computation 1- Under this computation, all other income declared by the
employee will be considered.
Computation 2 - Only salary income will be considered with house property
loss.
It may be noted that in the two computation, house property loss will be
considered.
Computation 1 Computation 2
` `
Salary (75,000 × 12) 9,00,000 8,60,000 8,60,000
Less: Standard deduction of 40,000
House property loss (-)25,400 (-)25,400
Bank interest 30,000 -
Dividend from Reliance Industries exempt -
Gross total income 8,64,600 8,34,600
Less: Deduction under Section 80C 1,00,000 1,00,000
Net income 7,64,600 7,34,600
Tax 65,420 59,420
Health and Education cess @4% 2,617 2,377
68,037 61,797
- TDS 3,090 -
TDS to be deducted 64,947 61,797
Space to write important points for revision
7.356 O Scanner CMA Inter Gr. I Paper 7 (2016 Syllabus)

2011 - June [8] (c) Discuss the liability towards deduction of tax at source
in the following cases :
(i) An Indian company pays dividends on preference shares to a
shareholder, of an amount of ` 10,000 on Sept. 30, 2018. (2 marks)
(ii) A foreign enterprise enters into a contract for the fabrication and
supply of components for machinery with X & Co. firm in India on Dec.
1, 2012. X & Co. in turn sub-contracts the work to Y & Co. and pays
` 20 lakhs on June 16, 2018. (2 marks)
(iii) A domestic company pays to a doctor a monthly retainership of
` 2,500/- for attending an outpatient clinic at its factory premises.
(2 marks)
Answer:
(i) Dividend paid on preference shares under Section 115O is exempt
from tax in the hands of share holder. Hence, no tax has to be
deducted at source.
(ii) As per Section 194C, Payment to contractor for carrying out any work
(including supply of labour for carrying out any work) in pursuance of
a contract between the contractor and specified person is liable to
deduct tax at source @
(i) 1%, if payee is an individual or a HUF and
(ii) 2%, if payee is any other person, of the sum paid or credited.
In this case, X & Co. is contractor and Y & Co. is subcontractor.
Hence, X will deduct tax at source while he will pay to Y & Co. @2%
(Since, Y& Co. is a Firm).
(iii) As per Section 194J, the company is liable to deduct @10% on sum
paid towards fees for professional or technical services only if the
aggregate amount of such fees exceeds ` 30, 000. In this case, total
amount paid to doctor does not exceed ` 30,000/. Hence, there is no
need to deduct tax at source.
Space to write important points for revision

2012 - June [6] (c) R Ltd., which had taken up a building on lease, sub-
leased the building along with the Furniture to T Ltd. on 01.04.2014. The
monthly lease rent is ` 10,000 for building and ` 4,000 for furniture upto
[Chapter  15] Tax Deducted at Source O 7.357

31.03.2017 and 50% increase in each, for the year ended 31.03.2019. The
entire amount was in arrears and only on 31.03.2018, the whole of the lease
amount was paid by T Ltd. to R Ltd.
Discuss the obligation of T Ltd. to deduct tax at source from the above
payment. What is the rate of TDS and the amount of tax to be deducted?
When should this TDS amount be remitted? (6 marks)
Answer:
TDS from rent [Section 194-I]
According to Section (194-I) , any person (other than Individual or
HUF)is liable to deduct TDS who pays to any resident in India, any income
by way of rent, amounting in aggregate to more than `1,80,000 in a financial
year.
T Ltd. has to deduct TDS if above condition is fulfilled. In the question
it is not clear whether R Ltd is Indian resident or not but we are assuming
that R Ltd is Indian company.
The obligation to deduct tax at source arises at the time of actual
payment of rent or at the time of its credit to the account of the payee,
whichever is earlier.
In each year, T Ltd. would have credited the lease rent payable to a
separate account and hence the TDS obligation is to be considered for each
year separately, for each of those years.
As far as AY 2019-20 is concerned, T Ltd. has to consider only the rent
for the FY 2018-19. The same is ` 21,000 × 12 = ` 2,52,000, which is more
than the limit of ` 1,80,000.
(i) If R Ltd has PAN then TDS will have to be deducted @10% by T Ltd.
(ii) If R Ltd do not has PAN then TDS will have to be deducted @ 20% by
T Ltd.

Note: No surcharge, or Health and education cess shall be added to the


TDS rates. Hence, tax will be deducted at source at the basic rate.
Assuming that the amount was credited as well as paid on 31-3-2019, the
TDS amount should be remitted on or before 7-4-2019.
Space to write important points for revision
7.358 O Scanner CMA Inter Gr. I Paper 7 (2016 Syllabus)

2013 - June [3] (c) State briefly whether the following transaction require
deduction of tax at source:
(i) Payment of royalty of ` 5 lacs by P. Limited, an Indian Company to
another Indian Company, Q. Limited
(ii) Payment of interest of ` 7,500 by D. Limited, an Indian Company to M.
Limited, an Indian Company for delayed payment of sale proceeds.
(iii) Payment of ` 1,00,000 by a partnership firm, resident in India to Mr. L,
resident contractor for manufacturing a product as per requirement of
the firm. The contractor used materials which were purchased by him
from a company. (3 marks)
Answer:
(i) As the amount of royalty paid to resident exceeds ` 30,000, tax is
required to be deducted at 10% under Section 194J.
(ii) Payment of interest by D. Ltd. to M. Ltd., resident required deduction
of tax at 10%, as the amount of interest exceeds ` 5,000. (Section
194A)
(iii) Under Section 194C work does not include manufacturing or
supplying of product according to the requirement or specification of
a customer by using material purchased from a person, other than
such customer. As L used the materials purchased from a third party,
the firm is not required to deducted tax at source.
Space to write important points for revision

2013 - June [5] (a) Abhishek & Co. a partnership firm incurred the following
expenses:
(i) Salary paid to staff ` 12,00,000 of which two employees were paid
salary in excess of ` 2,00,000 each. No tax was deducted at source
on those salary payments.
(ii) Interest paid to bank on working capital limit ` 27,500. No tax was
deducted at source.
(iii) Interest on capital paid to partners at 15% per annum of ` 1,50,000.
No tax was deducted on such interest payment.
[Chapter  15] Tax Deducted at Source O 7.359

(iv) Payments made to contractors for job work process ` 6,87,000. Two
parties to whom payments were ` 87,000 and ` 1,02,000 for which no
tax was deducted at source. The parties however have agreed to
admit the receipt in their income statement and pay tax thereon.
(v) Rent paid for machinery ` 20,000 per month for 8 months. No tax was
deducted at source on this amount
Determine the allowability or otherwise of the above said expenses due to
non-deduction of tax at source. (5 marks)
Answer:
(i) Salary paid to staff is a deductible expenditure even though no tax was
deducted at source on such payment. Section 40a(ia) disallowance
is not applicable for salary expenditure.
(ii) Interest paid to bank is subject to exemption given in Section
194A(3)(iii), hence the question of deduction of tax at source on
interest paid to bank does not arise.
(iii) Interest on capital paid to partners at 15% will be liable for
disallowance to the extent of 3% since Section 40(b) mandates
allowance only to the extent of 12% per annum. Allowable interest will
be ` 1,20,000/-. Section 194A(3)(iv) provides exception, hence no tax
deduction at source is required for the interest paid to partners of the
firm.
(iv) Payments made to contractor when exceeds in aggregate ` 75,000,
tax is deductible at source as per Section 194C. However, further
proviso to Section 40(a)(ia) carves out an exception viz. that if the
payee has admitted the said receipt in his return of income and paid
tax on such income, the payer is not deemed to be an assessee in
default under the first proviso to Section 201(1), the disallowance
envisaged in Section 40(a)(ia) will not apply. Therefore, if the
recipients / payees have paid tax on their income including the
contract receipt given in the question, the disallowance u/s 40(a)(ia)
will not apply.
7.360 O Scanner CMA Inter Gr. I Paper 7 (2016 Syllabus)

(v) Rent paid for machinery above ` 1,80,000 in a financial year warrants
deduction of tax at source under Section 194-I. Since the aggregate
payment was only ` 1,60,000 the expenditure does not attract tax
deduction provision contained in Section 194-I. Thus it is out of the
clutches provided in Section 40(a)(ia).
Space to write important points for revision

2015 - June [6] (b) State whether TDS/TCS provisions are attracted in the
following cases:
(i) A newspaper paid ` 2,00,000 to ex-cricketer of New Zealand for
writing series of articles during World Cup Cricket matches.
(ii) Mr. X purchased jewellery from a showroom for ` 5,50,000 on
10.10.2018 by cheque.
(iii) When Mr. Y purchased fly ash from a factory to use the fly ash for
manufacture of bricks.
(iv) A partnership firm having a turnover of ` 40 lakhs during the financial
year 2018-19 made contract payment of ` 35,000 to an individual for
a single bill. (4 marks)
Answer:
Liability to TDS/TCS
(i) As per Section 194E tax is deductible at source at 20% on the
payment made to non-resident foreign citizen sportsman.
(ii) When jewellery is purchased in cash the tax collection at source will
apply. In this case as the purchase was made by means of cheque,
TCS provisions will not apply.
(iii) When goods purchased are meant for manufacturing it is not liable for
tax collection at source. Only when it is for the purpose of trade, TCS
provisions will apply.
(iv) Tax is deductible at source in respect of each contract when it exceeds
` 30,000. [Clause (i)(k) of the Explanation to Section 194C will be
applicable to partnership firms.]
Space to write important points for revision
[Chapter  15] Tax Deducted at Source O 7.361

2015 - Dec [6] (c) Mrs. Zeenat is running a proprietary business whose
accounts are audited under section 44AB since her turnover always
exceeded ` 100 lakhs. She pays a monthly rent of ` 13,000 for the office
premises to Mr. Jack, the owner of the building, an individual. She also pays
` 10,000 per month to Mr. Jack for the use of furniture, fixtures and vacant
land appurtenant to the building. Is she liable to deduct tax at source on
these payments? If no tax is deducted at source, what would be the
consequence? (4 marks)
Answer:
Yes, Mrs. Zeenat is liable to deduct tax an source. TDS on rent is liable to be
deducted by the person making the payment if the total amount to be paid
during the year exceeds ` 1,80,000 p.a. This limit is ` 1,80,000 p.a. is per tax payer.
Exception to the above rule: No TDS on rent is liable to be deducted if the
payment is being made by an individual when is not required to get his tax
audit done u/s 44AB.
Consequence if no TDS deducted:
1. As per Section 40(a)(ia) any sum payable to a resident which is
subjected to deduction of tax at source would attract 30% disallowance
if it is paid without deduction of tax at source.
2. Levy of interest: 1% for every month or part of the month on the
amount of such tax from the date on which such tax was deductible to
the date on which such tax is deducted.
3. Levy of penalty: Penalty of an amount equal to tax not deducted could
be imposed u/s 271C.
Space to write important points for revision

2016 - June [2] (c) Nathan Gramin Bank, which does not have core banking
facility, has paid the following amounts as interest to Mrs. Hemalatha, a
resident individual on 31-03-2019:
Particulars Branch 1 (`) Branch 2 (`)
Interest on fixed deposit 6,000 7,000
Interest on recurring deposit 3,500 2,300
Total 9,500 9,300
What is the tax to be deducted at source?
Will the answer differ, if the bank has core banking facility? (3 marks)
7.362 O Scanner CMA Inter Gr. I Paper 7 (2016 Syllabus)

Answer:
Interests on fixed deposit as well as recurring deposit are to be considered.
Where there is no core banking facility, then the TDS obligation has to be
seen branch-wise.
If the total interest paid is below ` 10,000, then there will be no obligation to
deduct tax at source. Hence in the first instance, there will be no obligation
to deduct tax at source.
In case there is core banking facility, then the TDS obligation is to be
considered for each deductee, for the bank as a whole.
Since the interest exceeds ` 10,000, tax has to be deducted at 10%.
Space to write important points for revision

2016 - Dec [1] {C} (d) Answer the following question with brief
reason/working:
(iii) Balaji Ltd, a textile manufacturing company paid ` 15 lakhs as contract
payments to M/s. Ramesh Engineers (a partnership firm) for
construction of godown building. At what rate the tax is deductible at
source on such contract payment? (1 mark)
Answer:
Since the payment is made to a person other than an individual or HUF, the
rate of tax deduction under section 194C would be 2%.
Space to write important points for revision

2016 - Dec [5] (b) What is the rate of tax and the amount of tax to be
deducted at source from the following payments?
(i) Discount of ` 50,000 allowed to a customer
(ii) Sales commission of ` 12,000
(iii) House worth ` 60 lakhs given as prize in a TV show to a resident
individual
(iv) Purchase of rural agricultural land for ` 90 lakhs. (5 marks)
Answer:
Obligation to deduct tax at source
(i) Discount of ` 50,000 allowed to a customer.
There will be no liability to deduct tax at source. Only if it is
commission, there will be TDS obligation.
[Chapter  15] Tax Deducted at Source O 7.363

(ii) Sales Commission of ` 12,000.Tax is not deducted at source under


section 194-H., because the amount of sales commission is less than
` 15,000.
(iii) House worth ` 60 lakhs given as prize in a TV show to a resident
individual. As per section 194B, tax is required to be deducted at 30%
as the amount of winning exceeds ` 10,000. The amount TDS is ` 18
lakhs (i.e. 30% of ` 60 lakhs) and the same has to be recovered from
the winner before the house is handed over to him.
(iv) Purchase of rural agricultural land for ` 90 lakhs. There is no obligation
to deduct tax at source, where the property purchased from a resident
is rural agricultural land.
Space to write important points for revision

2017 - Dec [3] (b) Examine the applicability of TDS provisions for the
financial year 2018-19 and amount of tax, if any, to be deducted in the
following cases:
(i) Payment of fee for technical services of ` 22,000 and royalty of
` 25,000 to Mr. Ram who is having PAN.
(ii) Payment of ` 2,00,000 made to Mr. X for purchase of diaries made
according to specifications of M/s ABC Ltd. However, no material was
supplied for such diaries to Mr. X by M/s ABC Ltd.
(iii) Rent paid for plant and machinery ` 1,50,000 by a partnership firm
having sales turnover of ` 25,00,000 and net loss of ` 15,000.
(6 marks)
Answer:
(i) Payment of fee for technical services of ` 22,000. No. TDS required to
be deducted because the amount is less than ` 30,000. But in case of
royalty tax required to be deducted @ 10% i.e. ` 2,500.
(ii) There is no need of TDS in case of payment of ` 2,00,000 for
purchase of diaries.
(iii) Since the amount of Rent for plant & machinery is less than
` 1,80,000. There is no need of TDS.
Space to write important points for revision
7.364 O Scanner CMA Inter Gr. I Paper 7 (2016 Syllabus)

2018 - June [3] (b) State whether the following transactions attract tax
deduction at source (TDS) provisions and the rate of tax and the amount of
tax deductible in applicable cases:
(i) Interest on recurring deposit of ` 12,000 paid by a nationalized bank
to Mr. Dhoni.
(ii) Prize amount of ` 8,000 paid by Excellence Ltd. to Mr. Saha a winner
of crossword puzzle contest conducted by the company.
(iii) Commission of ` 21,000 paid to Kumble and Co. by Dravid Co. Ltd. for
purchase of raw materials.
(iv) Chandra Ltd. paid ` 40,000 per month as generator rent from 1st
August, 2018 and up to 31st March, 2019 to Mr. Shastri.
(8 marks)
Answer:
(i) TDS on interest other than “Interest on Securities” (Paid to
Resident Only):
Any person, not being an individual or a Hindu Undivided Family, who
is responsible for paying to a resident any income by way of interest
other than income by way of interest on securities, shall, at the time of
credit of such income to the account of payee or at the time of
payment thereof in cash or by issue of a cheque or draft or by any
other mode, whichever is earlier, deduct income-tax thereon at the rate
of 10% (TDS is also to be deducted from interest on Fixed Deposit and
Recurring Deposits with banks and Co-operative banks)
Where the amount of such income or, as the case may be, the
aggregate of the amount of such income credited or paid or likely to be
credited or paid during the financial year to the payee, does not
exceed:
(A) ` 10,000 on time deposit where payer is Banking Company.
(B) This threshold limit of ` 10,000 has been increased (with effect
from 1st April, 2018 to ` 50,000 if the recipient is a senior citizen.)
Since interest on recurring deposit of ` 12,000 paid by a
Nationalized bank to Mr. Dhoni is exceed the amount of `10,000.
Therefore, this transaction attract Tax Deduction of Source@ 10%
i.e. ` 1,200.
[Chapter  15] Tax Deducted at Source O 7.365

(ii) TDS on Winnings from lottery or Crossword Puzzle (Paid to


Resident as well as Non-Resident):
As per Section 194B, a person responsible for paying to any person
any income by way of winnings from lotteries or Crossword Puzzles
exceeding ` 10,000 is required, at the time of such payment, to deduct
income tax thereon at the rate of 30%. Where the winnings are wholly
in kind, the person responsible for paying shall, before releasing the
winnings in kind, ensure that tax has been paid in respect of the
winnings.
In the present case, since the amount paid by Excellence Ltd. to
Mr. Saha is ` 8,000 which is less than, ` 10,000 therefore, Excellence
Ltd. does not required to deduct TDS.
(iii) TDS on Commission or Brokerage (Paid to Resident only)
Any person, not being an individual or a Hindu undivided family, who
is responsible for paying, to a resident, any income by way of
commission (not being insurance commission referred to in Section
194D) or brokerage shall, at the time of credit of such income to the
account of the payee or at the time of payment of such income in cash
or by the issue of a cheque or draft or by any other mode, whichever
is earlier, deduct income tax thereon at the rate of 5%.
Provided that no deduction shall be made under this section in a case
where the amount of such income or as the case may be, the
aggregate of the amounts of such income credited or paid or likely to
be credited or paid during the financial year to the account of or to, the
payee, does not exceed ` 15,000. Since in the present case, the
amount of ` 21,000 paid by Dravid Co. Ltd. to Kumble & Co. a resident
which is exceed the amount ` 15,000, therefore, Dravid Co. Ltd.
should deduct TDS@ 5%, i.e. ` 1050.
(iv) TDS on Rent (Paid to Resident Only)
Any person, not being an individual or a Hindu Undivided Family, who
is responsible for paying to a resident any income by way of rent, shall,
at the time of credit of such income to the account of the payee or at
the time of payment thereof in cash or by the issue of a cheque or
draft or by any other mode, whichever is earlier, deduct income tax
thereon at the rate of:
7.366 O Scanner CMA Inter Gr. I Paper 7 (2016 Syllabus)

(a) 2% for the use of any machinery or plant or equipment, and


(b) 10% for the use of any land or building (including factory building)
or land appurtenant to a building (including factory building) or
furniture or fittings.
In the present case, Chandra Ltd. paid ` 40,000 per month as
generator rent for a period of 8 months (i.e. 1st August, 2017 to 31st
March, 2019)
Therefore, the TDS amount which should be charged is
40,000 × 8 months × 2/100
= 6,400.00
Space to write important points for revision

Table Showing Marks of Compulsory Questions


Year 14 14 15 15 16 16 17 17 18 18
J D J D J D J D J D
Practical 1
Total 1
16 TAX COLLECTED AT SOURCE
THIS CHAPTER INCLUDES
 Collection of Tax at Source

DESCRIPTIVE QUESTIONS

2008 - Dec [4] (b) Briefly sketch the responsibilities and liability of a person
liable to collect tax at source under the Income-tax Act, 1961. (5 marks)
Answer:
Responsibility & Liability of Tax Collector
1. To obtain Tax Collection Account No. [Section 206CA(1)]
2. To quote TCS No. in all returns, certificates and challans. [Section
206CA(1)]
3. To furnish quarterly returns in form No. 27EQ within stipulated time i.e.,
within fifteen days from the end of a quarter for the first three quarters
and 30th April for the last quarter.
4. Failure to Furnish TCS return: Penalty @ 100/- per day, during which
default continues, but not exceeding the amount of TCS.
[Sec. 272A(2)(g)]
5. Failure to deposit TCS in Government treasury may attract rigorous
imprisonment for a term of not less than 3 months, but which may extend
to 7 years, in addition to fine. [Section 276B & 276BB]
Space to write important points for revision

7.367
17 ADVANCE TAX
THIS CHAPTER INCLUDES
 Advance Payment of Tax  Due dates of Advance Tax
 Computation of Advance Tax  Credit for Advance Tax

Marks of Objective, Short Notes, Distinguish Between, Descriptive & Practical Questions

Legend
Objective Short Notes Distinguish Descriptive Practical

For detailed analysis Login at www.scannerclasses.com


for registration and password see first page of this book.

7.368
[Chapter  17] Advance Tax O 7.369

SHORT NOTES
2017 - Dec [8] Write short note on the following:
(d) Advance tax for senior citizens being resident/non-resident.
(5 marks)
Answer:
Advance Tax is not payable by:
An individual, Resident India, who does not have any Income chargeable
under the head “Profits and Gains of Business or profession” and is of the
age of 60 years or more at any time during the previous year.
Advance tax is payable by senior citizen being non resident.
Space to write important points for revision

DESCRIPTIVE QUESTIONS
2008 - Dec [5] (e) Is a representative assessee exempt from liability to pay
advance income-tax ? (2 marks)
Answer:
Person in whose hands assessed tax payable during a financial year is
` 10,000 or more are required to pay advance-tax.
The provisions relating to advance tax apply to all assessee. It is clearly
implicit therein that a representative assessee is not exempt from the liability
to pay advance tax.
Space to write important points for revision

2009 - June [4] (a) Discuss the provisions of the Income-tax Act, 1961 with
regard to interest payable by the Department on refund. (5 marks)
Answer:
Interest on refund: Section 244A
Interest on excess payment of advance-tax, tax deducted/collected at
sources (TDS and TCS), and any other tax or penalty becoming refundable,
shall be paid @ ½ % for every month or part of month.
7.370 O Scanner CMA Inter Gr. I Paper 7 (2016 Syllabus)

The period for which interest payable is:


(i) For refund out of advance and TDS from 1st April of relevant
assessment year to the date on which refund is granted. However, no
interest is payable, if the amount refund is less than 10% of the tax
determined u/s 143(1) or on regular assessment,
(ii) For all other tax or penalties, from the date of payment of tax/penalty
to the date on which refund is granted.
Delay in granting refund attributable to the assessee is excluded from
the period for which interest is payable.
Where the amount on which interest was payable is increased or
decreased due to regular assessment order, reassessment, rectification.
Appeal, revision or settlement commission order, interest will also be
increased or decreased.
Space to write important points for revision

2010 - June [5] (c) What are the due dates for payment of advance income
tax by corporate assessees? (4 marks)
Answer:
Advance tax shall be payable by a corporate assessee as per the following
schedule of installments:
Due date of instalment Amount payable
On or before 15th June of the Not less than 15% of advance tax liability.
previous year
On or before 15th September Not less than 45% of advance tax liability,
of the previous year as reduced by the amount, if any, paid in
the earlier instalment.
th
On or before 15 December of Not less than 75% of advance tax liability,
the previous year as reduced by the amount or amounts, if
any, paid in the earlier instalment or
instalments.
On or before 15th March of the The whole amount of advance tax liability
previous year as reduced by the amount or amounts, if
any, paid in the earlier instalment or
instalments.
Space to write important points for revision
[Chapter  17] Advance Tax O 7.371

2013 - June [6] (b) Enumerate the conditions for exemptions of senior
citizens from payment of advance tax u/s 207. (4 marks)
Answer:
Every assessee is required to pay Advance Tax if the tax liability for the
previous year is ` 10,000/- or more. From the financial year 2017-18.
Section 207 has been amended to provide that Advance Tax need not be
payable in the case of senior citizens if the following conditions are satisfied.
1. The tax payer is an individual.
2. He is resident in India.
3. He is at least 60 years of age at any time during the previous year (for
the F.Y. 2018-19 the condition can be satisfied in the case of an
individual who was born before April 1, 1954).
4. He does not have any income chargeable under the head “Profits and
gains of business or profession”.
If the above conditions are satisfied the concerned individual will not be
required to pay advance tax during the financial year (i.e. immediately
prior to the relevant assessment year).
Space to write important points for revision

2015 - Dec [1] {C} Answer the following question with brief reason/working:
(j) Is a company liable to pay advance tax when it is required to pay tax on
“book profit” under section 115JB? (2 marks)
Answer:
There is specific provision in Section 115JB(5) providing that all other
provisions of the Income-tax Act shall apply to every assessee, being a
company mentioned in that section.
Section 115JB is a self-contained code pertaining to MAT and by virtue of
sub-section (5) thereof, the liability for payment of advance tax would be
attracted.
Space to write important points for revision
7.372 O Scanner CMA Inter Gr. I Paper 7 (2016 Syllabus)

2017 - June [7] (b) Answer the following questions in the context of the
provisions relating to advance tax:
(i) Who is not liable to pay advance tax, in case of individual assesses?
(ii) State the due dates of instalment for payment of advance tax and the
amount of instalment for such assessees who are not covered under
the provisions of Section 44AD. (6 marks)
Answer:
(i) Advance Tax is not payable by:
 An Individual
 Resident in India
 Who does not have any income chargeable under the head
“Profits and Gains of Business or Profession”
 And is of the age of 60 years or more at any time during the
previous year.
(ii) Advance Tax is payable in the following installments:
In case of all Assessees except those who are covered under section
44AD
Due date of Instalment Amount Payable (`)
th
On or before 15 June Not less than 15% of the advance tax
liability.
th
On or before 15 September Not less than 45% of the advance tax
liability as reduced by the amount, if
any, paid in earlier instalment.
th
On or before 15 December Not less than 75% of the advance tax
liability as reduced by the amount, if
any, paid in earlier instalment.
th
On or before 15 March The whole of the advance tax liability
as reduced by the amount, if any,
paid in the earlier instalments.
Space to write important points for revision

2018 - Dec [7] (b) State the due dates for payments of advance tax, along
with the quantum of amount payable in each instalment. Present your
answer in the form of a table. (7 marks)
[Chapter  17] Advance Tax O 7.373

PRACTICAL QUESTIONS
2010 - Dec [3] (b) Mr. Ramesh, aged 62, paid ` 8,000, ` 12,000 and
` 30,000 and ` 35,000 on 15-6-2018, 12-09-2018 and 21-12-2018 and 11-
03-2019 respectively as advance tax instalments. He filed return of income
for the assessment year 2019-20 showing total income of ` 9,00,000; he is
entitled to tax credit of ` 2,700 on account of tax deducted at source.
Ascertain the interest, if any, chargeable under Section 234C of the Income-
tax Act, 1961. (5 marks)
Answer:
Computation of Interest u/s 234C
Particulars `
Tax on Income of ` 9,00,000
(Including Health and Education cess @ 4%) 93,600
Less: Tax deducted at source 2,700
Assessed Tax 90,900
15% of Assessed Tax 13,635
Tax paid before 15-06-2018 8,000
Shortfall 5,635
Interest @ 1% for 3 months 169
45% of assessed tax 40,905
Tax paid before 15-09-2018 20,000
Shortfall 20,905
Interest @ 1% for 3 months 627
75% of assessed tax 68,175
Tax paid before 15-12-2018 20,000
Shortfall 48,175
Interest @ 1% for 3 months 1,445
100% of assessed tax 90,900
Tax paid on or before 15-03-2019 85,000
Shortfall 5900
Interest @ 1% for 1 Month 59
Hence Total interest (169+627+1,445+59) 2,300
7.374 O Scanner CMA Inter Gr. I Paper 7 (2016 Syllabus)

Note:
1. As per provisions of Sec. 207(1) a resident senior citizen is not liable
to pay advance tax if he/she does not have any income from business
or profession during the previous year.
Space to write important points for revision

2012 - June [8] (a) Mr. Arvind (aged 45 years) has total income of
` 7,80,000 for the previous year ended 31.03.2019. He has credit by way of
tax deducted at source of ` 20,000. He paid advance tax of ` 10,000,
` 25,000 and ` 20,000 before 15th of June, September, December and March
during the financial year 2018-19.
Assume he would be filing his return of income on 31.10.2019 after paying
self assessment tax, compute interest payable by him under Section 234B
and 234C of the Income-tax Act, 1961. (7 marks)
Answer:
Computation of interest under Section 234B and 234 C for Mr. Arvind
for Assessment year 2019-20.
` `
Total income earned 7,80,000
Tax on above (Including Health and Education 71,240
cess at @ 4%)
Less: TDS 20,000
Advance tax liability for AY 2019-20 51,240
Advance tax due for June 2018 is 15% of ` 51,240 7,686
Less: Advance tax paid 10,000
Shortfall Nil
Interest u/s 234C @1% per month for 3 months Nil
Advance tax due in September 2018 is 45% of
` 51,240 23,058
Less: Advance tax paid 20,000
Shortfall 3,058
Interest @ 1% for 3 months 92
[Chapter  17] Advance Tax O 7.375

Advance tax due in December 2018 is 75% of


` 51,240 38,430
Less: Advance tax paid 45,000
Shortfall Nil
Interest u/s 234C @1% Nil
Advance tax due in March 2019 is 100% of 51,240
51,240
Less: Advance tax paid 65,000
Shortfall Nil
Advance tax payable (Assessed tax after TDS) 51,240
Advance tax paid 65,000
As more than 90% of assessed tax has been paid by way of advance tax,
there will be no liability to pay interest u/s 234B.
Interest u/s 234c = 92
Space to write important points for revision

2015 - June [2] (b) Vijay, a resident individual aged 59, is running a
wholesale business in fertilisers, whose turnover for the year ended
31.03.2019 is ` 70 lacs. Is he liable to pay advance tax, if he maintains
books of accounts and gets his accounts audited under section 44AB of the
Income-tax Act, 1961 (business income is ` 5.2 lacs)? Will your answer be
different if he opts for presumptive taxation? He has no other income.
(3 marks)
Answer:
Liability to pay advance tax:
Where the assessed tax payable by an individual who is not a senior citizen,
exceeds ` 10,000, he is liable to pay advance tax.
In the first situation, since the tax payable on total income exceeds ` 10,000
he is liable to pay advance tax. Where he opts for presumptive tax, the
obligation to pay advance tax will not arise at all.
Space to write important points for revision
7.376 O Scanner CMA Inter Gr. I Paper 7 (2016 Syllabus)

2016 - Dec [6] (a) Compute the amounts of advance tax installments,
payable in the financial year 2018-19 in respect of the following cases:
(i) A domestic company with tax liability of ` 3,90,000.
(ii) A super senior citizen (age above 80) having pension income of ` 3
lakhs and property income (computed) ` 5 lakhs. (4 marks)
Answer:
(i) Calculation of Installment of Advance Tax for a domestic company with
Tax Liability of ` 3,90,000
15% of Tax Liability upto 15th June ` 58,500
45% of Tax Liability upto 15th September ` 1,17,000 (1,75,500 – 58,500)
75% of Tax Liability upto 15th December ` 1,17,000 (29,25,00 – 1,75,500)
100% of Tax Liability upto 15th March ` 97,500. (3,90,000 – 2,92,500)
(ii) A super senior citizen (age above 80) having pension income of ` 3
lakhs and property income (computed) ` 5 lakhs. In the case of senior
citizens and super senior citizens as per section 207(2) no advance
tax is payable where they do not have income chargeable under the
head ‘profits and gains of business or profession’.
Thus advance tax liability = ‘NIL’.
Space to write important points for revision

Table Showing Marks of Compulsory Questions

Year 14 14 15 15 16 16 17 17 18 18
J D J D J D J D J D

Descriptive 2

Total 2
18 RETURN AND PAN
THIS CHAPTER INCLUDES
 Return of Income  Defective Return
 Return of Loss  Revised Return
 Belated Return
Marks of Objective, Short Notes, Distinguish Between, Descriptive & Practical Questions
Legend
Objective Short Notes Distinguish Descriptive Practical

For detailed analysis Login at www.scannerclasses.com


for registration and password see first page of this book.

7.377
7.378 O Scanner CMA Inter Gr. I Paper 7 (2016 Syllabus)

SHORT NOTES
2017 - June [8] (d) Revised return of income: Meaning and interplay of
Section 139(5). (5 marks)
Answer:
Revised Return: Section 139 (5)
Law effective from Assessment year 2017-18 [Finance Act, 2016]:
If any person having furnished a return under section 139(1) or under section
139(4), discovers any omission or wrong statement therein, then he may
furnish a revised return at any time before the expiry of one year from the
end of the relevant Assessment year or before the completion of
assessment, which ever is earlier.
Law upto Assessment Year 2016-17:
Upto Assessment Year 2016-17, the return filed under section 139(4) could
not have been revised under section 139(5).
As per finance Act, 2016 w.e.f. Assessment Year 2017-18, the return filed
under section 139(4) can be revised under section 139(5).
Notes:
(1) The revised return substitutes the original return from the date the
original return was filed. Once a revised return is filed, the original return
is deemed to have been withdrawn and the revised return is deemed to
have been filed on the date the original return was filed.
(2) As assessee can revise a return any number of times provided that the
revised return is filed with in the time prescribed under section 139(5).
Space to write important points for revision

2017 - Dec [8] Write short notes on the following:


(b) Who must sign the return of income in the following cases:
(i) Hindu undivided family when karta is bedridden.
(ii) Local authority.
(iii) Political party.
(iv) Limited Liability Partnership.
(v) Association of persons (5 marks)
(c) Revised return. (5 marks)
[Chapter  18] Return and Pan O 7.379

Answer:
(b) (i) If the karta is absent from India or is mentally incapacitated, then by
any adult member of the family.
(ii) In case of a Local Authority, by the Principal Officer thereof.
(iii) In case of Political Party, by the Chief Executive Officer of the Party.
(iv) In case of a Limited Liability Partnership:
(a) By the Designated partner thereof , or
(b) Where there is no designated partner as such, or where for any
unavoidable reason such designated partner is not able to
verify the return, by any partner thereof.
(v) In case of Association of Person by, any member or the Principal
Officer there of.
(c) Revised Return: Law Effective from Assessment year 2018-19
[Finance Act, 2017]
If any person having furnished a return under section 139(1) or under
section 139(4), discovers any commission or wrong statement therein,
then he may furnish a revised return at any time before the end of the
relevant Assessment year or before the completion of assessment,
which ever is earlier.
Space to write important points for revision

2018 - June [8] Write short notes on the following:


(a) Fee for delay in furnishing the return of income;
(d) Adjustments during the course of processing of return of income u/s
139(1). (5 marks each)
Answer:
(a) Fee for Default in Furnishing Return of Income (Introduced by Finance
Act, 2017)
(1) Without prejudice to the provisions of this Act, where a person
required to furnish a return of income under section 139 fails to do
so within the time prescribed in sub-section (1) of the said Section,
he shall pay, by way of fee, a sum of,:
7.380 O Scanner CMA Inter Gr. I Paper 7 (2016 Syllabus)

(a) ` 5,000, if the return is furnished on or before the 31st day of


December of the assessment year;
(b) ` 10,000 in any other case:
provided that if the total income of the person does not exceed ` 5
lakhs, the fee payable under this section shall not exceed ` 1,000.
(2) The provision of this section shall apply in respect of return of
income required to be furnished for the assessment year 2019-20
and future Assessment years.
(d) Scheme of processing of Returns:
(1) Where a return has been made under section 139, or in response to
a notice under section 142(1), such return shall be processed in the
following manner, namely:
(a) the total income or loss shall be computed after making the
following adjustments namely:
(i) any arithmetical error in the return; or
(ii) an incorrect claims, if such incorrect claim is apparent from
any information in the return;
(iii) disallowance of loss claimed, if return of the previous year
for which set off of loss is claimed was furnished beyond
the due date specified under section 139(1);
(iv) disallowance of expenditure indicated in the audit report but
not taken into account in computing the total income in the
return;
(v) disallowance of deduction claimed under sections 10AA,
80-IA, 80IAB, 80IB, 80IC, 80-ID or section 80-IE, if the
return is furnished beyond the due date specified under
section 139(1); or
(vi) addition of income appearing in form 26AS or Form 16A or
Form 16 which has not been included in computing the
total income in the return;
Provided further that the response received from the
assessee, if any, shall be considered before making any
adjustment, and in a case where no response is received
within thirty days of the issue of such intimation, such
adjustments shall be made.
[Chapter  18] Return and Pan O 7.381

(b) the tax and interest and fee under section 234F, if any shall be
computed on the basis of the total income computed under
clause (a);
(c) the sum payable by, or the amount of refund due to, the
assessee shall be determined after adjustment of the tax and
interest and fee under section 234F, if any, computed under
clause (b) by any tax deducted at source, any tax collected at
source, any advance tax paid any relief allowable under section
89/90/90A/91, any tax paid on self-assessment and any amount
paid otherwise by way of tax or interest and fee under section
234F;
(d) an intimation shall be prepared or generated and sent to the
assessee specifying the sum determined to be payable by, or
the amount of refund due to, the assessee under clause (c); and
(e) the amount of refund due to the assessee in pursuance of the
determination under clause (c) shall be granted to the assessee.
Space to write important points for revision

2018 - Dec [8] Write short notes on the following:


(a) Verification of return of income in the case of an individual, HUF and
political party (5 marks)
(b) Any five transactions where quoting PAN is mandatory (5 marks)

DESCRIPTIVE QUESTIONS
2009 - June [6] (b) Can income-tax return be filed through computer
readable media? (4 marks)
Answer:
Income - Tax Return Through Computer Readable Media [Section
139(1b]
1. Sub-section (1-B) of Section 139 of the Income-tax Act, 1961 enables
the taxpayer to file his return of income in computer readable media,
without interface with the Department.
7.382 O Scanner CMA Inter Gr. I Paper 7 (2016 Syllabus)

2. It provides an option to a person (both corporate and non-corporate),


required to furnish a return of his income.
3. Such person may, on or before the due date, furnish a return of income
in accordance with such scheme as may be notified by the CBDT, in
such form (including on a floppy, diskette, magnetic cartridge tape, CD-
ROM or any other computer readable media) and manner as may be
specified in that scheme.
Such return shall be deemed to be a return furnished under Section
139(1).
Space to write important points for revision

2009 - Dec [1] {C} (d) (i) Is it compulsory to file a claim for refund? discuss.
(2 marks)
Answer:
No, it is not compulsory to file a claim for refund.
Where an assessee has submitted any return of income and any refund of
tax is due, such refund shall be granted by the Assessing Officer on his own.
The assessee is not required to file any claim for such refund. Similarly, if
any refund arises due to an order of appeal, rectification of mistakes, revision
by CIT or appeal to the High Court, the refund shall be granted by the
Assessing Officer himself. In the case also, the assessee is not required to
file any claim for refund of tax. However, in any other case, for example,
where the tax has been deducted at source from the income of a person not
liable to pay tax, the assessee is required to file a claim of refund. Such claim
must be submitted in Form No. 30 and should be verified in the prescribed
manner. Form No. 30 must be accompanied by a return of income, if such
return has not been submitted earlier.
Space to write important points for revision

2010 - June [2] (c) What are the due dates for filing the return of income as
envisaged by Section 139(1) of the Income Tax Act, 1961? (4 marks)
[Chapter  18] Return and Pan O 7.383

Answer:
Due date for filing return of income
Different Situations Due date of
submission of
return
1. Where the assessee is a company September 30
2. Where the assessee is a person other than a
company.
(a) In case where accounts of the assessee are September 30
required to be audited under any law
(b) Where the assessee is a “working partner” in
a firm whose accounts are required to be
audited under any law
3. Assesses required to file Transfer Pricing Report
under Section 92E Pertaining to International
Transaction November 30
4. In any other case July 31
Note: Where the last day for filing return is a day on which the office is
closed, the assessee can file the return on the next day afterwards on
which the office is open and in such cases, the return will be considered
to have been filed within the specified time limit.
Space to write important points for revision

2010 - Dec [7] (b) Is e-filing of income-tax return mandatory for all
assessee? Also state the assessees for whom the same is mandatory.
(3 marks)
(c) Can a revised return of income be further revised? (2 marks)
Answer:
(b) E-filing of return is mandatory for:
(1) From the assessment year 2015-16 onwards any asessee filing ITR
1/2/2A (other than an individual of the age of 80 years or more at
anytime during the previous year) having a refund claim in the return
or having total income of more than ` 5,00,000 is required to furnish
the return of income electronically.
7.384 O Scanner CMA Inter Gr. I Paper 7 (2016 Syllabus)

(2) A firm or an individual or a Hindu Undivided Family (HUF) whose


books of account are required to be audited under section 44AB
shall furnish the return of income electronically.
(3) A person who is required to file ITR - 5 shall file the same.
(4) Every Company shall furnish the return of income.
(5) A resident assessee having any assets (including financial interest
in any entity) located outside India or signing authority in any
account located outside India or income from any source outside
India shall furnish the return of income electronically.
(6) Taxpayers claiming relief under section 90, 90A or 91 shall furnish
the return of income electronically.
(7) A taxpayer who is required to furnish a report of audit under sections
10(23C)(iv), 10(23C)(v), 10(23C)(vi), 10(23C)(via), 10A, 10AA,
12A(1)(b), 44AB, 44DA, 50B, 80-IA, 80-IB, 80-IC, 80-ID, 80JJAA,
80LA, 92E, 115JB or 115VW or to give a notice under section
11(2)(a) shall furnish the return electronically.
Answer:
(c) Revised Return:
Law Effective from Assessment Year 2018-19 (Finance Act, 2017):
If any person having furnished a return under section 139(1) or under
section 139(4), discovers any omission or wrong statement therein, then
he may furnish a revised return at any time before the end of the
relevant Assessment Year or before the completion of assessment,
whichever is earlier.
Space to write important points for revision

2011 - June [4] (c) What do you mean by annexure less return? In what
manner can the return of income be filed ? (4 marks)
Answer:
Annexure less Return of Income-
Section 139C empowers the Board (CBDT) to allow filing returns without any
annexure i.e. Annexure less return.
Accordingly, the board may make rules providing for class or classes of
persons who may not be required to furnish documents, statements,
[Chapter  18] Return and Pan O 7.385

receipts, certificates, reports or audit or any other documents, which are


otherwise under any other provisions of this Act required to be furnished,
along with the return.
However, such documents, statements receipts, etc. shall be furnished-
(a) As and when demanded by assessing officer or
(b) As provided in Section 139D
Space to write important points for revision

2011 - June [5] (c) What is the due date of filling of return of income in case
of a non-working partner of a firm whose accounts are subject to tax audit ?
Who is regarded as 'working partner' ? (3 marks)
Answer:
Due date of filling of return of income: Due date of filling of return of
income in case of a non working partner shall be 31st July of the assessment
year whether the accounts of the firm are required to audited or not.
Working partner: Any individual who is actively engaged in conducting the
affairs of the business or profession of the firm of which he is a partner and
is drawing remuneration from the firm is called ‘working partner’.
Space to write important points for revision

2012 - Dec [7] (c) State ten instances/transactions where Permanent


Account Number (PAN) has to be compulsorily quoted. (5 marks)
Answer:
The following are the transactions for which quoting of PAN is
mandatory.
1. Amount exceeding ` 10 lakh or valued by stamp valuation authority
referred to in section 50C at an amount exceeding ` 10 lakh;
2. Sale or purchase of a motor vehicle or vehicle, which requires
registration by a registering authority;
Note: The sale or purchase of a motor vehicle or vehicle does not
include two wheeled vehicles, inclusive of any detachable side-car
having an extra wheel, attached to the motor vehicle.
7.386 O Scanner CMA Inter Gr. I Paper 7 (2016 Syllabus)

3. A time deposit, exceeding ` 50,000 (fifty thousand rupees), with a bank;


4. A time deposit with post office, exceeding ` 50,000 (fifty thousand
rupees) or aggregating to more than ` 5 Lakh during the financial year;
5. A contract of a value exceeding ` 1,00,000 (one lakh rupees) for sale or
purchase of securities;
6. Opening an account (other than basic saving A/c or time deposit not
exceeding 50,000/-) with a banking company;
7. Payment to hotels and restaurants against their bills for an amount
exceeding ` 50,000 (Fifty thousand rupees) at any one time;
8. Payment in cash for purchase of bank drafts or pay orders or banker's
cheques from a bank for an amount aggregating ` 50,000 (fifty thousand
rupees) or more during any one day;
9. Deposit in cash aggregating ` 50,000 (fifty thousand rupees) or more,
with a bank during any one day;
10. Payment in cash in connection with travel to any foreign country of an
amount exceeding ` 50,000 (Fifty thousand rupees) at any one time.
Note:
a. "payment in cash in connection with travel" includes payment in
cash towards fare, or to a travel agent or for the purchase of
foreign currency;
b. "travel to any foreign country" does not include travel to the
neighboring countries or to such places of pilgrimage
11. Making an application to any bank or to any other company or institution,
for issue of a credit or debit card;
12. Payment of an amount of ` 50,000 (fifty thousand rupees) or more to a
mutual fund for purchase of its units;
13. Payment of an amount of ` 50,000 (fifty thousand rupees) or more to a
company for acquiring shares issued by it;
14. Payment of an amount of ` 50,000 (fifty thousand rupees) or more to a
company or an institution for acquiring debentures or bonds issued by
it;
15. Payment of an amount of ` 50,000 (fifty thousand rupees) or more to the
reserve bank of India (RBI), for acquiring bonds issued by it;
[Chapter  18] Return and Pan O 7.387

16. payment of an amount aggregating ` 50,000 (fifty thousand rupees) or


more in a year as life insurance premium to an insurer;
17. Sale or purchase, by any person, of goods or services of any nature any
amount exceeding ` 2,00,000.
18. A time deposit with a nidhi referred in section 406 of the companies act
2013, exceeding ` 50,000 (Fifty thousand rupees) or aggregating to
more than ` 5 Lakh during the financial year.
19. A time deposit with a NBFC which holds a certificate of registration under
section 45-IA of the RBI Act, 1934 to hold or accept deposit from public,
exceeding ` 50,000 (Fifty thousand rupees) or aggregating to more than
` 5 Lakh during the financial year.
Space to write important points for revision

2013 - June [8] (a) What are the consequences if a person fails to comply
with the provisions of Section 139A of the Income Tax Act, 1961?
(2 marks)
Answer:
As per Sec. 272 B (2) if a person fails to comply with the provisions of Sec.
139A, the assessing officer may direct such person to pay by way of penalty
a sum of ` 10,000/-.
Space to write important points for revision

2016 - June [1] {C} (d) Answer the following question with brief
reasons/working:
(iii) Every return of income filed by the assessee cannot be revised. Is the
statement correct? (1 mark)
Answer:
Effective from: A.Y. 2017-18
If any person, having furnished a return under section 139(1) or belated
return under section 139(4), discovers any omission or any wrong statement
therein he may furnish a revised return at any time before the expiry of one
year from the end of the relevant assessment year or before the completion
of the assessment whichever is earlier.
7.388 O Scanner CMA Inter Gr. I Paper 7 (2016 Syllabus)

Therefore Answer Should be:


Return furnished in pursuance of a notice issued under section 142(1)
cannot be revised.
Space to write important points for revision

PRACTICAL QUESTIONS
2011 - Dec [2] (a) Answer the following with reference to the provisions of
the Income-tax Act, 1961:
(ii) Return of income of a company was signed by the Company
Secretary. Is the return a valid return? (1.5 marks)
Answer:
Where the return of income of a company was signed by a company
secretary, although it is supposed to be signed by a Managing Director or a
Director (in the absence of a Managing Director), it is a defective return,
which can be rectified and not an invalid return.

Note:
1. There is an alternate view that the return of income of a company,
which is not signed and verified by the Managing Director or Director,
is not a defective return but an invalid return and thus, void-ab-initio.
2. In case of a company not resident in India, the return of income may be
signed and verified by any person who holds a valid power of attorney.
Since the question does not specify the residential status of a company,
it is possible to answer that the return is valid if the company is
nonresident and the Company Secretary holds a valid power of
attorney.
Space to write important points for revision

2012 - June [4] (b) State the due dates for filing of return of income under
Section 139(i) of the Income-tax Act, 1961 in the following cases, for the
assessment year 2019-20:
[Chapter  18] Return and Pan O 7.389

(i) By a non-working partner of a LLP, whose accounts are required to be


audited under Section 44AB;
(ii) By a company which has entered into international transactions with
a non-resident and is required to file audit report in this regard;
(iii) By a University whose gross fee receipts are ` 3 crores. (3 marks)
Answer:
Due dates for filing return of income
The due dates are as under:
(i) 31st July, 2019
(ii) 30-11-2019
(iii) 30th September, 2019
Space to write important points for revision

2014 - June [4] (a) Powell Ltd. filed its return of loss beyond the ‘due date’
prescribed in Section 139(1) read with Section 80 of the Act. It has business
loss of ` 20,50,000. It approaches you for advice regarding the course of
action to be taken to secure the benefit of carry forward of business loss for
set off against future profits. Advise suitably. (3 marks)
Answer:
Generally, return filed beyond the due date specified in Section 139 (1)
would deprive the tax payer the benefit of carry forward of business loss.
However, in extraordinary circumstances, the CBDT has the powers to
condone the delay in filing return having carry forward of business loss to the
future years.
[Associated Electro Ceramics vs. CBDT 201 ITR 501 (Kar)].
As per Circular No. 8 of 2001 monetary limits have been prescribed for
condonation of delay in filling loss returns. In cases where the refund is less
than ` 10,000, the Assessing Officer can condone the delay with the
approval of CIT, for refunds from `10,001 to ` 1,00,000 the Assessing Officer
can condone the delay with the approval of CCIT/DGIT. Where the refund
claim exceeds `1,00,000 only the CBDT has the power to condone the
delay.
7.390 O Scanner CMA Inter Gr. I Paper 7 (2016 Syllabus)

Therefore, the loss return of Powell Ltd. could be condoned by CBDT.


However, Powell Ltd. has to file a condonation petition to the CBDT to carry
forward the business loss.
Space to write important points for revision

2014 - Dec [6] (b) (i) Mr. A (non-resident) aged 66 has total income
(computed) ` 2,60,000 comprising income from house property and income
from other sources. Is he required to file his return of income for the
assessment year 2019-20? Also compute the total amount of tax payable by
him. (3 marks)
Answer:
Mr. A is a senior citizen but non-resident. The basic exemption limit
applicable to him is ` 2,50,000.
As the total income exceeded the basic exemption limit, he is liable to file his
return of income.
Mr. A is required to pay tax of ` 515
Upto ` 2,50,000 Nil
Next ` 10,000@5% 500
Add: Health & education @4% 20
Total tax liability 520
He is not eligible for rebate under section 87A because it is applicable to
resident assessee (ordinary or not ordinary) only.
Space to write important points for revision

2017 - Dec [7] (b) You are requested to state whether the following will
attract penalty provisions for the financial year 2018-19.
(i) Mr. Jayant whose turnover was ` 150 lakhs for the financial year 2018-
19 admitted income as per books of account as ` 6,50,000. The return
was filed without getting the books of account audited under section
44AB.
[Chapter  18] Return and Pan O 7.391

(ii) Mr. Maheswari received ` 5 lakhs by cash on 01.10.2018 for sale of an


apartment at Thane to Mr. Mahesh. As the agreement between the
parties got cancelled, Mr. Maheswari refunded the advance by account
payee cheque on 01.02.2019. (4×2 = 8 marks)
Answer:
(i) Yes, u/s 271 B failure to get accounts audited or furnish a report of
audit as required under section 44 AB:

½% of Total sales, turnover, or gross receipts, etc, or ` 1,50,000


whichever is less.
In the present case, Total penalty will be ` 75,000.
(ii) Yes, u/s 271D - Taking or accepting certain Loans and deposits or
specified sum in contravention of the provisions of Section 269SS:
Amount equal to loan or deposit or specified sum taken or accepted.
In the present case, Maximum penalty u/s 271D will be ` 5,00,000.
Space to write important points for revision

2018 - June [5] (b) State with one line reason, the due date for filing the
return of income in the following cases:
(i) Mr. Solkar, engaged in trade, has total turnover of ` 220 lakhs for the
year ended 31.03.2019.
(ii) Krish Srikanth, an advocate, has aggregate professional receipts of
12,40,000 opting to admit income under section 44ADA.
(iii) Mr. Abid Ali, having 5 heavy goods transport vehicles which are run
on hire, opting to admit income under section 44AE.
(iv) M/s Jayantilal & Mankad, a firm engaged in hotel business with annual
turnover of ` 130 lakhs preferring to offer income based on applicable
presumptive provisions.
(v) Vaman Kumar Charitable Trust registered under section 12AA having
total income of ` 12 lakhs. (before giving effect to the provisions of
Section 11 and 12, and before seeking accumulation of income for
application in the future years.) (6 marks)
7.392 O Scanner CMA Inter Gr. I Paper 7 (2016 Syllabus)

Answer:
Every person,:
(a) being a company or a firm, or
(b) being a person other then a company or a firm, if his total income or the
total income of any other person in respect of which he is assessable
under this Act during the previous year exceeded the maximum amount
which is not chargeable to tax.
All those tax payers, who have opted for the presumptive income
scheme as per Section 44AD, Section 44ADA, and Section 44AE of the
Income-Tax Act bile his return before;
(i) A person (other than a company) where accounts are required to be
audited under the Income-tax Act or under any other low;
The due date for filing the return is 30th September of the
assessment year.
(ii) Mr. Krish Sri Kanth, an advocate, who opting presumptive income
scheme u/s 44ADA shall file before 30th September, if Audit are
applicable and 31st July if Audit are not applicable.
(iii) Mr. Abid Ali, having goods transport vehicles opting presumptive
income scheme u/s 44AE shall file before 30th September, if Audit
are applicable and 31st July if Audit are not applicable.
(iv) M/s Jayantilal and Mankad, a partnership firm engaged in hotel
business with annual turnover of ` 130 lakhs, opting presumptive
income scheme U/S 44AD shall file before 30th September if Audit
are applicable and 31st July if Audit are not applicable.
(v) Vaman Kumar Charitable Trust registered under section 12AA
having total income of ` 12 Lakhs. (Before giving effect to the
provision of section 11 and 12) shall file return of income U/S
139(4A) before 30th September of the assessment year.
Space to write important points for revision
[Chapter  18] Return and Pan O 7.393

Table Showing Marks of Compulsory Questions

Year 14 14 15 15 16 16 17 17 18 18
J D J D J D J D J D

Descriptive 1

Total 1
19 ASSESSMENT PROCEDURE
THIS CHAPTER INCLUDES
 Refund of Tax  Reassessment
 Summary Assessment  Appeals
 Best Judgement Assessment  Revision
 Income Escaping Assessment  Rectification
 Penalties
Marks of Objective, Short Notes, Distinguish Between, Descriptive & Practical Questions

Legend
Objective Short Notes Distinguish Descriptive Practical

For detailed analysis Login at www.scannerclasses.com


for registration and password see first page of this book.

7.394
[Chapter  19] Assessment Procedure O 7.395

SHORT NOTES
2017 - June [8] Write short note on the following:
(b) Adjustments required to be made while processing the return of income,
u/s 143 (1). (5 marks)
Answer:
Section 143(1):
Scheme of Processing of Returns Adjustment to be made by software
to the returned income while processing under section 143(i):
Where a return has been made under section 139, or in response to a notice
under section 142(1), the total income or loss shall to computed after
making the following adjustments, namely:
(i) any arithmetical error in the return: or
(ii) an incorrect claim, if such incorrect claim is apparent from any
information in the return;
(iii) disallowance of loss claimed if return of the previous year for which
set off of less is claimed was furnished beyond the due date
specified under section 139(1);
(iv) disallowance of expenditure indicated in the audit report but not
taken into account in computing the total income in the return;
(v) disallowance of deduction claimed under section 10AA, 80-IA, 80-
IB, 80-IAB, 80-IC, 80-ID or Section 80-IE, if the return is furnished
beyond the due date specified under section 39(1);
(vi) addition of income appearing in Form 26AS or form 16A or form 16
which has not been included in computing the total income in the
return.
However, no such adjustments shall be made unless an
intimation is given to the assessee of such adjustments either in
writing or in electronic made.
7.396 O Scanner CMA Inter Gr. I Paper 7 (2016 Syllabus)

Further the response received from the assessee, if any, shall


be considered before making any adjustment, and in a case where
no response is received with in 30 days of the issue of such
intimation, such adjustments shall be made.
Space to write important points for revision

2018 - June [8] Write short note on the following:


(b) Scrutiny assessment (5 marks)
Answer:
Section 143(1D) provides that if a return is filed having a claim of refund and
the case is selected for scrutiny under section 143(2), then such return shall
not be processed under section 143(1). It means that refund shall not be
granted to assessee under section 143(1), if case is selected for scruting
Refund, if any, shall be granted in assessment under section 143(3). This
resulted in hard-ships to the assessees as their refund was upheld till the
scruting order Section 143(3) is passed.
Finance Act, 2017 provides that Section 143(1D) shall not apply to returns
filed for assessment year 2017-18 and future assessment years. That
means, for assessment year 2017-18 and future assessment years, if return
is filed having a claim of refund and case is selected for scruting under
section 143(2), then the refund shall be granted to the assessee under
section 143(1). The Income Tax Department cannot stop the refund till the
completion of assessment under section 143(3).
Space to write important points for revision

2018 - Dec [8] Write short note on the following:


(d) Best judgment assessment under section 144 (5 marks)
[Chapter  19] Assessment Procedure O 7.397

DESCRIPTIVE QUESTIONS

2009 - Dec [2] (d) A single letter of enquiry was issued by the Income Tax
Department to X of Mumbai. In this letter there was no specific mention of
any provision of the Income Tax Act, Can X be treated as an ‘Assessee’?
(2 marks)
Answer:
If a single letter of enquiry has been issued by the Income Tax Department
to X and no specific mention of any provision, X can be treated as an
‘Assessee’.
AS per Section 2(7) of Income Tax Act Assessee means a person by
whom any tax or any other sum of money (i.e., penalty or interest) is payable
under the act. The term also includes a person in respect of whom any
proceeding under the act has been taken (whether or not he is liable for any
tax, interest or penalty).
Space to write important points for revision

2010 - June [1] {C} (b) (iii) Non-allowing of interest to rectification order is
appealable — Discuss. (2 marks)
Answer:
When the I.T.O. rectifies and assessment under Section 154/155 and grants
refund but fails to grant interest on the refund, such rectification order has
the effect of reducing the amount payable to the Assessee and hence
appealable under Section 246 (1) (b). Hence, the statement is correct.
Space to write important points for revision

2011 - June [7] (b) What is a protective assessment under income-tax law ?
When is it resorted to and what is the procedure followed for the recovery of
tax in such cases? Can penalty be imposed in a protective assessment?
(4 marks)
7.398 O Scanner CMA Inter Gr. I Paper 7 (2016 Syllabus)

Answer:
When there are doubts about true ownership of the income, a protective
assessment is made. For example if there are doubts as to whether a rental
income belongs to Mr. A or Mr. B, then the assessing officer at his own
discretion may add the rental income to any one of them on a protective
basis. This is done ensure that in finality of the owner of the income, the
addition of income is not denied because of limitation of time.
Supreme Court in Lal ji Hardas Vs. ITO (43 ITR 387)
“In cases where it appear to the income tax authorities that certain income
has been received during the relevant assessment year but it is not clear
who has received that income and prima facie it appears that the income
may have been received either by A or B, or by both together, it would be
open to the relevant income tax authorities to determine the said question by
taking appropriate proceeding both against A and B’’.
Space to write important points for revision

2012 - Dec [5] (b) Briefly discuss the provisions of Section 142(2A) of the
Income-tax Act, 1961 relating to special audit. (4 marks)
Answer:
Special Audit [u/s 142 (2A)]-
1. Power of Assessing Officer: At any stage of the proceeding, the
Assessing Officer having regard to the nature and complexity of the
accounts of the assessee and the interests of the revenue, is of the
opinion that it is necessary so to do, he may, with the approval of the
Chief Commissioner or Commissioner, direct the assessee to get his
accounts audited by an accountant and to furnish a report of such audit.
2. Extension of Time Limit: Assessing Officer may suo moto or on the
application made by the assessee in behalf extend period for furnishing
the audit report upto 180 days from the date of directions.
3. Audit under Other Law: Assessing officer can direct the assessee to
get his accounts audited under this section even if the accounts are
already audited under this Act or under any other law.
[Chapter  19] Assessment Procedure O 7.399

4 Prior Approval: Prior approval of the Chief Commissioner or


Commissioner must be obtained before directing such audit.
5. Appointment: The Chief Commissioner/Commissioner shall nominated
the auditor.
6. Form of Report: The Audit Report containing prescribed particulars
must be furnished in the prescribed Form No. 6B.
7. Opportunity of being heard: The Assessing Officer shall not direct the
assessee to get the accounts so audited unless the assessee had been
given a reasonable opportunity of being heard.
8. Penalty: On failure to comply with direction issued u/s 142(2A): Penalty
for ` 10,000 u/s 271(1) (b) can be levied and Best judgement
assessment u/s 144 can be made.
Space to write important points for revision

2013 - June [4] (b) State the time limit prescribed for passing the following
orders under the Income Tax Act:
(i) An order of assessment by the Assessing Officer under Section
143(3);
(ii) An order of assessment by the assessing Officer under Section
143(3), where reference has been made to Transfer Pricing Officer for
determination of arm's length price in international transactions;
(iii) An order of assessment by the Assessing Officer under Section 147;
(iv) An order of revision by the Commissioner of Income Tax under
Section 263;
(v) An order by the Commissioner of Income Tax for granting registration
to a public charitable trust under Section 12AA. (5 marks)
Answer:
(i) As per Section 153 an assessment order under Section 143(3) is to
be passed within 21 months from the end of the assessment year in
which the income was first assessable.
(ii) Where a reference has been made to the Transfer Pricing Officer for
determination of arms length price in international transactions,
assessment order under Section 143(3) is to be passed within
33 months from the end of the relevant assessment year.
7.400 O Scanner CMA Inter Gr. I Paper 7 (2016 Syllabus)

(iii) An order of assessment under Section 147 is to be passed by the


Assessing Officer within 9 months from the end of the financial year in
which the notice under Section 148 was served.
(iv) An order for revision under Section 263 is to be passed by the
Commissioner within 9 months from the end of the financial year in
which such order u/s 263 is passed by the PC or CIT, as the case may
be.
(v) An order for granting registration to a public charitable trust under
Section 12AA is to be passed by the Commissioner within 6 months
from the end of the month in which application for registration was
received.
Space to write important points for revision

2013 - June [7] (b) In the course of assessment proceedings, the Assessing
Officer enhanced the value of closing stock and added the difference to the
total income. In the assessment year subsequent to this, the assessee wants
the A.O. to enhance, by the same amount, the value of the opening stock of
the year. Discuss the validity of the claim. (3 marks)
Answer:
The value of the closing stock of the preceding year must be the opening
stock of the succeeding year. Hence, if the value of the closing stock is
enhanced, the enhanced value should be taken as the value of opening
stock of the next year for purposes of the I.T. The claim of the assessee is
therefore valid.
Space to write important points for revision

2015 - Dec [1] {C} Answer the following question with brief reason/working:
(e) A foreign company cannot approach Dispute Resolution Panel in respect
of its assessment. Is this statement correct? (2 marks)
Answer:
This is a false statement. The Scheme of Dispute Resolution Panel is
available only to a foreign company and to any person who has entered into
an international transaction and whose assessment has been referred to
transfer pricing officer (Section 144C).
Space to write important points for revision
[Chapter  19] Assessment Procedure O 7.401

2015 - Dec [3] (c) State the scope and time limit for rectification of order
passed by the Assessing Officer under the Income-tax Act. (3 marks)
Answer:
Section 154 states that the Income tax assessing officer is empowered to
rectify any order of assessment or of refund or any other order passed by
him. Further, the AO is also empowered to amend any intimation or deemed
intimation u/s 143(1) or amend any intimation u/s 200A (1).
Rectification of an order can be made only within 4 years from the end
of financial year in which the order sought to be amended was passed.
In case of amendment made u/s 151; If an application for amendment
is made by the assessee, the AO shall pass an order within a period of 6
months from the end of the month in which the application is received.
Space to write important points for revision

2016 - June [6] (b) State the procedures to be adopted by the Income-tax
Authorities for rectification of mistakes under section 154 of the Income-tax
Act and the time limit for such rectification. (7 marks)
Answer:
Procedure for rectification of mistake u/s 154
An amendment of the following nature can be made only after the concerned
authority has given notice in this respect and also a reasonable opportunity
of being heard to the assessee or deductor or the collector.
(a) Amendment which enhances an assessment
(b) Amendment which reduces a refund
(c) Amendment which otherwise increase the liability of the assessee
or deductor or the collector.
If any amendment enhances the assessment or reduces a refund already
made, a notice of demand is served on the assessee or deductor or the
collector. Such notice is deemed to be a notice u/s 156.
If any amendment reduces the assessment, refund due to the assessee is
made unless it is withheld u/s 241.
7.402 O Scanner CMA Inter Gr. I Paper 7 (2016 Syllabus)

Time Limit for Rectification:


Period of limitation for making rectification as prescribed in sub-section (7)
of Section 154 is as follows:
No amendment under this section can be made after the expiry of 4 years
from the end of the financial year in which the order sought to be amended
was passed. It may be noted that an amendment is made when related order
is passed.
This period of limitation is not applicable in case the provision of Section 155
are applicable.
However, if a valid application has been made by the assessee for
rectification within the statutory time limit but is not disposed of by the
concerned authority within the time specified, it may be disposed of even
after the expiry of such time limit [Circular No. 73, dated 7th January, 1972].
This relief is, however, not admissible in case rectification proceedings are
initiated by the department itself.
Space to write important points for revision

PRACTICAL QUESTIONS
2009 - Dec [6] (b) A firm filed a Return of Income disclosing an income of
` 6,00,000 on June 30, 2018 in respect of Assessment Year 2017-18. The
Assessing Officer completed the assessment on September 30, 2018 under
Section 143(3) on an income of ` 9,00,000. Apart from questioning the merits
of the addition made by the Assessing Officer, on what other ground will you
challenge the assessment? (4 marks)
Answer:
Assessment u/s 143(3) can be made only when notice u/s 143(2) is served
on the assessee. Assessee can challenge the assessment on the grounds
of enhancement of income and assessment is also invalid as Assessing
Officer (AO) has not served the notice u/s 143(2).
Assessing Officer has to serve the notice within 6 months. Notice of Section
143(2) can be served upon assessee only when return has been furnishes.
Space to write important points for revision
[Chapter  19] Assessment Procedure O 7.403

Table Showing Marks of Compulsory Questions

Year 14 14 15 15 16 16 17 17 18 18
J D J D J D J D J D

Descriptive 2

Total 2
20 INCOME COMPUTATION AND
DISCLOSURE STANDARDS
THIS CHAPTER INCLUDES
 E-commerce Transactions  Income Computation and
 Equalization Levy Disclosure Standards (ICDS)
Marks of Objective, Short Notes, Distinguish Between, Descriptive & Practical Questions

Legend
Objective Short Notes Distinguish Descriptive Practical

For detailed analysis Login at www.scannerclasses.com


for registration and password see first page of this book.

7.404
[Chapter  20] Income Computation and Disclosure... O 7.405

SHORT NOTES
2017 - June [8] Write short note on the following:
(a) Features of ICDS (3 marks)
Answer:
Features of ICDS:
The following are the main features of ICDS:
(i) ICDS are to be followed by all assessees following the mercantile
system of accounting.
(ii) ICDS are applicable to the computation of income chargeable to
income tax under the head “Profit and gains of business or profession”
or “Income from other sources.”
(iii) This standard is not applicable for maintenance of books of account,
AS ICDS are meant for computation of income only, there is no need
for maintenance of books of account for this purpose.
(iv) In the case of conflict between the provision of the Income-Tax Act,
1961 and ICDS, the provisions of the Act shall prevail to that extent.
(v) Non compliance of ICDS will lead to be Judgement Assessment.
Space to write important points for revision

2017 - Dec [8] Write short note on the following:


(a) State the type of assesses to whom the Income Computation
Disclosure Standards (ICDS) apply? (5 marks)
Answer:
ICDS are applicable to the computation of income chargeable to income tax
under the head profits and gains of business or profession and income from
other sources.
Space to write important points for revision
7.406 O Scanner CMA Inter Gr. I Paper 7 (2016 Syllabus)

2018 - June [8] Write short note on the following:


(c) ICDS : Accounting policies (5 marks)
Answer:
Accounting Policies:
• Accounting policies adopted by a person shall be such so as to
represent a true and fair view of the state of affairs and income of the
business, profession or vocation.
• The treatment and presentation of transactions and events shall be
governed by their substance and not merely by the legal form.
• Marked to market loss or an expected loss shall not be recognised
unless the recognition of such loss is in accordance with the provisions
of any other income Computation and Disclosure standard.
Disclosure of Accounting Policies:
• All significant accounting policies adopted by a person shall be
disclosed.
• Any change in an accounting policy which has a material effect shall be
discloses (with quantum of the effect, if ascertainable) where such
amount is not ascertainable, the fact shall be indicated.
• Disclosure of accounting policies or of changes therein cannot remedy
a wrong or inappropriate treatment of the item.
Fundamental Accounting Assumptions:
The fundamental accounting assumptions i.e. Going concern, Consistency
and Accrual are assumed as followed. No specific disclosure is required, if
these assumptions are followed, however, if such assumption are not
followed, the fact shall be disclosed.
Space to write important points for revision

2018 - Dec [8] Write short note on the following:


(c) ICDS - I on “Accounting Policies” (5 marks)
21 OBJECTIVE QUESTIONS
2008 - Dec [1] {C} (a) Choose the correct answer:
(i) The following assessee is charged to income-tax in the assessment
year following the previous year:
(1) A non-resident business firm which shipped goods on 1.5.2018
at Visakhapatnam Port in Andhra Pradesh
(2) An employee left India to USA on 1.8.2018 with no intention of
returning
(3) ABC firm which discontinued its business on 1.9.2018
(4) An employee-assessee of a University who worked during
1.4.2016 to 30.03.2019.
(ii) The following is exempt income:
(1) Travel concession to employee
(2) Remuneration received for valuation of answer scripts
(3) Encasement of leave salary whilst in service
(4) Perquisites in India.
(iii) The following is not “plant” under Section 43(3) of Income-tax Act,
1961:
(1) Books
(2) Know-how
(3) Road in the factory building
(4) Electrical fittings.
(iv) The following is not a venture capital undertaking for the purposes
of Section 10(23F), if engaged in business of:
(1) Generation of power
(2) Telecommunications
(3) Providing infrastructural facility
(4) Dairy farming whose shares are not listed in a recognized stock
exchange. (1 x 4 = 4 marks)
Answer:
(i) (4)
(ii) (1)
7.407
7.408 O Scanner CMA Inter Gr. I Paper 7 (2016 Syllabus)

(iii) (3)
(iv) (4)
Space to write important points for revision

2008 - Dec [1] {C} (b) Fill up the blanks:


(iii) Assets held by a minor married daughter ________ (will/will not) be
clubbed in the hands of the individual. (1 mark)
(iv) Where a charitable trust is created on 01.04.2016, the application for
registration u/s 12A of the Income-tax Act, 1961 should be submitted
within ________ from. (1 mark)
(v) Under Section 44BBB (i) of the Income-tax Act, 1961, the presumptive
income is taken as _______ % of the eligible receipts in the hands of
eligible assessee. (1 mark)
(vii) Electronic furnishing of income-tax return in approved computer
readable media can be furnished under sub-Section _____ of Section
139 of the Income-tax Act, 1961. (1 mark)
(viii) From out of his agricultural income, X has paid interest of ` 10,000 on
education loan taken from nationalized bank last year. Deduction
available under Section 80E of the Income-tax Act, 1961 is
`_________ . (1 mark)
Answer:
(b) (iii) Will not
(iv) One year
(v) 10%
(vii) (1B)
(viii) Nil
Space to write important points for revision

2009 - June [1] {C} (a) State with reasons whether the following statements
are true or false (No. credit will be given for mere conclusions):
(i) For the purpose of transfer pricing provisions, arm's length price is
determined by the taxpayer in consultation with qualified accountant.
(ii) As per Section 194-C of the Income-tax Act, 1961, all Association of
Persons and Body of Individuals are liable to deduct tax at source from
specified payments made to resident contractors.
[Chapter  21] Objective Questions O 7.409

(iii) Where a person does basic operations in lands and later sells the
saplings grown by him in a nursery owned by him, the same will be
agricultural income. If the basic operations are not done by the
assessee and the saplings are sold in his nursery, the same will still be
regarded as agricultural income.
(iv) Short-term capital gains arising from sale of listed shares through a
recognized stock exchange, for which security transaction tax has
been paid, will be charged to tax at a concessional rate of 10%.
(v) For the purposes of computing minimum alternate tax (MAT) under
Section 115JB (2) of the Income-tax Act, 1961, the book profit need
not be increased, by inter alia, the amount of deferred-tax or provision
for deferred-tax debited to the profit and loss account.
(2 × 5 = 10 marks)
Answer:
(a) (i) False: The arm's length price in relation to an international transaction
is to be determined using the most appropriate method out of the
specified methods, having regard to the nature or class of transaction
or class of associated persons or functions performed by such persons
or such other relevant factors as may be prescribed.
(ii) False: The Finance Act, 2008 has amended the provisions of
Section 194-C. only those AOPs/BOIs, whose turnover during the
immediately preceding year exceeds the limit laid down in Section
44AB, are liable to deduct tax at source from specified payments
made to resident contractors.
(iii) True: The definition of agriculture income as per Section 2(1a) has
been amended. Agricultural income includes income derived from sale
of saplings grown in a nursery, It is not required that the basic
operations should have been done by the assessee only.
(iv) False: The Finance Act, 2008 has amended the same. Short-term
capital gains arising from sale of shares through a recognized stock
exchange for which security transaction tax has been paid, will be
charged to tax at a concessional rate of 15% from the assessment
year 2009-10 onwards.
7.410 O Scanner CMA Inter Gr. I Paper 7 (2016 Syllabus)

(v) False: The book profit has to be increased by certain items listed in
clause (a) of the Explanation to Section 115JB. Now a specific item
has been inserted so as to enjoin that the book profit shall be
increased by the amount of deferred tax or provision for deferred
taxation.
Space to write important points for revision

2009 - June [1] {C} (b) Fill up the blanks:


(ii) Service tax return can be revised within a period of days.
(iii) Interest on refund on Income-tax paid in excess is a receipt.
(iv) An assessee, after sale of house property, receiving arrears of rent
(is/is not) chargeable to tax; the same computed in the
stipulated manner, is chargeable to tax as (income from other
sources/income from house property). (1 × 3 = 3 marks)
Answer:
(b) (ii) 90
(iii) Revenue or taxable
(iv) is, Income from House Property
Space to write important points for revision

2009 - June [1] {C} (c) Choose the correct answers:


(ii) The following is not capital receipt;
(1) Dividend from investments
(2) Bonus shares
(3) Sale of technological know-how
(4) Compensation received for compulsory evacuation of place of
business.
(iv) Cash gifts received under Section 56(2)(vi) from non-relatives are not
taxable upto.
(1) ` 1,00,000
(2) ` 75,000
(3) ` 50,000
(4) ` 25,000
[Chapter  21] Objective Questions O 7.411

(v) Mr. A has three minor children deriving interest from bank deposits to
the tune of ` 2,000, ` 1,300 and ` 1,600 respectively. Exemption
available under Section 10(32) of the Income-tax Act, 1961 is
(1) ` 4,900
(2) ` 4,300
(3) ` 4,500
(4) None of the above.
(vi) Miss Femina, aged 17, is married to Mr. Masculine. Her mother alone
is alive. Income by way of interest on loans, of Miss Femina will be
(1) Assessed to tax in the hands of Mr. Masculine
(2) Assessed to tax in the hands or her mother
(3) Taxable in her own hands
(4) None of the above.
(vii) Mr. X gifts ` 60,000 to the HUF of Which he is member; said amount
will be treated as income of
(1) Mr. X
(2) The HUF
(3) None, as it is exempt
(4) None of the above. (1 × 7 = 7 marks)
Answer:
(ii) (4),
(iv) (3),
(v) (2), u/s 10 (32) ` 1,500 per child is exempt have 43,00 (1500  1300
 1500) is exempt.
(vi) (2),
(vii) (2)
Space to write important points for revision

2009 - Dec [1] {C} (a) Fill up the blanks:


(i) Compensation received from an insurer on account of damage to
the crops is _______ income.
(ii) Time limit for filing revised return when assessment has not been
completed is ______ from the end of the relevant assessment year.
(iii) A ________ means A Company which is not a domestic company.
7.412 O Scanner CMA Inter Gr. I Paper 7 (2016 Syllabus)

(iv) The deduction for amortisation of preliminary expenses under


Section 35D is allowable at _______ of the qualifying expenditure in
each of the _______ successive years beginning with the year in
which business commences.
(v) _______ is a non-recurring expenditure whereas _______ is
normally a recurring one.
(vi) The term business would include ______ and accordingly the term
business used in Section ________ would also include a
professional connection.
(vii) Any commission due or received by a partner of a firm from the firm
shall __________ be regarded as _________ under
Section ________ . (7 marks)
(b) State with reasons, whether the following statements are True or False:
(i) Depreciation is allowed only when it is claimed.
(ii) X is employed in complex Ltd. as a Chartered Accountant. The
annual membership fee of X paid by complex Ltd is not a perquisite
and hence not chargeable to tax.
(iii) In the case of a dealer in shares, Income by way of dividend is
taxable under the heads “profits and gains of business or
profession”.
(iv) In case of an artificial judicial person, no surcharge is payable where
the total income exceeds ` 1,00,00,000. (5 marks)
(c) Expand abbreviations:
(i) ITAT,
(ii) MAT; (2 marks)
Answer:
(a) (i) Agricultural income
(ii) 1 year
(iii) Foreign
(iv) Aggregate, 5
(v) Capital expenditure, revenue expenditure.
(vi) Trade, commerce or manufacture, 2(13)
(vii) not, salary, 15
(b) (i) False: Depreciation is mandatory to deduct. There is no need to
claim it.
[Chapter  21] Objective Questions O 7.413

(ii) False: It is a perquisite in the hand of employee. Hence, it is


chargeable to tax.
(iii) True: In the given case, dealer is trading in shares, so income by
way of dividend is taxable under the head “profit & gain of business
of profession”.
(iv) False: In case of an artificial judicial person, surcharge is payable
where the total income exceeds ` 1,00,00,000.
(c) (i) ITAT: Income Tax Appellate Tribunal
(ii) MAT: Minimum Alternate Tax
Space to write important points for revision

2010 - June [1] {C} (a) Fill up the blanks:


(i) Section 139(1) applies to all persons whether they are _____
or _____.
(ii) Receipts from TV serial shooting in Farm house _______ agricultural
income.
(iii) Section________defines various income which are chargeable to tax
under the head “Profits and gains of business or profession”.
(vi) The basis of chargeability under the head ‘income from house
property’ is _________.
(vii) _______ salary is taxable, while _______ against salary is not taxable.
(5 marks)
(c) State with reasons whether the following statements are True or False
(answers without reasoning will not receive any credit):
(i) An assessee owns 11 trucks. One truck is always kept as a spare
vehicle and is never plied on the road. Since only 10 vehicles are
plied on the road at any given point of time, the provisions of Section
44AE of the Income Tax Act, 1961, can be availed by the assessee.
(ii) Mr. Janak has received as gift, gold bullion bars worth ` 70,000 from
his friend on his birthday on 15.3.2019. The same is not to be treated
as income from other sources.
(iii) Mr. Saravanan follows mercantile system of accounting. On
13.3.2019, he has received from the State Government, in respect
of lands acquired, interest on enhanced compensation of ` 1,50,000
which includes a sum of ` 20,000 relatable to this year. The amount
assessable is ` 20,000.
7.414 O Scanner CMA Inter Gr. I Paper 7 (2016 Syllabus)

(iv) The period for setting off the MAT credit under Section 115JB is
seven years.
(v) The rate of TDS applicable for payment made on 28.02.2019 non-
individual sub-contractor, as per Section 194C, is 2%.
(vi) Only in the TDS certificate furnished by the deductor, quoting the
PAN of deductor is compulsory and not in other correspondences
between the deductor and the deductee. (1 × 6 = 6 marks)
Answer:
(a) (i) Resident/Non-Resident
(ii) is not
(iii) Section 28
(vi) Annual Value
(vii) Advance / Advance
(c) (i) FALSE: The benefit of computing business profit on presumptive
basis is available only where the assessee owns not more than 10
vehicles.
The criteria is number of vehicles owned and not number of vehicles
plied, hence the given statement is false.
(ii) TRUE: Under Section 56(2) (vii) of the Income-Tax Act, 1961, gift
of certain property received from non-relative is to be treated as
income from other sources. The term 'property' has been specifically
defined. Jewellery is covered by this definition, but gold bullion bars
are not covered. Hence the given statement is true.
(iii) FALSE: As per the amendment to Section 145A of the
Income-Tax Act, 1961, regardless of the method of accounting
followed by the assessee, interest on enhanced compensation is
assessable as income from other sources in the year of receipt.
Thus the entire interest of ` 1,50,000 is assessable in the current
assessment year 2018-19.
(iv) FALSE: Section 115AA has been amended by the Finance (No.2)
Act, 2009. Effective from AY 2011-12, the period for setting off the
tax credit has been increased to 10 years.
[Chapter  21] Objective Questions O 7.415

(v) TRUE: Section 194C has been substituted by a new Section w.e.f.
1-10-2009. As per the new section, there is no concession available
to sub-contractors, which was in vogue till 30-09-2009. The rate of
TDS is 2% w.e.f. 1-10-2009 in respect of contract payment made to
contractors/sub-contractors other than individuals/HUFs.
(vi) FALSE: As per Section 206AA, w.e.f. 1-4-2010 (A.Y.2010-11),
PAN of deductor is to be quoted not only in the TDS certificate but
also in all TDS statements, vouchers, bill and correspondence
between deductor and deductee.
Space to write important points for revision

2010 - Dec [1] {C} (a) Fill up the blanks:


(i) Medical insurance premium paid otherwise than in ............... is eligible
for deduction under Section 80D of the Income-tax Act, 1961.
(ii) Deemed Dividend is ..............(liable to tax/not liable to tax) under
Section 2(22)(e) of the Income-tax Act, 1961.
(iii) Expenditure incurred towards demerger is deductible in ...........equal
annual instalments under Section 35 DD of the Income -tax Act, 1961.
(iv) Arrear rent is taxable after deducting .............. % as per Section 25A
of the Income-tax Act, 1961.
(v) Amount received towards permission for putting up hoarding at the top
of the building is taxable under the head.................. .
(vi) The cost of acquisition of 100 bonus shares, where the original shares
(100 nos.) were acquired for ` 30,000 is..............
(vii) Income-tax rates are not prescribed by the ............ Act, but by the
.............. Act of each year.
(viii) It is obligatory to pay advance tax where the amount of tax is
`............or more.
(ix) While effecting the tax deduction at source, education cess and
special higher education cess totaling 4%................... (should/need
not) be also deducted from the amount due or payable to the
deductee. (1×9= 9 marks)
7.416 O Scanner CMA Inter Gr. I Paper 7 (2016 Syllabus)

(b) Choose the correct alternative:


(ii) The following is not taxable as income under the head “Salaries”:
(A) Commission received by a full-time director;
(B) Remuneration received by a partner;
(C) Allowances received by an employee;
(D) Free accommodation given to an employee.
(iii) Following Form Number is to be used for filing the return of income
by an individual having business income:
(A) Form No. 1;
(B) Form No. 2;
(C) Form No. 4;
(D) Form No. 4A.
(iv) Income received in India in previous year is taxable in the hands of:
(A) Resident;
(B) Not-resident;
(C) Non ordinarily resident;
(D) All above.
(v) Following is not a capital receipt:
(A) Dividend on investment;
(B) Bonus shares;
(C) Sale of know-how;
(D) Compensation received for vacating business place.
(1 × 5 = 5 marks)
(c) State with reasons whether the following statements are True of False
(Answers without reasons are not eligible for any credit):
(i) Political parties governed by Section 13A of the Income-tax Act,
1961 have to file their returns of income within the time limit
prescribed under Section 139(1) even if there is no income
chargeable to tax under the Act.
(ii) ‘Gross total income’ means aggregate of income computed under
various heads and after allowing deduction under Chapter VI-A.
(iii) Municipal tax in respect of staff quarters is deductible only if it is paid,
in computing business income.
[Chapter  21] Objective Questions O 7.417

(iv) Amount received under Keyman insurance policy is not exempt


under Section 10(10D) of the Income-tax Act, 1961.
(v) Market value of donation given in kind is also eligible for deduction
under Section 80G of the Income-tax Act, 1961.
(vi) Amount received under Reverse Mortgage Scheme is taxable as
income under the head ‘income from other sources’.
(vii) Advertisement in any souvenir, brochure, pamphlet or the like
published by a political party is not deductible under Section 37(2B)
of the Income-tax Act, 1961. (1×7= 7 marks)
Answer:
(a) (i) Cash
(ii) not liable to tax
(iii) 5
(iv) 30 (Standard deduction)
(v) Income from Other Sources
(vi) Nil
(vii) Income Tax, Finance
(viii) 10,000.
(ix) should
(b) (ii) ----B
(iii) ----C
(iv) ----D
(v) ----D
(c) (i) False: Every political party is obliged to file every year a return of its
total income, if income of political party exceeds maximum
exemption limit(without giving effect of Section 13A) in terms of
Sec. 139(4B). Hence if income of political party is less than
maximum exemption limit, than no need to file return of income.
(ii) False: ‘Gross total income’ means aggregate of income computed
under various heads and before allowing deduction under Chapter
VI-A.
(iii) True: Municipal Taxes paid on staff quarters is allowed as business
expense on paid basis u/s 43B.
(iv) True: Amount received under Key Man Insurance Policy is taxable
along with bonus if any received.
7.418 O Scanner CMA Inter Gr. I Paper 7 (2016 Syllabus)

(v) False: Donation given in kind is not deductible u/s 80G.


(vi) False: Amount received under reverse mortgage scheme is not
considered as income hence not taxable.
(vii) True: Not deductible as per Section 37(2B)
Space to write important points for revision

2011 - June [1] {C} (a) Fill up the blanks:


(i) Mr. Nathan acquired on 30-3-2019, a building for ` 10 lakhs when the
State stamp valuation authority adopted ` 15 lakhs for stamp duty
purpose. The amount taxable in the hands of Mr. Nathan u/s 56(2) will
be ______. (1 mark)
(ii) The due date for filling return of income u/s 139(1) in the case of
individual assessee having turnover above ` 100 lakhs is ________.
(1 mark)
(iii) Where an employee is paid fixed medical allowance of ` 1,000 per
month, a sum of ` ______is taxable per annum. (1 mark)
(iv) Salary paid to a working partner of a firm is chargeable to income-tax
in the hands of such partner under the head___________. (1 mark)
(v) Total tax payable on a lottery income of ` 3,00,000 as per Section
115BB is ` _________. (1 mark)
(vi) Payment of education loan ________(principal/interest) is deductible
under Section 80E. (1 mark)
(viii) Surcharge is payable by Indian Companies when the total income
exceeds _____________. (1 mark)
(ix) Maximum amount exempt in respect of Transport Allowance granted
to an employee for commuting between residence and place of work
is ` _________ per month. (1 mark)
(x) Claim of depreciation is _______(mandatory/optional) while computing
business income of the assessee. (1 mark)
(xi) Advance tax is payable in ________instalments by a non-corporate
assessee. (1 mark)
(xii) The Minimum Alternate Tax rate applicable to certain companies is
___________ percent of the book profits. (1 mark)
[Chapter  21] Objective Questions O 7.419

(b) Choose the correct alternative:


(i) An individual (aged 28 years) born in India left for employment in
France on 30.10.2018. His visit outside India is for the first time.
His residential status for the assessment year 2019-20 will be
(a) Resident and Ordinarily Resident
(b) Resident but Not Ordinarily Resident
(c) Non-resident
(d) None of the above (1 mark)
(ii) Xmas Charities received anonymous donation of ` 5 lakhs. The said
sum is
(a) Exempt from tax
(b) Partly exempt and partly taxable
(c) Taxable @ 30%
(d) None of the above (1 mark)
(iii) Mr. A let out his house property to Mr. B, Mr. B sub-let the property
to Mr. C. The income from sub-letting in the hands of Mr. B will be
(a) Exempt income
(b) Taxable under the head 'house property'
(c) Income from other sources or business income, depending on
facts
(d) Income under the head 'capital gains' (1 mark)
(iv) Miss Padmaja received ` 50,000 on 13.10.2018 being the
unrealized rent pertaining to an earlier year, consequent to
Court/Tribunal decree. The property to which the unrealized rent
relates was sold in the year 2013-14. The unrealized rent amount
received now is,---
(a) Fully taxable as income from other sources
(b) 50% taxable as income from other sources
(c) Fully taxable as income from house property
(d) 70% taxable as income from house property (1 mark)
7.420 O Scanner CMA Inter Gr. I Paper 7 (2016 Syllabus)

(c) State with reasons whether the following statements are true or false:
(i) Where Mr. VKS (an Indian resident) received ` 50,000 as dividend
from a foreign company, the same is taxable.
(ii) Farm house rent received from tenant of agricultural land is
chargeable to tax as income from house property.
(iv) Income from transfer of self-generated goodwill of a profession is not
chargeable to tax under the head "Capital gains". (2 marks each)
Answer:
(a) (i) ` 5,00,000
(ii) 30/09/2019
(iii) ` 12,000
(iv) Profits and Gains of Business or Profession
(v) ` 93,600 (31.20% of 3,00,000)
(vi) Interest
(viii) 1crore
(ix) NIL
(x) Mandatory
(xi) Four
(xii) 18.5%
(b) (i) (a) Resident and ordinarily Resident
(ii) (c) Taxable @ 30%
(iii) (c) Income from Other Sources or Business income, depending on
facts
(iv) (c) Fully Taxable as Income from House Property
(c) (i) True: Dividend received from an Indian company is exempted from
tax u/s 10(23) but dividend received from a foreign company will be
taxable.
(ii) False: Income from form house is considered as agriculture income.
The sole purpose of such form house should be for use of dwelling
for the cultivators or use as store houses.
(iv) True: Income from transfer of self generated goodwill of a
professional is not chargeable to tax since there is no cast of
acquisition and no capital gain. Supreme Court Judge DC Sriniwas
setty.
Space to write important points for revision
[Chapter  21] Objective Questions O 7.421

2011 - Dec [1] {C} Choose the correct answer:


(a) FOR RAJASTHALI Ltd., a company engaged in the manufacture of
automotive parts, the rate of depreciation on acquiring machinery from
UK on 1 January, 2017 to be used in India for the first time for AY 2019-
20 will be:
(a) 15%
(b) 25%
(c) 7.5%
(d) None of these
(b) ARUN JAIDEV Ltd. has taken office premises on lease from Mr. X. The
monthly rental amounts to ` 15,000 on which service tax is levied @
15% thereby resulting in a total monthly expense of ` 17,250. TDS u/s
194I on this transaction is applicable on:
(a) ` 15,000
(b) ` 16,500
(c) ` 16,854
(d) Nil
(c) PREM BANSAL Ltd. makes a payment of ` 35,000 in cash to a
transporter for plying of goods carriages in a single day. The
disallowance under Section 40A(3) will be:
(a) ` 35,000
(b) ` 20,000
(c) ` 15,000
(d) Nil
(d) S. S. CORPORATE SECURITIES Ltd., an existing company incurred an
expenditure of ` 1,00,000 in FY 2018-19 in connection with issue of
shares to increase its share capital for a period of 10 years. The amount
allowed as deduction under Section 37(1) for AY 2019-20 will be:
(a) ` 10,000
(b) ` 20,000
(c) ` 1,00,000
(d) Nil
7.422 O Scanner CMA Inter Gr. I Paper 7 (2016 Syllabus)

(e) PELF FINSTOCK Ltd. filed its Return of Income Tax for AY 2019-20 on
30th March, 2020. The notice for making scrutiny assessment under
Section 143(3) can be served on the assessee upto:
(a) 30th September, 2019
(b) 31st December, 2019
(c) 30th March, 2020
(d) 30th September, 2020
(f) Interest under Section 234B is payable by an assessee due to the
following reason:
(a) Defaults in furnishing return of income
(b) Defaults in payable of advance tax
(c) Defaults in instalments of advance tax
(d) Defaults due to non-deduction of tax at source
(g) The basic exemption limit for a Non-resident woman who is 70 years old
for the Assessment Year 2019-20 is:
(a) ` 1,60,000
(b) ` 2,40,000
(c) ` 1,90,000
(d) ` 2,50,000
(h) The charging Section of the Income Tax Act is:
(a) Section 1
(b) Section 2
(c) Section 3
(d) Section 4
(i) Deduction of profit and gains derived from export under Section 10AA
available to SEZ units is for____years.
(a) 10
(b) 5
(c) 8
(d) None of these
(k) Walmart Ltd., is in the business of setting up and operation cold chain
facilities. It commenced its operations on 1.4.2018 and claimed
deduction of all expenditure under Section 35AD. The net loss for the
year ending 2018-19 is ` 50,00,000. The number of years for which this
loss can be carried forward is:
[Chapter  21] Objective Questions O 7.423

(a) 8 years
(b) 4 years
(c) Infinitely
(d) Cannot be carried forward
(l) Mr. RAKESH BEHARI is a trader and deals in purchase and sales of
cloth. He is required to get his accounts audited under Section 44AB if
his total sales/turnover exceeds:
(a) ` 40,00,000
(b) ` 10,00,000
(c) ` 15,00,000
(d) ` 100,00,000
(m) MAT credit under Section 115JAA is allowed to be carry forward and set-
off for_______years.
(a) 10
(b) 8
(c) 7
(d) Infinitely
(o) The ceiling limit for exemption of Gratuity under Section 10(10) received
from the employer is:
(a) ` 10,00,000
(b) ` 3,50,000
(c) ` 5,00,000
(d) ` 3,00,000
(p) On 17th January, 2019, M/s. Naina sold a house property and earned a
long term capital gain of ` 1,02,50,000. She invested a sum of
` 50,00,000 in bonds specified in Section 54EC on 8th March, 2019. She
further invested a sum of ` 50,00,000 in same bonds on 8th May, 2019
Taxable Income of M/s. Naina for the Assessment Year 2019-20 will be:
(a) ` 50,00,000
(b) ` 52,50,000
(c) ` 1,00,50,000
(d) ` 2,50,000
7.424 O Scanner CMA Inter Gr. I Paper 7 (2016 Syllabus)

(q) PRARTHANA Ltd. has to make a payment of ` 10,00,000 to Mr. PIYUSH


for certain services. PRARTHANA Ltd. is required to deduct tax at
source at the rate of 10%. However, Mr. PIYUSH does not provide
Permanent Account Number (PAN) to PRARTHANA Ltd. for deduction
of tax from ` 10,00,000. The rate at which tax will be deducted by
PRARTHANA Ltd. will be:
(a) 10%
(b) 20%
(c) 30%
(d) No TDS should be deducted due to non-availability of PAN
(r) Incomes which accrue or arise outside India but are received directly in
India are taxable in case of:
(a) Resident only
(b) Non-Residents
(c) All Assessees
(d) Resident but not Ordinarily Resident
(s) Where the Assessing Officer is aggrieved by an order of the
Commissioner of Income-tax (Appeals), further appeal in respect of
same lies to:
(a) Income-tax Appellate Tribunal
(b) Dispute Resolution Panel
(c) Central Board of Direct Taxes
(d) Income-tax Settlement Commission
(t) A company is required to pay 75% of its advance tax liability by which
date:
(a) 15 March of the previous year
(b) 15 December of the previous year
(c) 15 September of the previous year
(d) 15 June of the previous year
(u) S. Ltd. is an Indian Company which is 100% subsidiary of H. Ltd., a
Foreign Company. H. Ltd. sells its products to S. Ltd. at $ 15 per unit. At
the same time, it sells its products to an unrelated party at $ 20 per unit.
The arm's length price as per Section 92C in this transaction will be:
[Chapter  21] Objective Questions O 7.425

(a) $ 15
(b) $ 20
(c) $ 15.75
(d) $ 21
(v) R. Ltd. is registered in UK. Its control and management is wholly situated
in India. R. Ltd. shall be:
(a) Resident in India
(b) Non-resident in India
(c) Resident but not ordinarily resident
(d) Resident in India if other certain conditions are satisfied
(x) Circulars issued by the Central Board of Direct Taxes is binding on:
(a) Assessee only
(b) Income-tax authority only
(c) Both assessees and income tax authorities
(d) On everyone except the Judiciary
(y) Additional depreciation under Section 32(1)(iia) of the Income-tax Act,
1961 is available to:
(a) Plant & Machinery
(b) Plant & Machinery and Buildings
(c) Plant & Machinery and intangible assets
(d) All depreciable assets under the Income-tax Act, 1961
(1 × 24 = 24 marks)
Answer:
(a) (C) (Rate of depreciation on machine is 15%. Half of the rate, if used for
less than 180 days.)
(b) (D) (As per Section 194I, any person who is responsible for paying to
a resident any income by way of rent shall deduct income tax
thereon @ 10% for the use of any land or building provided amount
exceed ` 1,80,000. Hence, no TDS liability u/s 194I.
(c) (D) (As per Section 43A (3), any payment in cash made to a transporter
is deductible expenditure up to ` 35,000.)
(d) (D) (Expenses relating to increasing authorised Share capital shall be
treated as capital expenditure. hence not deductible u/s 37(1).)
7.426 O Scanner CMA Inter Gr. I Paper 7 (2016 Syllabus)

(e) (D) (Notice u/s 143(3) can be served on the assessee within 6 months
from the end of the financial year in which return is furnished i.e. 30th
September, 2020).
(f) (B) (Interest is paid by assessee under Section 234B @ 1% p.m., if he
defaults in paying Advance Tax.)
(g) (D) (Basic exemption limit for non residents is ` 2,50,000.)
(h) (D) (Section 4 is charging Section under the Income Tax Act, 1961.)
(i) (D) (Section 10AA, for first 5 years 100% and next 5 years 50% for
further 5 years 50% subject to certain conditions)
(k) (c) (loss from specified business covered under Section 35AD can be
carried forward for any number of years immediately succeeding the
assessment year in which such loss is first computed.)
(l) (D) (As per Section 44AB, every assessee whose gross turnover
exceeds ` 1crore year is required to get his accounts audited by any
practicing chartered Accountant.)
(m) (A) (Section 115JAA, MAT credit is allowed to be carry forward and
set-off for 15 years)
(o) (A) (The ceiling limit for exemption of Gratuity under Section 10(10)
received from the employer is ` 10,00,000 for the employees whose
contract of service is terminated on or after 24/05/2010.)
(p) (B) (1,02,50,000 - 50,00,000 = 52,50,000) (As per Section 54EC if any
assessee transfers its any long term capital asset and acquires
specified bond within the specified time then he can claim a
deduction of an amount to the maximum of the amount he has
invested in acquisition of new asset.)
(q) (B) (Rate of TDS will be 20%, if assessee has not furnished PAN.)
(r) (C) (Incomes which accrue or arise outside India but are received
directly in India are taxable in case of all Assessees)
(s) (A) (Where the Assessing Officer is aggrieved by an order of the
Commissioner of Income-tax (Appeals), further appeal in respect of
same lies to Income-tax Appellate Tribunal.)
(t) (B) (A company is required to pay 75% of its advance tax liability by 15
December of the previous year.)
(u) (A) (Section 92C, The arm’s length price in this transaction will be ` 15.)
[Chapter  21] Objective Questions O 7.427

(v) (A) (A foreign company whose control & management is wholly situated
in India is termed as Resident in India.)
(x) (B) (circulars issued by the CBDT is binding on income tax authority
only.)
(y) (A) (Additional depreciation u/s 32(1)(iia) is available on Plant &
Machinery)
Space to write important points for revision

2012 - June [1] {C} (a) Choose the correct alternative:


(i) The basic exemption limit for a non-resident woman above the age of
60 years for the assessment year 2019-20 is
(a) ` 1,90,000
(b) ` 2,00,000
(c) ` 2,40,000
(d) ` 2,50,000
(ii) When a person retires from a profession and receives any amount
towards self-generated goodwill, it is
(a) Taxable as income from profession
(b) Exempt income under Section 10
(c) Not taxable since there is no cost of acquisition
(d) Taxable as capital gain
(iii) Mr. Xavier received ` 2,00,000 from the prospective employer before
joining duty in order to resign from the present employer.
Subsequently, he joined the new employer. The amount received is
(a) Taxable as income from business
(b) Taxable as salary income
(c) Exempt from tax u/s 10
(d) Exempt being capital receipt
(iv) Jain introduced his motor car costing ` 3,00,000 acquired in April,
2016 into the business newly commenced by him from 01.04.2017.
The actual cost of car for the purpose of depreciation would be
(a) ` 3,00,000
(b) ` 2,16,750
7.428 O Scanner CMA Inter Gr. I Paper 7 (2016 Syllabus)

(c) ` 1,50,000
(d) ‘Nil’ not eligible for depreciation
(v) For non-Government employee governed by Payment of Gratuity Act,
1972, the monetary limit for exemption is
(a) ` 5,00,000
(b) ` 3,50,000
(c) ` 10,00,000
(d) Limitless
(vi) Unrealised rent of ` 50,000 was received in June, 2017. The property
was sold before April, 2018. How much of unrealized rent is taxable?
(a) ` 50,000
(b) ` 35,000
(c) ` 30,000
(d) Not taxable
(vii) A registered charitable trust meant for educational purpose has annual
aggregate receipt of ` 80,00,000. Its income after expenses is
` 20,00,000. The income liable to income-tax would be
(a) ` 80,00,000
(b) ` 60,00,000
(c) ‘Nil’
(d) ` 30,00,000
(viii) Pravin received ` 2,000 per month as transport allowance. The
amount eligible for exemption would be
(a) ` 1,000 p.m.
(b) ` 200 p.m.
(c) ` 1,600 p.m.
(d) ‘Nil’ fully taxable
(ix) Jayant working in a college received ` 2,000 per month as research
allowance for pursuing research. The taxable portion of allowance
would be
(a) ` 2,000 p.m.
(b) ‘Nil’
[Chapter  21] Objective Questions O 7.429

(c) ` 1,000 p.m.


(d) ` ‘Nil’ fully exempt
(x) Pankaj was provided accommodation in a hotel by the employer for 10
days consequent to his transfer from Mumbai to Kolkata. The cost of
accommodation was ` 30,000 to the employer. The value of perquisite
is
(a) ‘Nil’
(b) ` 30,000
(c) ` 15,000
(d) ` 20,000
(xi) A & Co., a partnership firm, contributed ` 1,00,000 towards family
planning programme among the employees. The amount eligible for
deduction would be
(a) ` 1,00,000
(b) ` 20,000
(c) ` 1,50,000
(d) ‘Nil’ not eligible for deduction
(xii) A sum of ` 50,000 was written off as bad debt in the assessment year
2010-11 and was disallowed. During financial year 2018-19 ` 20,000
was recovered. Out of the recovery how much is taxable?
(a) ` 20,000
(b) Nil
(c) ` 30,000
(d) ` 70,000
(xiii) Pon Ltd. incurred ` 35,00,000 towards voluntary retirement
compensation paid to its employees in the financial year 2018-19. How
much is deductible for the assessment year 2019-20 out of the said
payment?
(a) ` 7,00,000
(b) ` 5,00,000
(c) ` 35,00,000
(d) ` 3,50,000 (1 × 13 = 13 marks)
7.430 O Scanner CMA Inter Gr. I Paper 7 (2016 Syllabus)

(b) Fill up the blanks:


(i) Salary expenditure not recorded in the books is taxable as
unexplained expenditure and at the same time it is _____
(deductible/not deductible) for computing total income as per
Section 69C.
(ii) Amount received under keyman insurance policy including bonus
thereon is ______ (income/exempted income) under the Income-tax
Act, 1961.
(iii) Income of minor child when subjected to clubbing, a sum of
` ________ is exempt.
(iv) Deposit in public provident fund in the name of minor child is
______ (deductible/not deductible) under Section 80C in the hands
of contributing parent.
(v) An individual can avail the benefit of exemption in respect of leave
travel concession offered by his employer _______ in a block of four
years.
(vi) The maximum amount of exemption in respect of encashment of
earned leave is ` _______ .
(vii) A charge created consequent to the decree of a Court/Tribunal of
law is ________ (application/diversion) of income.
(viii) Amount recovered by an employer from the employees towards the
latter’s share of provident fund contribution is ______ (income/not
an income) of the assessee-employer.
(ix) Loss from non-speculation business _______ (can/cannot) be set
off against profits derived from speculation business.
(x) Salary foregone is _______ (taxable/not taxable) in computing the
income from salaries in the hands of the concerned employee.
(xi) The monetary ceiling limit for exemption for gratuity received w.e.f.
21.05.2011 under the Payment of Gratuity Act, 1972 is ` _______.
(xii) Fixed medical allowance of ` 2,000 per month paid by an employer
is ______ (taxable/exempt) in the hands of the employee.
( 1 × 12 = 12 marks)
[Chapter  21] Objective Questions O 7.431

Answer:
(a) (i) (d)
(ii) (c)
(iii) (b)
(iv) (a)
(v) (c)
(vi) (a)
(vii) (c)
(viii) (d)
(ix) (d)
(x) (a)
(xi) (a) (As per Section 10(14) read with rule 2BB, which states
maximum amount of allowance exempt from tax is to the extent the
allowance is utilized for specified purpose. The question is silent
about utilization of allowance).
(xii) (b)
(xiii) (a)
(b) (i) Not deductible
(ii) income
(iii) 1500
(iv) Deductible
(v) Twice
(vi) 3,50,000
(vii) Diversion
(viii) Income
(ix) Can
(x) Taxable
(xi) 10,00,000
(xii) Taxable
Space to write important points for revision

2012 - Dec [1] {C} (a) Choose the most appropriate alternative:
(i) The basic exemption limit for a non-resident super senior citizen above
the age of 80 years for the assessment year 2019-20 is
7.432 O Scanner CMA Inter Gr. I Paper 7 (2016 Syllabus)

A. ` 2,00,000
B. ` 2,50,000
C. ` 5,00,000
D. ` 1,60,000
(ii) When patent is transferred, the amount received is
A. Not chargeable to tax
B. Chargeable under the head ‘Profits and gains of business or
profession’
C. Taxable under the head ‘Income from other sources’
D. Adjusted towards the patent value in the block of assets
(iii) Rent derived from land located outside India, used exclusively for
agricultural purpose is
A. Agricultural income
B. Exempt income
C. Taxable as business income
D. Taxable under the head ‘Income from other sources’
(iv) Ram introduced his building costing ` 10,00,000 acquired in April,
2011 into the business newly commenced by him from 01.04.2018.
The actual cost of building for the purpose of depreciation for the
assessment year 2019-20 would be `
A. 10,00,000
B. 5,90,490
C. 6,56,100
D. None of these.
(v) Motor car with more than 1.6 litres cubic capacity is given to the
employee both for official and personal use with expenditure on
running and maintenance met by the employer. The car was self
driven by the employee. The perquisite value shall be
A. ` 1,200 p.m.
B. ` 1,800 p.m.
C. ` 2,400 p.m.
D. Nil
(vi) The liability for payment of advance tax would arise only when the tax
liability of the taxpayer after reducing tax deductible at source exceeds
[Chapter  21] Objective Questions O 7.433

A. ` 10,000
B. ` 5,000
C. ` 20,000
D. None of these
(vii) Amount is received under the notified reverse mortgage scheme, is
A. Taxable as ‘Income from other sources’
B. Exempt from tax
C. 50% taxable
D. Taxable as capital gain
(viii) Azhar, being an employee in a transport company, received ` 10,000
per month by way of allowance to meet his personal expenses in the
course of running such transport system from one place to another.
The amount chargeable to tax is
A. Nil
B. ` 3,000 p.m.
C. Fully taxable
D. Fully exempt
(ix) Rajiv received scholarship of ` 20,000 to meet the cost of education.
The amount of scholarship chargeable to tax is `.
A. Nil
B. ` 20,000
C. ` 10,000
D. ` 5,000
(x) Sita received family pension of ` 10,000 per month in the capacity of
widow of the deceased spouse who died in the course of operational
duties in Indian army. The amount of family pension chargeable to tax is
A. ` 1,20,000
B. ` 1,05,000
C. Nil
D. ` 30,000
(xi) A religious trust registered under Section 12AA received ` 2,00,000 by
way of anonymous donation. The total amount of donation received
during the year was ` 15,00,000. The amount of anonymous donation
chargeable to tax is
7.434 O Scanner CMA Inter Gr. I Paper 7 (2016 Syllabus)

A. ` 2,00,000
B. ` 1,00,000
C. ` 75,000
D. Nil. Not taxable
(xii) Fees paid to Registrar of companies for increasing the authorized
capital of a company is
A. Capital expenditure
B. Fully deductible
C. 50% deductible
D. 75% deductible (1 x 12 = 12 marks)
(b) Fill up the blanks:
(i) Interest received on delayed payment of enhanced compensation
shall be deemed to be______________(income/not an
income/interest relating to the concerned year alone is income) of the
year in which it is received.
(ii) Gift received from a trust registered under Section 12AA
is_________ (included/not included) in the taxable income of an
individual.
(iv) Dividend received from a company having only agricultural income
is______ (agricultural income/non-agricultural income/50% taxable)
in the hands of its shareholder.
(v) There are two schools of Hindu Law, one in Mitakshara and the other
is____.
(vi) The depreciation allowable in respect of an asset used for the
purpose of business for less than 180 days shall be restricted
to______ (50%/25%/75%) of the normal rate of depreciation.
(vii) The rate of TDS will be__________in all cases, if PAN is not
furnished by the deductee.
(viii) The minimum alternate tax u/s 115JB for the A.Y. 2019-20 shall
be______% of book profit [Basic rate excluding surcharge, education
cess, etc.].
(ix) The minimum penalty levied u/s 271A for not maintaining books of
account, documents as required u/s 44AA is___________.
[Chapter  21] Objective Questions O 7.435

(x) Prima facie, revision of any order can be made by the Commissioner
of Income-tax within_________ years.
(xi) Unabsorbed loss under the head ‘Capital gains’ shall be carried
forward for a period of____________assessment years immediately
following the assessment year in which such loss was incurred.
(xii) Loss from gambling____________(can/cannot) be carried forward
and set off in subsequent years under profits from gambling.
(1 x 11 = 11 marks)
Answer:
(a) (i) (B) ` 2,50,000. Basic exemption limit for all non resident assessee
is ` 2,50,000.
(ii) (D) Patent is an intangible asset. As per Section 2(11) of the
Income Tax Act, there is a block for intangible assets, and if the
asset is transferred, the amount should be adjusted in the same
block of assets.
(iii) (D) Taxable under the head ‘Income from Other Sources’. As per
Section 10(1) of the Income Tax Act, agricultural income in India
is fully exempt from tax. Foreign agricultural income will not fall
under Section 10(1), hence taxable under the head ‘Income
from Other Sources’.
(iv) (c) 6,56,100
(10,00,000 – 10% of 10,00,000 = 9,00,000
9,00,000 – 10% of 9,00,000 = 8,10,000
8,10,000 – 10% of 8,10,000 = 7,29,000
7,29,000 – 10% of 7,29,000 = 6,56,100)
(v) (c) ` 2,400 pm. As per Rule 3(2)(A) of the Income Tax Act, if
employer is providing car with more than 1600 cc capacity to his
employee and running and maintenance is borne by employer,
the taxable value of perquisite is ` 2,400 pm
(vi) (A) `10,000
As per Section 208 of Income Tax Act, 1961, liability of
advance tax arises when estimated tax liability of tax payer
exceeds `10,000.
(vii) (B) Exempt from tax.
7.436 O Scanner CMA Inter Gr. I Paper 7 (2016 Syllabus)

(viii) (B) ` 3,000 pm.


As per Section 10(14), allowance given to transport employees
are exempt to the extent of least of following
i. 70% of allowance or
ii. ` 10,000. p.m.
(ix) (A) Nil, The scholarship granted to meet the cost of education is
exempt from tax u/s 10 (16).
(x) (C) Nil, the family pension received by the family members of armed
forces is exempted from tax u/s 10(19).
(xi) (D) Nil. Not Chargeable. The anonymous donation received for
religious purposes is excluded from the ambit of taxable
donation.
(xii) (A) Capital Expenditure. A fee paid to ROC is non-recurring
expenses. Hence it is capital expenditure.
(b) (i) Income
(ii) Not included
(iv) Non agricultural income
(v) Dayabhaga
(vi) 50%
(vii) 20%
(viii) 18.5%
(ix) ` 25,000
(x) 4
(xi) 8
(xii) Cannot
Space to write important points for revision

2013 - June [1] {C} (a) Choose the most appropriate alternative:
(i) The basic Exemption limit for a female below the age of 60 years for
the assessment year 2019-20 is
(a) ` 1,80,000
(b) ` 1,90,000
[Chapter  21] Objective Questions O 7.437

(c) ` 2,00,000
(d) ` 2,50,000
(ii) Under Rule 7A of the Income Tax Rules, the following %age of income
from manufacture of Rubber shall be deemed to be business income
and liable to tax
(a) 15%
(b) 25%
(c) 35%
(d) 50%
(iii) Interest is payable to assessee on Refund under the Income Tax Act,
1961 at the rate of
(a) 5%
(b) 6%
(c) 9%
(d) 12%
(iv) The maximum penalty leviable for failure to keep or maintain books
of account or document as required u/s. 44AA of the Income Tax Act,
1961 is
(a) ` 25,000
(b) ` 75,000
(c) ` 1,00,000
(d) ` 1,50,000
(v) Tax on non-monetary benefit paid by the employer is
(a) Fully taxable
(b) Taxable to the extent of 50%
(c) Taxable to the extent of 60%
(d) Fully exempted from Tax.
(vii) The maximum amount of deduction from Gross Total Income available
to an individual for interest on savings bank deposit is
(a) ` 5,000
(b) ` 7,500
(c) ` 10,000
(d) ` 12,000
7.438 O Scanner CMA Inter Gr. I Paper 7 (2016 Syllabus)

(viii) Loss from activity of owing and maintaining race horses can be carried
forward for : (Assessment years)
(a) 4
(b) 6
(c) 8
(d) 5
(ix) Annual value of house property if not let out is taken as _______.
(x) No tax is deductible if the amount of rent credited or paid during the

I
financial year does not exceed rupees _______ u/s 194 of the
Income Tax Act, 1961.
(xi) When entire net consideration has been invested by an individual
towards subscription of shares of an eligible company the exemption
u/s. 54GB of the Income Tax Act,1961 would be
(a) NIL
(b) 10% of capital gain
(c) 50% of capital gain
(d) 100% of capital gain
(xii) Amount received towards share application money when not properly
explained it is
(a) taxable u/s. 68
(b) exempt u/s. 10
(c) fully taxable but deduction at 50% u/s. 57 (iii) is allowable
(d) None of the above
(xiii) Book profit u/s. 115 JB of a domestic company was ` 52 lakhs. The tax
liability of the company for the assessment year 2019-20 would be
(a) 18.54% including cess
(b) 19.24% including cess
(c) 20% including cess surcharge at 5%
(d) 19.431% including cess and surcharge at 2%
(1 × 12 = 12 marks)
Answer:
(i) D. ` 2,50,000
(ii) C. 35%
[Chapter  21] Objective Questions O 7.439

(iii) B. 6%
(iv) The correct answer is `25,000 u/s. 271A.
(v) D. Fully exempted from tax
(vii) C. ` 10,000
(viii) A. 4 Assessment years
(ix) Nil
(x) ` 1,80,000
(xi) D. 100% of Capital Gain
(xii) A. Taxable u/s 68
(xiii) B. 19.24% including cess
Space to write important points for revision

2013 - June [1] {C} (b) Fill up the blanks:


(i) Any sum paid to an approved university, college or other institutions
u/s. 35(1) (iii) of the Income Tax Act, 1961 the allowable deduction is
_______(100%/125%)
(ii) Interest payable to a partner by a firm shall not exceed _______
(18%/12%) per annum.
(iii) An assessee _______ (can/cannot) spread over the arrears of rent
over the past several years.
(iv) Chapter VI-A deduction _______ (shall/shall not) be allowed in respect
of income from short term capital gain.
(v) Dividend receives by an Indian Company on shares of a Foreign
Company is _______ (taxable/exempted)
(vi) Salary received by Mr. P a foreign national and a non resident out-side
India for services rendered in India for 150 days is _______
(chargeable/not chargeable) to tax in India.
(vii) Deduction for provision for bad and doubtful debts made by a public
financial institution is allowed upto _______% of total income before
allowing such deduction and deduction under chapter VIA.
7.440 O Scanner CMA Inter Gr. I Paper 7 (2016 Syllabus)

(viii) Z. Ltd. awarded three contracts for repair work of ` 22,000, ` 23,000
and ` 30,000 respectively to L. Ltd. in the year 2018-19. Z. Ltd. is
______ (required/not required) to deduct tax at source under Section
194C of the Income Tax Act 1961.
(ix) In case of slum sale of any undertaking indexation benefit is ______
(allowed/not allowed) for the purpose of computation of capital gain.
(x) Annual value of any one palace in the occupation of a former ruler is
_______.
(xi) A charitable trust must apply atleast _______ percent of its income
towards its objects.
(xii) The time limit for issue of notice to assess the income in relation to
assets located out side India for reassessment purposes is _______
years from the end of the relevant assessment year.
(1 × 12 = 12 marks)
Answer:
(i) 100%
(ii) 12%
(iii) Cannot
(iv) Chapter VIA deduction shall be allowed in respect of short term
capital gain, generally. However, in case it is short term capital gain
in respect of transaction in equity shares in a company chargeable
to STT (u/s 111A) deduction under Chapter VIA shall not be
allowed. The question does not mention the type of transaction.
Hence, both answers would be possible.
(v) Taxable
(vi) Chargeable
(vii) 5
(viii) Not Required
(ix) Not allowed
(x) Exempt
(xi) 85
(xii) 16
Space to write important points for revision
[Chapter  21] Objective Questions O 7.441

2013 - Dec [1] {C} (b) Choose the most appropriate alternative:
(i) Travel expenditure of the patient and the attender for medical
treatment abroad is fully exempted, if gross total income before
including reimbursement of foreign travel expenditure is
(A) ` 2,00,000 (B) ` 2,50,000 (C) ` 3,00,000 (D) ` 5,00,000
(ii) Audit of accounts u/s 44AB of the Income-tax Act, 1961 is mandatory
for a person carrying on profession where his gross receipts exceed
(A) ` 40,00,000(B) ` 60,00,000 (C) ` 20,00,000 (D) ` 50,00,000
(iii) Long-term capital gain arising from sale of listed shares in a
recognized stock exchange (STT paid) is exempt under Section
_________ of the Income-tax Act, 1961:
(A) 10(35) (B) 10(37) (C) 10(38) (D) 10(36)
(iv) Deduction in respect of interest on deposits in savings account is
allowed under Section 80 TTA of the Income-tax Act, 1961 to the
maximum extent of
(A) ` 5,000 (B)`10,000 (C) ` 15,000 (D) ` 20,000
(v) An assessee who has no income from business or profession will not
be required to pay any advance tax if the said assessee is a/an
(A) Firm (B) AOP (C) Senior citizen(D) Indian Company
(1 x 5 = 5 marks)
Answer:
(i) a
(ii) d
(iii) c
(iv) b
(v) c
Space to write important points for revision

2014 - June [1] {C} (a) Fill up the blanks :


(i) Mr. A, a senior citizen, has total income of ` 8 lacs, earned by way of
interest from secured debentures. The advance tax payable by him is
` ________.
(ii) A partnership firm will be treated as non-resident, only if the ________
of the control and management of its affairs is situate outside India.
7.442 O Scanner CMA Inter Gr. I Paper 7 (2016 Syllabus)

(iii) An employee of a partnership firm is treated as ”specified employee”


if the income under the head ”Salaries”, excluding non-monetary
perquisites exceeds ` ________.
(iv) The maximum amount of retrenchment compensation exempt u/s 10
(10B) in the hands of a person, when received from a private scheme
not approved by the Board, is ` ________.
(v) Where any unrealized rent, earlier allowed as deduction is realized
subsequently, the deduction available therefor is ________.
(vi) In the case of a payee not having PAN for whom tax is to be deducted
at source u/s 194A, the rate applicable is ________. (6 marks)
(b) Choose the most appropriate alternative:
(i) For an employee in receipt of fixed medical allowance, the maximum
amount which is exempt is `
(A) 12,000 (B) 15,000 (C) 18,000 (D) Nil
(ii) Disallowance for expenditure incurred in relation to exempt income
is made under Section
(A) 14A (B) 14 (C) 80A (D) 10(33)
(iii) Where any land is located within aerial distance of 7 kms. from
municipal limits, to be regarded as capital asset u/s 2(14), the
population of the municipality as per last census done before
1.3.2013 should be more than
(A) 9 lacs (B) 8 lacs (C) 10 lacs (D) None of these
(iv) To avail exemption u/s 54, an individual should purchase a new
residential house within ________ years from the date of sale
(A) 2 (B) 3 (C) 1 (D) 4
(v) For an assessee engaged in manufacturing activity, additional
depreciation u/s 32(1)(iv) for second-hand machinery costing ` 3
lacs, installed on 12.05.2013 is `
(A) 30,000 (B) 45,000 (C) 60,000 (D) Nil
(5 marks)
Answer:
(a) (i) Zero
(ii) Whole
[Chapter  21] Objective Questions O 7.443

(iii) 50,000
(iv) 5,00,000
(v) 30% deduction u/s 25A is available
(vi) 20%
(b) (i) (D) Nil
(ii) (A) 14A
(iii) (C) 10 Lacs
(iv) (A) 2
(v) (D) Nil
Space to write important points for revision

2014 - Dec [1] {C} (a) Fill up the blanks:


(i) Deduction under Section 80G for donation to National Children’s Fund
is ______ percent.
(ii) Life Insurance premium paid in excess of ________ percent of the
actual capital sum assured is not deductible under Section 80C, in
respect of policies issued on or after 01.04.2013.
(iii) Commodities transaction tax is _________ even if it is incurred in the
course of business.
(iv) Buy back of unlisted shares by a company is _______ in the hands of
the shareholder.
(v) Rebate under Section 87 is to be calculated __________ the levy of
education cess.
(vi) Rate of income-tax applicable for foreign institutional investors in
respect of income from notified bonds and government securities
is__________.
(vii) The due date for furnishing Annual Information Return is _______.
(viii) Sale of gold coin in excess of _______ is liable for tax collection at
source. (1 × 8 = 8 marks)
7.444 O Scanner CMA Inter Gr. I Paper 7 (2016 Syllabus)

(b) Choose the most appropriate alternative:


(i) Deduction for investment in new plant or machinery under Section
32AC is applicable for
(A) all assessees
(B) companies
(C) partnership firms
(D) individuals
(ii) Income of securitization trust from the activity of securitization is
(A) exempt [Section 10(23 DA)]
(B) taxable at 20%
(C) taxable at 5%
(D) taxable at the regular rates
(iii) Royalty paid by State Government undertaking to the State
Government is
(A) deductible
(B) inadmissible
(C) 50% deductible
(D) 20% deductible
(iv) Time limit for setting up undertaking for generation of power to avail
deduction under Section 80-IA is available upto
(A) 31.03.2019
(B) 31.03.2018
(C) 31.03.2015
(D) 31.03.2016
(v) The maximum deduction under Rajiv Gandhi Equity Savings Scheme
is
(A) ` 10,000
(B) ` 50,000
(C) ` 1,00,000
(D) ` 25,000 (1×5 = 5 marks)
Answer:
(a) (i) 100
(ii) 10
[Chapter  21] Objective Questions O 7.445

(iii) Deductible
(iv) Exempt
(v) Before
(vi) 20%
(vii) 31st August
(viii) ` 2 lakhs
Space to write important points for revision

Answer:
(b) (i) (B) Companies
(ii) (A) Exempt
(iii) (B) Inadmissible
(iv) (C) 31.03.2015
(v) (D) 25,000
Space to write important points for revision

2015 - June [1] {C} (a) Fill up the blanks:


(i) A company incorporated outside India is said to be resident in India,
if control and management is _________ situated in India.
(ii) A foreign company is liable to surcharge at 5%, if the total income
exceeds ` _________.
(iii) A Zero coupon bond is a long-term capital asset, if it is held for more
than ________ months before transfer.
(iv) If statement of deduction of tax at source is not filed within due date,
the deductor is liable to a fee of ` _________ per day of default or the
amount of tax deductible, whichever is less.
(v) Maximum amount of exemption under section 10 (10C) of the Income-
tax Act in respect of compensation received for voluntary retirement is
` _________.
(vi) A manufacturing company investing more than ` ________ in new
plant and machinery in the previous year 2018-19 is entitled to
investment allowance @ 15%.
7.446 O Scanner CMA Inter Gr. I Paper 7 (2016 Syllabus)

(vii) An assessee can contest an order of the Income-tax Appellate


Tribunal on any substantial question of law of filing appeal to the
jurisdictional High Court within ________ days from the date of receipt
of the said order.
(viii) Royalty payable by Government to a non-resident is liable to be taxed
at ________ % on gross amount of royalty. (1 × 8 = 8 marks)
Answer:
(i) Wholly
(ii) `10 crores
(iii) 12 months
(iv) 200
(v) ` 5 lacs
(vi) ` 25 crores
(vii) 120
(viii) 25
Space to write important points for revision

2015 - June [1] {C} (b) Choose the most appropriate alternative:
(i) Subject to fulfilment of other conditions, remuneration received by a
foreign national as an employee of a foreign enterprise for services
rendered by him during his stay in India is exempted from income tax,
if his stay in India does not exceed a period of
(A) 30 days
(B) 60 days
(C) 90 days
(D) 120 days
(ii) Long-term capital gain on off-market sale of shares of a listed
company without availing of indexation benefit is taxed at
(A) 5%
(B) 10%
(C) 15%
(D) 20%
[Chapter  21] Objective Questions O 7.447

(iii) If dividend declared by an Indian company on or after 1st October,


2018 amounts to ` 50,00,000, then dividend distribution tax (without
considering surcharge and education cess) is
(A) ` 5,00,000
(B) ` 6,25,000
(C) ` 7,50,000
(D) ` 8,82,353
(iv) Deduction under section 80 JJAA in respect of employment of new
workmen can be claimed by a company for an amount equal to
(A) 15% of additional wages to new workmen
(B) 20% of additional wages to new workmen
(C) 25% of additional wages to new workmen
(D) 30% of additional wages to new workmen
(v) A return of income for Assessment Year 2019-20 filed within the due
date specified in Section 139(1) can be revised by the assessee at any
time before expiry of
(A) 31st March, 2019
(B) 31st March, 2020
(C) 31st December, 2018
(D) 31st December, 2019 (1 × 5 = 5 marks)
Answer:
(i) (C) 60 days
(ii) (B) 10%
(iii) (D) ` 8,82,353
(iv) (D) 30% of additional wages to new workmen
(v) (B) 31st March, 2020
Space to write important points for revision

2016 - June [1] {C} (a) Fill up the blanks:


(i) Assessee’s own contribution to the National Pension Scheme is
eligible for a maximum deduction of ` ___________.
(ii) Any payment received from an account opened under Sukanya
Samriddhi Account Rules, 2014 is ___________.
7.448 O Scanner CMA Inter Gr. I Paper 7 (2016 Syllabus)

(iii) A charitable trust in order to be eligible for exemption under section 11


must not have more than ___________ % of aggregate receipts from
any activity in the nature of trade, commerce or business.
(iv) The amount of deduction towards health insurance premium paid by
an individual (not being a senior citizen) is limited to ` ___________.
(v) Fee under section 234E for delay in filing of quarterly TDS/TCS return
is ` ___________ per day. (1 × 5 = 5 marks)
Answer:
(i) ` 50,000
(ii) Exempt [Sec. 10(11A)]
(iii) 20
(iv) 25,000
(v) 200
Space to write important points for revision

2016 - June [1] {C} (b) Choose the most appropriate alternative:
(i) A senior citizen having total income consisting of pension and let out
property income aggregating to ` 6 lakhs must have paid advance tax
during the financial year 2018-19 of
(a) NIL
(b) 90% of ` 28,840
(c) 90% of ` 44,290
(d) 90% of ` 39,140
(ii) Mr. Ramji is employed in ABC Ltd who maintained a hospital for
treatment of employees. During the financial year 2018-19, the value
of medical benefit availed by Ramji’s family from the hospital was
` 2,10,000. The amount of medical perquisite chargeable to income
tax would be
(a) ` 2,10,000
(b) ` 1,05,000
(c) ` 21,000
(d) NIL
[Chapter  21] Objective Questions O 7.449

(iii) Mr. Laxman occupied his apartment till December 2018 and thereafter
occupied the quarters provided by the employer. The apartment of
Mr. Laxman was let out at ` 20,000 per month from 1st January, 2019.
The annual value of the property would be
(a) ` 60,000
(b) ` 2,40,000
(c) ` 1,80,000
(d) Nil
(iv) When a company paid ` 5 lakhs to Indian Institute of Technology to
carry on research in a field unrelated to the activity of the company,
the amount eligible for deduction paid by way of donation would be
(a) ` 5,00,000 (100%)
(b) ` 6,25,000 (125%)
(c) ` 7,50,000 (150%)
(d) ` 7,50,000 (150%)
(v) Mr. A has loss from regular business of ` 8 lakhs and income from
speculation business of ` 11 lakhs. His total income chargeable to tax
would be
(a) ` 3,00,000
(b) ` 11,00,000
(c) ` 7,00,000
(d) ` 2,50,000 (1 × 5 = 5 marks)
Answer:
(i) (a) NIL
(ii) (d) NIL
(iii) (a) ` 60,000
(iv) (a) ` 5,00,000 (100%)
(v) (a) ` 3,00,000
Space to write important points for revision
7.450 O Scanner CMA Inter Gr. I Paper 7 (2016 Syllabus)

2016 - June [1] {C} (c) State without indicating reason whether the following
statements are true or false:
(i) Share of a private limited company held for 15 months before its sale
is a long-term capital asset.
(ii) A return of income filed without payment of self-assessment tax is a
defective return.
(iii) Profit from growing and manufacturing tea in India is fully exempted
from income tax under section 10(1) of the Income-tax Act.
(iv) Tax is required to be deducted at source from salary at the time of
payment and not at the time of crediting salary to the account of the
employee.
(v) Capital gain arising from compulsory acquisition of a property under
law is taxable in the year of receipt of compensation or part thereof.
(1 × 5 = 5 marks)
Answer:
(i) False
(ii) True
(iii) False
(iv) True
(v) True

Space to write important points for revision

2016 - Dec [1] {C} (a) Fill up the blanks:


(i) The maximum amount deductible under section 88 TTA in respect of
interest on savings bank account is ` _________.
(ii) Monetary limit for exemption in the case of encashment of earned
leave on superannuation received by private sector employees is
` _________.
(iii) When unrealized rent of ` 50,000 in respect of a let-out property is
realized subsequently, the amount liable to tax would be
` _________.
[Chapter  21] Objective Questions O 7.451

(iv) Interest on enhanced compensation received by Mr. A, a resident


individual is ` 4,00,000 of which 75% pertains to earlier financial years.
The amount of such interest to be included in the total income under
the head ‘income from other sources’ is ` _________.
(v) Medical expenditure of ` 40,000 was incurred by Mr. A on his mother
(being a senior citizen). The amount eligible for deduction under
section 80D would be ` _________. (1 × 5 = 5 marks)
Answer:
(i) ` 10,000
(ii) ` 3,00,000
(iii) ` 35,000
(iv) ` 2,00,000
(v) ` 40,000
Space to write important points for revision

2016 - Dec [1] {C} (b) Choose the most appropriate alternative:
(i) When a person having agricultural lands sells the seeds taken from
such lands in a nursery, which is part of the said lands, the income
from such sale is treated as
(A) Business income
(B) Agricultural income
(C) Income from other sources
(D) None of the above
(ii) An employer has paid medical insurance premium of ` 12,000 in
respect of a salaried employee drawing annual salary of ` 6 lakhs.
The amount of perquisite charged in the hands of employee is
(A) Nil
(B) ` 6,000
(C) ` 12,000
(D) None of the above
7.452 O Scanner CMA Inter Gr. I Paper 7 (2016 Syllabus)

(iii) The rate of depreciation for a block of assets consisting of buildings


used as factory is
(A) 2.5%
(B) 5%
(C) 10%
(D) None of the above
(iv) In case of a Hindu Undivided Family, where the return of income
cannot be signed by the Karta, the same can be signed by
(A) the next senior-most male member.
(B) Karta’s wife.
(C) any male member of the family.
(D) any adult member of the family.
(v) In case of an individual or HUF, to determine whether certain TDS
provisions are attracted, what has to be seen is whether the person is
subject to tax audit under section 44AB in
(A) the immediately preceding financial year.
(B) current year.
(C) last two continuous financial years.
(D) None of the above. (1 × 5 = 5 marks)
Answer:
(i) (b) Agricultural Income.
(ii) (c) NIL.
(iii) (c) 10%.
(iv) (d) Any adult member of family.
(v) (b) The Immediately Preceding Financial Year.
Space to write important points for revision

2016 - Dec [1] {C} (c) Match the following:


(i) Securities Transaction Tax (a) Maximum limit ` 50 lakhs
(ii) Contribution of Employer to (b) Includible as Salary income of
Pension Fund of Central employee
Government
[Chapter  21] Objective Questions O 7.453

(iii) Donation in kind (c) N o t d e d u c t i b l e w h i l e


computing income from
property
(iv) Ground rent (d) Deductible as business
expenditure
(v) Bonds specified in Section (e) Not eligible for deduction
54EC under section 80G
(1 × 5 = 5 marks)
Answer:
(i) (d)
(ii) (b)
(iii) (e)
(iv) (c)
(v) (a)
Space to write important points for revision

2017 - June [1] {C} (a) Find the most suitable alternative for the following:
(i) The number of identities included in the definition of persons is
(a) five
(b) six
(c) seven
(d) eight
(ii) A trust shall not be considered as charitable trust for according the
benefits of Section II, when the commercial activities in the previous
year exceed ` ______.
(a) 10 lakhs
(b) 25 lakhs
(c) 15 lakhs
(d) 30 lakhs
(iii) Deduction available under section 24(a) is ______ of NAV.
(a) 30%
(b) 50%
7.454 O Scanner CMA Inter Gr. I Paper 7 (2016 Syllabus)

(c) 15%
(d) 70%
(iv) Expenditure incurred by a businessman for ready to use software is
entitled to benefit of
(a) 15% as depreciation
(b) 30% as depreciation
(c) 60% as depreciation
(d) 100% as revenue expenditure
(v) The basic exemption limit for a resident super senior citizen above the
age of 80 is
(a) ` 2,00,000
(b) ` 2,50,000
(c) ` 5,00,000
(d) None of the above
(vi) The provisions relating to interest on delay in payment of refund are
given in section
(a) 234A
(b) 234B
(c) 244A
(d) 244B
(vii) Which of the following can be corrected while processing the return of
income under section 143(1)?
(a) Any arithmetical error in the return
(b) Any mistake in the return of income
(c) Any error of principle in the return of income
(d) Any claim by the taxpayer which is against law
(viii) Notice under section 156 is given for
(a) failure to submit return
(b) tax demand
(c) deferment of tax
(d) None of the above
[Chapter  21] Objective Questions O 7.455

(ix) As per Section 271H, where a person fails to file the statement of tax
deducted/collected at source i.e. TDS/TCS return on or before the due
dates prescribed in this regard, then he shall be liable to pay penalty
under section 271H. Maximum penalty that can be levied is ` ______.
(a) 1,00,000, but not exceeding the amount of TDS/TCS.
(b) 2,00,000
(c) 3,00,000
(d) 3,00,000
(x) The threshold exemption limit for Equalization levy is `
(a) ` 5 lakh
(b) ` 3 lakh
(c) ` 2 lakh
(d) ` 1 lakh (1 × 10 = 10 marks)
Answer:
(i) (c) Seven
(ii) There is no option in respect of correct answer as it should be 20% of
gross receipt.
(iii) (a) 30%
(iv) (d) 100% as revenue expenditure
(v) (c) ` 5,00,000
(vi) (c) 244A
(vii) (a) Any arithmetical error in the return
(viii) (b) Tax Demand
(ix) (a) 1,00,000 but not exceeding the amount of TDS/TCS
(x) (d) ` 1 Lakh
Space to write important points for revision

2017 - June [1] {C} (b) Match the following:


(i) Section 87A (A) ` 5,000
(ii) Section 80GG (B) ` 2,500 (or) Actual Tax (w.e.l.)
(iii) Sukanya Samrudhi Scheme (C) ` 1,500
(iv) Minor child exemption (D) 30% deduction
(v) Arrears of rent (E) Section 80C
(1×5=5 marks)
7.456 O Scanner CMA Inter Gr. I Paper 7 (2016 Syllabus)

Answer:
(i) Section 87A (B) ` 2,500 (or) Actual Tax (w.e.l.)
(ii) Section 80GG (A) ` 5,000
(iii) Sukanya Samrudhi Scheme (E) Section 80C
(iv) Minor child exemption (C) ` 1,500
(v) Arrears of rent (D) 30% deduction
Space to write important points for revision

2017 - June [1] {C} (c) State whether true or false:


(i) An Indian company is always resident in India.
(ii) Salary received by a member of Parliament is exempt.
(iii) Income of a self-occupied property cannot be negative.
(iv) Preliminary expenditure are allowed deduction in 10 equal instalments.
(v) Capital gain arises from the transfer of any capital asset.
(1×5=5 marks)
Answer:
(i) True
(ii) False
(iii) False
(iv) False
(v) True
Space to write important points for revision

2017 - June [1] {C} (d) Fill in the blanks:


(i) In case of an Indian citizen who leaves India during the previous year
for employment outside India, the period of 60 days shall be
substituted by _______ days.
(ii) Scholarship received by a student was ` 2,000 p.m. He spends
` 16,000 for meeting the cost of education. The Balance ` 8,000
is _______.
[Chapter  21] Objective Questions O 7.457

(iii) Generally, income is taxable under the head, house property only
when the assesse is the _______ of such house property.
(iv) Salary, bonus, commission or remuneration due to or received by a
working partner from the firm is taxable under the head _______.
(v) Period for holding bonus shares or any other financial asset without
any payment shall be reckoned from the date of _______.
(1×5=5 marks)
Answer:
(i) 182 days
(ii) Exempt
(iii) Owner
(iv) Profits and gains of Business or profession
(v) Allotment
Space to write important points for revision

2017 - Dec [1] {C} (a) Choose the most appropriate alternative:
(i) When Mr. Balu paid royalty to Dr. Peter of Sweden for use of know-
how in India, such payment is
(a) exempt from tax.
(b) accruing in India.
(c) accrues in Sweden.
(d) received in India.
(ii) In the case of foreign company with total income of more than ` 1
crore but less than ` 10 crores the surcharge liveable is at
(a) 5%
(b) 12%
(c) 2%
(d) 1%
(iii) Mr. Hari resident in India received ` 11 lakhs by way of dividend from
Indian companies. Such dividend is
(a) exempt from tax.
(b) taxable at regular rates.
7.458 O Scanner CMA Inter Gr. I Paper 7 (2016 Syllabus)

(c) taxable at maximum marginal rate.


(d) taxable at 10%.
(iv) When an employee receives money on closure of national pension
system trust it is
(a) chargeable to tax.
(b) exempt from tax.
(c) 40% is exempt from tax.
(d) 60% is exempt from tax.
(v) When employer contributes to approved superannuation fund it is
chargeable to tax as perquisite when the contribution exceeds
(a) ` 1,50,000
(b) ` 1,00,000
(c) ` 50,000
(d) ` 20,000
(vi) When the shares are held in unlisted company, it is treated as long
term capital asset when the holding period exceeds
(a) 36 months.
(b) 24 months.
(c) 12 months.
(d) 6 months.
(vii) Long term capital gain arising from transfer of unlisted securities in the
hands of non-resident/foreign company is chargeable to tax at
(a) 10%
(b) 20%
(c) 30%
(d) 40%
(viii) Interest on housing loan taken by individual being his first residential
house is eligible for deduction under section 80EE up to a maximum
of
(a) ` 30,000
(b) ` 50,000
(c) ` 1,50,000
(d) ` 2,00,000
[Chapter  21] Objective Questions O 7.459

(ix) A start-up can claim deduction under section 80-IAC for _________
consecutive years beginning from the year in which the eligible start-
up was incorporated.
(a) 1
(b) 2
(c) 3
(d) 5
(x) When the return of income for the assessment year 2019-20 is filed
under section 139(4), the assessee can revise the return on or before
(a) 31.03.2020
(b) 31.12.2020
(c) 31.03.2021
(d) 31.12.2021 (1 × 10 = 10 marks)
Answer:
(a) (i) (b)
(ii) (c)
(iii) (d)
(iv) (c)
(v) (a)
(vi) (b)
(vii) (a)
(viii) (b)
(ix) (c)
(x) (a)
Space to write important points for revision

2017 - Dec [1] {C} (b) Match the following:


(i) Additional depreciation for plant used for more (a) 40%
than 180 days
(ii) Basic exemption limit of income for resident (b) ` 10,000
individual being senior citizen
(iii) Rate of tax for LLP (c) ` 3,00,000
7.460 O Scanner CMA Inter Gr. I Paper 7 (2016 Syllabus)

(iv) Depreciation for computers (d) 30%


(v) Exemption in respect of Post office SB interest (e) 20%
(1 × 5 = 5 marks)
Answer:
(b)(i) (e) Additional depreciation for plant used for more than 180 days is
20%
(ii) (c) Basic exemption limit of income for resident individual being senior
citizen is ` 3,00,000.
(iii) (d) Rate of tax for LLP is 30%.
(iv) (a) Depreciation for computers is 40%.
(v) (b) Exemption in respect of Post office SB interest is ` 10,000.
Space to write important points for revision

2017 - Dec [1] {C} (c) State whether the following are True or False:
(i) Interest on deposit certificates issued under Gold Monetization
Scheme, 2015 is exempt from tax.
(ii) The monetary limit of ` 5 lakhs in respect of gratuity received by an
employee covered by Payment of Gratuity Act, 1972 is exempt from
tax.
(iii) Medical insurance premium paid by son for parents who are senior
citizens is deductible up to a maximum of ` 35,000.
(iv) In order to avail carry forward loss from house property, the return of
income must be filed before the due date specified in Section 139(3).
(v) 30% of the additional employee cost incurred by the employer is
deductible under section 80JJAA. (1 × 5 = 5 marks)
Answer:
(c)(i) The Statement is true: Interest on deposit certificates issued under
Gold Monetization scheme 2015 is exempt from tax u/s 10(15)(vi).
(ii) The Statement is false: The Monetary limit is ` 10 lakhs.
(iii) The Statement is false: Medical insurance premium paid by son for
parents who are senior citizen is deductible upto a maximum of
` 50,000.
[Chapter  21] Objective Questions O 7.461

(iv) The Statement is false: In order to avail carry forward loss from
house property, if Return of income is furnished after the due date.
(v) Statement is true: 30% of the additional employee cost incurred by
the employer is deductible u/s 80JJAA.
Space to write important points for revision

2017 - Dec [1] {C} (d) Fill up the blanks:


(i) When a director of a company received 30 lakhs by way of non-
compete fee, it is taxable under the head __________.
(ii) When unrealized rent is received based on court decree but at the
time of receipt the property was not owned by the assessee, it is
taxable under the head _________.
(iii) When Mr. Ashwin received ` 20,000 as scholarship for meeting the
cost of education it is ________.
(iv) The Income Computation Disclosure Standards (ICDS) will apply only
when the assesse adopts __________ method of accounting.
(v) Speculation loss can be carried forward for a maximum period of
______ (number of) years after the year of such loss.
(1 × 5 = 5 marks)
Answer:
(d) (i) Profit and Gains of Business or Professional.
(ii) Income from House Property
(iii) Exempt
(iv) Mercantile
(v) Four
Space to write important points for revision

2018 - June [1] {C} (a) Choose the most appropriate alternative:
(i) Which of the following is not a case of deemed ownership of house
property ?
(a) Transfer to spouse for inadequate consideration
(b) Transfer to minor child for inadequate consideration
(c) Co-owner of a Property
(d) None of the above.
7.462 O Scanner CMA Inter Gr. I Paper 7 (2016 Syllabus)

(ii) Where assessment has not been completed, belated income tax return
for the A.Y. 2019-20 can be filed up to:
(a) 31.03.2020
(b) 31.02.2020
(c) 31.03.2021
(d) Cannot be filed belatedly.
(iii) An individual estimates that he is required to pay ` 1,00,000 as
advance tax. By 15th of December, how much amount must be paid by
the individual ?
(a) ` 30,000
(b) ` 75,000
(c) ` 1,00,000
(d) Nil.
(iv) Section 80 RRB the Income -tax Act, 1961 deals with deduction from
gross total income in respect of income by way of
(a) Interest on debentures of a government company
(b) Royalty income on authors
(c) Royalty on patents
(d) Royalty from text-books.
(v) Preliminary expenses that can be amortized under the Income-tax Act
1961 has to be restricted to _______________ of the cost of the
Project.
(a) 5%
(b) 15%
(c) 20%
(d) None of the above.
(vi) Maximum Marginal Rate for the A.Y. 2019-20 is ___________.
(a) 34.5%
(b) 33.99%
(c) 35.88%
(d) None of the above.
[Chapter  21] Objective Questions O 7.463

(vii) Rebate u/s 87A can be claimed by


(a) Any resident
(b) Resident Individual
(c) Any person
(d) Any person other than non resident
(viii) As per Section 115 BBDA dividend from Indian companies is taxable
in the hands of certain recipients at _______________ when the
aggregate dividend exceeds ` ____________.
(a) 10%, 1 lakh
(b) 15%, 10 lakhs
(c) 10%, 10 lakhs
(d) 5%, 5 lakhs
(ix) ICDS VIII deals with _____________.
(a) Government Grants
(b) Securities
(c) Revenue recognition
(d) Construction Contract
(x) Income escaping assessment is covered under section ________.
(a) 144
(b) 156
(c) 143 (3)
(d) 147 (1 × 10 = 10 marks)
Answer:
(i) (c) Co-owner of a property
(ii) (a) 31-03-2020
(iii) (b) ` 75,000
(iv) (c) Royalty on patents
(v) (a) 5%
(vi) (c) 33.535%
(vii) (b) Resident Individual
(viii) (c) 10%, 10 lakhs
(ix) (b) Securities
(x) (d) 147
Space to write important points for revision
7.464 O Scanner CMA Inter Gr. I Paper 7 (2016 Syllabus)

2018 - June [1] {C} (b) Match the following :


(i) ALTERNATE MINIMUM TAX (A) SECTION 44AD
(ii) RETURN BY WHOM TO BE VERIFIED (B) SECTION 263
(iii) REVISION BY COMMISSIONER (C) SECTION 140
(iv) PRESUMPTIVE TAX (D) SECTION 80EE
(v) ` 50,000 (E) SECTION 115JC
(1× 5 = 5 marks)
Answer:
(b)
(i) Alternate Minimum Tax (E) Section 115JC
(ii) Return by Whom to be Verified (C) Section 140
(iii) Revision by Commissioner (B) Section 263
(iv) Presumptive Tax (A) Section 44AD
(v) ` 50,000 (D) Section 88EE
Space to write important points for revision

2018 - June [1] {C} (c) State whether True or False :


(i) All incomes that accrue to a minor child will be included in the total
income of that parent whose total income is greater.
(ii) Caution money forfeited by the assessee is taxable in the year of
forfeiture under the head capital gains.
(iii) Paintings are not considered as personal effects in the context of
“capital asset” definition.
(iv) In the hands of a manufacturer, factory building newly constructed is
not eligible for additional depreciation.
(v) Income from assets acquired by spouse out of pin money or
household savings is not subject to clubbing. (1× 5 = 5 marks)
[Chapter  21] Objective Questions O 7.465

Answer:
(i) False
(ii) False
(iii) True
(iv) True
(v) True.
Space to write important points for revision

2018 - June [1] {C} (d) Fill in the blanks :


(i) Deduction under section 80GGB in respect of house rent paid is
applicable to _____________.
(ii) Unabsorbed depreciation shall be allowed to be carried forward for any
number of years and such carried forward unabsorbed depreciation
may be set off against any income, other than _____.
(iii) Income referred to in Sec. 68 to Sec. 69D shall be taxable @
_____________. (Excluding SC and Cess)
(iv) _______________ received by an electoral trust shall be exempted.
(v) Income from sub-letting of a house property by a salaried employee is
taxable under the head _____________. (1 × 5 = 5 marks)
Answer:
(i) Individual (Note: Deduction for house rent paid is under Section
80GG, not under Section 80GGB.)
(ii) Income from salary, winning from lotteries, cross word puzzles etc.
(iii) 60%
(iv) Any voluntary contributions.
(v) Income from other sources.
Space to write important points for revision

2018 - Dec [1] {C} (a) Choose the most appropriate alternative:
(i) Short-term capital gain on sale of listed shares (STT paid) in a
recognized stock exchange is chargeable to income-tax @ ____%.
(a) 10
(b) 15
7.466 O Scanner CMA Inter Gr. I Paper 7 (2016 Syllabus)

(c) 20
(d) 30
(ii) When the total income of an individual exceeds ` 50 lakhs, the
surcharge is payable @
(a) 5%
(b) 7%
(c) 10%
(d) 12%
(iii) When the amount is withdrawn from National Pension System Trust,
it is chargeable to tax to the extent the withdrawal exceeds _______%
of the contribution of the assessee.
(a) 10
(b) 25
(c) 15
(d) 20
(iv) Ms. Jothi (aged 23) got married and left India to join her husband in
the United Kingdom on 10.06.2018. She had never left India earlier.
Her residential status for the assessment year 2019-20 is:
(a) Resident and ordinarily resident
(b) Resident but not ordinarily resident
(c) Non-resident
(d) None of the above
(v) While computing TDS on salary paid to employees, the losses given
below to the applicable extent would be considered by the employer:
(a) Loss from business
(b) Loss from house property
(c) Long-term capital loss
(d) Short-term capital loss
(vi) When tax is not deducted at source on annual rent of ` 2 lakhs paid to
landlord by a company, the amount liable for disallowance under
section 40(a) (ia) is
(a) Nil
(b) ` 2,00,000
(c) ` 20,000
(d) ` 60,000
[Chapter  21] Objective Questions O 7.467

(vii) When the assessee has loss, from house property, the maximum
amount of such loss eligible for set of against other permissible
incomes would be
(a) ` 30,000
(b) ` 1,50,000
(c) ` 2,00,000
(d) No Limit
(viii) When a capital asset was acquired on 01.04.1981 and sold in June,
2018, the cost of acquisition or the fair market value of the asset as on
_____, at the option of the assessee is to be adopted for indexation
purpose:
(a) 01.04.2012
(b) 01.04.2002
(c) 01.04.1992
(d) 01.04.1982
(ix) When a motor car is sold for ` 12 lakhs by a dealer to a buyer holding
PAN the amount of tax collectible as source shall be_______.
(a) ` 12,000 (1%)
(b) ` 24,000 (2%)
(c) ` 1,20,000 (10%)
(d) Nil
(x) Cash donation given to a charitable trust (approved under section
80G) is eligible for deduction under that section, when the amount of
donation does not exceed `________.
(a) 2,000
(b) 5,000
(c) 7,000
(d) 10,000 (1 × 10 = 10 marks)
2018 - Dec [1] {C} (b) Match the following (Sufficient to give the
corresponding item in column 3 for column 1; reproducing columns 2 and 4
are not required):
1 2 3 4
(i) Depreciation on patents (A) 40%
7.468 O Scanner CMA Inter Gr. I Paper 7 (2016 Syllabus)

(ii) Amount received by an individual (B) Valuation of inventories


as a loan in a reverse mortgage
(iii) Interest partner on capital (C) 25%
(iv) Depreciation on solar power (D) Exempted, since there is
generating system no transfer
(v) ICDS II (E) Allowed up to 12% p.a.
(1 × 5 = 5 marks)
2018 - Dec [1] {C} (c) State whether following statements are True or False:
(i) Cost of self-generated goodwill of business is deemed to be Nil.
(ii) Reimbursement of ordinary medical expenses by the employer is fully
exempted.
(iii) Where capital gain arises to an individual from the transfer of a capital
asset, being immovable property under a joint development
agreement, the capital gain is chargeable to tax in the previous year
in which the certificate of completion for whole or part of the project is
issued by the competent authority.
(iv) In order to avail carry forward of unabsorbed depreciation, the
assessee must furnish the return of income within the due date
specified in section 139(1).
(v) In order to claim exemption under section 54B, the agricultural land,
which is transferred, must have been used by the assessee or his
parents for at least 3 years prior to the date of transfer.
(1 × 5 = 5 marks)
2018 - Dec [1] {C} (d) Fill in the blanks.
(i) The total income computed will have to be rounded off to the nearest
multiple of ` ________.
(ii) Domestic company means a/an ________ company.
(iii) Additional depreciation on factory building for ` 30 lakhs, acquired by
a manufacturer on 1st Dec., 2018 is ________.
(iv) Unabsorbed depreciation can be carried forward for ________ years.
(v) An assessee, who receives leave encashment during continuation of
his service, can also claim ________. (1 × 5 = 5 marks)
[Chapter  21] Objective Questions O 7.469

Table Showing Marks of Compulsory Questions

Year 14 14 15 15 16 16 17 17 18 18
J D J D J D J D J D

Objective 11 13 13 15 15 25 25 25 25

Total 11 13 13 15 15 25 25 25 25
IMPORTANT AMENDMENTS

IMPORTANT AMENDMENTS IN THE INCOME TAX ACT APPLICABLE


FROM 01.04.2017
The Finance Act, 2017, which has made a history of being a enacted law on
31st March, 2017 , has made far reaching changes in Direct Taxes. Specially
in case of provisions of cash transaction which are going to affect almost
every person of the society belonging to business and non-business class.
I am trying to make to make summary of important changes which are
applicable from 01.04.2017;
1 Limit for Cash Expenditure both, revenue and capital expenditure
reduced from `. 20,000/- to ` 10,000/- per day in aggregate per person.
Capital expenditure paid other than account payee cheque and other
banking channel beyond ` 10,000/- will not be taken into account for
depreciation purposes. However, the cash payment limit for fright etc.
remains the same at ` 35,000/-
2 No person shall receive an amount of 2 Lac or more, by cash (Sec.
269ST).
(a) in aggregate from a person in a day; or
(b) in respect of a single transaction; or
(c) in respect of transactions relating to one event or occasion.
3 No TCS on cash sales exceeding ` 2 Lac.
4 In case of turnover Less than ` 2 Crores:
 Non Cash Business Turnover: Sales through Digital, Online, cheque,
Bank etc. Net Profit will be taken as 6% of Turnover/Gross Receipt.
 Cash Business Turnover: Net Profit will be taken as 8% of
Turnover/Gross Receipt.
5 Basic Exemption Limit is ` 2,50,000/-
 5% Slab for Income from ` 2.50 lakh to ` 5 Lakh. Tax Rebate U/s
87A
 ` 2500/- for total income upto ` 3.50 Lakh.

7.470
Important Amendments O 7.471

 Surcharge @ 10% of the tax for Individuals having total income


exceeding ` 50 Lac but below ` 1 Crore.
 Surcharge @ 15%, if total income exceeds ` 1 crore.
6 Payment of Rent – ` 50,000/- per month by any Individual or HUF (not
covered in Tax Audit) – deduct TDS @ 5%.
7 Capital Gain in respect of Land & Buildings –
 Period for long term Capital Gain is reduced from 3 years to 2 years.
 Base year shifted to 01.04.2001 for all assets including Immovable
property.
Earlier it was 01.04.1981
8 Corporate Tax Rate for the accounting year 2017-18 for companies
whose annual turnover upto ` 50 crores (in the account year 2015-16)
is reduced to 25%.
9 No change in slab for firm @ 30%.
10 Donations in cash exceeding ` 2000/- will be not be eligible for deduction
under section 80G. Trusts accepting 80G donations may advise their
donors to give donations exceeding ` 2000 vide cheque / RTGS / digital
modes.
11 Sale of unquoted shares to be taxed at (deemed) fair value.
12 In absence of PAN of the buyer of specified goods, the rate of TCS will
be twice of the extent rate or 5%, whichever is higher.
13 From financial year 2017-18, if Return is not filed within due date, late
filing fee of ` 5,000/- for delay up to 31st December, and ` 10,000
thereafter.
14 Every person who is eligible to obtain AADHAR number should quote
such number, on or after 1 July 2017, in the Return of income.
Furthermore, every person who has been allotted PAN as on 1st July
2017 must intimate the AADHAR number to the Tax Authority, failing
which, PAN allotted to such person shall be deemed to be invalid.
Be careful linking of AADHAR with PAN is not possible, if name as
AADHAR and PAN is not same. Hence, please step forward to rectify if
required.
7.472 O Scanner CMA Inter Gr. I Paper 7 (2016 Syllabus)

15 Where Sec.12AA registered trusts modify their objects clause, they need
to apply within 30 days to CIT for approval of the modified clauses.
SI.No. Financial Year (FY) Assessment Year (AY) Cost Inflation
Index
1 2001-02 2002-03 100
2 2002-03 2003-04 105
3 2003-04 2004-05 109
4 2004-05 2005-06 113
5 2005-06 2006-07 117
6 2006-07 2007-08 122
7 2007-08 2008-09 129
8 2008-09 2009-10 137
9 2009-10 2010-11 148
10 2010-11 2011-12 167
11 2011-12 2012-13 184
12 2012-13 2013-14 200
13 2013-14 2014-15 220
14 2014-15 2015-16 240
15 2015-16 2016-17 254
16 2016-17 2017-18 264
17 2017-18 2018-19 272
18 2018-19 2019-20 280
June - 2018
CMA Inter Gr. I
Paper - 7
Direct Taxation

All questions are compulsory. In Question No. 1.


all sub-questions are compulsory.

1. (a) Choose the most appropriate alternative:


(i) Which of the following is not a case of deemed ownership of
house property ?
(a) Transfer to spouse for inadequate consideration
(b) Transfer to minor child for inadequate consideration
(c) Co-owner of a Property
(d) None of the above.
(ii) Where assessment has not been completed, belated income tax
return for the A.Y. 2019-20 can be filed up to:
(a) 31.03.2020
(b) 31.02.2020
(c) 31.03.2021
(d) Cannot be filed belatedly.
(iii) An individual estimates that he is required to pay ` 1,00,000 as
advance tax. By 15th of December, how much amount must be
paid by the individual ?
(a) ` 30,000
(b) ` 75,000
(c) ` 1,00,000
(d) Nil.
(iv) Section 80 RRB the Income -tax Act, 1961 deals with deduction
from gross total income in respect of income by way of
(a) Interest on debentures of a government company
(b) Royalty income on authors
(c) Royalty on patents
(d) Royalty from text-books.
7.473
7.474 O Scanner CMA Inter Gr. I Paper 7 (2016 Syllabus)

(v) Preliminary expenses that can be amortized under the Income-


tax Act 1961 has to be restricted to _______________ of the
cost of the Project.
(a) 5%
(b) 15%
(c) 20%
(d) None of the above.
(vi) Maximum Marginal Rate for the A.Y. 2019-20 is ___________.
(a) 34.5%
(b) 33.99%
(c) 35.88%
(d) None of the above.
(vii) Rebate u/s 87A can be claimed by
(a) Any resident
(b) Resident Individual
(c) Any person
(d) Any person other than non resident
(viii) As per Section 115 BBDA dividend from Indian companies is
taxable in the hands of certain recipients at _______________
when the aggregate dividend exceeds ` ____________.
(a) 10%, 1 lakh
(b) 15%, 10 lakhs
(c) 10%, 10 lakhs
(d) 5%, 5 lakhs
(ix) ICDS VIII deals with _____________.
(a) Government Grants
(b) Securities
(c) Revenue recognition
(d) Construction Contract
(x) Income escaping assessment is covered under
section ________.
(a) 144
(b) 156
(c) 143 (3)
(d) 147 (1 × 10 = 10 marks)
Question Paper O 7.475

(b) Match the following :


(i) ALTERNATE MINIMUM TAX (A) SECTION 44AD

(ii) RETURN BY WHOM TO BE VERIFIED (B) SECTION 263

(iii) REVISION BY COMMISSIONER (C) SECTION 140

(iv) PRESUMPTIVE TAX (D) SECTION 80EE

(v) ` 50,000 (E) SECTION 115JC


(1× 5 = 5 marks)
(c) State whether True or False :
(i) All incomes that accrue to a minor child will be included in the
total income of that parent whose total income is greater.
(ii) Caution money forfeited by the assessee is taxable in the year
of forfeiture under the head capital gains.
(iii) Paintings are not considered as personal effects in the context
of “capital asset” definition.
(iv) In the hands of a manufacturer, factory building newly
constructed is not eligible for additional depreciation.
(v) Income from assets acquired by spouse out of pin money or
household savings is not subject to clubbing. (1× 5 = 5 marks)
(d) Fill in the blanks :
(i) Deduction under section 80GGB in respect of house rent paid
is applicable to _____________.
(ii) Unabsorbed depreciation shall be allowed to be carried forward
for any number of years and such carried forward unabsorbed
depreciation may be set off against any income, other
than _____.
(iii) Income referred to in Sec. 68 to Sec. 69D shall be taxable
@ _____________. (Excluding SC and Cess)
7.476 O Scanner CMA Inter Gr. I Paper 7 (2016 Syllabus)

(iv) _______________ received by an electoral trust shall be


exempted.
(v) Income from sub-letting of a house property by a salaried
employee is taxable under the head _____________.
(1 × 5 = 5 marks)
2. (a) (i) Compute the tax liability of Sri A. Harichandraprakash whose
total income is
(a) ` 49,62,500
(b) ` 51,00,000
Note : (Source of income is Salary only) (5 marks)
(ii) Explain the following concepts
 Tax Planning
 Tax Avoidance
 Tax Evasion (3 marks)
(b) Two brothers Rama and Shankar are co-owners of a house property
with equal shares. The property was constructed during the
Financial year 1999-2000. The property consists of 8 identical units
and is situated at Salem. During the Financial Year 2018-19 each
owner occupied 1 unit for residence and balance 6 units were let out
at a rent of ` 14,000 per unit per month. The municipal value of
property is ` 9,00,000 and municipal tax are 10% of municipal value,
paid during the year. The other expenses are as follows:
(i) Repairs ` 90,000
(ii) Insurance premium paid ` 15,000
(iii) Interest payable on loan taken ` 3,50,000
One of the let out remained vacant for 4 months during the year.
Rama could not occupy his unit for 6 months as he was transferred
to Bangalore. He does not own any other house. The other income
of Rama and Shankar are ` 3,50,000 and ` 1,80,000 respectively for
the Financial Year 2018-19.
Question Paper O 7.477

The co-owners received during the year ` 1,40,000 as unrealized


rent for 2015-2016 and ` 50,000 as arrears of rent.
Compute the income under the head “Income from House Property”
and total income of the two brothers for the Assessment Year 2019-
20. (7 marks)
3. (a) Mr. Ashwin of Chennai sold a vacant site for ` 30 lakhs to Mr. Raina
on 01.05.2018. The value of land for stamp duty purposes was ` 25
lakhs. The vacant site was acquired in April, 2001 for ` 3 lakhs. The
fair market value of the vacant site on 01.04.2002 was ` 4 lakhs.
The entire sale consideration plus a housing loan of ` 38 lakhs from
a nationalized bank was availed for acquiring a residential building
for ` 68 lakhs in Pune on 01.07.2018. The stamp duty paid for the
purpose of acquisition was ` 2,90,000. The property was let out for
a monthly rent of ` 10,000 from 01.07.2018. Interest on housing loan
during the year till its closure, amounted to ` 2,80,000.
Mr. Ashwin sold yet another vacant site for ` 28 lakhs on
21.01.2019. This vacant site was acquired in October, 2016 for ` 20
lakhs. He utilized the entire sale proceeds realized in January 2019
for repaying the housing loan.
His other incomes are (i) Income from business (computed)
` 3,90,000 and (ii) Bank interest of ` 60,000 from term deposits and
` 15,000 from SB account.
Compute the total income of Mr. Ashwin for the assessment year
2019-20.
Cost inflation index : F.Y. 2002-03 = 105; F.Y. 2016-17 = 264; F.Y.
2018-19 = 280. (7 marks)
(b) State whether the following transactions attract tax deduction at
source (TDS) provisions and the rate of tax and the amount of tax
deductible in applicable cases:
7.478 O Scanner CMA Inter Gr. I Paper 7 (2016 Syllabus)

(i) Interest on recurring deposit of ` 12,000 paid by a nationalized


bank to Mr. Dhoni.
(ii) Prize amount of ` 8,000 paid by Excellence Ltd. to Mr. Saha a
winner of crossword puzzle contest conducted by the company.
(iii) Commission of ` 21,000 paid to Kumble and Co. by Dravid Co.
Ltd. for purchase of raw materials.
(iv) Chandra Ltd. paid ` 40,000 per month as generator rent from 1st
August, 2018 and up to 31st March, 2019 to Mr. Shastri.
(8 marks)
4. (a) Mr. Kamal employed in Rajini Mfg. Co. Ltd. Mumbai as General
Manager furnishes the following information for the year ended
31.03.2019:
Particulars Amount (`)

Basic salary (per month) 50,000

Dearness Allowance (eligible for retirement benefits) 80% of basic salary

House Rent Allowance (per month) 10,000

Rent paid by him ` 15,000 per month for 6 months and ` 20,000 per
month for balance 6 months (at Mumbai)

City Compensatory Allowance (per month) 2,500

Medical reimbursements (annual) 13,000

Gymkhana club annual membership fee reimbursed by employer 20,000

Mobile phone bill reimbursed by the employer 37,500


(Used for both official and personal use)

Motor car (cubic capacity of engine 2.2 litres ) owned by the 85,800
employee but the maintenance expenses fully met by the employer
(Motor car was used both for personal and official use)

Cash gift paid by the employer in appreciation of performance on 30,000


01.01.2019
Question Paper O 7.479

Contribution to recognized provident fund :

Employee 1,20,000

Employer 90,000

Contribution to National Pension Trust:

Employee 60,000

Employer 55,000

Medical insurance premium paid by means of uncrossed cheque 18,000

You are requested compute the total income of Mr. Kamal for the
assessment year 2019-20. (10 marks)
(b) State with brief reasons whether the following are agricultural
income either in whole or in part:
(i) Purchase of standing sugarcane crop by Mr. Amin for ` 2 lakhs
and after cutting the canes, selling them for ` 2,50,000.
(ii) Income from milk dairy run by Mr. Raj in his agricultural lands
` 50,000.
(iii) Income from sale of plants ` 1,00,000 earned by Mr. Jain who
maintains a nursery by name Soundarya Nursery.
(iv) Income from sale of rubber ` 3,20,000 realised by Mr. Ram Nair
who owns rubber estate and cultivates rubber.
(v) Income from gracing of cattles allowed in the land owned by Mr.
Richard ` 60,000. (1 × 5 = 5)
5. (a) Ahuja Industries Ltd. engaged in manufacturing activity and
generation of power, gives you the following information for the year
ended 31st March, 2019:
7.480 O Scanner CMA Inter Gr. I Paper 7 (2016 Syllabus)

Description Op. WDV Acquisition/ New Sold during


(01.04.2017) Date acquisition the year
used from

Plant ` 5,00,000 ` 60,000 01.11.2018 ` 80,000


(01.05.2018) (01.01.2019)

Windmill – ` 60,00,000 01.09.2018 –


(01.06.2018)

Computer ` 3,00,000 ` 90,000 01.11.2018 ` 40,000


(01.10.2018) (Office use) (01.03.2019)

Patent – ` 4,00,000 01.12.2018 –


(01.12.2018)
Compute the depreciation and additional depreciation for the
assessment year 2019-20. The computation must be such that the
same is most beneficial to the assessee. (9 marks)
(b) State with one line reason, the due date for filing the return of
income in the following cases:
(i) Mr. Solkar, engaged in trade, has total turnover of ` 220 lakhs
for the year ended 31.03.2019.
(ii) Krish Srikanth, an advocate, has aggregate professional
receipts of 12,40,000 opting to admit income under section
44ADA.
(iii) Mr. Abid Ali, having 5 heavy goods transport vehicles which are
run on hire, opting to admit income under section 44AE.
(iv) M/s Jayantilal & Mankad, a firm engaged in hotel business with
annual turnover of ` 130 lakhs preferring to offer income based
on applicable presumptive provisions.
(v) Vaman Kumar Charitable Trust registered under section 12AA
having total income of ` 12 lakhs. (before giving effect to the
provisions of Section 11 and 12, and before seeking
accumulation of income for application in the future years.)
(6 marks)
Question Paper O 7.481

6. (a) The Profit & Loss Account of ABC & Associates, a partnership firm
for the previous year 2018-19 is given below :
Particulars ` ` Particulars `

Establishme 96,00,000 Gross Profit 156,40,000


nt and other
expenses

Interest to Profit on 2,80,000


partners sale of
@15% e q u i t y
shares (Sold
after 2 years
through
recognized
s t o c k
exchange)

A 1,80,000

B 2,40,000

C 1,20,000 5,40,000

Salary to Rent from 1,20,000


w o rk i n g house
partners property

A 4,80,000

B 3,60,000 8,40,000
7.482 O Scanner CMA Inter Gr. I Paper 7 (2016 Syllabus)

Interest on 20,000
bank deposit

Profit on 2,40,000
sale of
e q u i t y
shares (after
10 months
through
recognized
s t o c k
exchange)

Net Profit 53,20,000

1,63,00,000 1,63,00,000
Additional information:
(i) Establishment expenses include bonus ` 2,40,000 which was
paid on 30-12-2019.
(ii) The firm is eligible for deduction under section 80-IC.
(iii) Establishment expenses also included securities transaction tax
of ` 2,000.
Compute the tax liability of the firm for the assessment year 2019-
20. Assume that no extension of time has been granted u/s 139 (1)
for filing the return of income. (10 marks)
(b) Aswini’s accounts are not required to be audited under section
44AB. He furnished his return of income for Assessment Year 2019-
20 on 1st August, 2019. He has the following losses during the
previous year 2018-19:
Loss from house property let out: ` 12,000
Loss from business: ` 60,000
Unabsorbed depreciation: ` 15,000
Short-term capital loss from sale of shares: ` 8,000
Question Paper O 7.483

State, with reason, whether Aswini is entitled to carry forward above


losses and unabsorbed depreciation. (5 marks)
7. (a) Discuss the taxability or otherwise in the hands of the recipients:
(i) PQR Private Limited issued 15,000 shares at ` 150 per share
(face value ` 100 per share). The fair market value of the share
is ` 130 per share.
(ii) Mr. Sakshitha received a sum of ` 92,000 being proceeds at the
time of maturity of a life insurance policy (taken 5 years back)
and ` 1,10,000 being proceeds of maturity value of a Key-man
insurance policy.
(iii) Nilay, a member of his father’s HUF, gifted a house property to
the HUF. The stamp duty value of the house is ` 8 lakhs.

(iv) Rashmi received a cell phone worth ` 60,000 as gift from her
friend on the occasion of her birthday.
(v) On the occasion of her marriage Tripti received cash gifts of
` 1,30,000, which includes ` 60,000 from her friends.
(2 × 5 = 10)
(b) ABC & Co., a partnership firm, consisted of 4 equal partners up to
31.03.2018. It had accumulated business losses of ` 8 lakhs and
unabsorbed depreciation of ` 6 lakhs relating to assessment year
2017-18. On 01.04.2018 one partner retired. The firm, for the
previous year ended 31st March, 2019, made a turnover of ` 150
lakhs. The firm wishes to opt for presumptive taxation.
The entire sale proceeds were realized through banking channel.
Compute the total income of the firm for the assessment year 2019-
20. (5 marks)
8. Write short notes on any three of the following:
(a) Fee for delay in furnishing the return of income;
(b) Scrutiny assessment
(c) ICDS : Accounting policies
(d) Adjustments during the course of processing of return of income u/s
139 (1). (5 × 3 = 15)
7.484 O Scanner CMA Inter Gr. I Paper 7 (2016 Syllabus)

December - 2018
CMA Inter Gr. I
Paper - 7
Direct Taxation
Question No. 1. which is compulsory and any five from Question Nos. 2 to 8.
1.(a) Choose the most appropriate alternative:
(i) Short-term capital gain on sale of listed shares (STT paid) in a
recognized stock exchange is chargeable to income-tax @ ____%.
(a) 10
(b) 15
(c) 20
(d) 30
(ii) When the total income of an individual exceeds ` 50 lakhs, the
surcharge is payable @
(a) 5%
(b) 7%
(c) 10%
(d) 12%
(iii) When the amount is withdrawn from National Pension System
Trust, it is chargeable to tax to the extent the withdrawal exceeds
_______% of the contribution of the assessee.
(a) 10
(b) 25
(c) 15
(d) 20
(iv) Ms. Jothi (aged 23) got married and left India to join her husband
in the United Kingdom on 10.06.2018. She had never left India
earlier. Her residential status for the assessment year 2019-20 is:
(a) Resident and ordinarily resident
(b) Resident but not ordinarily resident
(c) Non-resident
(d) None of the above
Question Paper O 7.485

(v) While computing TDS on salary paid to employees, the losses


given below to the applicable extent would be considered by the
employer:
(a) Loss from business
(b) Loss from house property
(c) Long-term capital loss
(d) Short-term capital loss
(vi) When tax is not deducted at source on annual rent of ` 2 lakhs paid
to landlord by a company, the amount liable for disallowance under
section 40(a) (ia) is
(a) Nil
(b) ` 2,00,000
(c) ` 20,000
(d) ` 60,000
(vii) When the assessee has loss, from house property, the maximum
amount of such loss eligible for set of against other permissible
incomes would be
(a) ` 30,000
(b) ` 1,50,000
(c) ` 2,00,000
(d) No Limit
(viii) When a capital asset was acquired on 01.04.1981 and sold in
June, 2018, the cost of acquisition or the fair market value of the
asset as on _____, at the option of the assessee is to be adopted
for indexation purpose:
(a) 01.04.2011
(b) 01.04.2001
(c) 01.04.1991
(d) 01.04.1981
(ix) When a motor car is sold for ` 12 lakhs by a dealer to a buyer
holding PAN the amount of tax collectible as source shall
be_______.
(a) ` 12,000 (1%)
7.486 O Scanner CMA Inter Gr. I Paper 7 (2016 Syllabus)

(b) ` 24,000 (2%)


(c) ` 1,20,000 (10%)
(d) Nil
(x) Cash donation given to a charitable trust (approved under section
80G) is eligible for deduction under that section, when the amount
of donation does not exceed `________.
(a) 2,000
(b) 5,000
(c) 7,000
(d) 10,000 (1 × 10 = 10 marks)
(b) Match the following (Sufficient to give the corresponding item in
column 3 for column 1; reproducing columns 2 and 4 are not
required):
1 2 3 4
(i) Depreciation on patents (A) 40%
(ii) Amount received by an individual (B) Valuation of inventories
as a loan in a reverse mortgage
(iii) Interest partner on capital (C) 25%
(iv) Depreciation on solar power (D) Exempted, since there is
generating system no transfer
(v) ICDS II (E) Allowed up to 12% p.a.
(1 × 5 = 5 marks)
(c) State whether following statements are True or False:
(i) Cost of self-generated goodwill of business is deemed to be Nil.
(ii) Reimbursement of ordinary medical expenses by the employer is
fully exempted.
(iii) Where capital gain arises to an individual from the transfer of a
capital asset, being immovable property under a joint development
agreement, the capital gain is chargeable to tax in the previous
year in which the certificate of completion for whole or part of the
project is issued by the competent authority.
Question Paper O 7.487

(iv) In order to avail carry forward of unabsorbed depreciation, the


assessee must furnish the return of income within the due date
specified in section 139(1).
(v) In order to claim exemption under section 54B, the agricultural
land, which is transferred, must have been used by the assessee
or his parents for at least 3 years prior to the date of transfer.
(1 × 5 = 5 marks)
(d) Fill in the blanks.
(i) The total income computed will have to be rounded off to the
nearest multiple of ` ________.
(ii) Domestic company means a/an ________ company.
(iii) Additional depreciation on factory building for ` 30 lakhs, acquired
by a manufacturer on 1st Dec., 2018 is ________.
(iv) Unabsorbed depreciation can be carried forward for ________
years.
(v) An assessee, who receives leave encashment during continuation
of his service, can also claim ________. (1 × 5 = 5 marks)
2. (a) Mr. Barun furnishes you the following information for the year ended
31st March, 2019:
SI. Particulars `
No.
(i) Pension received in India from a former employer in United 1,80,000
Kingdom (UK)
(ii) Income from business in Singapore (Controlled from India) 1,00,000
(iii) Interest on company deposit in Singapore (credited in bank 80,000
account held there)
(iv) Profit from business in Kolkata controlled from UK 2,00,000
(v) Income from tea cultivation in Sri Lanka 3,00,000
(vi) Income from property in Singapore but received in Malaysia 2,50,000
Compute the total income of Mr. Barun, where he is (i) an ordinarily
resident in India; (ii) a resident but not ordinarily resident in India,
and (iii) a non-resident. (9 marks)
7.488 O Scanner CMA Inter Gr. I Paper 7 (2016 Syllabus)

(b) Mr. Chaturvedi, Delhi has 3 house properties in various parts of


India. The details are given below:
Location of property Delhi Chandigarh Kolkata
Usage Self occupied Let out Let out
Amount ` Amount ` Amount `
Rent received NIL 3,60,000 1,80,000
Fair rent 2,40,000 3,00,000 1,50,000
Municipal value 2,10,000 2,40,000 1,20,000
Standard rent 1,80,000 2,10,000 90,000
Municipal tax - Due 20,000 40,000 30,000
Municipal tax - paid by NIL NIL 20,000
the assessee
Interest on moneys 2,80,000 1,40,000 1,50,000
borrowed
Note: All the properties were acquired/constructed after 01.04.2011.
You are required to compute the income of Mr. Chaturvedi
chargeable under the head “Income from house property” for the
assessment year 2019-20. (6 marks)
3. (a) Mr. Subramani is Senior Manager (Finance) of VKS Steel Ltd. The
particulars of his emoluments for the year ended 31.03.2019 are
given below:
Basic Salary ` 60,000 per month
Dearness Allowance ` 40,000 per month (30% is for
retirement benefit)
Annual performance Incentive ` 1,80,000
House Rent Allowance ` 10,000 per month
Mr. Subramani pays rent of ` 20,000 per month for a flat occupied
from 1st November, 2018 at Erode, Tamil Nadu.
Question Paper O 7.489

He received gift voucher of ` 6,000 from the employer on the


occasion of his marriage anniversary.
The employer provided him a motor car (cubic capacity of the engine
exceeds 1.6 litres) without chauffeur with effect from 1st December,
2018. Running and maintenance expences of ` 30,000 were fully
borne by the employer. The car is used by Mr. Subramani both for
official and private purposes.
The employer paid the following premiums for Mr. Subramani:
(i) Medical insurance premium ` 12,000
(ii) Life insurance premium ` 15,000
(iii) Accident insurance premium ` 10,000
Tax on employment paid to Erode Municipal Corporation by Mr.
Subramani ` 5,000.
Compute the income chargeable to tax under the head “Salaries” in
the hands of Mr. Subramani for Assessment Year 2019-20.
(9 marks)
(b) Mr. Manish, a resident in India, has the following incomes for the
year ended 31st March, 2019:
Income from sale of tea grown and
manufactured in India ` 4,00,000
Income from growing and manufacturing
rubber in India ` 5,00,000
Income from agricultural operations in Sri Lanka
(cultivated paddy) ` 1,00,000
Income derived from sale of coffee grown,
cured, roasted and grinded in India ` 2,00,000
Determine the quantum of income which is regarded as agricultural
income and non-agricultural income in the hands of Mr. Manish for
the assessment year 2019-20. (6 marks)
7.490 O Scanner CMA Inter Gr. I Paper 7 (2016 Syllabus)

4. (a) The summarised financial position of Purva India (P) Ltd. as on


31/12/2018 is as under:
Liabilities Amount ` Assets Amount `
Equity share capital of 8,00,000 Land 6,00,000
` 10 each
Preference share capital 1,00,000 Building (WDV as per 3,00,000
Income tax Act)
Reserves 2,00,000 Machinery (WDV as per 4,00,000
Income tax Act)
Loan Creditors 6,00,000 Current Assets 10,40,715
Creditors 6,00,000
Provision of Dividend 40,715
Distribution Tax
23,40,715 23,40,715
Additional Information:
The Company went into liquidation on the balance sheet date; all
current assets and building realized at book value. The realized
money was applied towards payment of outside liabilities including
Dividend Distribution Tax, and there after the preference
shareholders.
Mr. Utkarsh is a holder of 10% equity shares and 20% preference
shares of the company. Equity shares were originally acquired by
him 16.08.2003 at face value. However, he had subscribed to
preference shares on 01.04.2018, which were issued at par. He
received a part of land (MV ` 5,00,000) and cash (for preference
share) ` 20,000.
Compute the capital gain in hands of the company and Mr. Utkarsh.
(8 marks)
Question Paper O 7.491

(b) Ms. Pinky submits the following particulars for the year ended 31st
March, 2019:
Sl. Particulars `
No.
(i) Loss from let out residential building-computed 3,00,000
(ii) Arrear rent from a commercial building received
during the year (commercial property had been sold
in June, 2016) 40,000
(iii) Textile business discontinued from 31st October
2017–Brought forward business loss of Assessment
Year 2015-16 60,000
(iv) Profit from chemical business of current year 5,50,000
(computed)
(v) Bad debt written off in the Assessment Year 2014-
15 relating to textile business recovered during the
year consequent to Court decree 1,00,000
(vi) Long-term capital gain on sale of shares (STT paid)
in recognized stock exchange on 23.05.2018 90,000
(vii) Speculation business in oil seeds–profit 3,00,000
(viii) Winning from lottery (Gross) 11,00,000
(ix) Loss from the activity of owning and maintaining
race horses 2,10,000
You are required to compute the total income of Ms. Pinky and also
ascertain the amount of losses that can be carried forward.
(7 marks)
7.492 O Scanner CMA Inter Gr. I Paper 7 (2016 Syllabus)

5. (a) Mr. Bhushan, engaged in manufacture of chemicals, furnishes his


Manufacturing, Trading and Profit & Loss Account for the year
ended 31st March, 2019 as under:
Particulars ` Particulars `
To Opening stock 3,40,000 By Sales 1,14,00,000
To Purchases 1,00,20,000 By Closing stock 19,00,000
To Manufacturing 10,40,000
Expenses
To Gross Profit 19,00,000
1,33,00,000 1,33,00,000
To Salary 4,30,000 By Gross Profit 19,00,000
To Bonus 80,000 By Discount 25,000
To Bank term loan 90,000 By Agricultural Income 1,50,000
interest
To Factory rent 1,20,000 B y D i v i d e n d f r o m 75,000
Indian Companies
To Office rent 2,10,000
To Administration 3,30,000
Expenses
To Net Profit 8,90,000
21,50,000 21,50,000
Additional Information:
(i) The total turnover of Mr. Bhushan for the Financial Year 2017-18
was ` 132 lakhs.
(ii) Salary includes ` 1,80,000 paid to his daughter. The excess
payment considering her qualification and experience is
ascertained as ` 40,000.
(iii) Factory rent was paid to his brother. Similar portions are let out
to others by him for a rent of ` 96,000 per annum.
Question Paper O 7.493

(iv) No tax was deducted at source from the office rent paid during
the year.
(v) Purchases include ` 70,000 paid by cash to an agriculturist for
purchase of grains (being raw material).
(vi) Depreciation allowable under section 32 of the Income-tax Act,
1961 amounts to ` 45,000 for assets held as on 01.04.2018.
During the year, a machinery costing ` 5,00,000 was acquired on
01.07.2018 and was put to use from 15.10.2018.
(vii) Administration expenses include commission paid to a purchase
agent of ` 12,000 for which no tax was deducted at source.
(viii) The following expenses debited above were not paid till
31.03.2019 and up to the ‘due date’ for filling the return specified
in section 139(1)
(I) Term loan interest of ` 35,000;
(II) Demurrages to Indian Railways for using their clearing yard
beyond stipulated hours (disputed by the assessee) forming
part of manufacturing expenses ` 30,000.
Compute the income of Mr. Bhushan chargeable under the head
“Profits and gains of business or profession” for the Assessment
Year 2019-20: (10 marks)
(b) Mr. Raghavan, aged 57, is a person with disability. He furnishes you
the following information for the year ended 31.03.2019.
(i) Income from business (computed) ` 7,00,000
(ii) Dividend from an Indian company ` 10,50,000
(iii) Interest on Saving bank account with a nationalized bank
` 17,000
(iv) Medical insurance premium paid by account payee cheque
For self ` 20,000
For brother, wholly dependent on ` 15,000
him
Compute his total income for the Assessment Year 2019-20.
(5 marks)
7.494 O Scanner CMA Inter Gr. I Paper 7 (2016 Syllabus)

6. CMA Anup Banerjee is in practice as Cost Accountant. He follows


mercantile basis of accounting. His income & expenditure account for
the year ended 31st March, 2019 is given below:
Expenditure ` Receipts `
Salary and stipends 10,50,000 Professional fees 45,00,000
Bonus to staff 1,00,000 Share of profit from a 2,00,000
partnership firm
Meeting Conference and 2,50,000 Interest on fixed 27,000
seminars deposit in a bank
(Net of TDS)
Fees to consultants 1,50,000 H o n o r a r i u m f o r 54,000
valuation of answer
papers of various
institutes (Net of TDS)
Travelling and conveyance 4,60,000
Rent for office premises 6,00,000
Provision for bad debts 40,000
Depreciation 1,45,000
Provision for income tax 7,02,000
Excess of income over 12,84,000
expenditure
47,79,000 47,79,000
Other information:
(i) Depreciation as per the Income - tax Act ` 2,00,000.
(ii) Salary and stipends include ` 40,000 paid to one trainee for
passing CMA final examination with rank.
(iii) Bonus to staff was paid in November, 2019.
Question Paper O 7.495

(iv) In the financial year 2017-18, a sum of ` 15,000 was due to a


consultant, which was allowed. The said amount was paid on 14th
May, 2018 in cash.
Compute the total income of CMA Anup Banerjee for the
Assessment Year 2019 -20. He has not opted for presumptive
taxation scheme under section 44ADA. The due date for
furnishing the return of income under section 139(1) may be taken
as 31st October, 2019. (15 marks)
7. (a) Explain with reasons, the taxability of the following transactions
under the head “Income from other sources”:
(i) Veena received interest of ` 5,00,000 on additional
compensation on account of compulsory acquisition of land
acquired few years back. Year-wise break up of interest
received:
` 1,20,000 for the Financial Year 2016-17, ` 2,40,000 for the
Financial Year 2017 -18 and ` 1,40,000 for the Financial Year
2018-19. (3 marks)
(ii) Gopal has shareholding (with voting rights) of 12% in Krishna
Pvt. Ltd., a closely held company. He received loan of
` 2,50,000 from the company on 1st May, 2018, for which he
furnished adequate security to the company. The accumulated
profit of the company at that time was ` 1,75,000. Gopal repaid
the loan on 30th Sept., 2018. (3 marks)
(iii) Family pension of ` 60,000 received by Sreelekha, window of
Late Vikram. (1 mark)
(iv) Vasant, whose salary income is ` 4,00,000 has received a
cash gift of ` 60,000 from a charitable trust registered under
section 12AA for meeting his medical expenses. (1mark)
(b) State the due dates for payments of advance tax, along with the
quantum of amount payable in each instalment. Present your
answer in the form of a table. (7 marks)
7.496 O Scanner CMA Inter Gr. I Paper 7 (2016 Syllabus)

8. Write short notes on any three of the following:


(a) Verification of return of income in the case of an individual, HUF and
political party
(b) Any five transactions where quoting PAN is mandatory
(c) ICDS - I on “Accounting Policies”
(d) Best judgment assessment under section 144 (5 × 3=15 marks)

You might also like